0% found this document useful (0 votes)
5K views907 pages

Pyq Fmge

Copyright
© © All Rights Reserved
We take content rights seriously. If you suspect this is your content, claim it here.
Available Formats
Download as PDF, TXT or read online on Scribd
0% found this document useful (0 votes)
5K views907 pages

Pyq Fmge

Copyright
© © All Rights Reserved
We take content rights seriously. If you suspect this is your content, claim it here.
Available Formats
Download as PDF, TXT or read online on Scribd
You are on page 1/ 907

ARISE

MEDICAL ACADEMY
Building Better Doctors

FMGE RE C A L L’ S WiTH EXPLANATiONS

RISE
FMGE iNSiGHT
2017-DEC
2018-JUNE,DEC
2019-JUNE,DEC
2020-AUG,DEC
2021-JUNE
2020-AUG,DEC
2019-JUNE,DEC
2018-JUNE,DEC Arise Medical Academy

2017-DEC Add us on FB Page : - /arisemedacademy


www.arisemedicalacademy.com
ARISE
MEDICAL ACADEMY
Building Better Doctors

ARISE - Hyderabad
+91 7680929292
040 2351 5252
# 2ⁿ Floor, Above Indian Bank,
Opp. Olive Hospital,
Kakatiya Nagar Colony,
Mehdipatnam, Hyderabad - 500 028
arisemedicalacademy@gmail.com

ARISE - Kerala (Kannnur)


+91 8136932666
+86 15778411369
# 2ⁿ Floor, Kingdom Tower,
Manna, Taliparamba,
Kannur, Kerala.
arisemedicalacademy.kerala@yahoo.com

ARISE - Delhi
+91 9100728811

Goutham Nagar,

arisemedicalacademy@gmail.com
Follow us on
f cy d
Arise Medical Academy
Add us on FB Page : - /arisemedacademy
www.arisemedicalacademy.com
MCI DEC 2017
1. Which of the following is a Selective estrogen receptor modulator -
a. Fulvestrant
b. Flutamide
c. Tamoxifen
d. Clomiphine citrate

Explanation :
a. Fulvestrant – selective estrogen receptor downregulators (or) pure
estrogen antagonists
b. Flutamide - selective antagonist of androgen receptor
c. Tamoxifen – selective estrogen receptor modulator. Used as a major
indication for treatment of breast cancer. Has beneficial effect on bone
and lipid profile but increase the risk of endometrial cancer and
thromboembolism.
d. Clomiphine citrate – binds to both ER- alpha and ER – beta and acts
as a pure estrogen antagonists.
2. which of the following Stone is insensitive to alkaline PH of urine?
a. Xanthine stones
b. Cystine stones
c. Struvite stones
d. Calcium stones

Explanation:
a. Xanthine stones – due to deficiency of xanthine oxidase leading to
xanthine uria. May be either radio opaque or radio lucent .
b. Cystine stones – Due to the defect in renal absorption of amino acid
cysytine leading to cystinuria. Radio opaque due to the presence of
sulphur. They are hexagonal shaped or benzene like crystals.
c. Struvite stones – aka triple phosphate stones. Formed mainly after
infection by proteus, which causes alkaline urine leads to precipitation
of phosphate crystals. Drugs like acetazolamide can also cause
struvite stones.
d. Calcium stones - most common type of stone is calcium oxalate
stones. They are radio opaque.
3. Which of the following is Ocp induced liver malignancy ?
a. Hepatic adenoma
b. Hepatocellular carcinoma
c. Hemangioma
d. Focal nodular hyperplasia
Explanation :
a. Hepatic adenoma: Most common symptomatic benign liver tumour.
Most common associated factors are ocp and steroids. High risk of
rupture and malignant transformation.
4. Which of the following methods of contraception is used within 1 st 6 months of
lactation?
a. Combined Ocps
b. Copper T- A380
c. Progesterone only pills
d. Condoms

Explanation:
a. Combined ocps – are contraindicated during lacation .
b. Copper T-A380 – can also be used during lactation but most common
complication is vaginal bleeding.
c. Progesterone only pills - used mainly within 72 hrs of intercourse to
reduce the risk of pregnancy and is considered as safe during
lactation.
d. Condoms – not much effective because of its high failure rate 2-14
per HWY.
5. Prosopagnosia means ?
a. Inability to write
b. Inability to calculate
c. Inability to identify face
d. Inability to speak
Explanation :
a. Inability to write – agraphia
b. Inability to calculate – acalculia
c. Inability to identify faces – prosopagnosia
d. Inability to speak - aphasia
6. Layers of spinal anaesthesia ?
a. Skin>subcutaneous>supraspinatus ligament>interapinatus
ligament> ligamentum flavum
b. Skin> subcutaneous>supraspinatus ligament >ligamentum
flavum>interspinous ligament
c. Skin>subcutaneous>interspinous ligament> supraspinatus
ligament>ligamentum flavum
d. Skin >supraspinatus ligament >subcutaneous >interspinous ligament
>ligamentum flavum
7. Which of the following is the correct order of endometrial blood supply?
a. Arcuate>radial>basal>spiral
b. Radial>arcuate >spiral > basal
c. Radial >basal > arcuate > spiral
d. Arcuate > radial > spiral > basal
8. SIADH is caused by ?
a. Head trauma
b. Tb meningitis
c. Small cell carcinoma of lung
d. All of the above
Explanation :
SIADH : syndrome of inappropriate ADH.
CAUSES: Tumors like- head and neck cancer
Small cell cancer of lung
Bronchial carcinoma
Carcinoid tumour
Head trauma
Drugs like – carbamazapine
Clofibrate
Chlorpropamide
Vincristine
Infections like – pneumonia
TB
Meningitis
SIADH is characterized by gain of water and hence dilutional hyponatremia is seen.
Rapid correction of hyponatremia produces “central pontine myelinosis”.
Rx: doc is – demiclocycline
9. RDA of calcium in elderly women?
a. 600
b. 1200
c. 1800
d. 2400
Explanation :
Elderly women – 1200mg
Pregnancy – 1300 - 1500mg
10. Calculate the volume of fluid to be replaced in a burn patient of TBS 40% of 50 kg
within 1 st 8 hrs?
a. 4 L
b. 8 L
c. 12 L
d. 16 L
Explanation :
Volume of fluid to be given for burns patient is calculated by parkland formula.
Parkland formula= TBSA% * weight (kg) * 4
= 40% * 50*4
= 8L
In 1st 8 hrs- 50% of fluid should be given ie, 4L
In next 16 hrs – remaining 50% of fluid should be given .
11. Sun burst appearance in osteosarcoma is due to?
a. Abnormal calcium deposits
b. Periosteal reaction
c. Bony destruction
d. Highly vascular
Explanation :
Osteosarcoma:
2nd most common malignant bone tumour, arise from metaphysis of lower end of
femur. Gross appearance shows mutton leg appearance . periosteal reaction is
present.
X ray findings- sun ray appearance
Codmans triangle
Alkalaine phosphatase is highly increased.
Rx: pre- operative chemotherphy ,limb saving surgery , post – operative
chemotherphy .
12. Motor molecule are all of the following except?
a. Kinesin
b. Dyenin
c. Myosin
d. Actin
13. P value in null hypothesis?
a. 0.005 %
b. 0.05%
c. 0.5%
d. 5%
Explanation :
Test of significance:
 state the null hypothesis ( ie, state that both groups are same)
 state the level of significance (alpha – 0.05)
 choose the appropriate statistical test
 find “P” value
 if “P” value is <0.05 , reject null hypothesis ie, both groups are different .
 if “P” value is >0.05 ,accept null hypothesis ie, both groups are same.

14. Framingham heart study is an example of?


a. Retrospective cohort
b. Prospective cohort
c. Case control study
d. Cross sectional study
Explanation:
Framingham heart study was initiated to study the relationship between the number
of risk factors and subsequent development of cardiovascular disease. Is an
example of cohort study.
15. In Which of the following insentives to mother is 1000Rs and Asha 400rs?
a. Rural area ,low performing states
b. Urban area , low performing states
c. Rural area , high performing states
d. Urban area , high performing states
Explanation : RURAL
URBAN
Mother Asha Mother Asha
Low 1400 600 1000 400
performing
states
High 700 600 600 400
performing
states

16. Which of the following is regarding the work of Asha?


a. Conduct delivery
b. Provides resource for anganwadi workers
c. Builds house hold toilets
d. For immunization of children
Explanation :
ASHA: ACREDITED SOCIAL HEALTH ACTIVIST
 Asha must be a resident of a village preferably the age group of 25-45.
 1 Asha works for 1000 population.
 Minimum education required for Asha is- 8 th pass
 Training of Asha is done by ANM and AWW for minimum duration of – 23
days .
 Impact indicators of Asha:- infant mortality rate
 Child malnutrition
 Number of TB/Leprosy case detection.
 Escort services of RCH
 Helps in construction of household toilets.

17. Which of the following cancer is highest reported to death worldwide ?


a. Ovarian ca
b. Breast ca
c. Stomach ca
d. Lung ca
Explanation :
The most common causes of cancer death are cancers of:
 Lung (1.69 million deaths)
 Liver (788 000 deaths)
 Colorectal (774 000 deaths)
 Stomach (754 000 deaths)
 Breast (571 000 deaths)
Reference : http://www.who.int/mediacentre/factsheets/fs297/en/.

18. Vaccine contraindicated in pregnancy?


a. Rubella
b. Yellow fever
c. BCG
d. Hep b
Explanation :
All live vaccines are contraindicated in pregnancy except, yellow fever vaccine and
OPV. Women are advised not to receive the rubella vaccine during pregnancy as a
safety precaution based on the theoretical possibility of a live vaccine causing
disease, in this case, ‘congenital rubella syndrome’.
19. Black colour triage indicates ?
a. Unsalvageable
b. Highly prioritized
c. Can wait for some time
d. Ambulatory care
Explanation : red – highly prioritized
Yellow – can wait for some time
Green – ambulatory care
Black - moribund/ dead
20. Which of the following is the correct order of Chain of survival ?
a. Early access> early chest compression>early airway management
>early defibrillator>early hospital admission
b. Early chest compression> early hospital admission >early access
>early airway management > early defibrillator
c. Early access>early airway management>early chest compression
> early defibrillator > early hospital admission
d. Early access>early defibrillator > early hospital admission > early
airway management > early chest compression
21. Which of the following hormone helps in the regulation of 1,25
dihydroxycholecalciferol ?
a. Renin
b. Angiotensin
c. Parathormone
d. Vit D
Explanation :

22. Ranolazine is not contraindicated in giving with which of the following drugs?
a. Ketoconazole
b. Diltiazem
c. Morphine
d. Clozapine
Explanation:
Dose of ranolazine should be limited to 500 mg twice daily in patients on
MODERATE CYP3A INHIBITORS ,including diltiazem , verapamil , erythromycin ,
fluconazole .
Do not use ranolazine with STRONG CYP3A INHIBITORS, including ketoconazole,
itraconazole, clarithromycin etc.
23. Glycemic index of glucose?
a. 0.5
b. 1
c. 1.5
d. 2
24. Corocoid process of scapula is a example of?
a. Pressure epiphysis
b. Traction epiphysis
c. Atavistic epiphysis
d. Aberrant epiphysis
Explanation :

25. Imbecile?
a. Less than 20
b. 25-34
c. 25-49
d. 50-70
Explanation : less than 20 – profound
0 -24 - idiot
25 – 49 – imbecile
50 – 70 – moron
26. Keratic precipitate is seen in which layer ?
a. Epithelium of cornea
b. Decemets membrane
c. Stomach
d. Endothelium of cornea
Explanation:
Keratic precipitate (KP) is an inflammatory cellular deposit seen on corneal
endothelium. Acute KPs are white and round in shape whereas old KPs are faded
and irregular in shape. Mutton-fat KPs are large in shape and are greasy-white in
color and are formed from macrophages and epithelioid cell. They are indicative of
inflammatory disease. Mutton fat Kps are due to granulomatous iridocyclitis. Another
variant called red KPs may be seen in hemorrhagic uveitis.
27. All of the following shows Increased anion gap are all except?
a. Aspirin poisoning
b. Uremia
c. Diarrhoea
d. Diabetic acidosis
Explanation : diarrhoea is eg of normal anion gap.
28. Respiratory papillomatosis is caused by?
a. HPV
b. EBV
c. H. Influenza
d. Klebsiella
Explanation :
Recurrent respiratory papillomatosis (RRP), which is caused exclusively
by human papilloma virus (HPV), is a rare condition characterized by recurrent
growth of benign papillomata in the respiratory tract. The papillomata can occur
anywhere in the aerodigestive tract but most frequently in the larynx, affecting both
children and adults. The management of this entity remains still challenging since no
specific definitive treatment exists.
29. Positive swartz sign is seen in ?
a. Meneier’s disease.
b. Active otosclerosis
c. Acoustic neuroma
d. Malignant otitis externa
Explanation:
Otosclerosis is an abnormal bone growth in the middle ear that causes hearing
loss. Swartz sign is a pink reflex , seen on the promontory through intact tympanic
membrane ,in the area of oval window. It indicates active focus with increased
vacularity. Otosclerosis usually occurs during pregnancy. Management for active
otosclerosis is surgery and inactive otosclerosis is sodium fluoride theraphy.
30. which of the following complement deficiency leads to Meningococcal meningitis?
a. C3b
b. C1
c. C5-C9 [MAC]
d. C5a

Explanation :
Recurrent meningococcal meningitis is caused by neisseria infections due to
deficiency of MAC complex (c5-9).
C1 esterase deficiency leads to hereditary angioedema.
C3b – is a opsonin ,helps in opsonisation of the pathogens.
C5a – chemokine.
31. In which of the following cattle tract appearance is seen?
a. ARMD (age related macular degeneration )
b. Diabetic retinopathy
c. Retinal detachment
d. CRAO
Explanation :
Atherosclerosis related thrombosis at the level of lamina cribrosa is most common
cause (80%) followed by embolization from heart and carotid artery (20%) producing
a sudden painless loss of vision.
C/F : sudden painless loss of vision ,cherry red spot at fovea, segmentation of blood
in blood vessels producing cattle tract sign .
Rx : ocular massage is most effective method
Decrease IOP by giving manitol
Inhalation of carbogen (95% of O2 and 5% of CO2).
32. Treatment for nasopharyngeal carcinoma?
a. Surgery
b. Chemotheraphy
c. Radiotheraphy
d. Both chemo and radiotheraphy
Explanation :
Mc tumour of head and neck which give rise to secondaries with occult primary . Mc
site is fossa of Rossenmuller . Mc type is non keratinizing undifferentiated carcinoma
followed by keratinazing carcinoma. Mc manifestation is upper neck swelling usually
unilateral due to cervical lymphadenopathy . earliest lymphnode to be involved is
retropharyngeal lymphnode. Mc cranial nerve palsy in nasopharyngeal carcinoma is 5
th followed by 6 th and also 9 th and 10 th CN. Treatment of choice is irradiation
(external beam radiotheraphy) as it is highly radiosensitive. In stage 1 and 2 only
radiotherapy is done and in 3 rd and 4 th stage chemoradiation is the treatement.
33. At 24 weeks of gestation the fundus height is at the level of ?
a. Pubic symphysis
b. Umbilicus
c. Xiphoid sternum
d. Manubrium of sternum

Explanation:
At 12 weeks fundus is above pubic symphysis
At 24 weeks fundus is at the level of umbilicus
At 36 weeks fundus is at the level of xiphi sternum
At 38 weeks fundus is just below the level of xiphi sternum
34. All of the following is true regarding TOF EXCEPT?
a. Ventricular septal defect
b. Normal position of aorta
c. Pulmonary stenosis
d. Right ventricular hypertrophy

Explanation:
 Tetralogy of fallot is a cyanotic congenital heart defect.
 A hole in the wall between your heart’s main pumping chambers (ventricular septal
defect or VSD).
 A valve between your heart and lungs that is too narrow (pulmonary stenosis or PS)
 A right heart chamber with walls that are too thick (right ventricular hypertrophy)
 A major blood vessel (aorta) that is misplaced or moved (overriding aorta).

35. All of the following are involved in triple arthodesis except?


a. Talotibular
b. Subtalar joint
c. Calcaneocuboid joint
d. Talonavicular joint
Explanation:
Triple arthrodesis is a surgical procedure whose purpose is to relieve pain in
the rear part of the foot, improve stability of the foot, and in some cases correct
deformity of the foot, by fusing of the three main joints of the hindfoot: the subtalar
joint, calcaneocuboid joint and the talonavicular joint.
36. Spontaneous or post trauma watery nasal discharge is due to?

a. Allergic rhinitis
b. CSF rhinorrhea
c. atrophic rhinitis
d. Rhinosporidiosis
Explanation:
It is a flow of CSF from nose (due to CSF leakage from sub arachnoid space into
nasal cavity). Usual sites of CSF leak are from cribriform plate >frontal sinus > floor
of the anterior cranial fossa. Mc cause of CSF rhinorrhoea is iatrogenic trauma
during surgery.
C/F: unilateral clear watery discharge dripping on looking down, which increases on
coughing , sneezing or exertion.
37. Optic nerve diameter of adult is?
a. O.5 mm
b. 1 mm
c. 1.5 mm
d. 2 mm
38. Which of the following is the carbonic anhydrase inhibitor is anti glaucogenic?
a. Dorsalamide
b. Lantoprost
c. Brimonidine
d. Dipivefrine

Explanation :
Dorsalamide is a carbonic anhydrase inhibitor which is used in the management of
glaucoma to decrease the IOP by decreasing the production of aqueous humour.
Should not be given to patients who are allergic to sulphur.
39. 1 st trimester miscarriage is most commonly due to?
a. Chromosomal abnormalaities
b. Incompetent cervix
c. Fibroids
d. Infections

Explanation :
The most common cause of recurrent 1 st trimester abortion is chromosomal
abnormalities of which mc is balanced translocation in mother. Mc cause of
recurrent 2 nd trimester abortion is incompetent cervix. Infections can’t cause
recurrent abortions.
40. Quintan fever is caused by?
a. Tularaemia
b. Coxiella burnetti
c. H.ducreyi
d. Bartonella
Explanation:
Trench fever/quintan fever is a clinical syndrome caused by infection with
Bartonella Quintana. The disease is classically a 5-day fever. The onset of
symptoms is sudden with high fever, severe headache, back pain and leg pain and a
fleeting rash. Recovery takes a month or more. Relapses are common.
41. Most common site of carcinoma esophagus?
a. Upper 1/3
b. Middle 1/3
c. Lower 1/3
d. Cricopharyx.
Explanation :
Mc site of esophageal carcinoma in india – middle 1/3 rd
Mc site of esophageal carcinoma in worldwide – lower 1/3 rd
Mc type – squamous cell carcinoma
42. Recurrent laryngeal nerve doesn’t supply which of the following laryngeal
muscles?
a. Cricoarytenoid
b. Thyroarytenoid
c. Cricothyroid
d. Interarytenoids
Explanation :
All the laryngeal muscles are supplied my recurrent laryngeal nerve except
cricothyroid which is supplied by external laryngeal nerve , branch of superior
laryngeal nerve .

43. A boy with fever with chills , Swollen tonsil and it is pushed medially is due to
which of the following?
a. Quinsy
b. Lushcka tonsil
c. Parapharyngeal abcess
d. Retropharyngeal abcess

Explanation :
Quinsy/peritonsillar abcess is the collection of pus between the fibrous capsule of the
tonsil. Mc occurs as a complication of acute tonsillitis and generally unilateral.
Caused by GABHS /mixed flora. Managed by i.v fluids ,antibiotics and analgesics. If
not responding to antibiotics, then interval tonsillectomy is done after 6 weeks of
quinsy.

44. Karyotype of down syndrome is ?


a. Trisomy 13
b. Trisomy 18
c. Trisomy 21
d. Trisomy 22

45. From which of the following parafollicular “c” cells are derived from ?
a. Ultimobrachial body
b. Neuroectoderm
c. splanchnic Mesoderm
d. pharyngeal arches
Explanation :
Parafollicular “c” cellas are derived from neural crest > 4 th ventral endodermal
pouch.
46. Which of the following organ undergoes liquefactive necrosis?
a. Cerebrum
b. Kidney
c. Small intestine
d. Pancreas

Explanation :
Brain – undergoes liqefactive necrosis due to ischemia /abcess formation in brain .
All other organs of the body undergo coagulative necrosis eg.kidney ,small intestine
etc.due to coagulation of proteins.
Pancreas and breast are eg of fat necrosis.
47. Launge hauman classification is for ?
a. Shoulder joint
b. Hip joint
c. Knee joint
d. Ankle joint

Explanation :
The Lauge-Hansen classification system is used for the classification of the ankle
injuries based on injury mechanisms which have predictable patterns and imaging
findings. Along with the Weber classification, these systems are useful tools for
describing and classifying ankle injuries.

Classification
The Lauge-Hansen system uses two-word descriptors:

 the first word describes the position of the foot at the time of injury (i.e. supination
or pronation)
 the second word describes the deforming force direction (i.e. abduction,
adduction, or external rotation)

48. Which of the following is the most cochlear toxic drug ?

a. kanamycin
b. gentimicin,
c. amikacin,
d. streptomycin
Explanation :
Neomycin has the greatest cochleotoxic effect of all antibiotics.
Kanamycin are close to neomycin in cochleotoxic potential and are capable of
causing profound, permanent hearing loss while sparing balance.
Streptomycin tends to cause more damage to the vestibular portion than to the
auditory portion of the inner ear.

49. Creatinine is formed from which of the following amino acids.?


a. Aspartate, glycine,lucine
b. Alanine ,glutamate ,methionine
c. Aspartate ,lycine , glutamate
d. Arginine ,glycine, methionine

50. Wernike korsakoff is due to the deficiency of?


a. Vit b2
b. Vit b12
c. Vit b1
d. Vit –b6

Explanation :
Wernicke encephalopathy and koraskoff psychosis is due to deficiency of vit B1
thymine usually seen in alcoholics.
51. Identify the structure shown in arrow.
a. Asterion
b. Pterion
c. Inion
d. Glabella
Explanation :

52. GCS score for dead patient is?


a. 0
b. 3
c. 5
d. 15

Explanation :
GCS : max score is 15 and minimum score is 3 . GCS for a dead patient is 3.
53. Best to evaluate adequate fluid resuscitation in a burn patient?
a. Urine output
b. Thirst
c. Fall in Blood pressure
d. Shock

Explanation :
The key for monitoring fluid resuscitation in burns patient is urine output. It
should be between 0.5 and 1 ml/kg body weight per hour. If the urine output is below
this ,the infusion rate should be increased by 50%.
54. 21 teeth present at the age of?

a. 5
b. 6
c. 10
d. 12
Explanation :

The complete set (ie,20) primary teeth have to come in by the time the child is
2 1/2 to 3 yrs old. Rule of thumb is that ~ 4 teeth will erupt for every 6 months of life.
Lower teeth erupt before upper teeth with teeth in both jaws usually erupting in pairs.
Primary teeth are smaller and whiter in color than permanent teeth that will follow.
After age 4 ,the jaw and facial bones of child begin to grow and creating the space
between the primary teeth. Between the age of 6 and 12, a mixture of booth primary
teeth and permanent teeth reside in mouth .
55. Another Term of lesbianism ?
a. Fetischism
b. Eonism
c. Urolangia
d. Tribadism
Explanation :
Lesbianism /tribadism – gratification of sexual desire of a woman by another women.
Urolangia – sexual excitement by sight or odor of urine or faeces
Eonism /transvestism – pleasure of wearing clothes of opposite sex.
Fetischism – sexual gratification by article of opposite sex.
56. Movement of superior rectus muscle?
a. Elevation , intortion, adduction
b. Depression, extortion, adduction
c. Elevation , extortion, abduction
d. Depression , intortion, abduction

Explanation:
Muscles Function
Primary secondary tertiary
Abduction - -
Lateral rectus
Inferior rectus Adduction - -
Inferior rectus Depression Extortion Adduction
Superior rectus Elevation Intortion Adduction
Inferior oblique Extortion Elevation Abduction
Superior oblique Intortion Depression Abduction

57. A Man with bulla on face and neck, flaccid bulla, spreading and nikolsky sign
positive is seen in ?
a. Bullous pemphigoid
b. Pemphigus vulgaris
c. Pemphigus violaceous
d. Dermatitis herpetiformis
Explanation :
Disease Biopsy features DIF
 Subepidermal oedema/blister.
Bullous pemphigoid Linear IgG,
 Mixed perivascular infiltrate with eosinophils. C3 in BMZ
(lamina
lucida).
Dermatitis  Subepidermal vesicles. Granular IgA
herpetiformis  Mixed papillary infiltrate with eosinophils. in tips
of papillae.

 Suprabasal acantholytic vesicle (tombstone


Pemphigus vulgaris IgG within
pattern). suprabasal
 Mixed perivascular infiltrate with eosinophils. intercellular
spaces.
Bulla spread and nikolsky sign is positive.
Pemphigus  Subcorneal acantholytic vesicle (tombstone IgG within
foliaceus pattern). upper
 Mixed perivascular infiltrate with eosinophils. epidermal
intercellular
spaces.

58. waxy hyaline membrane deposits in alveoli is seen in?


a. ARDS
b. Interstitial lung disease
c. Asthma
d. Emphysema
Explanation :

Acute Respiratory distress syndrome (ARDS) of the newborn is an acute


lung disease caused by surfactant deficiency, which leads to alveolar collapse and
noncompliant lungs. Previously known as hyaline membrane disease, this condition is
primarily seen in premature infants younger than 32 weeks’ gestation. RDS has
improved with the increased use of antenatal steroids to improve pulmonary maturity,
early postnatal surfactant therapy to replace surfactant deficiency.
59. Vitamin k undergoes -?
a. Gamma decarboxylation of arginine
b. alpha decarboxylation of arginine
c. Gamma carboxylation of glutamate
d. Alpha carboxylation of glutamate
Explanation :
Vitamin – K : is the coenzyme for gamma carboxylation of glutamate in
postsynthetic modification of calcium binding proteins. Vitamin K is also important in
synthesis of bone calcium binding proteins. Menadione is a water soluble form of Vit
– k.
60. Which of the following drug is the antidote for methanol poisoning ?
a. Flumazenil
b. Fomepizole
c. Pralidoxime
d. Protamine sulphate
Explanation :
Fomepizole is used as an antidote in confirmed or suspected methanol or
ethylene glycol poisoning. Fomepizole is a competitive inhibitor of alcohol
dehydrogenase, the enzyme that catalyzes the initial steps in the metabolism of
ethylene glycol and methanol to their toxic metabolites.
61. GFR is best measured by which of the following substances?
a. Inulin
b. Paraaminohippuric acid
c. Insulin
d. ADH
Explanation :
Kidney function is determined by measuring glomerular filtration rate (GFR)--
the volume of plasma that the kidneys filter through the glomeruli per unit time. The
“gold standard” for measuring GFR is through the use of inulin. Para aminohippuric
acid is used to measure the renal plasma flow.

62. Which of the following is most common cause for oral cancer ?
a. Alcohol
b. Betel nut chewing
c. Sun exposure
d. Infections like EBV,HPV etc.
Explanation :
Most cases of oral cancer (~90%) in south east asia are linked to tobacco
chewing and tobacco smoking. In India mc form of tobacco chewing is betel liquid
which usually consist of betel leaf ,areca nut , lime and tobacco.

63. In which of the following route fentanyl should not be given ?

a. Intramuscular
b. Intravenous
c. Transdermal
d. Intrathecal
Explanation :
Fentanyl can be given as slow iv or im , it can also be given intrathecal
administration.
64. Site of gluconeogenesis is ?
a. Muscle
b. Liver
c. Kidney
d. Intestine
Explanation:
Gluconeogenesis is the synthesis of glucose from non carbohydrate
compounds. It takes place in liver and kidney . The liver accounts for 90% of
gluconeogenesis in the body and the remaining 10% occurs in kidney and other
tissues of the body. It takes place in both cytoplasm and mitochondria of liver.
65. Hand foot mouth disease is caused by ?
a. HINI
b. Adenovirus
c. Rotavirus
d. Coxsackie virus
Explanation:
Coxsackie virus is single strand + ve sense RNA virus ,which spread
by feco oral route. HFMD, caused by coxsackieviruses, usually causes fever,
malaise, rash, and small blisters that ulcerate. The most frequent locations for
the blisters/ulcers are on the palms of the hand, soles of the feet, and in the mouth.
There is no specific treatment or vaccine available for coxsackievirus infections.
66. Period of viability of the fetus is?
a. 210
b. 220
c. 230
d. 240
Explanation :
Previously ,28 weeks of geststion is considered as a period of viability ie,210 days.
Now it has been changed to 22 weeks to consider fetus as viable. The best option
here is 210 days.

67. Cephalic index of 75- 80 represents?


a. Aryans
b. Negroes
c. Europeans
d. Mangolians
Explanation :
Types of skull Cephalic index Race
Dolico cephalic 70 - 75 Aryans, Negroes
Mesaticephalic 75 - 80 Europeans , Chinese
Brachycephalic 80 - 85 Mongols

68. Pyronaridine belongs to which class of drugs?


a. Antibiotic
b. Anti malarial
c. Anti viral
d. Anti fungal
Explanation :
Pyronaridine has high potency against Plasmodium falciparum, including
chloroquine-resistant strains. Studies in various animal models have shown
pyronaridine to be effective against strains resistant to other anti-malarials, including
chloroquine. Resistance to pyronaridine appears to emerge slowly and is further
retarded when pyronaridine is used in combination with other anti-malarials, in
particular, artesunate. Pyronaridine toxicity is generally less than that of chloroquine,
though evidence of embryotoxicity in rodents suggests use with caution in
pregnancy.
69. G6PD deficiency leads to ?
a. Shock
b. Galactosemia
c. Hemolytic anemia
d. Hereditary fructose intolerance
Explanation :
G6PD deficiency is an inherited condition in which the body doesn't
have enough of the enzymeglucose-6-phosphate dehydrogenase, or G6PD, which
helps red blood cells (RBCs) function normally. This deficiency can cause hemolytic
anemia, usually after exposure to certain medications, foods, or even infections.
70. Identify the structure shown in image indicated by arrow.
a. 4 th ventricle
b. Cerebral aqueduct
c. Cerebellum
d. Thalamus
Explanation : The arrow in the above image indicates 4 th ventricle.

71. Which of the following is the Antibody for neonatal lupus ?


a. Anti centromere antibody
b. Anti ds DNA
c. Anti Ro
d. Anti mitochondrial antibody
Explanation :
Anti centromere antibody – CREST SYNDROME
Anti ds DNA – SLE
Anti RO – sjogren’s syndrome , neonatal lupus
Anti mitochondrial antibody – primary biliary cirrhosis

72. Definition of antiport system ?


a. Two molecules simultaneously enter the cell via same channel
b. One molecules enter and one molecules leave the cell via same
channel simultaneously .
c. Two molecules simultaneously leaves the cell via same channel .
d. Only one molecules leave the cell .
Explanation :
An antiport is an integral membrane transport protein that simutaneously
transports two different molecules, in opposite directions, across the membrane.
73. All of the following comes under COAD except ?
a. Asthma
b. Chronic bronchitis
c. Emphysema
d. None of the above
Explanation :
COAD – chronic obstrcutive airway disease includes all the 3 ie, asthma,
chronic bronchitis , emphysema. COAD is associated with chronic obstruction of the
flow of air out of the lungs. This obstruction is generally permanent and progressive
over time. People with this condition often cough up large amounts of sputum for
several months of the year because the inflammation stimulates the production of
mucus. This mucus further narrows the already inflamed airways. The ideal way to
prevent this condition from happening and to stop it getting worse is to not smoke.

74. Which of the following pass through guyon’s canal ?


a. Radial nerve
b. Median nerve
c. Ulnar nerve
d. Auxillary nerve
Explanation :

75. Germ tube formation is seen in which of the following ?


a. Candida albicans
b. Cryptococcus neoformans
c. Histoplasmosis
d. Chromoblastomycosis
Explanation :
Candida albicans has evolved mechanisms to evade innate immunity, which is
an important reason that candidiasis is a major complication in AIDS patients. C.
albicans resists macrophage phagocytosis via a mechanism that does not stimulate
apoptosis in macrophages (22). C. Albicans are defective in making germ tubes and
cannot escape the macrophages following phagocytosis and are killed.
76. Ovarian follicular reserve is best accessed by which of the following ?
a. Urine LH
b. Antral follicle count
c. Endometrial biopsy
d. Measuring HCG
Explanation :
Ovarian reserve plays a crucial role in achieving pregnancy following any
treatment in subfertile women. The estimation of ovarian reserve is routinely
performed through various ovarian reserve tests (ORTs) in an effort to predict the
response and outcome in couples prior to In Vitro Fertilization and counsel them.
Most widely used tests are basal follicle stimulating hormone and anti-Mullerian
hormone and antral follicle count. Here the best answer is antral follicle count.
77. All of the following is true regarding crohn’s disease except?
a. Skip lesions
b. Feeling of mass or fullness in the abdomen
c. more common in women than in men
d. recurrence is uncommon
Explanation :
Crohn's disease is a Inflammation can affect any part of the digestive system,
from the mouth to anus. Common symptoms can include:

 diarrhoea
 abdominal pain
 fatigue (extreme tiredness)
 unintended weight loss
 blood and mucus in your faeces (stools).

People with Crohn's disease sometimes go for long periods without symptoms or
with very mild symptoms. This is known as remission. Remission can be followed by
periods where symptoms flare up and become particularly troublesome.
78. Which of the following is not correct regarding reduced ORS ?
a. Sodium 75
b. Trisodium Citrate 10
c. Potassium chloride 20
d. Glucose 90
Explanation :

79. Folinic acid is a antidote for which of the following?


a. 6 – mercaptopurine
b. Methotrexate poisoning
c. Cisplatin toxicity
d. Cyclophosphamide
Explanation :
Methotrexate inhibits dihydrofolate reductase and also inhibits thymidylate
synthase. Methotrexate toxicity can be reduced by administration of leucovorin
(folinic acid).
80. Which of the following is not a component of superior mediastinum ?
a. Esophagus
b. Thoracic duct
c. SVC
d. Desending aorta
Explanation :
Superior mediastinum extends upwards terminating at the superior thoracic
aperture.it includes arch of aorta , SVC , vagus nerve , phrenic nerve ,thymus,
trachea, esophagus and thoracic duct. Desending aorta is a content of posterior
part of inferior mediastinum.
81. Identify the given image below?

a. Colorectal carcinoma
b. Diffuse esophageal spasm
c. Intussception
d. Crohn’s disease

Explanation :
The “apple core sign” was first described in barium enema examinations of the
colon. The apple core appearance is the visual manifestation of an annular lesion of
the bowel with irregular overhanging edges and shouldered margins . The apple core
sign is classically seen in cases of colon carcinoma.
82. Which of the following k+ sparing diuretics that doesn’t cause gynecomastia ?
a. Spironolactone
b. Triamterene
c. Amiloride
d. Eplenorone
Explanation :
Amiloride acts by inhibiting the epithelial sodium channel (ENaC). Its mechanism is
different to spironolactone ,which is an aldosterone receptor antagonist. Therefore
amiloride doesn’t cause gynecomastia. Eplenorone has similar action like
spironolactone but it have less adverse effects like gynecomastia and vaginal
bleeding when compared with spironolactone.
83. Lid lag sign of hyperthyroidism is?
a. Moebius sign
b. Vongraefe’s sign
c. Dalyrymptes sign
d. Stellwag’s sign
Explanation :

84. In case of hanging , during autopsy which of the following cavity should be
opened 1st?
a. Cranial cavity
b. Thoracic cavity
c. Abdominal cavity
d. Pelvic cavity

85. CEA is a marker for the follow up of which of the following ?


a. Pheochromocytoma
b. Colorectal carcinoma
c. Choriocarcinoma
d. Prostatic cancer
Explanation:
Carcinoembryonic antigen (CEA) is a glycoprotein, which is present in
normal mucosal cells but increased amounts are associated with adenocarcinoma,
especially colorectal cancer. CEA therefore has a role as a tumour marker CEA may
be elevated in colorectal cancer, which is where it is most clinically useful. However,
it may also be elevated in a wide variety of other malignant and benign conditions.
86. Heberdan arthropathy is a feature of ?
a. Osteoarthritis
b. Osteoporosis
c. Psoriatic arthritis
d. Osteogenesis imperfect
Explanation :
Heberden’s nodes seen in distal interphalangeal joints is a characteristic feature of
idiopathic osteoarthritis. Bouchard’s nodes is also a feature of osteoarthritis seen in
proximal interphalangeal joints.

87. Tb of phalanges is called as


a. Caries sicca
b. Spina ventosa
c. Pot spine
d. Military TB
Explanation :
Tuberculous dactylitis, also known as spina ventosa, is a rare skeletal manifestation
of tuberculosis where the short tubular bones (i.e. phalanges, metacarpals,
metatarsals) are affected.
88. Identify the part in the image indicated by
arrow?

a. Sphincter of oddi
b. Pancreas
c. 3 rd part of duodenum
d. Jejunum
Explanation :

89. Distribution of rash in chickenpox is ?


a. Centripetal distribution
b. Centrifugal distribution
c. Spread downwards
d. Spread upwards

Explanation :
90. Most common tumour to occur in HIV patients is ?
a. Cervical cancer
b. Hodgkin’s lymphoma
c. Kaposi scarcoma
d. Nasopharyngeal carcinoma
Explanation :

Infection with HIV weakens the immune system and reduces the body's ability to
fight viral infections that may lead to cancer. The viruses that are most likely to cause
cancer in people with HIV are:

 Kaposi sarcoma-associated herpesvirus (KSHV), also known as human


herpesvirus 8 (HHV-8), which causes Kaposi sarcoma is the most common
cancer to occur in HIV patients and some subtypes of lymphoma.

 Epstein-Barr virus (EBV), which causes some subtypes of non-


Hodgkin and Hodgkin lymphoma.

 Human papillomaviruses (HPV), high-risk types (16 & 18) of which


cause cervical cancer, most anal cancers, and oropharyngeal, penile, vaginal,
and vulval cancer.

 Hepatitis B virus (HBV) and hepatitis C virus (HCV), which both cause liver
cancer.

91. Visual Crowding is a phenomenon seen in which of the following ?


a. Astigmatism
b. Amblyopia
c. Presbyopia
d. Senile cataract

Explanation :
Amblyopia is a developmental visual disorder which is characterized by
reduced visual acquity (uncorrectable by lense in the absence of any detectable
defect in eye or visual pathway). Crowding phenomenon is a feature of an amblyopic
eye. It is partial loss of sight , best detected and treated by 3 or 4 yrs of age for best
results. Occlusion of normal eye to encourage the use of amblyopic eye is the most
effective treatment.
92. Man with Lepromatous leprosy who had undergone chemotherapy develops
various nodules is a example of which type of hypersensitivity ?
a. Type 2
b. Type 3
c. Type 4
d. Type 5
Explanation :
Leprosy is a chronic granulomatous infection principally affecting the skin
and peripheral nerves caused by the obligate intracellular organism Mycobacterium
leprae. The disease causes skin lesions and neuropathy. Complications secondary to
the neuropathy can result in deformity and disability. The immunological response
mounted by the host dictates the clinical phenotype that develops. People with
leprosy show a spectrum of clinical types. Tuberculoid disease is the result of high
cell-mediated immunity with a largely Th1 type immune response. Lepromatous
leprosy however is characterized by low cell-mediated immunity with a humoral Th2
response. Type 1 reactions are delayed hypersensitivity(type 4) reactions that occur
in borderline disease. Type 2 or erythema nodosum leprosum (ENL) reactions occur
in borderline lepromatous and lepromatous disease (type -3).

93. Site of hordeolum externa is ?


a. Meibomian glands
b. Gland of zeis
c. Wolfring’s gland
d. Glands of Moll

Explanation :
A hordeolum is a common disorder of the eyelid. It is an acute focal infection
(usually staphylococcal) involving either the glands of Zeis (external hordeola, or
styes) or, less frequently, the meibomian glands (internal hordeola). In the early stages
of disease the gland become swollen, hard and painful. Usually the whole edge of the
lid is edematous. Subsequently abscess is formed which points near the base of one
of the cilia. Pain is considerable until pus is evacuated.
94. Intermittent ring causing dysphagia is seen in which of the following disease?
a. Diffuse esophageal spasm
b. Schatzki’s ring
c. Esophageal web
d. Streakhouse syndrome
Explanation :
Schatzki rings are fixed smooth, benign, circumferential, and narrow ring of
tissue in the lower end of the esophagus. Schatzki rings are a common cause of
intermittent dysphagia. Typically, the dysphagia is intermittent or short-lived. Food may
become stuck in the esophagus but can be forced down by drinking liquids or may be
regurgitated. The dysphagia may not recur for months or years. Patients may have
associated symptoms of gastroesophageal reflux. Barium esophagography is the
preferred method of detection of Schatzki rings.
95. Identify the procedure given in the picture?

a. Robotic surgery
b. Laproscopic cholecystitis
c. Endoscopic
d. LAZER ablation

Explanation :

Robotic surgery, or robot assisted surgery, is one of the developments that is at the
forefront of this shift. Robotic surgery involves the use of surgical robots that can be
controlled by doctors through computers. This results in less pain and blood loss for
the patient, and quicker healing time. Even higher accuracy can be achieved when
computer-generated images are used to guide surgeries. One such possibility is the
use of a real time MRI scan to guide surgeries.
96. Most common site of zollinger Ellison syndrome is?
a. Cardiac part of stomach
b. Pylorus part of stomach
c. Duodenum
d. Jejunum

Explanation :
Zollinger-Ellison syndrome (ZES) is a disease of the gastrointestinal system.
People who have ZES develop tumors known as gastrinomas in the pancreas and
duodenum (the first section of the small intestine). The gastrinomas caused by ZES
secrete the hormone gastrin. Because gastrin creates excessive stomach acid, 90
percent of patients with ZES develop stomach and duodenal ulcers.
97. Mummification is seen in which conditions?
a. Moist and humid environment
b. Lead poisoning
c. Desiccation of body in hot and dry weather
d. Electric burns

Explanation :
Mummification is a another modification of putrefaction. Hot and dry climate
favours it. Dehydration and drying causes evaporation of water and shrinkage of
cadaver. The natural appearances and body features are preserved. The mummified
body is odourless, dry and leathery brown. The time required is 3 months to 1 yr. It
begins in the exposed parts of the body like face, hands, feet and then extends to
internal organs. Arsenic and Antimony poisoning favours mummification .
98. In Game keeper’s thumb which of the following is damaged?
a. medial collateral ligament
b. Ulnar collateral ligament
c. anterior ligamnet
d. posterior ligament
Explanation :
Ulnar collateral ligament of thumb is damaged

99. Carotid sinus massage leads to ?


a. Reflux tachycardia
b. Reflux bradycardia
c. Bradypnea
d. Tachypnea

Explanation :
Carotid sinus massage is performed for three reasons:
• To break junctional tachycardia (IAVNRT and AVRT) by blocking AV nodal
conduction (and cause Bradycardia).
• To reveal on the P wave pattern of an atrial arrhythmia by reducing the frequency of
AV nodal conduction during the tachycardia so that the QRS complexes do not mask
the artrial activity.
• To test for carotid sinus hypersensitivity, which is a severe fall in blood pressure or
heart rate in response to carotid sinus stimulation.
100. Incidence is
a. No. Of new case
b. No. Of existing case
c. 1st case in community
d. 1st case that comes to doctor

Explanation :
Incidence is defined as the no. of new cases occurring in a defined population during
specific period of time. It can be determined from cohort study .
Incidence = no of new case /population at risk *1000
Incidence is a RATE . (expressed per 1000)
101. Invertogram to be done in a new born:
a. Immediately
b. After 2 hrs
c. After 4 hrs
d. After 6 hrs

Explanation :
Invertogram is done for detection of imperforate anus. It is done after six hours
of birth in infant u sufficient air may have collected in large intestine to cast a X-ray
shadow.
102. For hyponatremia select the electrocardiographic finding with which it is most
commonly associated?
a. No known electrocardiographic abnormalities
b. Prolonged QT interval
c. Short QT interval
d. Widened QRS complex

Explanation :
Hypokalemia typically increases automaticity of myocardial fibers, which results
in ectopic beats or arrhythmias. Electrocardiography in hypokalemia reveals
flattening of the T wave and prominent U waves. Hyperkalemia decreases the rate of
spontaneous diastolic depolarization in all pace maker cells. It also results in slowing
of conduction. One of the earliest electrocardiographic signs of hyperkalemia is the
appearance of tall, peaked T waves. More severe elevations of the serum potassium
result in widening of the QRS complex. At serum sodium levels compatible with life,
neither hyponatremia nor hypernatremia results in any characteristic
electrocardiographic abnormalities.
103. Over 75% of the strength of the intact abdominal wall lies in the –
a. Skin
b. Subcutaneous tissue
c. Aponeurosis
d. Peritoneum

Explanation:
Aponeurosis of rectus sheath is very strong and provides maximum strength of intact
abdominal wall.
104. In amblyopia, the treatment of choice is:
a. Surgery
b. Orthoptic exercise
c. Spectacles
d. Conventional occlusion

Explanation :
Amblyopia is a developmental visual disorder which is characterized by
reduced visual acquity (uncorrectable by lense in the absence of any detectable
defect in eye or visual pathway). Crowding phenomenon is a feature of an amblyopic
eye. It is partial loss of sight , best detected and treated by 3 or 4 yrs of age for best
results. Occlusion of normal eye to encourage the use of amblyopic eye is the most
effective treatment. Occlusion therapy must be carried out for as short a period as
may be expected to give improvement in the acuity of the amblyopic eye and yet
avoid the risk of occluded eye itself becoming amblyopic from lack of visual output.
105. Paranoid schizophrenia is seen with which of the following substance abuse?
a. Cocaine
b. Amphetamine
c. Barbiturates
d. Opiods
Explanation :
Schizophrenia like syndrome may be seen with amphetamine use, mescaline
,phencyclidine abuse, LSD abuse , huntington’s chorea, homocystinuria ,
hemochromatosis , wilson’s disease, acute intermittent porphyria. Thought disorder
in paranoid schizophrenia are bizzre delusions ,1st rank symptoms , and formal
thought disorder. Insight is absent.

106. Which of the following is not a component of carpel tunnel ?


a. Median nerve
b. Flexor digitorum profundus
c. Flexor pollicis longus tendon
d. Ulnar nerve
Explanation :

107. Which antiviral agent most often causes anemia, leucopenia, or


thrombocytopenia?
a. Zidovudine
b. Ribavarin
c. Acyclovir
d. Amantadine
Explanation:
The most serious side effect of zidovudine is bone marrow suppression.
Ganciclovir (Cytovene) may also cause these adverse effects, but acyclovir (Zovirax)
usually does not. Didanosine and zalcitabine (other dideoxynucleosides used for
HIV) are less likely to cause bone marrow suppression than zidovudine and are
more frequently associated with pancreatitis and sensorimotor neuropathy.
108. Maximum disinfectant is seen with?
a. Ethylene oxide
b. 2% glutaraldehyde
c. Alcohol 70- 80%
d. Cresol

Explanation:
Cresol emulsions are very powerful disinfectant . Also known as “all
purpose general disinfectant “.
109. Hippus is seen in which poisoning ?
a. Abrus
b. Aconite
c. Datura
d. Alcohol
Explanation :
The condition called hippus is also called pupillary athetosis, or spasmodic,
rhythmic dilation and constriction of the pupils.
Aconite- in late stages Hippus is seen.
Alcohol- pupils are earlier dilated, later constricted,
Barbiturates- pupils are earlier constricted, later dilated.
110. Superior vena caval syndrome is most commonly caused by ?
a. Lung cancer
b. pheochromocytoma
c. cirrhotic liver
d. rib fractures
Explanation :
SVCS is more common for people who have lung cancer, non-Hodgkin lymphoma,
or cancers that spread to the chest. But cancer can cause SVCS in other ways:
 A tumor in the chest may press on the superior vena cava.

 A tumor may grow into the superior vena cava, causing a blockage.

 If cancer spreads to the lymph nodes surrounding the superior vena cava, the
lymph nodes may enlarge and press on or block the vein.

 A blood clot may appear in the vein. This is caused by a pacemaker wire or
an intravenous catheter, which is a flexible tube placed in a vein to take out or
put in fluids

111. Common cause for delayed puberty in girls?


a. Turner syndrome
b. Constitutional delay
c. Primary ovarian failure
d. Addison’s disease
Explanation:
A girl who has not started to have breast development by the age of 13 is considered
to be delayed. Girls with delayed puberty are simply late maturers, but once they
start, puberty will progress normally. This is called constitutional delayed puberty and
is more common in boys than girls. Often, this is something that is inherited from the
parents, so it is more likely to occur if the mother started her periods after age 14
(the average is about 12 ½) or if the father was a "late bloomer." Decreased body fat
is a major cause of pubertal delay in girls. Here the best option is constitutional
delay.
112. Treatment of choice for atropic rhinitis is?
a. Vidian neurectomy
b. McGovern’s technique
c. Young’s operation
d. FESS
Explanation:
Atrophic rhinitis is defined as a chronic nasal disease characterised by
progressive atrophy of the nasal mucosa along with the underlying bones of
turbinates. The common symptoms may include foetor, ozaena, crusting/nasal
obstruction, epistaxis, anosmia / cacosmia and secondary infection with maggot
infestation. The mainstay of treatment is conservative (for example, nasal irrigation
and douches; nose drops (e.g. glucose‐ glycerine, liquid paraffin); antibiotics and
antimicrobials; vasodilators and prostheses). Surgical treatment -(Young’s operation)
aims to decrease the size of the nasal cavities, promote regeneration of normal
mucosa, increase lubrication of dry nasal mucosa and improve the vascularity of the
nasal cavities.
113. Most common site of pheripheral artery aneurysm is ?
a. Iliofemoral artery
b. Internal iliac artery
c. Subclavian artery
d. Popliteal artery
Explanation :
About 70% of peripheral arterial aneurysms are popliteal aneurysms; 20% are
iliofemoral aneurysms. Aneurysms at these locations frequently
accompany abdominal aortic aneurysms, and > 50% are bilateral. Rupture is
relatively infrequent. Peripheral arterial aneurysms occur in men much more often
than in women (> 20:1); mean age at presentation is 65. Aneurysms in arm arteries
are relatively rare; they may cause limb ischemia, distal embolism, and stroke.
114. Which organ assessment should be done before CECT?
a. Liver
b. Kidney
c. Heart
d. Brain
Explanation:
Renal function should be assessed before taking CECT. Iodinated contrast media is
highly nephrotoxic.
115. Polyuria is a feature of all of the following except:
a. Lithium toxicity
b. Hypocalcemia
c. ADH deficiency
d. Hypokalemia

Explanation:
Polyuria is defined, as 24-hour urine volume greater than 3 litres.
Hypocalcaemia does not cause polyuria. Hypercalcaemia causes polyuria by solute
diuresis. Solute diuresis is also seen with glucose, mannitol, radiocontrast agent,
urea, resolving ATN.
Lithium causes nephrogenic diabetes insipidus and hence polyuria. ADH deficiency
causes central diabeted insipidus. The various causes are postoperative (removal of
pituitary), trauma supra-or intrasellar tumour, Sheehan’s syndrome, infections etc.

116. Which one of the following vectors transmits scrub typhus?


a. Louse
b. Ticks
c. Mite
d. Flea

Explanation:
Scrub typhus is caused by Orientia tsutsugamushi, which is maintained in nature
by transovarian transmission in trombiculid mites, mainly of genus Leptotrombidium.
After hatching, the infected larval mites (chigger) inoculate organisms into the skin
while feeding. Clinical features are fever, headache, myalgia, cough, gastrointestinal
symptoms, regional lymphadenopathy etc. Doxycycline or chloramphenicol is used for
treatment.

117. Food poison is a example of which kind of epidemics?


a. Seasonal trends
b. Multiple source epidemics
c. Probagated epidemics
d. Point source epidemics
Explanation:
Seasonal trends - eg. Measles ,URI
Multiple source epidemics – eg. Contaminated well in a village
Probagated epidemics – eg. HIV (person to person spread)
Point source epidemics – eg.food poisoning , Bhopal gas tragedy.

118. Muscle stretch is due to the action of which of the following ?


a. Muscle spindle
b. microfilaments
c. Golgi tendon organ
d. Sarcomere
Explanation:
Muscle spindle is present in skeletal muscle. It has a nuclear bag fibre and nuclear
chain fibre. It helps in the contraction of skeletal muscles. Its function is to change
the length of the muscle.
119. Pre exposure schedule of rabies vaccine is?
a. 0, 3, 7, 14, 21, 28, 90
b. 3, 7 , 21
c. 0, 3, 7, 14 ,28
d. 0, 7 , 28
Explanation:
Hydrophobia is pathognomic.
Causative agent is – Lyssavirus type-1.
Types of rabies – street virus and fixed virus.
Vaccine schedule :
post exposure prophylaxis : 0, 3, 7, 14 ,28, 90
pre exposure prophylaxis : 0,7,28
post exposure prophylaxis in those vaccinated previously : 0,3,7
120. Which of the following drug cause uterine relaxation ?
a. Dinoprost
b. Carbetocin
c. Methergine
d. Atosiban
Explanation:
Preterm birth is strongly associated with neonatal death and long-term
neurological morbidity. The purpose of tocolytic drug administration is to postpone
threatening preterm delivery for 48 hours to allow maximal effect of antenatal
corticosteroids and maternal transportation to a center with specialized neonatal
care facilities. There is uncertainty about the value of atosiban (oxytocin receptor
antagonist) and nifedipine (calcium channel blocker) as first-line tocolytic drugs in the
management of preterm labor.
121. A 3 yr old boy was brought to ER with multiple coalescing lesions on his face
and extremites.

a. Herpes simplex
b. Impetigo
c. Dermatitis herpetiformis
d. Phemphigus.
Explanation :
Impetigo is the most common bacterial infection in children. primarily caused
by Streptococcus pyogenes or Staphylococcus aureus. Impetigo is classified as either
nonbullous (impetigo contagiosa) (about 70% of cases) or bullous. nonbullous
impetigo commonly have honey-colored crust with a moist erythematous base as
shown in the above image. Bullous impetigo is considered to be less contagious than
the nonbullous form.

122. Skin of the parotid gland is supplied by which of the following nerve?
a. Greater auricular nerve
b. Auriculotemporal nerve
c. Mandibular nerve
d. Vagus nerve
Explanation :
The parotid gland receives sensory and autonomic innervation. The autonomic
innervation controls the rate of saliva production.
Sensory innervation is supplied by the auriculotemporal nerve (gland) and the great
auricular nerve (fascia and skin). The parasympathetic innervation to the parotid
gland has a complex path. It begins with the glossopharyngeal nerve.
The auriculotemporal nerve then carries parasympathetic fibres from the otic
ganglion to the parotid gland. Parasympathetic stimulation causes an increase in
saliva production. Sympathetic innervation originates from the superior cervical
ganglion, part of the paravertebral chain.
123. Which of the following tumour is rathke’s pouch derivative ?
a. Glioblastoma multiforme
b. Craniopharyngioma
c. Vestibular schwanoma
d. Oligodentroglioma
Explanation :
A craniopharyngioma is a benign (noncancerous) tumor arising from small
nests of cells near the pituitary stalk. Adamantinomatous (ordinary)
craniopharyngioma occurs in children and tends to be less solid than papillary
craniopharyngioma. Papillary craniopharyngioma occurs in adults and is a more solid
tumor. Increased pressure within the brain causes many of the symptoms associated
with this tumor. Other symptoms result from pressure on the optic tract and pituitary
gland Surgery to remove the tumor is usually the first step in treatment. If
hydrocephalus (excess water in the brain) is present, a shunt (drainage system) may
be placed during surgery. The shunt will help remove excess cerebrospinal fluid from
the brain and ease the pressure.
124. For diagnosis of schizophrenia symptoms should be present for how long
period?
a. 2 weeks
b. 1 month
c. 6 month
d. 1 year
Explanation :
Two or more of the following ,each present for a significant portion of time of during 1
month
 Delusions
 Hallucinations
 Disorganized speech
 Negative symptoms ie, affective flattening, alogia or avolition .
Continous signs of disturbance persist for atleast 6 months.
This 6 month period must include atleast 1 month of symptoms.
125. Leucocyte rolling over endothelial cells of the blood vessel is carried out by
which of the following?
a. Selectins
b. Integrins
c. PECAM 2
d. E- Cadherins
Explanation:
Rolling is a process of transient adhesion of leucocytes with the endothelial
cells. Selectins are the important molecules responsible for it. They interact with
complementary molecules resulting in transient adhesion.
126. Which of the following tumour is most common in post renal transplant
patients?
a. Non Hodgkin’s Lymphoma
b. Hepatocellular carcinoma
c. Non melanoma skin cancer
d. Kaposi’s sarcoma
Explanation :
There is an overall 3–5-fold increase in cancer risk in transplant recipients
compared with the general population, with skin cancers and lymphoma particularly
prevalent. Cancers in transplant recipients are often more aggressive than those in
the general population, with poor prognosis, particularly for gastrointestinal tumours
and lymphomas.
127. In ERG ‘A’ wave corresponds to?
a. Rods and cones
b. Pigment epithelium
c. Nerve bundle layer
d. Artefact
Explanation:
Electroretinography: A test in which the electrical potentials generated by the retina
of the eye are measured when the retina is stimulated by light. Abbreviated as ERG.
In an ERG, an electrode is placed on the cornea at the front of the eye that
measures the electrical response of the rods and cones, the visual cells in the retina
at the back of the eye. An ERG may be useful in the evaluation of hereditary and
acquired disorders of the retina. The instrument used to conduct ERG is an
electroretinograph, and the resultant recording is called an electroretinogram
waves Function
A wave Activity of rods and cones

B wave A response arising from


bipolar cell layers
C wave Retinal metabolism of the
pigment
128. Which of the following is not associated with cephal hematoma?
a. Usually involves parietal and occipital bones
b. Bleeding is sub periosteal
c. Swelling is present at birth
d. Usually reabsorbed in 2-3 months.
Explanation:
Cephalhaematoma is a collection of blood between pericranium and skull
bones following forceps delivery or normal labour due to rupture of small emissary
veins or fracture of scalp bones. It appears 12 to 24 hours afterbirth.
Cephalhaematoma is never present at birth. It appears a few hours after birth and at
times on the second day. It causes hyperbilirubinaemia due to release of bilirubin from
collected blood. Caput succedaneum is present at birth.
129. Culex is a vector for which of the following diseases?
a. Japanese encephalitis
b. Yellow fever
c. Brugian filariasis
d. Chickengunya
Explanation :
Japanese encephalitis :
Causative agent : Group- B Arbovirus
Host factors : Pigs are “Amplifier host” .
Cattles and Buffalos are ‘mosquito attractants’.
Birds are also involved
Man is a ‘Incidental Dead End Host’.
Vectors of JE: Culex tritaeniorhynchus (most important vector)
Culex vishnuii
Incubation period of JE: 5- 15 days (9-12 days in mosquito)
Case fatality rate : 20 – 40%
Vaccines : nakayama strain /Beijing strain
SA-14-14-2 strain
130. Which of the following structure is not visualized in hysteroscopy ?
a. endocervix
b. Uterine surface
c. Tubal ostia
d. Endometrial cavity
Explanation:
Hysteroscopy permits full visualization of the endometrial cavity and endocervix and
is helpful in diagnosing focal lesions that are missed with endometrial sampling.
Uterine surface, which is the outermost layer of uterus ie, perimetrium cannot be
visualized by hysteroscopy.
131. Presenting case of infant with intussception , management to be done is?
a. fluid resuscitation + NG tube decompression + antibiotic +
hydrostatic reduction
b. fluid resuscitation + antibiotic + surgery
c. fluid resuscitation + NG tube decompression + surgery
d. fluid resuscitation + NG tube decompression + antibiotic
Explanation:
Intussusception is the most common cause of intestinal obstruction in
children between ages 6 months to 3 years old. A segment of intestine invaginates
into the adjoining intestinal lumen, causing bowel obstruction. Mc site is ileocolic.
The symptoms includes classic triad of vomiting, abdominal pain, and red current
jelly stools. Intussusception is managed by hydrostatic and pneumatic reduction of
the obstructed bowel. If manual reduction is not possible or perforation is present, a
segmental resection with an end-to-end anastomosis is performed.
132. Chancroid is caused by ?
a. Campylobacter jejuni
b. N. Gonorrhoea
c. H.Ducreyi
d. Treponema Pallidum
Explanation:
Chancroid (soft chancre)- caused by haemophilus ducreyi. Single or multiple
painful ulcers with deep ulceration. The ulcer is soft ,non-indurated, dirty looking. It
produces painful, inguinal lymphadenitis k/a inflammatory bubo. Management is
with ceftriaxone i.m or azithromycin 1 gm single dose.
133. Drug of choice for acute gout is ?
a. Naproxen
b. Probenacid
c. Allopurinol
d. Febustat
Explanation:
Naproxen is a NSAIDS, is used to relieve pains from various conditions such
as headaches, muscle aches , tendonitis , dental pains ,mensural cramps. It also
reduces pain caused by arthiritis , bursitis ,and gout attacks. Probenacid and
allopurinol is used for chronic gout attacks.
134. Identify the above given image :

a. Pithris pubis
b. Scabei
c. Head louse
d. Tick
135. Which of the following is not a metabolites of tyrosine?
a. Melanin
b. Melatonin
c. norepinephrine
d. T3 and t4
Explanation :
Tyrosine is required for the formation of thyroid hormones, melanin and
catecholamines such as epinephrine, norepinephrine, dopamine. Melatonin is a
metabolite of tryptophan.
136. Incubation period of hepatitis – B is
a. 10- 14 days
b. 30- 180 days
c. 15- 60 days
d. 3- 35 days
Explanation :
Type Causative agent Incubation period Common modes of
transmission
Hep -A Enterovirus -72 15 - 45 days Fecal – oral ,
sexual
(picornavirus)
Hep -B Hepadnavirus 30 – 180 days Sexual ,perinatal,
percutaneous
Hep -C Hepacivirus 15 – 160 days percutaneous
(flavivirus)
Hep –D Viriods like 30 – 180 days Sexual , perinatal ,
percutaneous
Hep -E Calcivirus 15 – 60 days Fecal – oral
(alpha virus like)

137. Haemorragic conjunctivitis is caused by ?


a. Cornebacterium diphtheria
b. Adenovirus
c. Enterovirus 70
d. Chlamydia
Explanation:
Acute haemorragic conjunctivitis is a dark red spots on conjunctiva caused by
Enterovirus 70.
Pseudomembranous conjunctivitis is caused by adenovirus.
True membranous conjunctivitis is caused by infection with cornebacterium
diphtheria.
Swimming pool conjunctivitis is caused by Chlamydia.
138. Which of the following drugs is preferred in patients with hypertriglyceridemia?
a. Simvastatin
b. Colestipol
c. Gemfibrozil
d. Alirocumab
Explanation :
Fibric acid derivatives such as gemfibrozil ,clofibrate, fenofibrate, bezafibrate
stimulate PPAR – alpha and it increase LPL and decrease triglycerides and hence
preferred in patients with hypertriglyceridemia.
Cloestipol is a bile acid resins is bad for hypertriglyceridemia.
Simvastatin is a HMG coA reductase inhibitors are the doc for dyslipidemia.
139. Which of the following is not the characteristic feature of kwashiorkor?
a. Edema
b. Pluckable hair
c. Preserved appetite
d. Flaky paint dermatosis
Explanation :
Protein energy malnutrition is a spectrum of conditions ranging from mild
undernutrition to extreme forms of malnutrition. It includes kwashiorkor and
marasmus.

140. Which of the following is the tumour marker for hepatocellular carcinoma?
a. CEA
b. a – feto protein
c. B- HCG
d. calcitonin
Explanation :
CEA Colon cancer
Pancreatic cancer
Lung and stomach cancer
AFP Yolk sac tumour
Hepatocellular carcinoma
B – HCG Trophoblastic tumors like choriocarcinoma , seminoma etc.
Calcitonin Medullary carcinoma of thyroid

141. H.pylori is a risk factor for causing which of the following cancers?
a. GIST
b. Gastric adenocarcinoma
c. Squamous cell carcinoma of esophagus
d. Hepatic adenoma
Explanation :
H.pylori is a gram –ve flagellated bacteria producing enzymes like phospholipase
and urease. H.pylori is associated with peptic ulcer disease, gastric cancer and
gastric mucosa associated lymphoma.
142. Which of the following class of drugs is not used in treatement of uterine
fibroids?
a. Nafarelin
b. Tranexemic acid
c. Misoprostol
d. Mifepristone
Explanation :
Nafarelin – GnRH agonist is used to decrease the size of fibroids.
Tranexemic acid is a non hormonal drug which can be used to reduce the blood loss
over short periods.
Mifepristone is also used to treat fibroids.
143. Which of the following nerve passes through cavernous sinus ?
a. Occulomotor
b. Abducens
c. Trochlear
d. Trigeminal V1
Explanation :

144. Schober’s test is done for?


a. Flexion of lumbar spine
b. Pain with motion of hip
c. Neck pain and stiffness
d. Chest expansion
Explanation :
Schober’s test is measure of flexion on lumber spine. This test is done in ankylosing
spondylitis

145. Presumptive treatment is given in


a. Leprosy
b. Tuberculosis
c. Malaria
d. AIDS
Explanation :
Presumptive treatment is given for Malaria when API is  2.
• For leprosy, multidrug therapy is used.
• Short course chemotherapy is given for tuberculosis as DOTS regime.
• Universal precaution with body fluids is necessary in case of AIDS.
146. Dancing carotid is seen in ?
a. Thyrotoxicosis
b. Av fistula
c. Hypothyroidism
d. Blow out carotid
Explanation:
A jerky carotid pulse characterized by full expansion followed by quick collapse. Also
known as Dancing carotid. Prominent carotid pulsations due to the wide pulse
pressure in aortic regurgitation (Corrigan's sign). It is also seen in thyrotoxicosis.
147. Blood supply to the Lower end of esophagus is supplied by ?
a. Inferior thyroid artery
b. Left gastric artery and inferior phrenic artery
c. Left pulmonary artery
d. Short gastric artery
Explanation:
Blood supply of esophagus :
 The cervical portion is supplied by the inferior thyroid artery
 The thoracic portion is supplied by bronchial and esophageal branches of the
thoracic aorta
 The abdominal portion is supplied by ascending branches of the left phrenic
and left gastric arteries.

148. PR interval in ECG corresponds to ?


a. Ventricular repolarization
b. Atrial depolarization and AV nodal conduction
c. Late repolarization of papillary muscle
d. IDO- ventricular rhythm
Explanation :
PR interval – normal duration : 0.12- 0.2 sec
This represents the time that it takes for the electrical impulse
generated in the sinus node to travel through the atria and across the atrioventricular
node to the ventricles.
149. Blood supply to the dorsum of penis is?
a. Cavernosal artery
b. Common penile artery
c. Internal pudendal artery
d. Inferior gluteal artery
Explanation :
The arterial blood supply of penis happens at the expense of external and internal
pudendal arteries. The external pudendal artery starts from an internal wall of
femoral artery on 2.5-2.7 cm below inguinal ligament. The internal pudendal artery is
main source of blood supply of penis of the man.
150. Drug of choice for ovulation induction is
a. Asoprisnil
b. Tamoxifen
c. Levonorgestrel
d. Clomiphene citrate
Explanation :
It is non steroidal triphenylethylene compound related to DES. It binds to estrogen
receptors in the hypothalamus , so the negative feedback of estrogen on
hypothalamus is abolished resulting in increased pulsatile GnRH frequency which
causes a rise in FSH and LH. It is used in case of anovulations, pcos, anovulatory
DUB with infertility. It is the drug of choice for postpill amenorrhoea. Most common
side effect of clomiphene citrate is multiple pregnancy.
151. Most common cause of facial nerve palsy is ?
a. Infection
b. Idiopathic bell’s palsy
c. Head trauma
d. Stroke
Explanation :
Facial nerve (7th cranial nerve) palsy is often idiopathic (formerly called Bell
palsy). Idiopathic facial nerve palsy is sudden, unilateral peripheral facial nerve
palsy. Symptoms of facial nerve palsy are hemifacial paresis of the upper and lower
face. Tests (eg, chest x-ray, serum ACE level) are done to diagnose treatable
causes. Treatment may include lubrication of the eye, intermittent use of an eye
patch, and, for idiopathic facial nerve palsy, corticosteroids.
152. Curdy white discharge from vagina is due to ?
a. Candidial vulvivaginitis
b. Trachomonas vaginitis
c. Bacterial vanginosis
d. Atrophic vaginitis
Explanation:
Candidal vaginitis is most common during pregnancy , in OCP users , corticosteroid
theraphy, diabetes and in all immune compromised and immunodeficient states.
Caused by – candida albicans , candida galbrata , candida tropicales.
Symptoms: Vaginal vulvar pruritus, burning, or irritation (which may be worse during
intercourse) and dyspareunia are common, as is a thick, curdy white, cottage
cheese–like vaginal discharge that adheres to the vaginal walls. Symptoms and
signs increase the week before menses. Erythema, edema, and excoriation are
common.
Diagnosis : Vaginal pH - <4.5
wet mount – budding yeast cells and filamentous elements.
Treatment : topical miconazole / cotrimazole
Systemic fluconazole
Both partners should be treated.
153. VMA in urine is used to diagnose which of the following conditions?
a. Adrenal adenoma
b. Cushing syndrome
c. Pheochromocytoma
d. Addison’s disease
Explanation :
Vanillylmandelic acid (VMA) is one of the breakdown products (metabolites)
of catecholamines. This test measures the amount of VMA that is passed into the
urine, typically over a 24-hour period, to detect excess epinephrine and
norepinephrine. It is used to detect tumors called neuroblastomas and
other neuroendocrine tumors. Neuroendocrine tumors, such as neuroblastomas
and pheochromocytomas, can sometimes produce large amounts of
catecholamines, resulting in greatly increased concentrations of the hormones and
their metabolites.
154. Horrock’s apparatus is used to ?
a. Disinfect the action of chlorine in water
b. Chlorine demand estimation of water
c. Remove the hardness of water
d. Indicate the quality of water
Explanation :
It is used to find out the dose of bleeching powder required for disinfetion of water
ie, chlorine demand estimation of water. Bleeching powder contains 33% of available
chlorine. Orthotoludine – arsenite test is used for testing residual chlorine at the end
of 1 hr contact. Chlorine acts best as a disinfectant when the PH of water is around
7. The standard prescribed for chloride is 200 mg/ litre. Maximum permissible level is
600 mg/litre.
155. Which of the following drug is used in the treatment of post menopausal
osteoporosis?
a. Clomiphene citrate
b. fluoxetine
c. veralipride
d. Raloxifene
Explanation :
Raloxifene is SERM , that act as both estrogen agonist and antagonists
depending on the tissue. Raloxifene has estrogen like action on bones and lipids and
estrogen antagonists action on breast and endometrium. Due to its agonist effect on
bones, it reduces the risk of fracture by 50% , specially in vertebra by increasing
bone mineral density by 2-3%. 1st line treatment of post menopausal osteoporosis is
bisphosphonates. Major adverse effect is predisposition of thromboembolism.
156. Which of the following cell is the Sole efferent fibre from cerebellum?
a. Stellate cells
b. Golgi cells
c. Purkinje cells
d. Granules cells
Explanation :
The three-layered cerebellar cortex contains six main neuronal cell types:
stellate and basket cells in the molecular layer, Purkinje cell somata in the
Purkinje cell layer, and granule cells, Golgi cells, and unipolar brush cells in the
granular layer. The cerebellar nuclei and the lateral vestibular nucleus constitute the
sole output (efferent) of the cerebellum, and play a central role in cerebellar circuitry
and function. Purkinje cells represent the sole output neuron of the cerebellar cortex
and thus changes in their function have significant impact on the function of the
cerebellum as a whole. Understanding pathologic processes within these cells is
crucial to develop strategies to prevent their loss and thus it reduce ataxia. Only
excitatory cell of cerebellum is granules cells.
157. Which part of eye is used in biometric scans ?
a. Lens
b. Iris
c. Cornea
d. Retina
Explanation :
Biometrics is the science of human recognition. In biometrics, “Iris and Retinal
scanning” technologies are known as ocular based identification technologies,
means that they rely on unique physiological characteristics of eye to identify an
individual. The structure of the iris is unique to the individual and can be used for
identification purposes. Iris recognition and retinal scanning are both very reliable
modalities for biometric identification.
Iris recognition – It takes the reading of Retinal scanning – It is the analysis of
the characteristics of the iris blood vessels at the back of eye.

158. Identify the structure marked below

a. Azygous vein
b. Hemiazygous vein
c. Minor fissure of right lung
d. Major fissure of left lung
Explanation :

159. Lac operon coding region is done by ?


a. Inducer
b. Promoter
c. Regulator
d. Operator
Explanation :
The lac operon is an operon, or group of genes with a single promoter (transcribed
as a single mRNA). The genes in the operon encode proteins that allow the bacteria
to use lactose as an energy source.
The promoter is the binding site for RNA polymerase, the enzyme that performs
transcription.
The operator is a negative regulatory site bound by the lac repressor protein.
The lac repressor is a protein that represses (inhibits) transcription of the lacoperon.
160. Which of the following condition doesn’t cause gynecomastia ?
a. Leprosy
b. Tb
c. Klinefelter syndrome
d. Hyperthyroidism
Explanation :
Gynecomastia is triggered by a decrease in the amount of the hormone
testosterone compared with estrogen. The hormones testosterone and estrogen
control the development and maintenance of sex characteristics in both men and
women. The pathologic causes of gynecomastia are diverse and may
include Klinefelter syndrome, certain cancers, endocrine disorders such as
hyperthyroidism , metabolic dysfunction, various medications, or may occur due to a
natural decline in testosterone production. Gynecomastia is the most obvious clinical
manifestation of hormone dysfunction in leprosy.
161. True statement about bronchopulmonary segment is
a. It is according to the segmental bronchus.
b. It is surgically unresectable
c. Pul. Artery is intersegmental
d. 10 segments in each lung
Explanation :
A bronchopulmonary segment is a division of the lung. These segments are the
largest subdivisions of a lung lobe. Each supplied by a tertiary (segmental) bronchus.
Segments are pyramidal-shaped, with their apices facing the lung root and their bases
at the pleural surface. Supplied independently by a segmental bronchus and a
segmental pulmonary Artery .10 segments on the right side and 8-10 segments on
the left side. They are surgically resectable.
162. Which of the following is the anaerobic branched filamentous non acid fast
bacilli ?
a. Clostridium tetani
b. Nocardia
c. Actinomyces
d. Atypical mycobacteria
Explanation:
Actinomyces : It is a anaerobic gram +ve and filamentous branching bacilli. It is
non- acid fast bacilli , while Nocardia is partially acid fast. Actinomyces normal flora
is in female genital tract , gingival. Transmission is endogenous due to tooth pick
injuries. Most common organism is Actinomyces Israeli that produce indurated
swelling and draining sinus that contains yellow sulphur granules. Most common site
is lower jaw. It also produces solitary brian abcess and also cause
Actinomycetoma. Doc for Actinomyces infection is penicillin.
163. Thalassemia in pregnancy is diagnosed by ?
a. Nestrof test
b. Hba2>3.5 %
c. Mutational analysis test
d. Chorionic villus sampling
Explanation :
Prenatal diagnosis of thalassemia can be done by chorionic villus sampling or
by fetal blood sampling. Both tests are conducted under ultrasound guidance.
Chorionic villus sampling involves obtaining some cells from the placenta for
testing. This is done during the 10th to 12th week of pregnancy.
Fetal blood sampling involves obtaining a small amount of fetal blood from the
umbilical cord for testing. This procedure is done at about 18 - 20 weeks of
pregnancy.

164. Fox - Fordyce spots are?


a. Seen due to blockade of apocrine gland
b. Variant of Sebaceous gland in lips
c. Pilo sebaceous units
d. Type of Eccrine sweat glands
Explanation :
Fordyce spots are visible sebaceous glands without hair follicles. They are
present in 80–95% of adults. They are probably present at birth but become bigger
and more visible from about puberty onwards. They are present on most of the body,
and sometimes are visible on the penis, scrotum, lips, or inner (buccal) cheeks.
These are completely normal and are not a sexually transmitted infection (STI).
165. Which of the following is seen in catarrhal stage of typhoid ?
a. High leucocyte
b. Eosinophilia
c. Leucopenia with relative lymphocytosis
d. Thrombocytopenia
Explanation :
Typhoid is a bacterial infection that can lead to a high fever, diarrhea, and vomiting.
It can be fatal. It is caused by the bacteria Salmonella typhi.
Clinical features is divided into 4 stages :
1st week – prodromal symptoms similar to URT infection.
Step ladder pattern temperature
Relative bradycardia (faget’s sign)
Malaise , headache, abdominal pain
Leucopenia with relative lymphocytosis
2nd week - macula- popular “Rose spot” rash
Constipation , confusional state , hepatospleenomegaly
Positive Widal test
3rd week - “week of complications”
Haemolytic anemia , meningitis ,acute cholecystitis , UTI , intestinal perforation and
haemorrhage . Dehydration is a significant risk. Thrombocytopenia with risk of
bleeding. Death can occur at this stage from overwhelming toxaemia.
4th week – “week of convalescence”

In the untreated patient the fourth week ceases the fever, mental state
and abdominal distension slowly improve over a few days, but intestinal
complications may still occur. Convalescence is prolonged, and most relapses occur
at this stage.
Treatement :

The only effective treatment for typhoid is antibiotics. The most commonly used
are ciprofloxacin (for non-pregnant adults) and ceftriaxone. Other than antibiotics, it
is important to rehydrate by drinking adequate water.In more severe cases, where
the bowel has become perforated, surgery may be required.

166. Obesity is not seen in which of the following disease ?


a. Prader-willi syndrome
b. Acromegaly
c. Bardet – Biedl syndrome
d. Carpenter syndrome
Explanation :
Acromegaly , a disorder of IGF-1 which causes excessive growth of the hands, feet,
jaw and other internal organs in adults. In acromegaly , glucose does not suppress
growth hormone. MRI shows pituitary tumours in 90% of acromegalic patients. The
best confirmatory test for acromegaly is the oral glucose suppression test. Obesity is
not seen with acromegaly.
Gigantism , abnormally high linear growth due to excessive action of IGF-1 before
the closure of the epiphyseal growth plate in children.
167. Copycat suicide is mainly carried out by ?
a. School going Children
b. Newborns
c. Adults
d. Adolescent

168. Colipase enzyme is required for ?


a. Bile
b. Pancreatic lipase
c. Gastric secretions
d. Salivary enzymes
Explanation :
Colipase is a protein that functions as a cofactor for pancreatic lipase, with which
it forms a stoichiometric complex. It also binds to the bile-salt covered triacylglycerol
interface thus allowing the enzyme to anchor itself to the water-lipid interface.
169. Neonatal sepsis is caused by
a. Strep. Pyogenes
b. Strep. Agalactiae
c. Salmonella species
d. Strep.pneumoniae
Explanation :
Group – B streptococcal (strep.agalactiae) are normal commensals of the female
genital tract and are important cause of neonatal sepsis and meningitis.
170. Which of the following is necessary in the process of exocytosis?
a. Calcium
b. Magnesium
c. Sodium
d. Potassium
Explanation :
Neurotransmitters are transmitted by exocytosis. They are chemical
messages that are transported from nerve to nerve by synaptic vesicles. Synaptic
vesicles are membranous sacs formed by endocytosis of the plasma membrane at
pre-synaptic nerve terminals. The synaptic vesicle awaits a signal, an influx of
calcium ions brought on by an action potential, which allows the vesicle to dock at
the pre-synaptic membrane.
171. SERM that is associated with endometrial hyperplasia is ?
a. Raloxifene
b. Clomiphene citrate
c. Tamoxifen
d. Ormeloxifene
Explanation :
SERM – tamoxifen , doloxifene , toremifene
Possess estrogen antagonistic activity in breast and blood whereas agonistic activity
in bone, uterus, and liver.
Their major indication is in the treatment of breast carcinoma.
They have beneficial effect on bone and lipid profile but increases the risk of
endometrial carcinoma and thromboembolism.
172. Treatment for hyperkalemia is ?
a. Salbutamol
b. Calcium
c. Hemodialysis
d. All of the above
Explanation :

173. Vitamin required for the regulation of the synthesis of collagen is ?


a. Vit. A
b. Vit. C
c. Vit. E
d. Vit. K
Explanation :
Collagen contains specific amino acids – Glycine, Proline, Hydroxyproline and
Arginine. These amino acids have a regular arrangement in each of the three chains
of these collagen subunits. These amino acids are found at specific locations relative
to glycine and are modified post-translationally by different enzymes, both of which
require vitamin C as a cofactor.
174. Breast cancer of 4 cm with axillary lymph node involvement without distant
metastasis is in which stage?
a. 1
b. 2a
c. 3a
d. 3b

Explanation :
175. Glutaraldehyde is not used as a disinfectant for which of the following ?
a. Thermometer
b. Endotracheal tube
c. Cystoscope
d. Bronchoscope
Explanation :
Glutaraldehyde is used for cystoscopes, bronchoscopes, rubber tubes, face masks,
and endotracheal tubes. Wiping with alcohol was at least as efficient as immersion in
a phenolic and is recommended for thermometer disinfection in hospitals.
176. Which of the following is associated with rheumatic heart disease?
a. Roth’s spots
b. Pulsus paradoxus
c. Koplik’s spots
d. McCallum plaques
Explanation :
Rheumatic heart disease describes a group of short-term (acute) and long-term
(chronic) heart disorders that occur as a result of acute rheumatic fever. Damage of
the heart valves is a common result of rheumatic fever. It is usually seen in children
who are 5 to 15 years old. Mural endocardial lesions can be seen as MacCallum
plaques in rheumatic heart disease. These plaques appear as map-like areas of
thickened, roughened, and wrinkled part of the endocardium in the left atrium.
177. Burning sensation in extremites is due to ?
a. Pheripheral neuropathy
b. Poliomyelitis
c. Syringomyelia
d. Multiple sclerosis
Explanation :
Peripheral neuropathy is damage to the peripheral nerves and results in a
tingling, painful, or burning sensation in the extremities. It most commonly occurs in
the legs. Other symptoms of peripheral neuropathy can include weakness and
numbness. Poorly controlled diabetes is one of the most common causes of
peripheral neuropathy, but a number of conditions can be responsible for damage to
the peripheral nerves.
178. Burned airways, 1st line management is ?
a. Tracheostomy
b. Elective intubation
c. Fluid replacement
d. Prophylactic corticosteroid
Explanation :
In cases of thermal burns to the airway, many burn surgeons prefer ET intubation
over blind insertion techniques, but supraglottic airways are acceptable if intubation
isn’t available or can’t be established quickly.

179. Amniocentesis can be done earliest in which of the following weeks?


a. 8 to 10 wks
b. 10 to 12 wks
c. 14 to 16 wks
d. 18 to 20 wks
Explanation :

Amniocentesis is an invasive, diagnostic antenatal test. It involves taking a sample of


amniotic fluid in order to examine fetal cells found in this fluid. Amniocentesis is
usually performed between 14 and 20 weeks. Some medical facilities may
perform amniocentesis as early as 11 weeks. Amniocentesis can be used
later in the third trimester for a few reasons.
180. Malignant hyperthermia is due to defect in ?
a. Ryanodine receptor
b. T- tubule defect
c. Na+/k+ ATP ase defect
d. Mutation of SERCA pump.
Explanation :
Malignant hyperthermia (MH) is a dominantly inherited disorder of skeletal muscle
that leads to a life threatening adverse reaction upon exposure to potent volatile
anesthetics (halothane, isoflurane, sevoflurane, desflurane, etc.) and the skeletal
muscle relaxant succinylcholine. The anesthetic drugs trigger an uncontrolled
calcium (Ca2+) release from the sarcoplasmic reticulum (SR) through the ryanodine
receptor (RYR1) causing a rapid and sustained rise in myoplasmic Ca2+ which
cause prolonged muscle contraction. DOC is dantrolene.
181. Identify the image given

a. Ulcus serpens
b. Acanthancochal ulcer
c. Dentritic ulcer
d. Pseudodentritic ulcer
Explanation :
Viral ulcer – caused by herpes simplex
usually attacks the epithelium and
stroma. In epithelium these dentritic
ulcers are seen. These are called true
dentrites and are like branch of tree.
Corneal sensation is reduced and so it is
regarded as most painless ulcer. Steroid
are C/I in dentritic ulcer . If given, it can
lead to geographical ulcer.
182. Delayed onset of puberty in female age is seen at?
a. 12 yrs
b. 13 yrs
c. 14 yrs
d. 15 yrs
Explanation :
Puberty starts when the pituitary gland begins to produce two hormones ,
luteinizing hormone (called LH) and follicle-stimulating hormone (called FSH), which
cause the ovaries to enlarge and begin producing estrogens. The growth spurt starts
shortly after breasts begin to develop, and the first menstrual cycle begins about 2-3
years later. A girl who has not started to have breast development by the age of 13 is
considered to be delayed.
183. Microsporum affects ?
a. Skin , hair and nail
b. Skin and hair
c. Skin and nail
d. Hair and nail
Explanation :
Microsporum spp. mostly infect the hair and skin, except for Microsporum
persicolor which does not infect hair.
184. Who level of Hb conc. Of anemia in pregnancy is ?
a. 10 gm/dl
b. 11 gm/dl
c. 12 gm/dl
d. 13 gm/dl
Explanation :
Anemia is a condition of low circulating haemoglobin (Hb) in which concentration
has fallen below a threshold lying at two standard deviations below the median of a
healthy population of the same age, sex and stage of pregnancy. The WHO
definition for diagnosis of anemia in pregnancy is a Hb concentration of less than 11
g/dl (7.45 mmoL/L) and a hematocrit of less than 33%.
185. Radiological test for staging of endometrial carcinoma invasion is?
a. MRI
b. CECT pelvis
c. TAS
d. TVS
Explanation :
A dedicated MRI protocol is crucial for accurate MRI evaluation of endometrial
carcinomas. In endometrial carcinoma, it is important to accurately assess invasion
depth and preoperative staging. Fusion of T2-weighted magnetic resonance imaging
(T2WI) and diffusion-weighted magnetic resonance imaging (DWI) may contribute to
the improvement of anatomical localization of lesions.
186. For how long immunusuppresive therapy should be given post renal
transplant?
a. Lifelong
b. 3 months
c. 6 months
d. 9 months

187. Organism causing whipple’s disease ?


a. Actinomycetes
b. Giardia
c. M. Chelonie
d. M. Kansasii
Explanation :
T. whipplei is one of the actinomycetes, and is a distant relative of Mycobacterium
avium-intracellulare, and Mycobacterium paratuberculosis explaining in part why
Whipple's disease is similar to the diseases caused by these bacteria. The disease is
common in farmers and those exposed to soil and animals, suggesting that the
infection is acquired from these sources.
188. What is the ideal time to check for diabetic retinopathy in type 1 IDDM ?
a. 1 yr
b. 3 yr
c. 5 yr
d. 7 yr
Explanation :
For type – 1 DM , pt can wait for 5 yrs to check the retina
For type – 2 DM , it is necessary to check retina immediately as it is asymptomatic.
189. Most common cause of secondary amenorrhoea is?
a. Turner syndrome
b. Pregnancy
c. Pelvic radiation
d. Mullerian agenesis
Explanation :
Secondary amenhorrea is defined as a absence of mensuration of 3 normal cycles
or for 6 months or more in a woman with previous normal mensural pattern in
absence of pregnancy. Most common cause of secondary amenorrhea is
pregnancy.
190. Floor of femoral triangle is formed by ?
a. Inguinal ligament
b. Adductor longus
c. Pectineus muscle
d. Sartorius muscle
Explanation :
Borders of femoral triangle :
Superior border – formed by inguinal ligament
Lateral border – formed by the medial border of Sartorius muscle
Medial border – formed by medial border of adductor longus muscle
Roof – formed by fascia lata
Floor – formed by pectineus, iliopsoas muscle, and adductor longus muscle.

191. Which of the following is not a sign of placental separation?


a. Gush of blood flow
b. Firm and globe shaped uterus
c. Uterus rises in the abdomen
d. Shortening of the length of umbilical cord
Explanation :

Childbirth involves three stages:

First stage: Begins from the onset of true labor and lasts until the cervix is
completely dilated to 10 cm.

Second stage: Continues after the cervix is dilated to 10 cm until the delivery
of your baby.

Third stage: Delivery of your placenta. The third stage is the delivery of the
placenta and is the shortest stage. The time it takes to deliver your placenta
can range from 5 to 30 minutes.
Signs of placental separation :
 A gush of blood comes from vagina
 Umbilical cord gets longer
 Uterus rises over the abdomen
 Uterus becomes round in shaped.

192. Most common site for the development of laryngeal web is?
a. Anterior
b. Posterior
c. Subglottic
d. Supraglottic
Explanation :
Laryngeal web is a condition in which child’s larynx (voice box) contains a layer
of web-like tissue. This partially obstructs child’s windpipe, causing frequent
shortness of breath and other symptoms. Laryngeal webs occur in the glottic level
and affect the vocal cords ™More than 90% of laryngeal webs webs are located
anteriorly and extend toward the arytenoids. Other types include the posterior glottic
web, causing interarytenoid vocal cord fixation; subglottic webs, which may occur
with or without cricoid cartilage involvement and subglottic stenosis; and supraglottic
webs.
193. Rise in JVP with inspiration is seen with all except ?
a. Pericardial effusion
b. Pleural effusion
c. Constrictive pericarditis
d. Cardiac tamponade
Explanation :
JVP will decrease during inspiration in the normal state. The JVP will rise during
inspiration (kussmaul’s sign )in the presence of pericardial constriction , right
ventricular infarction , pericardial effusion, or rarely, cardiac tamponade.

194. Normal refractive power of neonates are ?


a. Myopia
b. Pseudomyopia
c. Hypermetropia
d. Anisometropes
Explanation :
The eye of the newborn is hypermetropic by 00+2 to +3D. A – P diameter of eyeball
is 16.5 mm. Corneal diameter is 10mm. At 2 - 4 months there is a development of
fixation reflex , and at 6 -8 months depth perception starts developing.
195. Intraabdominal pressure during gynaecological procedure is ?
a. 5 – 10 mg
b. 10 – 20 mg
c. 20 – 30 mg
d. 40 – 50 mg
Explanation :
The deliberate induction of carbon dioxide pneumoperitoneum during laparoscopic
surgery could be a possible source of cardiovascular collapse. An IAP from 8 to 12
mm Hg is recommended for laparoscopic surgery, to avoid complications caused by
hemodynamic derangements.

196. Which of the following is the marker for hepatocellular carcinoma ?


a. Alpha feto protein
b. Beta HCG
c. TSH
d. Thyroglobulin
Explanation :
Hepatocellular carcinoma (HCC) is one of the most frequent malignant tumors.
Serum tumor markers, as the effective method for detecting hepatocellular
carcinoma for a long time, could be divided into 4 categories: oncofetal antigens and
glycoprotein antigens; enzymes and isoenzymes; genes; and cytokines. Serum
alpha fetoprotein (AFP) is the most widely used tumor marker in detecting patients
with hepatocellular carcinoma, and has been proven to have capability of prefiguring
the prognosis.
197. Which of the following statement is false regarding gastrinoma?
a. Associated with MEN 1 syndrome
b. Most common presentation is Gastric outlet obstruction
c. Mc site is duodenum
d. It is relatively silent
Explanation :
Gastrinomas are neuroendocrine tumours that are usually found in the first part of
the small intestine or in the pancreas, which produce the hormone gastrin. The signs
and symptoms of gastrinomas that most patients experience are abdominal pain,
heartburn, nausea, blood in the vomit, bleeding from the back passage and
diarrhoea.
198. Damage to which of the following nerves lead to aphonia, aspiration, vocal
cord paralysis?
a. inferior laryngeal nerve
b. superior laryngeal nerve
c. Internal laryngeal nerve
d. Recurrent laryngeal nerve

Explanation :

199. Esmolol is a short acting B- blocker because?


a. Blood esterase activity
b. Rapid renal clearance
c. Rapid hydrolysis
d. Increased bioavailablity
Explanation :
Esmolol (Brevibloc) is an intravenous, short-acting, titratable, cardioselective beta
blocker with a very rapid onset and offset of action (t1/2 = 9.2 minutes). Esmolol-
induced beta blockade can be maintained as long as infusion is continued. It exhibits
neither intrinsic sympathomimetic activity nor significant membrane-stabilizing
activity. It is rapidly metabolized by an esterase in the erythrocyte cytosol to an
inactive acid metabolite. Its hemodynamic and electrophysiologic effects are similar
to those of other beta blockers.
200. Route of spread of TB to kidney is through?
a. Lymphatic
b. Hematogenous
c. Both
d. None
Explanation :
Renal tuberculosis (renal TB) is a type of extra-pulmonary tuberculosis that
affects the kidney. It is caused by Mycobacterium tuberculosis, which is a common
cause of respiratory infection especially in the low socio-economic regions of the
underdeveloped and developing countries. The kidneys can get infected through the
spread of infection from other parts of the body especially the lungs via the blood.
This type of infection of kidneys that spreads from other sites in the body that are
already affected is referred to as secondary tuberculosis.
201. A 36 yr old man who was taking some drug develops a patch of redness and
swelling of the skin. which of the following is possible diagnosis?
a. TEN
b. Fixed drug eruption
c. Pemphigus vulgaris
d. Steven Johnson syndrome
Explanation:
A fixed drug eruption is an allergic reaction to a medicine that characteristically
recurs in the same site each time when a particular drug is taken. It presents as well
defines, round or oval patches of redness and swelling of skin, sometimes
surrounded by blisters. In mucosal sites, extensive ulceration can occur. The hands,
feet, lips, eyelids, genitalia, and perianal regions are common sites.
202. Choice of contraception in patients with history of gestational diabetes ?
a. Combined contraceptive pills
b. DMPA
c. IUD
d. Barrier method
Explanation :
Women with prior gestational diabetes are at risk of developing overt diabetes
predominantly type- 2 after pregnancy often during their reproductive cycles.
Contraception is essential component to be able to plan pregnancy when glucose
status is normalized and may also confer protection from developing diabetes by
preventing a subsequent pregnancy. IUD is a very effective and reversible
contraceptive method without metabolic disturbances and therefore is an ideal
contraceptive for women with prior GDM.
203. Laboratory findings shows high PTH , high ALP, low calcium and low PO4.
Which of the following is the possible diagnosis of the patient?
a. Paget’s disease of bone
b. Osteoporosis
c. Vit. D deficiency
d. Hypoparathyroidism
Explanation :
Among the patients with severe vit .D deficiency, prevalence of hyperparathyroidism
is high (38%). Severe vit. D deficiency was associated with elevated levels of
alkaline phosphatase, a marker for increased bone turnover.

204. A 9 yr old school going girl has a difficulty in focusing on one task, day
dreamer, easily distracted, talk nonstop. Possible diagnosis is ?

a. Conduct disorder
b. ADHD
c. AUTISM
d. Pervasive development disorder

Explanation :

ADHD is one of the most common conditions in childhood. It impacts how kids
function in school and in everyday life. symptoms include :

 Difficulty paying attention (inattention)


 Being overactive (hyperactivity)
 Acting without thinking (impulsivity)

There is no cure for ADHD, currently available treatments may help reduce
symptoms and improve functioning. ADHD is commonly treated with medication,
education or training, therapy, or a combination of treatments.
205. False statement regarding peutz jegher syndrome is?
a. Associated with mucosal, lip and skin melanosis
b. It is autosomal disorder
c. Has multiple polyp in intestine that require surgery to stop bleeding
d. It is a malignant condition
Explanation :
Peutz-Jeghers syndrome (PJS) is an autosomal dominant inherited disorder
characterized by benign intestinal hamartomatous polyps in association with a
distinct pattern of skin and mucosal macular melanin deposition. People with Peutz-
Jeghers syndrome have a high risk of developing cancer during their lifetimes.
Cancers of the gastrointestinal tract, pancreas, cervix, ovary, and breast are among
the most commonly reported
206. B – blocker ointment is used for which of the following conditions?
a. Superficial burns
b. Infantile hemangiomas
c. Contaminated wounds
d. Clean wounds
Explanation :
The treatment of infantile hemangiomas changed from the use of oral
corticosteroids to oral propranolol . Topical beta blockers are now used for lesions
with both deep and superficial components and those that are amblyogenic. When
initiated in the proliferative phase of the lesion, the effectiveness of the treatment can
be seen within days.

207. NRHM tracking system is for ?


a. Mother and child linkage
b. Tracking leprosy cases by ASHA
c. Tracking HIV patients
d. Monitoring TB patients.
Explanation :
Mother and Child Tracking System (MCTS) is an initiative of Ministry of
Health & Family Welfare (national rural health mission) for ensuring delivery of
full spectrum of healthcare and immunization services to pregnant women and
children up to 5 years of age. Generation of work plans of ANMs, sending
regular alerts to the service providers as well as beneficiaries about the services
and to monitor delivery of services will go a long way in ensuring quality service
delivery, micro birth planning, ensuring universal immunization and will have
positive impact on important health indicators like Infant Mortality Rate and
Maternal Mortality Ratio.—
208. Equilibrium potential of chloride ions in the cell according to nerst equation?
a. +60mv
b. -70mv
c. +80 mv
d. – 90 mv
Explanation :
Nerst equation gives the equilibrium potential or isoelectric potential. Equilibrium
potential is the membrane potential at which equilibrium is reached. ie, there is no
net flux of that ion.
E Na+ = +60 mv (tendency of sodium is to diffuse in till potential reaches +60 mv)
E K+ = -90 mv tendency of potassium is ti diffuse out till potential reches -90mv)
E cl- = - 70 mv . This is the closest to the RMP.
209. In which of the following eating disorder amenorrhoea is present?
a. Anorexia nervosa
b. Bullemia nervosa
c. Schizophrenia
d. Binge eating disorders
Explanation :
Anorexia nervosa Bulimia nervosa
Mainly refers to starving on purpose to It refers to binge eating followed by
create a skeletal like body. purging, resulting in an average or over
weight.
Very less eating Attempts to counteract the effects of
over eating by self induced vomiting.
Restrictive type is commoner
Binge eating – 25- 50% Binge eating is required for diagnosis
May result in conditions such as May result in heart failure and damage
Amenorrhoea , osteoporosis, infertility to esophagus and teeth.
etc.
210. Fetal biophysical profile doesn’t include?
a. Non- stress test
b. Fetal breathing movements
c. To estimate placental function
d. Fetal tone
Explanation :

211. Identify the organism in a slide image given below:

a. Chlonorchis sinensis
b. Schistosoma hematobium
c. Fasciola hepatica
d. Paragonimus westermani
Explanation :

Schistosoma hematobium causes schistosomiasis or bilharziasis . Symptoms


associated are allergic dermatitis due to cercariae known as swimmers itch .
Symptoms due to eggs are egg granuloma and hematuria . It also causes bladder
cancer.
212. In modern management techniques, in which of the following analysis
Benefits are measured in monetary terms ?
a. Cost effective analysis
b. Cost benefit analysis
c. System analysis
d. Network analysis
Explanation :
Cost benefit analysis :
It is a management technique where economic benefits of any programme are
compared with cost of that programme. The benefits are expressed in monetary
terms. The main draw back of this technique is that all benefits in field of health
cannot be expressed in monetary terms.
213. Which of the following is a short acting synthetic opoid?
a. Loperamide
b. Buprenorpine
c. Remifentanil
d. Tramadol
Explanation :
Remifentanil is a potent ultra short-acting synthetic opioid analgesic drug. It is
given to patients during surgery to relieve pain and as an adjunct to an anaesthetic.
Remifentanil is a specific mu-type-opioid receptor agonist. Hence, it causes a
reduction in sympathetic nervous system tone, respiratory depression and analgesia.
214. True statement regarding erythropoietin is ?
a. Senses hypoxia
b. Transfers oxygen to tissues
c. Stimulates bone marrow
d. Mainly produced by liver
Explanation :
Erythropoietin (EPO) is a hormone produced by the kidney that promotes the
formation of red blood cells by the bone marrow. Erythropoietin stimulates the bone
marrow to produce more red blood cells. The resulting rise in red cells increases the
oxygen-carrying capacity of the blood. Only about 10% of erythropoietin is produced
in the liver.
215. Common site of carcinoid tumour is ?
a. Appendix
b. Ileum
c. Rectum
d. Lungs
Explanation :
Carcinoid tumors arise from neuroendocrine cells and may develop in almost
any organ. The gastrointestinal tract accounted for 54.5% of the tumors. Within the
gastrointestinal tract, the small intestine was the most common site (44.7%),
followed by the rectum (19.6%), appendix (16.7%), colon (10.6%), and stomach
(7.2%).
216. Which among the following cell is activated form of macrophage?
a. Stellate cells
b. Langerhan cells
c. Gaint cell
d. Epitheloid cells
Explanation :
Epithelioid cells are cells of the mononuclear phagocyte system found in certain
granulomas mainly associated with intense immunological activity. These cells show
little phagocytic activity.
217. Function of SGLT 1 is to transport ?
a. Sodium
b. Glucose
c. Potassium
d. Bicarbonate
Explanation :
SGLT -1 , is a high-affinity Na+/glucose co-transporter . SGLT1 transports glucose
and galactose across the luminal (gut) side of enterocytes, and is the first step in the
absorption of sugars from nutrients. In the kidney, SGLT1 is located on the apical
(urine) side of the proximal tubule, and facilitates the reabsorption of urinary glucose
from the glomerular filtrate.
218. Hypervirulent klebsiella pneumonia causes?
a. URTI
b. Conjunctivitis
c. endopthalmitis
d. Bronchitis
Explanation :
Hypervirulent Klebsiella pneumonia (hvKP) infection have been reported
worldwide . Such strains are notorious for their capacity to cause serious and
metastatic infections in young and healthy individuals, such as pyogenic liver
abscesses and endophthalmitis.
219. Most common site of Puerperal infection is?
a. Uterus
b. Placenta
c. Vagina
d. Umbilical cord
Explanation :
Puerperium is a period of after child birth. It starts after delivery and extends 6-8 wks
after child birth.
Puerperal pyrexia – rise of temp reaching 100.4 ‘ F (38’C) on separate occations at
24 hrs apart within 1st 10 days following delivery.
Most common site of puerperal infection – placental site.
Most common manifestation of puerperal infection – endometritis
Most common cause of puerperal sepsis – streptococcus
Most common route of infection – direct spread
220. Main function of DNA POLYMERASE 3 is?
a. To initiate replication
b. To fill the gap after removing primer
c. Helps in proof reading
d. Endonuclease activity
Explanation :
DNA polymerase 3 is responsible for replication but it doesn’t start the process of
replication. It will read the parent DNA from 3’ to 5’. Specific feature of DNA
polymerase 3 is to
1) Proof reading /rechecking
2) Exonulease activity
DNA polymerase 1 helps to fill the gap once after removing the primer by RNAse H.
221. Most common site of innocent murmur is ?
a. Apex
b. Left lower sternal border
c. Above clavicles
d. 2nd rt intercoastal space
Explanation :
Still's murmur is by far and away the most common innocent heart murmur.
These signs are unreliable in under age 1 year due to higher Incidence of Congenital
Heart Disease.

222. Afferents of cremasteric reflex are present in ?


a. Genitofemoral
b. Iliohypogastric
c. Ilioinguinal
d. Pudendal
Explanation :
The cremasteric reflex is dependent upon the nerve roots L1 and L2.
Genitofemoral nerve carries both the afferent and efferent components of reflex. This
reflex is elicited by stroking the superior and medial part of the thigh in a downward
direction. The normal response in males is a contraction of the cremasteric muscle
that pulls up the scrotum and testis on the side stroked. In children, this reflex may
be over exaggerated and this can lead to mistaken of diagnosis of undescented

testis. Upper and lower motor neuron disorders can cause an absence of
cremasteric reflex.
223. Chance of metastasis to liver in colon cancer is?
a. Less than 25%
b. 25- 50%
c. 75%
d. 90 %
Explanation:
More than 50% of the patients with colorectal cancer (CRC) have or will develop
metastasis, with a quarter having distant metastatic disease at the time of diagnosis,
most frequently in the liver. Liver metastasis is the leading cause of cancer-related
morbidity and mortality in colorectal cancer. Left sided colon cancer has 70%
increased risk of isolated liver metastases. The only potentially curative treatment
for liver metastasis is liver resection, but only 15% to 20% of the patients are suitable
for surgical resection.
224. False statement regarding crohn’s disease is?
a. Skip lesions are seen
b. Presence of non caseating granuloma
c. Intestinal resection is curative
d. Ileum is the most common site to be affected.
Explanation :
225. Which of the following is not a absolute contraindication of OCP?
a. Breast cancer
b. Liver disease
c. Age <30 yrs
d. Congenital hyperlipdemia
Explanation :

226. Which of the following is not a time resistant strategy ?


a. Aim
b. Goal
c. Objective
d. Mission
Explanation :
Goal : ultimate desired state towards which objectives and resources are directed .It
is not constrained by time or exisiting recources and shows “all or none
phenomenon”.
Objective : is planned en – point of all activities . It is concerned with problems itself.
Mission : is a disruption of fundamental principle of existence of a programme. It is
usually time bound.
Aim/ Target : a discrete activity which helps measure extent of attainment of
objectives. It is a concept of degree of achievement.
227. A 70 yr old patient underwent TURP for BPH under spinal anaesthesia. One
hr later he develops vomiting and altered sensorium. The most possible cause is
?
a. Rupture of bladder
b. Hyperkalemia
c. Over dosage of spinal anaesthetic agent
d. Water intoxication

228. Weight of respired lung to total body weight is?


a. 1/35th
b. 1/70th
c. 1/50th
d. 1/90th
Explanation :
Character Unrespired Lungs Respired Lungs
Weight in relation to 1/70 1/35
body weight
Volume Small Large and covers the
heart
Consistency Liver Like: Dense firm Soft, spongy, elastic,
non crepitant Crepitant
Extension Up to the level of 4th and Up to the level of 6th and
5th rib 7th rib
Specific gravity 1.04 0.94
Margin Sharp Rounded
color Uniform reddish Mottled/ marbled
appearance
Air vesicle Not inflated Inflated
Section Little froth less blood Abundant frothy blood
exudates on pressure exudates
Breslow life test Whole or part sinks Expanded

229. Which of the following statement is correct regarding fast breathing in a child?
a. > 60 /min in a child > 12 months
b. >50 /min in a child of age between 2 months to 1 yr
c. > 40/min in a child of > 2 months
d. > 70 /min in a child of > 18 months
Explanation :
According to ARI control programme, a case of pneumonia is considered only if
child has fast breathing.
Upto 2 months - >60/min
2- 12 months - > 50/min
>12 months - >40/min
230. Which of the following hormone is increased in ZES?
a. Histamine
b. Pepsin
c. Gastrin
d. Secretin
Explanation :
Zollinger-Ellison syndrome is a rare disorder that occurs when one or more
tumors form in the pancreas and duodenum. The tumors, called gastrinomas,
release large amounts of gastrin that cause the stomach to produce large amounts
of acid. Patients with ZES show gastric acid hypersecretion, which induces refractory
peptic ulcers, severe gastroesophageal reflux disease, diarrhea, and even death.
The plasma concentration of gastrin is measured by RIA or ELISA , and can be used
to diagnose gastrin-secreting tumors, atrophic gastritis, gastric ulcers, and pernicious
anemia. Zollinger-Ellison syndrome treated with medications to reduce gastric acid
secretion and with surgery to remove gastrinomas.
231. Febuxostat is used in the treatment of?
a. Gout
b. Rheumatoid arthritis
c. Polyarteritis nodosa
d. gaint cell arteritis
Explanation :
Febuxostat is used to lower uric acid levels in people with gout. It inhibits xanthine
oxidase . Febuxostat works by reducing the amount of uric acid made by the body.
Increased uric acid levels can cause gout.
232. Dense bodies in smooth muscle is analogues to which part of skeletal
muscle?
a. I band
b. M line
c. Z line
d. A band
Explanation :
Smooth muscle fibers are spindle-shaped and have a single nucleus; they range
from about 30 to 200 μ m and they produce their own connective tissue,
endomysium. Although they do not have striations and sarcomeres, smooth muscle
fibers do have actin and myosin contractile proteins, and thick and thin filaments.
These thin filaments are anchored by dense bodies. A dense body is analogous to
the Z-discs of skeletal and cardiac muscle fibers and is fastened to the sarcolemma.
233. Excessive vomiting is an example of which type of acid base disorder?
a. Metabolic acidosis
b. Repiratory acidosis
c. Metabolic alkalosis
d. Respiratory alkalosis
Explanation:
In a case of vomiting , H+ ions are lost and because of this, bicarbonate ions
remains and the patient develops metabolic alkalosis.
234. Under mental illness act, detaining of mentally ill person is punishable for how
many yrs?
a. 1 yr
b. 2 yr
c. 5 ys
d. 7 yr
Explanation :
The mental heath act (1987), repels the Indian lunacy act (1912).
Defines, “mentally ill person” as a person who is in need of treatment by reason of
any mental disorder other than mental retardation for treatment as in patient a
person can represent himself or represented by family or friends in writing to the
director of a psychiatric hospital. For a preliminary diagnosis , he can be kept in
hospital for 10 days, if more time is required then it has to permit by magistrate who
can give extension of 10 days twice.
Punishment : detains or keeps mentally ill patient otherwise in accordance with the
act is punishable with upto 2 yrs imprisonment or Rs.10000 fine or both.
235. In establishing causal association, most essential criterion is?
a. Consistency of association
b. Temporal association
c. Duration of association
d. Strength of association
Explanation :
The Bradford-Hill criteria are widely used in epidemiology as providing a framework
against which to assess whether an observed association is likely to be causal.
1. Strength of the association.
According to Hill, the stronger the association between a risk factor and outcome, the
more likely the relationship is to be causal.
2. Consistency of findings.
Have the same findings must be observed among different populations, in different
study designs and different times?
3. Specificity of the association.
There must be a one to one relationship between cause and outcome.
4. Temporal sequence of association.
Exposure must precede outcome.
5. Biological gradient.
Change in disease rates should follow from corresponding changes in exposure
(dose-response).
6. Biological plausibility.
Presence of a potential biological mechanism.
7. Coherence.
Does the relationship agree with the current knowledge of the natural history/biology
of the disease?
8. Experiment.
Does the removal of the exposure alter the frequency of the outcome?

236. Which of the following is the most common form of tularaemia?


a. Fever
b. Lymphadenopathy
c. Ulcer
d. Chills
Explanation :
Ulceroglandular - This is the most common form of tularemia and usually occurs
following a tick or deer fly bite or after handing of an infected animal.
Glandular - Similar to ulceroglandular tularemia but without an ulcer.

Oculoglandular - This form occurs when the bacteria enter through the eye. This
can occur when a person is butchering an infected animal and touches his or her
eyes. Symptoms include irritation and inflammation of the eye and swelling of lymph
glands in front of the ear.

Oropharyngeal - This form results from eating or drinking contaminated food or


water.

Pneumonic - This is the most serious form of tularemia. Symptoms include cough,
chest pain, and difficulty breathing. This form results from breathing dusts or
aerosols containing the organism. It can also occur when other forms of tularemia
(e.g. ulceroglandular) are left untreated and the bacteria spread through the
bloodstream to the lungs.

237. Nyhas classification of direct inguinal hernia is ?


a. Type - 1
b. Type - 2
c. Type - 3 a
d. Type – 3 b
Explanation :

238. Which of the following statement is true regarding intermittent claudication ?


a. Pain starts even at rest
b. Pain begins when the patient starts to walk
c. Pain relieved after few mins of walking
d. Patient was unable to walk
Explanation :
Claudication is pain and/or cramping in the lower leg due to inadequate blood flow to
the muscles. It is commonly referred to as "intermittent" claudication because it
comes and goes with exertion and rest. In severe claudication, the pain is also felt at
rest. Intermittent claudication can be due to temporary artery narrowing due to
vasospasm (spasm of the artery), permanent artery narrowing due
to atherosclerosis, or complete occlusion (closure) of an artery to the leg. Diagnostic
tests include blood pressure measurements to compare the arms and legs, Doppler
ultrasonography on the legs, duplex Doppler/ultrasound exam of the extremities to
visualize arterial blood flow, an ECG, and arteriography. The prognosis with
intermittent claudication is generally favorable because the condition often stabilizes
or improves in time. Conservative therapy is advisable.
239. Which of the following is a malignant cerebellar tumour in children?
a. Craniopharyngioma
b. Ependymoma
c. Medulloblastoma
d. Oligodentroglioma
Explanation :
Medulloblastoma is a fast-growing, high-grade tumor located in the cerebellum—the
lower, rear portion of the brain. It is unusual for medulloblastomas to spread outside
the brain and spinal cord. the exact cause of medulloblastoma is not known .The
most common symptoms of medulloblastoma include behavioral changes, changes
in appetite, symptoms of increased pressure on the brain (eg, headache, nausea,
vomiting, and drowsiness, as well as problems with coordination). Unusual eye
movements may also occur. Treatment consists of surgical removal of as much
tumor as possible, radiation, and then chemotherapy.
240. A patient was brought to the hospital with a history of RTA 8 hrs back. A few
drops of blood were noted at the external urethral meatus. He had not passed
urine and his bladder palpable per abdomen. The probable diagnosis is.?
a. Rupture bladder
b. Urethral injury
c. Anuria due to hypovolemia
d. Extravasation of urine in the retroperitoneum due to fracture pelvis
Explanation :
Membranous urethra is injured in this case. Common cause is due to fractured pelvis
due to trauma or RTA. Blood is seen at urethral meatus. Investigation of choice is
RGU. Managed with supra pubic catheter and drain the urine.
241. Thymidylate synthase inhibitor is?
a. Cladribine
b. Lomustine
c. 5 – flurouracil
d. Paclitaxel
Explanation :
5 – flurouracil inhibits thymidine synthase. Used for – head and neck, breast , and
ovarian cancers. It is also used topically for basal cell cancers and keratosis. It
causes single strand breaks affecting the action of both DNA and RNA. S/E – bone
marrow suppression.
242. For 100 mt sprint energy/ATP that are required is from?
a. Muscle
b. Liver
c. Blood
d. Lungs
Explanation :

The 100-meter sprint requires intense, all-out effort for a very short time. Cells break
down carbohydrates and fats to provide energy, then store that energy in the form of
adenosine triphosphate (ATP). ATP is often termed the cell's energy currency.
Because the breakdown of stored ATP releases energy almost instantaneously, & it
is the primary energy source as a sprinter explodes out of the blocks. However,
muscles only store enough ATP for two to three seconds of maximal power
output. To rapidly replenish ATP, muscle cells break down another high-energy
compound called creatine phosphate (CP). This is called the ATP-CP or phosphagen
energy system, sometimes referred to as the alactic anaerobic system because it
does not require oxygen.

243. In which of the following conditions large ovaries is not seen?


a. PCOS
b. Mucinous cystadenoma
c. Turner syndrome
d. Fibroma
Explanation :
Turner syndrome – genotype- 45XO
Normal pathway of development is female soin the absence of y chromosomes the
fetus becomes female but for normal ovarian development 2X chromosomes are
required. Hence the presence of only one chromosome will make the gonads streak
like. It is the most common cause of primary amenorrhoea and delayed puberty in
females.
244. Moderately increased risk of invasive breast cancer is associated with which
of the following?
a. Fibroadenoma
b. Ductal ectasia
c. Apocrine metaplsia
d. Atypical ductal hyperplasia
Explanation :
Of the epithelial hyperplasias, only florid hyperplasia and atypical hyperplasia are
associated with increased risk of breast cancer.
245. Which type of Anemia seen in rheumatoid arthiritis is ?
a. Haemolytic nemia
b. Aplastic anemia
c. Anemia of chronic disease
d. Sideroblastic anemia
Explanation :

RA can be associated with different types of anemia, including anemia of chronic


inflammation and iron deficiency anemia. Chronic inflammation can lower the
production of red blood cells in your bone marrow. This can lead to the release of
certain proteins that affect how the body uses iron. Inflammation can affect the way
the body produces erythropoietin, a hormone that controls the production of red
blood cells.

246. Which of the following statement is true regarding aminocaproic acid?


a. Is a irreversible inhibitor of plasminogen
b. Is a derivative and analogue of amino acid arginine.
c. Used as a fibrinolytic agent
d. C/I in bleeding disorders
Explanation :
Overdose of fibrinolytic agent is treated by EPSILON AMINOCAPROIC ACID
(EACA) & TRANEXEMIC ACID. EACA is a derivative and analogue of the amino
acid lysine, and effective inhibitor of plasminogen. EACA has been used in
numerous clinical situations to control bleeding and has been claimed to be an
effective agent in subarachnoid hemorrhage, genitourinary bleeding from many
causes, and in dental surgery in hemophiliacs. It may also be effective in several
less well studied situations such as prophylaxis of bleeding episodes in
hemophiliacs, control of menorrhagia, gastrointestinal bleeding, obstetrical bleeding
and in bleeding following cardiac and thoracic surgery. Major side effects from EACA
include hypotension, cardiac arrhythmias, rhabdomyolysis, and generation of
thrombi.

247. “PAD sign “ is a feature of which disease?


a. Ca. Head of pancreas
b. Pyloric stenosis
c. Chronic pancreatitis
d. Duodenal peptic ulcer
Explanation :
Pancreatic calcifications may be seen concurrently in approximately in 2% of
patients who have chronic pancreatitis complicated by pancreatic carcinoma. Upper
GI barium studies reveal an extrinsic impression of the mass on the posteroinferior
aspect of the antrum of stomach. This is known as “Antral Pad sign”.

248. Correct Sequence of hematopiesis is ?


a. Placenta >liver > yolk sac
b. Placenta> bone marrow> yolk sac
c. Yolk sac > bone marrow> liver
d. Yolk sac > liver> bone marrow
Explanation :
Erythropoiesis begins in the yolk sac from 3rd week of intrauterine life.
From 3rd month – liver
From 4th month – bone marrow
249. Which of the following is not associated with pain in sub mucosal fibroids?
a. Torsion
b. Large size
c. Associated with endometriosis
d. Degeneration
Explanation :
A patient with fibroid usually doesn’t complain pain. If a fibroid patient presenting
with pain it deserves to be evaluated against :
 Malignancy
 It is being extruded from body as a polyp
 Associated endometriosis
 Torsion of a pedunculated fibroma
 Degeneration
250. Most common cause of pulsatile proptosis is?
a. craniopharyngioma
b. Cavernous fistula
c. Prolactinoma
d. Meningitis
Explanation :
Proptosis – forward displacement or bulging of eyeball. Usually caused by carotid-
cavernous fistula which may occur after trauma and cause arterial blood to get into
the superior or inferior ophthalmic veins that empty into the sinus. In addition to
pulsatile proptosis may also get palsy of CN 6 (most common) or CN 3 or 4 less
commonly, along with loss of vision.
251. Type of liver disease seen in kwashiorkor is ?
a. Fatty liver
b. Cirrhotic liver
c. Hepatitis
d. Liver abcess
Explanation :
Kwashiorkor, also called protein malnutrition, condition caused by
severe protein deficiency. It is common in young children weaned to a diet
consisting chiefly of cereal grains, cassava, plantain, and sweet potato or similar
starchy foods. The latter term comes from the reddish orange discoloration of the
hair that is characteristic of the disease. Other symptoms include dry skin and skin
rash, potbelly and edema, weakness, nervous irritability, anemia, digestive
disturbances such as diarrhea, and fatty infiltration of the liver.
252. Opportunistic screening for HTN & DM starts at the age of ?
a. 30 yrs
b. 40 yrs
c. 50 yrs
d. 60 yrs
Explanation :
As per the guidelines issued by the National Program for Prevention and Control
of Cancer, Diabetes, Cardiovascular disease and Stroke, opportunistic screening for
hypertension and diabetes in ≥30 years of age is to be done by the health workers at
the subcenter level (the most peripheral health center) and referred to the
Community Health Centre (CHC).
253. Immediate effect of insulin on DKA type - 1 DM patient is?
a. High bicarbonate
b. Glycosuria
c. Hyperkalemia
d. Increased PH

254. Which statement is false about carcinoid tumour?


a. Presenting with flushing, wheeze, diarrhoea
b. Most common site is ileum
c. Conservative management is curative
d. Present at multiple sites
Explanation :
Carcinoid tumors are of neuroendocrine origin and derived from primitive stem cells
in the gut wall, but they can be seen in other organs, including the
lungs, mediastinum, thymus, liver, pancreas, bronchus, ovaries, prostate, and
kidneys. The etiology of carcinoid tumors is not known, but genetic abnormalities are
suspected.
Sometimes these tumours can produce hormones, which can cause a collection of
symptoms known as ‘carcinoid syndrome'. This can include:
 flushing
 severe diarrhoea
 loss of appetite
 weight loss
 increased heart rate
 wheezing
 sudden changes in your blood pressure.

Surgery, if feasible, is the only curative therapy. If the tumor has metastasized
(most commonly, to the liver) and is considered incurable, there are some promising
treatment modalities, such as radiolabeled octreotide or the radiopharmaceutical for
arresting the growth of the tumors and prolonging survival in patients with liver
metastases.

255. Gynecomastia in a newborn is due to the effect of which hormone?


a. Maternal Estrogen
b. Fetal estrogen
c. B – HCG
d. AFP
Explanation :
Neonatal gynecomastia is caused by the passage of maternal hormones (both
estrogen and progesterone) through the placenta during pregnancy. In infants the
breast enlargement usually progresses during the first 2 months of life. Also
sometimes is observed secretion of the breasts. The breasts should not be red, hot
and soft in neonatal gynecomastia. Diagnosis is made by physical examination ,
there are laboratory tests that are needed. No treatment is necessary because the
mother's hormones are out of the baby's bloodstream , so will disappear from the
breast tissue.
256. Important Step at waste management in hospital is?
a. Segregation
b. Transportation
c. Dumping
d. Generation
Explanation :
Segregation is one of the most important steps to successfully manage HCW.
Segregating hazardous from nonhazardous waste also reduces the great risks of
infecting workers handling HCW. Actually, the part of the HCW that is hazardous and
requires special treatment could be reduced to some 2-5% if the hazardous part
were immediately separated from the other waste. About 10-25% of the HCW is
hazardous, treatment and disposal costs could be greatly reduced if a proper
segregation were performed.
257. Causative agent of recurrent juvenile laryngeal papillatosis is?
a. EBV
b. Respiratory syncytial virus
c. HPV 6 and 11
d. Adenovirus
Explanation :
Age group affected – 2- 4 yrs. Caused by HPV serotype of 6 and 11. It is multiple in
nature. Symptoms include hoarseness of voice. Managed with co2 lazer therapy.
Serotype 11 is more aggressive form which may transform to malignancy.
258. P53 inhibitor acts in which phase of cell cycle?
a. G1 phase
b. G1- S phase
c. M phase
d. S – G2 phase
Explanation :
P53 is tumour suppressor gene & mc gene mutated in most human cancers. P53 is
also known as the “Guardian of Genome”. P53 is a check point at both G1 – S phase
and G2 – M phase.
259. Schilling’s test is positive for which of the following conditions?
a. Iron deficiency anemia
b. Thalasemia
c. AML
d. Vit b12 deficiency
Explanation :
The Schilling test is used to diagnose the site of the disorder causing cobalamin
deficiency. The Schilling test is performed to evaluate vitamin B12absorption.
Ingested vitamin B12 combines with intrinsic factor (produced in the stomach) and is
absorbed in the distal ileum.
260. Best parameter to determine the fetal age in 1st trimester is?
a. Crown rump length
b. LMP
c. Level of AFP
d. Gestational sac mean diameter
Explanation :
Estimation of the gestational age by sonographic measurements of fetal parameters
is usually done by measuring mean sac diameter (MSD), Crown-rump length (CRL),
biparietal diameter (BPD), head circumference (HC), femoral length (FL) and
abdominal circumference (AC) depending on the trimester of pregnancy. MSD and
CRL in 1st trimester and BPD, HC, FL and AC in 2nd and 3rd trimester. Gestational
age was calculated by MSD using Rempen normogram and Hadlock normograms
were used for the rest of fetal parameters. Gestational age by LMP was also
calculated at the same time. Correlation of different fetal parameters in estimating
gestational age in different trimesters was done by Pearson correlation. Pearson
correlation showed that the CRL was the best fetal parameter (correlation coefficient
of 0.909) in the first trimester.
261. All are true about gastric carcinoma except?
a. H.pylori infection increase risks
b. Most common site is at fundus
c. Vit.C protects
d. More in low socioeconomic group
Explanation :
Stomach cancer usually begins in the mucus-producing cells that line the stomach.
This type of cancer is called adenocarcinoma. The most common site of gastric
cancer is antrum of stomach. Symptoms include postprandial heaviness in the
abdomen (earliest symptom), weight loss (mc symptom), vomiting and anorexia.IOC
is endoscopy with biopsy and brush cytology.mets most commonly to liver, followed
by lungs, bone, ovary.
262. Gastric lavage is not indicated in all cases of acute poisoning ideally because
of?
a. Danger of cardiac arrest
b. Danger of respiratory arrest
c. Fear of aspiration
d. Inadequate ventilation
Explanation :
The main danger of consuming a poison, other than reducing the effects of poison
itself on the body due to absorption, is death due to aspiration pneumonitis. It is
because of this that gastric lavage is not indicated in all cases of acute poisoning.
263. Km of an enzyme is?
a. The substrate concentration that produces half maximum velocity
b. The normal physiological substrate concentration
c. Numerically identical for all isozymes that catalyze a given reaction
d. Dissociation constant
Explanation :
The substrate concentration that produces half maximal velocity (v termed the K value
or Michaelis constant”
264. Unmet need of family planning are all except?
a. Women who are sexually active but not using any methods of
contraception
b. Women who doesn’t want to have any children in future
c. Women not using contraception but want to have children in future
d. Women who are sexually inactive
Explanation :
Unmet need for family planning is an important indicator for assessing the potential
demand for family planning services. Currently married women who are not using
any method of contraception but who do not want any more children are defined as
having an unmet need for limiting and those who are not using contraception but
want to wait two or more years before having another child are defined as having an
unmet need for spacing.
265. A infant with respiratory distress, excessive drooling of saliva and inability to
swallow. On physical examination the child abdomen was distended. Chest x- ray
reveals tracheal compression and deviation & Absence of a gastric air bubble.
What is the possible diagnosis?
a. ARDS
b. Esophageal atresia with tracheoesophageal fistula
c. Spontaneous pneumothorax
d. Achalasia
Explanation :
Tracheoesophageal fistula (TEF) is a developmental anomaly characterized by an
abnormal connection between the trachea and the esophagus that usually
accompanies esophageal atresia and in which the esophagus is closed off at some
point. Abnormal development of the septum during this time can lead to TEF.
The most common abnormality is type C TEF with esophageal atresia, in which the
upper section of the esophagus terminates in a blind pouch, and the lower section
ascends from the stomach and connects with the trachea by a short fistulous
tract. Abdominal X-rays may show the absence of air in the stomach and lower GI
tract (types A and B) or distention (types C and D). Esophagoscopy identifies
strictures or fistulas in the upper pouch; bronchoscopy confirms the presence of a
fistula. Upright positioning
Treatment - Suction catheter placement to reduce secretions
Diet - Nothing by mouth (NPO)
Gradual introduction of oral feedings postoperatively, initially via a nasoduodenal
tube and then orally after anastomoses heal.

266. True statement about frank breech is ?


a. Most commonly associated with congenital anomaly
b. Has hip extended and knee flexed
c. ECV done before 34 weeks
d. Has hip flexed and knee extended
Explanation :
There are three types of breech presentation: complete, incomplete, and frank.

Complete breech is when both of the baby's knees are bent and his feet and bottom
are closest to the birth canal.

Incomplete breech is when one of the baby's knees is bent and his foot and bottom
are closest to the birth canal.

Frank breech is when the baby's legs are folded flat up against his head and his
bottom is closest to the birth canal.

267. False statement about aminoglycoside is?


a. Used to treat gram +ve bacterial infection
b. Acts by inhibiting protein synthesis
c. kidney failure and hearing loss are the most common side effects
d. Poorly absorbed by oral intake
Explanation :
Aminoglycosides are active against most gram-negative aerobic and facultative
anaerobic bacilli but lack activity against anaerobes and most gram-positive bacteria,
except for most staphylococci. Aminoglycosides are thought to work by inhibiting
protein synthesis inside bacteria. Impairment of kidney function and hearing loss are
the most common side effects of aminoglycosides. Aminoglycosides are not well
absorbed when given by mouth, so need to be given in injection by IV. Eg.
tobramycin , gentamicin, amikacin, Streptomycin, neomycin, and kanamycin.
268. Migratory motor complex of GIT is seen in which condition?
a. Full fed state
b. Fasting
c. While eating
d. 3 hrs after food
Explanation :
Migratory motor complex are waves of electrical activity that sweep through the
intestines in a regular cycle during fasting. An increase in gastric, biliary and
pancreatic secretion is also seen in conjunction with the motor activity. The periodic
nature of the migrating motor complex is thought to be controlled from the central
nervous system and may be implemented in part by the enteric hormone motilin. The
MMC occurs every 45–180 minutes during the interdigestive phase (i.e., between
meals) and is responsible for the rumbling experienced when hungry.
269. For DNA fingerprinting blood collected to be in which of the following?
a. Glass bottles
b. Syringes containing Na F
c. EDTA vials
d. Testubes with Na Cl
Explanation :
DNA fingerprinting is a technique, by which an individual can be identified at
molecular level. Scientists have chosen repeating sequences in the DNA, which are
present in all individuals on different chromosomes, and are known to vary from
individual to individual except in identical twins. These are used as genetic markers
to identify the individual. Any biological material such as a drop of blood, saliva,
semen, and any body part such as bones, tissue, skull, teeth, hair with root etc can
be used as specimen for DNA fingerprinting. Blood samples (2-3 ml) can be
collected in the sterile blood collection material (EDTA vials)
270. False statement about fibroadenoma of breast is?
a. It increases a risk of malignancy
b. Surgery in a treatment of choice if size is > 5cm
c. Surgery done on patients request
d. Popcorn calcification is seen
Explanation :
A fibroadenoma is a noncancerous tumor in the breast that’s commonly found in
women under the age of 30. The tumor consists of breast tissue and stromal, or
connective, tissue. Fibroadenomas can occur in one or both breasts.
271. Confirmatory test for Diagnosis of gout is ?
a. Urine analysis
b. Blood test
c. Synovial fluid analysis
d. X- ray of joints
Explanation :
Diagnosis of gout :
Synovial fluid examinations involve taking fluid samples from a joint through a
needle and examining them under a microscope for urate crystals. This test can
confirm the diagnosis but isn’t always practical – it can be difficult and sometimes
uncomfortable to draw fluid from a small joint such as the big toe.
A blood test can measure the amount of urate in your blood. The critical serum level
of urate (the saturation point) is around 360 μmo/L. A raised level of urate strongly
supports a diagnosis of gout but can’t confirm it – not everyone with a raised level of
urate will develop crystals in their joints, and it’s possible for urate levels in the blood
to be normal at the time of an acute attack.
X-rays of joints will reveal joint damage if you have long-standing and poorly
controlled gout. However, x-rays are rarely helpful in confirming the diagnosis
because they’re usually normal in the early years of having gout.
272. Which of the following is not a cause of haemolytic anemia in newborn?
a. Enzyme deficiency
b. ABO/ Rh incompatablity
c. Breast feeding
d. Hemoglobinopathies
Explanation :
Hemolytic disease of the newborn (HDN) is
a blood problem in newborns. It occurs when
your baby's red blood cells break down at a
fast rate. It’s also called erythroblastosis
fetalis.
273. A 4 yr old child presents to ER with inspiratory stridor. X- ray reveals clear
lung fields and respiratory tract obstruction. Which is the most common site of
obstruction that cause inspiratory stridor is?
a. Nasopharynx
b. larynx
c. Trachea
d. Bronchus
Explanation :
Stridor is a symptom, not a diagnosis or a disease, and the underlying cause must
be determined. It may be inspiratory (most common), expiratory, or biphasic,
depending on its timing in the respiratory cycle, and the three forms each suggest
different causes, as follows:
 Inspiratory stridor suggests a laryngeal obstruction
 Expiratory stridor implies tracheobronchial obstruction
 Biphasic stridor suggests a subglottic or glottic anomaly

274. Which of the following is not the adverse effects of morphine?


a. Midryasis
b. Orthostatic hypotension
c. Bradycardia
d. Depression
Explanation:

275. Relaxation of mesangial cells of kidney is brought about by ?


a. PGF2
b. Endothelin
c. Vasopressin
d. CAMP
Explanation :
Agents causing contraction or relaxation of mesangial cells: -

Contraction Relaxation

Endothelins ANP

Angiotensin II Dopamine

Vasopressin PGE2

Nor-epinephrine cAMP Q

Platelet -activation factor

Platelet-derived growth factor

Thromboxane A2

PGF2

LT-C4 and D4

Histamine

276. In breast feeding , hind milk portion is richer in ?


a. Proteins
b. Vitamins
c. Fats
d. Lactose
Explanation :
Hind milk is rich in fat and provides more energy and satisfies the hunger of the baby.
It comes towards the end of the feed. Fore milk is secreted at the start of the feed and
is rich in protein, sugar, vitamins, mineral and water that satisfies the thirst of the baby.
Colostrums is secreted in first three days after delivery and contains more antibodies,
cells and high amounts of vitamins A, D, E and K. In transitional milk the
immunoglobulin and protein content decreases while fat and sugar content increases.
Preterm milk of mother who delivers prematurely contains more proteins, sodium iron,
immunoglobulin and calories.
277. Kiesselbach’s area has been damaged in a fist fight. The main artery to the
area is?
a. Anterior ethmoidal
b. Posterior ethmoidal
c. Spheno – palatine
d. Greater – palatine
Explanation :
o Spheno-palatine artery is a branch of maxillary artery and is the chief artery to the
nose.
o Antero-inferior part of the nasal septum has the Little’s area (Kiesselbach’s plexus),
which is an important site of arterial anastomosis between the branches of:
spheno-palatine, anterior ethmoidal and the labial branch of facial.
o This is a common site for nasal bleeding (epistaxis).

278. Increased suicidal tendency is associated with alteration in the brain levels of?
a. Dopamine
b. Serotonin
c. GABA
d. Noradrenaline
Explanation :
Decreased serotonin and its metabolite levels in brain is associated with depression
whereas elevated levels of 5 – HT are associated with anxiety.
279. Vaccine given in adolescence is ?
a. Hep - B
b. Td/Tdap
c. MMR
d. Yellow fever
Explanation :
280. Commonest post operative complication of intubation is?
a. Aspiration pneumonitis
b. Sore throat
c. Vocal cord granuloma
d. Tracheal stenosis
Explanation :
Most common post – operative complication of intubation is sorethroat which usually
subsides in 2-3 days without any specific treatment.
281. Hormone responsible for diabetogenic state in pregnancy is?
a. B- HCG
b. AFP
c. HPL
d. Maternal progesterone
Explanation :
Human placental lactogen a/k as human chorionic somatotropin that is secreted by
syncytiotrophoblast. It is mainly responsible for diabetogenic state in pregnancy, as it
antagonises the action of insulin. It leads to maternal lipolysis and increased levels of
circulating free fatty acids thus provides a source of energy for maternal metabolism
and fetal nutrition. It is a potent angiogenic hormone therefore,may play important in
fetal vasculature formation.
282. All of the following are the causes of metabolic encephalopathy except?
a. Carbon monoxide poisoning
b. Electrolytic imbalance
c. Diabetic neuropathy
d. Increased catecholamines levels
Explanation :
Metabolic encephalopathy (toxic metabolic encephalopathy) is a broad category
that describes abnormalities of the water, electrolytes, vitamins, and other chemicals
that adversely affect brain function. Other causes of metabolic encephalopathy
include carbon monoxide or cyanide poisoning, which prevents hemoglobin from
carrying oxygen in the bloodstream and results in tissue anoxia. Toxic metabolic
encephalopathy can also include medication side effects or drug ingestions affecting
the chemical transmitters in the brain. Called neurotransmitters, dopamine,
GABA, serotonin, acetylcholine, and glutamine help nerve endings pass electrical
signals between brain cells.
283. Adult Polycystic disease of kidney may have cysts in all of the following
organs except?
a. Liver
b. Lungs
c. Pancreas
d. Spleen
Explanation :

284. Most common inherited cause of mental retardation is ?


a. Down syndrome
b. Hypothyroidism
c. Fragile x- syndrome
d. Neurofibromatosis
Explanation :
Fragile x- syndrome – due to defects in FMR 1 gene that is present in long arm of X-
chromosome and is associated with CGG repeats.
On karyotyping , the x – chromosomes appears broken. It is the important inherited
cause of mental retardation. Patient presents with – long face, large mandible, large
ears , large testes.
285. Which of the following drug is used in treatment of cachexia?
a. Cyprohepatidine
b. Megestrol acetate
c. 5- flurouracil
d. Cetrorelix
Explanation :
Cachexia is a complex metabolic syndrome associated with many chronic or end-
stage diseases, especially cancer, and is characterized by loss of muscle with or
without loss of fat mass. Among the treatments proposed in the literature for cancer-
related cachexia, some proved to be ineffective, namely, cyproheptadine, hydrazine,
metoclopramide, and pentoxifylline. Among effective treatments, progestagens are
currently considered the best available treatment option for cancer-related cachexia.
286. Cerebellar degeneration is seen in ?
a. Renal cell carcinoma
b. Hodgkin lymphoma
c. Thymoma
d. Ca.stomach
Explanation :
Paraneoplastic syndromes :

Syndrome Associated tumours

Retinal degeneration Small –cell carcinoma of lung

Opsoclonus myoclonus Ovarian, lung

Neuroblastoma (in children)

Sensory neuropathy Small-cell carcinoma of lung

Hodgkin’s disease

Limbic encephalitis Small-cell carcinoma of lung, Hodgkin’s disease

Myelitis Small-cell carcinoma of lung

Cerebellar degeneration Small – cell carcinoma of lung, Ovarian, Breast

Hodgkin’s disease

Subacute motor neuronopathy Hodgkin’s disease, Small – cell carcinoma of


lung

Sensorimotor peripheral Small – cell carcinoma of lung, Breast


neuropathy
Other carcinoma
Lambert – Eaton myasthenic Small-cell carcinoma of lung
syndrome

Dermatomyositis / Polymyositis Lung, breast, ovary

Guillain – Barre Hodgkin’s disease


Flip

287. Which of the following is a bony lesion in vitamin – A intoxication?


a. Bone pain
b. Bone demineralization
c. Hyperostosis
d. All of the above
Explanation :

Vitamin A. intoxication produces following bony manifestation -


Bone demineralization,
Bone pain ,
Hypercalcemia ,
Hyperostosis

Vitamin D intoxication (delayed effect) causes persistent hypercalcemia and hyper


phosphatemia which may produce:
1). Urinary Lithiasis.
2). Metastatic calcification which affects kidneys, bronchi, pulmonary alveoli,
muscles, arteries and gastric mucosa & Renal failure leading to death.
288. Identify the given image

a. Candida tropicalis
b. Pityriasis rosea
c. Tuberous sclerosis
d. Roseola infantum
Explanation :
Acute symmetrical self limiting papulosquamatous disorder. Seen specifically on the
trunk in children and young adults common during spring and fall.1st manifestation is
1 2-6 cm annular lesion – herald patch with fine scaling, fine cigarette paper like
collarette of scales usually over the chest. Typical truncal presentation is chirstmas
tree pattern /fir tree pattern. Hnging curtain sign is seen. Associated with HHV 7. As
mentioned previously, it is self limiting and hence no treatment is required. If pruritic
, antihistamines and tropical steroids and UV – B phototheraphy.
289. Chorionic component in early stages of pregnancy is composed of?
a. Cytotrophoblast +syncytotrophoblast + somatic mesoderm + fetal
blood vessels
b. Cytotrophoblast + syncytotrophoblast
c. Syncytotrophoblast + somatic mesoderm + fetal blood vessels
d. None of the above
Explanation :
Placenta
The placenta is a fetomaternal organ. The fetal portion of the placenta is
known as the villous chorion. The maternal portion is known as the decidua
basalis. The two portions are held together by anchoring villi that are
anchored to the decidua basalis by the cytotrophoblastic shell.
Function
The placental membrane separates maternal blood from fetal blood. The fetal part
of the placenta is known as the chorion. The maternal component of the placenta is
known as the decidua basalis.

Although the placental membrane is often referred to as the placental barrier, many
substances, both helpful and harmful, can cross it to affect the developing embryo.

Structure

· Primary chorionic villi are solid outgrowths of cytotrophoblast that protrude


into the syncytiotrophoblast.

· Secondary chorionic villi have a core of loose connective tissue, which grows
into the primary villi about the third week of development.

· Tertiary chorionic villi contain embryonic blood vessels that develop from
mesenchymal cells in the loose connective tissue core. These blood vessels
connect up with vessels that develop in the chorion and connecting stalk and begin
to circulate embryonic blood about the third week of development.
290. Earliest markers for down syndrome is?
a. Unconjugated estriol
b. Ultra sound markers ( nuchal translucency)
c. AFP
d. Amniocentesis
Explanation :
ultrasound (also called sonography) is to confirm the gestational age of the fetus. It
can also pick up problems of a serious medical nature, such as blockage of the small
intestine or heart defects. Knowing these defects exist as early as possible will
benefit the treatment of the child after birth.

Studies showed that there was a strong association between the size of a collection
of fluid at the nack of the fetal neck, called nuchal transluceny, and the risk of Down
syndrome. Early attempts to use a measurement of the nuchal area were limited by
a wide variety in measurement techniques. Recently, standardized guidelines on
measuring nuchal translucency along with specific training and certifications have
been instituted, making this ultrasound measurement useful as part of the first
trimester screen.

291. Pulmonary eosinophilia is caused by ?


a. Wuchereria bancofti (microfilaria)
b. Eccinococcus granulosus
c. Ascaris lumbricoides
d. Taenia solium
Explanation :
Simple pulmonary eosinophilia (SPE), also known as Loeffler syndrome, is a rare,
temporary (transient) respiratory disorder characterized by the accumulation of
eosinophils in the lungs. They are usually produced in response to allergens,
inflammation or infection and are particularly active in the respiratory tract. Most
cases of SPE are believed to be due to an allergic reaction to drugs and
infection caused by Wuchereria bancrofti, a filarial infection.
292. which of the following is the most common complication of extracapsular
cataract surgery ?
a. Vitreous hemorrage
b. Opacification of posterior capsule
c. Retinal detachment
d. None

Explanation :

Patient presents with painless loss of vison. Post operatively, opacification of


posterior capsule occurs in 10- 50% of cases after 3-5 yrs.
293. Specific congenital abnormality associated with DM is?
a. ASD
b. VSD
c. Caudal regression syndrome
d. TOF
Explanation :
Sacral agenesis may be expected in 1% of offspring of diabetic mothers. In
16% of cases of caudal regression syndrome, the mother is diabetic; rarely, the
father is diabetic instead. The syndrome appears sporadically but is 100 times more
common in one sibling of twin births.

294. Which of the following joint disease/radiograph finding relationships is correct?


a. Osteoarthritis – erosions with overhanging margins
b. Rheumatoid arthritis – ankylosis
c. Gout – osteophytes
d. Psoriatic arthiritis – “bamboo spine”
Explanation :
Rheumatoid arthritis is associated with ankylosis of the joint space caused by the
development of pannus, which overgrows and destroys the articular cartilage
295. Sudden and irresistible force compelling a person to commit an offence
consciously is known as ?
a. Illusion
b. Obsession
c. Twilight state
d. Impulsiveness
Explanation :

Impulse: This is a sudden and irresistible force compelling a person to the conscious
performance of some action without motive or fore-thought. A sane person is
capable of controlling an impulse, but an insane person may do things on impulse.
Types:
a. KleptomaniaQ- An irresistible desire to steal articles of little value.
b. Pyromania- an irresistible desire to set things to fire.
c. Mutilomania- An irresistible desire to mutilate animals.
d. Dipsomania- an irresistible desire for alcoholic drinks at periodic intervals.
e. Sexual impulses- including sexual perversions.
f. Suicidal and homicidal impulses.
g. Twilight state is seen in visual hallucinations, Epilepsy, Head injury, Hysteria,
Punch-drunkenness.

296. Sphingomyelin and phosphatydil choline are precursors of?


a. Surfactant in lung
b. Mucopolysaccharides
c. Cholesterol
d. Ion transport proteins
Explanation :

Surfactant is dipalmitoyl phosphatidylcholine. Surfactant is secreted by type II


pneumocytes by exocytosis.

297. All of the following are produced by hepatocytes except?


a. Von Wille brand factor
b. Prothrombin
c. Factor 7
d. Fibrinogen
Explanation :

VWF →synthesized in endothelial cells


Protein synthesized by liver:-
Antithrombin III, fibrinogen, Factor II, VII, IX and X (vit.K. Dependent) protein-C (vit.
K.
dependent)

298. Most common cause of shock in surgical patient is ?


a. Anaphylactic shock
b. Compressed cardiogenic shock
c. Neurogenic shock
d. Hypovolumic shock
Explanation :

The most common cause of shock in the surgical or trauma patient is loss of
circulating volume from hemorrhage. The clinical and physiologic response to
hemorrhage has been classified according to the magnitude of volume loss.
a. Loss of up to 15% of the circulating volume (700 to 750 mL for a 70-kg patient)
may produce little in terms of obvious symptoms, while
b. Loss of up to 30% of the circulating volume (1.5 L) may result in mild tachycardia,
tachypnea, and anxiety.
c. Hypotension, marked tachycardia [i.e., pulse >110 to 120 beats per minute (bpm)],
and confusion may not be evident until more than 30% of the blood volume has been
lost;
d. Loss of 40% of circulating volume (2 L) is immediately life threatening, and
generally requires operative control of bleeding.

299. Chemical peeling is done by?


a. Trichloroacetic acid
b. Phosphoric acid
c. Carboxylic acid
d. Kojic acid
Explanation :

CHEMICAL PEEL
1). Chemical peel produces a controlled partial thickness injury to the skin.
2). Following the insult to the skin, a wound healing process ensues that can
regenerate epidermis from surrounding epithelium and adnexal structures, decrease
solar elastosis, and replace and reorient the new dermal connective tissue.
3). The result is an improved clinical appearance of the skin, with fewer rhytids and
decreased pigmentary dyschromia.
Peeling Agents
1). Glycolic acid.
2). Jessner’s solution.
3). Pyruvic acid.
4). Resorcinol.
5). Salicylic acid.
6). Trichloracetic acid.
7). Deep chemical peels.
8). Combinations peels: Salicylic acid +Trichloracetic acid.
300. WHO defines adolescent age between ?
a. 10 – 19 yrs
b. 10 – 14 yrs
c. 10 – 25 yrs
d. 9 – 14 yrs
Explanation :

Adolescence

Adolescence is defined as the period from the onset of puberty to the termination of
physical growth and attainment of final adult hight, i.e. adolescence is considered as
a period of transition from childhood to adulthood. Adolescence is usually the period
10 to 20 yr.

Puberty

Puberty is the biological process in which a child become adult, i.e. biological
process which occurs during transition from childhood to adulthood. Changes include
appearance of secondary sex characteristics and development of reproductive
capacity.

WHO/UN definitions

Adolescent 10-19 years

Early adolescent 10-13 years

Middle adolescent 14-15 years

Late adolescent 16-19 years

Youth 15-24 years


FMGE JUNE 2018
ANATOMY JUNE 2018
1. Coracoid process of the scapula is which type of epiphysis ?

a) Pressure
b) Traction
c) Aberrant
d) Atavistic

Atavistic epiphysis: A bone that is independent phylogenetically but is now fused with
another bone. These types of fused bones are called atavistic, e.g., the coracoid process of
the scapula, which has been fused in humans.

2. Pointing index finger is seen in which nerve injury

a) Axillary
b) Radial
c) Median
d) Ulnar

“Pointing finger” deformity is caused due to injury to the median nerve in the mid-forearm by
paralysis of flexor digitorum superficialis.

3. NOT a content of carpal tunnel

a) Flexor pollicis longus


b) Flexor digitorum profundus
c) Ulnar nerve
d) Median nerve

The carpal tunnel contains the median nerve, the four tendons of the flexor digitorum
profundus, the four tendons of the flexor digitorum superficialis, and the flexor
pollicis longus (FPL) tendon. Carpal tunnel syndrome (CTS) is the result of median nerve
compression within the carpal tunnel.

4. Which of the following structure passes through Guyon's canal

a) Flexor carpi radialis


b) Radial nerve
c) Ulnar nerve
d) Median nerve
The ulnar nerve and ulnar artery pass through the Guyon canal as they pass from distal
forearm to the hand.
Roof - Formed by palmar carpal ligament.
Floor - Formed by flexor retinaculum.

5. NOT a content of superior mediastinum

a) Pulmonary trunk
b) Thymus
c) Oesophagus
d) Arch of aorta

Contents of Superior Mediastinum:

 Organs: thymus, trachea, esophagus.


 Arteries: aortic arch, brachiocephalic trunk, left common carotid artery, left
subclavian artery.
 Veins and lymphatics: superior vena cava, brachiocephalic veins, the arch of the
azygos, thoracic duct.

6. Abdominal part of esophagus is supplied by

a) Bronchial artery
b) Abdominal aorta
c) Right gastric artery and inferior phrenic artery
d) Left gastric artery and inferior phrenic artery

The abdominal esophagus is supplied by the left gastric artery (a branch of the coeliac
trunk) and left inferior phrenic artery.

7. Which of the following artery is a direct branch of abdominal aorta?

a) Superior suprarenal artery


b) External iliac artery
c) Internal iliac artery
d) Testicular artery
8. Brunner's gland is a histological finding of which structure in the. gastro-intestinal
tract ?

a) Esophagus
b) Stomach
c) Duodenum
d) Liver

Brunner Gland: - Present in submucosa of proximal duodenum, secretes alkaline


secretions to neutralize acid.

9. Secondary ossification centre appears before birth at

a) Upper end of femur


b) Lower end of femur
c) Lower end of tibia
d) Lower end of fibula

Secondary ossification centers develop at later times. Ossification of the distal end of the
femur, to form the condyles and epicondyles, begins shortly before birth.
10. A 44 year old male came in to your clinic post hernia repair and wants you to
evaluate his condition. While eliciting the cremasteric reflex, you notice that the
scrotum did not ascend on the side of the surgery, suggesting an iatrogenic injury
to the:-

a) Sacral plexus
b) Femoral branch of genitofemoral nerve
c) Genital branch of genitofemoral nerve
d) Femoral nerve

11. In the cremasteric reflex, it is noticed that the scrotum did not ascend on the side
of the surgery, suggesting an iatrogenic injury to the Genital branch of
genitofemoral nerve.

The cremasteric reflex is a superficial reflex found in human males that is elicited when
the inner part of the thigh is stroked. Stroking of the skin causes the cremaster muscle to
contract and pull up the ipsilateral testicle toward the inguinal canal.

 Cremaster muscle Lies in scrotum


 Sensory supply to cremaster:
 Laterally -Femoral Branch of genitofemoral nerve
 Medially - Ilioinguinal nerve
 Motor supply to cremaster:
 Genital branch of genitofemoral nerve.

12. Dorsum of first webspace is supplied by which nerve ?

a) Superficial peroneal
b) Deep peroneal
c) Sural
d) Posterior tibial

The superficial peroneal nerve innervates the peroneus longus and peroneus brevis muscles
and the skin over the antero-lateral aspect of the leg along with the greater part of the
dorsum of the foot (with the exception of the first web space, which is innervated by the
deep peroneal nerve).

13. Nerve damaged in fracture neck fibula :

a) Superficial peroneal
b) Deep peroneal
c) Common peroneal
d) Tibial
The common fibular nerve is most commonly damaged by a fracture of the fibula, or the
use of a tight plaster cast. The anatomical course of the common fibular nerve causes it to
wrap round the neck of the fibular, and so any fractures of the fibular neck can cause nerve
palsy.

14. Identify the marked area in the image below

a) Nasion
b) Glabella
c) Pterion
d) Lambda
15. A 43-year-old male came in with complaints of headache, vomiting, blurring of
vision, and drowsiness. His son claimed that he had a fall a few hours ago. If you
suspect an increased intracranial pressure as the root cause of this, which cranial
nerve do you first check since it is commonly injured in cases of raised ICP?

a) 3rd
b) 4th
c) 5th
d) 6th

Mc nerve involved in increased ICT Abducens (VI).


Reasons :

 Longest intradural course


 Taking a sharp bend over apex of the petrous part of temporal bone.
 Passing through the cavernous sinus, not supported laterally by any dura mater.

16. Sensory innervation of cornea is by nerve

a)3rd
b)4th
c)5th
d) 6th

The cornea is one of the most sensitive tissues of the body, as it is densely innervated with
sensory nerve fibres via the ophthalmic division of the trigeminal nerve by way of 70–80
long ciliary nerves.

17. Masseter is supplied by nerve

a) Mandibular
b) Facial
c) Glossopharyngeal
d) Hypoglossal

Along with the other three muscles of mastication (temporalis, medial pterygoid, and lateral
pterygoid), the masseter is innervated by the anterior division of the mandibular division
(V3) of the trigeminal nerve.
18. Deepest nucleus of cerebellum:-

a) Dentate
b) Emboliform
c) Fastigial
d) Globose

The four deep cerebellar nuclei are in the center of the cerebellum, embedded in the white
matter. Dentate, Emboliform, Globose, Fastigial from lateral to medial location in the
cerebellar white matter.

An easy mnemonic device to remember their names and positions relative to the midline is
the phrase "Don't Eat Greasy Food", where each letter indicates the lateral to medial
location in the cerebellar white matter.

PSM JUNE 2018

1. A 25 year old young male was travelling from abroad to India. At the airport he was
diagnosed with some disease condition for which India has quarantine. Disease
usually not seen in a country but brought from abroad is called as?

a) Endemic
b) Epidemic
c) Zoonotic
d) Exotic

 Exotic: Disease usually not seen in a country but brought from abroad.
 Zoonotic: A disease which is transmitted to man from animals e.g. Rabies, plague.
 Epidemic: when disease has exceeded its normal expectancy.
 Endemic: Constant or continuous presence of a disease in a population.

2. Incidence is best measure by.

a) Cross sectional study


b) Cohort study
c) Case control study
d) Double blind study

We do fellow up, useful parameters obtained e.g. Incidence, relative risk, Attributable risk,
population attributable risk.
Case control - odds ratio
Cross sectional - prevalence
Ecological study Group characteristic

3. Primary prevention aims to prevent disease or injury before it ever occurs.


Primary prevention in myocardial infarction are all of the following EXCEPT?

a) Maintenance of normal body weight


b) Change in life style
c) Change in nutritional habits
d) Screening for hypertension

All screening & diagnostic tests comes under secondary level of prevention.
Hence, Screening for hypertension is a secondary level of prevention. It aims to reduce
the prevalence of a disease by shortening its duration.
All other options given protects against development of MI. Hence, primary level of
prevention.

4. Installation and usage of sanitary latrines by general public constitutes which


level of prevention:

a) Health promotion
b) Specific protection
c) Early diagnosis and treatment
d) Disability limitation and rehabilitation

 Include: safe water


 Installation of sanitary latrines
 Environmental: Improving in housing condition
 Modification: Insect / Rodent control

Level of prevention Modes of intervention


Primary level Health promotion & specific protection
Secondary level Early diagnosis & treatment
Tertiary level Disability limitation & rehabilitation
5. A microbiologist is preparing BCG vaccine in the laboratory. Which of the
following is diluent used with BCG vaccine?

a) Normal saline
b) Distilled water
c) Dextrose
d) Colloids

Lyophilized vaccines & their diluents:

BCG vaccine → Normal saline

Yellow fever vaccine → cold saline

Japanese Encephalitis → phosphate buffer saline

HiB → DPT / saline

Measles, MMR → Distilled /sterile water

6. A 'Case’ in TB is defined as:

a) X-ray positive
b) Sputum positive
c) Mantoux positive
d) History of cough

Case of tuberculosis:
A definite case of TB (defined below) or one in which a health worker (clinician or other
medical practitioner) has diagnosed TB and has decided to treat the patient with a full
course of TB treatment. Note. Any person given treatment for TB should be recorded as a
case.
TB is diagnosed by sputum smear
CBNAAT
ALC, LPA, LED - FM
CXR, TST
All test used in revised national tuberculosis control programme.
7. Cyclopropagative life cycle is defined as the disease agent which undergoes
cyclical change, and multiplies in the body of the arthropod. This type of life cycle
is seen in which of the following disease?

a) Malaria
b) Plague
c) Filarial
d) Yellow fever

Biological transmission of arthropod Borne disease:


Transmission Propagative Cyclo propagative Cyclo development
Definition (Only (Multiplication + (Only development)
multiplication) development)
Examples Plague Malaria Filaria
Yellow fever Guinea worm

8. Causative agent for kala-azar/ visceral leishmaniasis:

a) Leishmania donovani
b) Leishmania tropica
c) Leishmania braziliensis
d) None of the above

 Disease transmitted vector is sandfly.


 Transaminases of Leishmania donovani, the causative organism of kala-azar.
 Leishmania tropica - caused cutaneous leishmaniasis commonly known As oriental
sore or delhi boil or Baghdad boil.
 Leishmania braziliensis muco cutaneous leishmaniasis all three transmitted By sand
fly.

9. Which of the following is true for DOTS?

a) Treatment under supervision


b) Every patient given the same treatment
c) Streptomycin is given to all the patients
d) Daily regimen

Treatment under supervision→ YES it is a directly observed treatment, Short - Course


(DOTS). Pts has to consume medication in front of DOTS provider.
10. Sensitive parameter of combined obstetric and pediatric care in the country is:

a) Post neonatal mortality rate


b) Infant mortality rate
c) Neonatal mortality rate
d) Perinatal mortality rate

Perinatal mortality rate: its death occur during 28 weeks to till day post delivery

11. Number of live births per 1000 women in the reproductive age reproductive age
group in a year refers

a) Gross reproduction rate


b) Total fertility rate
c) Net reproduction rate
d) General fertility rate

General fertility rate is defined as number of live births per thousand women in the age
group (15-49 years) in a given year.

Total fertility rate → Total children born to a women or couple during there life span

Gross reproduction rate → Total number of girl children born to a women in entire
reproduction
Net reproduction → Total girl children produced during entire reproduction life span taking
into a amount mortality

12. Maximum biological value is of:

a) Pulses
b) Soyabean
c) Milk
d) Egg

Biological value tells N2 absorption V5N2 Excretion

 Egg – 94
 Milk – 84
 Soya bean – 73
13. Mode of disposal of sewage ground water:

a) Activated sludge process


b) Soakage pit
c) Oxidation pond
d) All of the above

The activated sludge process is still the most widely used biological treatment
process for reducing the concentration of organic pollutants in sewage. It's aerobic
oxidation process used as a secondary treatment. Heart of this method its Aeration tank.
Well-established design standards based on empirical data have evolved over the years.

14. Value of free residual chlorine in Drinking water should be:

a) 0.2 mg/L
b) 0.3 mg/L
c) 0.4 mg/L
d) 0.5 mg/L

It's should be minimum 0.5 mg/I (ppm) in drink water

 ≥ 1.0 mg /I swimming pool


 ≥ 2.0 mg/I kill cyclops
 Measured by chloroscope

15. Mark the correct statement regarding Ayushman Bharat.

a) Umbrella scheme includes Pradhan Mantri Jan Aarogya Yojna and subcenters.
b) ↑Benefit to 40 % of poor and vulnerable population
c) Covers only tertiary health care centres
d) Cashless ,paperless service by only government hospitals.

Ayushman Bharat

 Under Ministry of Health and Family Welfare


 Umbrella scheme
 Health and wellness centre (HWC)
 Earlier subcentres were there
 Comprehensive PHCs
 Mid level Heath provider -Nurse Ayurvedic practitioner
 Pradhan Mantri Jan Arogya Yojana (PMJAY)
 23 September 2018
 ↓ Out of pocket expense
 ↑Benefit to 40 % of poor and vulnerable population
 Cover secondary and tertiary health
 Coverage of 5 lakh/family
 No restriction of family size
 Cashless ,paperless service by government and private hospitals

16. The frequently occurring value in a data is:

a)Median
b) Mode
c) Standard deviation
d)Mean

 The mode is the value that appears most frequently in a data set.
 Median is middle value
 The mean is the average or the most common value in a collection of numbers.
 Standard deviation measure of dispersion.

17. Problem village is all EXCEPT:

a)Where no water source in a distance of 1.6 km from community


b)Water is more than depth of 15
c) There is excess of Na, K, salts
d) Risk of guinea worm infection

Guinea worm Eliminated from India Feb 2000.

18. A 25 year old female is using vaginal sponge as a barrier contraceptive during
sexual intercourse. Which of the following Spermicide is used in contraceptive
TODAY?

a)NET-EN
b)DMPA
c)Nonoxynol - 9
d)Nor-ethinosterol

 "TODAY" is vaginal sponge is barrier type of contraceptive.


 NON OYYNOL - 9 → kill the sperm by rupture of plasma membrane of Acrosomal cap of
sperm.
 Failure rate of today → 9-20 /Healthy Women Years.
 TODAY also associated with toxic shock syndrome .

19. Bagassosis occurs with:

a)Silica fibers.
b)Carbon particles
c)Sugarcane fibers
d)Cotton

One of these lesser-known illnesses is Bagassosis. Defined as the inflammation of the


lung caused by the inhalation of dust from bagasse (sugar cane dust), the disease was
initially discovered more than 50 years ago in India.

20. Cytotoxic drug is disposed in:

a) Yellow bag
b) Red bag
c) Blue bag
d) Black bag

 Yellow bag
 Human anatomical waste
 Animal waste
 Soiled waste ( cotton /cloths)
 Chemical waste
 Discarded medicine ( expired medicine, cytotoxic drug)
 Liquid chemical waste
 Micro biological, lab waste
 Red category → Solid contaminated (plastic / rubber)
 White category → wasted sharp (needle, Sx blades, and scalpels)
 Blue category → glasses, metallic body implants

21. A 24 year old reproductive age female visited to the rural PHC for IUCD insertion
to prevent pregnancy. Which of the following is the most common side effect of
IUCD insertion?

a) Ectopic pregnancy
b) Pelvic infection
c) Pain
d) Bleeding

 Mc side effect bleeding (give to female iron tablet for 2 months).


 Pain is 2nd mc. but its mc cause of removal of IUCD.
22. The Factories Act, 1948 is a social legislation which has been enacted for
occupational safety, health and welfare of workers at work places. What is the
maximum number of work hours/ week, prescribed under the Factories act?

a) 42 hours/week
b) 48 hours/week
c) 54 hours/week
d) 60 hours/week

 The Factories Act. 1948 is a social legislation which has been enacted for occupational
safety, health and welfare of workers at work places.
 Its 9 hours a day & maximum 48 hours. week.
 60 hours , week then paid extra.

23. A 36 year old male presented to the OPD with difficulty in breathing while walking,
back pain, unable perform physical activity and excessive laziness. On
examination his body weight is 106 kgs. Which of the following is height
independent obesity index?

a) Corpulence index
b) Quetlet index
c) Brocca index
d) Ponderal index

Corpulence Index = Actual weight / Desirable weight


So, Corpulence Index is “Height independent index” of obesity; cutoff ≤ 1.2.

24. Cafeteria approach' is related with:

a) Diet program
b) Child and maternal health
c) National vector borne disease control programme
d) Contraception

Family planning & contraception - "Cafeteria approach" has new name "GATHER"
G: Grade the couples
A: Ask them their contraception requirement
T: Tell them about available methods
H: Help them choose best method for them
E: Explain
R: Returns

25. A 42 year old obese female presented to the rural PHC. Which of the following is
the appropriate statistical test to find out obesity as a significant risk factor for
breast cancer?

a) Student's paired ‘t’ test


b) Student's unpaired 't' test
c) Chi-square test
d) Wilcoxon's signed rank test.

 Obesity is qualitative variable : breast cancer also qualitative.


 So, appropriate statistical test to find out obesity as significant risk factor for breast
cancer is Chi-square test.

26. An important measure of communicability of a disease is:

a) Incidence rate
b) Case fatality rate
c) Prevalence rate
d) Secondary attack rate

SAR =( Number of secondary case in 1 incubation period / Total Susceptible ) x 100


Measles SAR – 80 %
Chicken pox – 90%
Petrussis – 90 %
Mumps – 86%

27. Which one of the following is FALSE regarding confounding factor in


epidemiological studies?

a) Associated both with exposure and disease


b) Distributed equally between study and control groups
c) Independent risk factor for disease in question
d) Source of bias is interpretation

Confounding factor Any factor associated with both exposure, outcome and has an
independent effect in causation of outcome is confounder.
BEST METHOD TO REMOVE CONFOUNDING:-

 MATCHING [MC used/ simplest method]- Removes confounding bias by distributing


known confounding factors equally in two groups
 RANDOMISATION (2ND BEST)
 RESTRICTION
 STRATIFICATION
 STATISTICAL MODELING / MULTIVARIATE ANALYSIS
 STRATIFIED RANDOMIZATION (BEST METHOD).

28. Denominator in calculation of case fatality rate is:

a)Total number of deaths due to all causes


b)Total number of hospital admissions
c)Total number of cases due to the disease concerned
d)Total number of deaths due to the disease concerned

Case Fatality :

 Refers to proportion of fatal cases among those who have the disease.
 Provides an index of the deadliness of a particular disease within a specific population.
CFR = ( Number of deaths due to disease / Number of cases of disease ) x 100

29. An epidemic is the rapid spread of disease to a large number of people in a given
population within a short period of time. A well of contaminated water resulting in
an epidemic of acute watery diarrhoea is a typical example for which of the
following?

a) Common source, single exposure epidemic


b) Common source, continuous exposure epidemic
c) Slow epidemic
d) Propagated epidemic

 A well of contaminated water resulting in an epidemic of acute watery diarrhea is an


example of Common source, continuous exposure epidemic as there is
continuous and repeated exposure.
 Multiple peaks known as SECONDARY WAVES are seen.
ORTHO JUNE 2018
1. A 65 years old male brought to casualty with history of fall. The patient is in severe
pain, there is tenderness on the greater trochanter and also the patient is not able
to walk. The lateral border of the foot touches the bed. What is the most common
complication in the given clinical scenario?

a)Non union
b)Ischemic necrosis
c)Malunion
d)Pulmonary complications

The given clinical scenario depicts the Fracture neck of femur (intertrochanteric)

Fractured neck of femur presents with the following clinical features:

 Painful movements of the hip joint


 Shortening and external rotation of the affected limb
 Extra-capsular (inter-trochanteric) fracture is more common than intra-capsular.
 Most common complication of Extra-capsular fracture of neck of femur of neck is
mal-union.
 In inter-trochanteric fractures, there is usually gross deformity and pronounced lateral
rotation.
 In Intra-capsular (neck of femur fracture)
 Most common complication is avascular necrosis > > non-union.

2. 8 years old boy sustained fracture to the hand while playing with his sister. In
which of the following condition, the Triangular relation of the elbow is
maintained?

a) Fracture ulna
b) Anterior dislocation of elbow
c) Posterior dislocation of elbow
d) Supracondylar fracture of humerus

 Triangular relation of elbow includes medial epicondyle, lateral epicondyle and tip of
olecranon.
 The relationship is destroyed by dislocations of elbow
 It is maintained in supracondylar fracture of Humerus (occurs above elbow joint)
3. Which of the following statement about tubercular osteomyelitis is not true?

a)It is a type of secondary osteomyelitis


b)Sequestrum is uncommon
c)Periosteal reaction is characteristic
d)Inflammation is minimal

 Tuberculous osteomyelitis is a type of secondary osteomyelitis.


 Sequestrum is uncommon in tubercular osteomyelitis.
 Periosteal reaction is common in Osteomyelitis, but there is little or no
periosteal reaction in tubercular osteomyelitis.
 In tubercular osteomyelitis, Inflammation is rare.

4. Most common site of osteomyelitis in children

a) Diaphysis
b) Metaphysis
c) Epiphysis
d) Growth plate

 Overall Most common location of osteomyelitis is metaphysis. It is due to high


vascularity, hair-pin loop system of blood supply. more prone to trauma and
monocyte macrophage system is weaker.
 Most common part of osteomyelitis is lower end of Femur.
 Most common organism of osteomyelitis is staphylococcus aureus.

5. Primary osteoarthritis affect all except:

a) Hip Joint
b) Knee joint
c) Distal interphalangeal joint
d) Metacarpophalangeal joint

 Osteoarthritis involves DIP and PIP with sparing of MCP.


 Rheumatoid arthritis involves PIP and MCP with sparing of DIP.
 Osteoarthritis commonly involves knee joint in India.
 Hip joint is rarely involved in osteoarthritis.
MICRO JUNE 2018
1. A liquid media used to suppress commensal bacteria while allowing the pathogen
to remain viable and grow. Which of the following is an enrichment media?

a) Alkaline peptone water


b) Monsour's taurocholate Tellurite peptone water
c) Selenite F broth
d) All of the above

Enrichment media -

 LIQUID culture media


 These media are used to suppress commensal bacteria while allowing the pathogen to
remain viable and grow.
 Example –
 Alkaline peptone water - V. cholerae
 Selenite F-broth - Salmonella, shigella
 Monsour's taurocholate tellurite peptone water - V. cholerae.

2. Biological indicator for determining efficacy of autoclaving is

a) Pseudomonas aeruginosa
b) Clostridium perfringenes
c) Bacillus stearothermophilus
d) Salmonella typhi

There are three indicators that may be used to detect the efficacy of the
autoclave process:
(1) Physical: pressure and temperature recording devices,
(2) Chemical: indicators that change color after being exposed to specific
temperatures, such as temperature sensitive tape. The color change upon exposure
to the given temperature, and
(3) Biological: Bacillus stearothermophilus spores are used, due to its
resistance to heat, for the testing that measures the biological performance of
the autoclave process.

3. Yaws is caused by:

a) T.Pertunae
b) T. Pallidum
c) T. Caraetum
d) Hemophilus
Yaws is an infectious disease caused by a spiral-shaped bacterium (spirochete) known
as Treponema pertenue. Yaws is usually transmitted by direct contact with the infected skin
sores of affected individuals.

4. Babesiosis is most commonly transmitted by:

a) Pigs
b) Rats
c) Sand fly
d) Ticks

The deer tick (Ixodes dammini or scapularis) is the most common vector that transmits
babesiosis. In extremely rare cases, babesiosis may be transmitted following a blood
transfusion with blood that is contaminated with the microorganism.

5. Which bacteria shows Quellung reaction?

a) Pneumococcus
b) Gonococcus
c) Staphylococcus
d) All of the above

The quellung reaction, also called the Neufeld reaction, is a biochemical reaction in which
antibodies bind to the bacterial capsule of Streptococcus pneumoniae, Klebsiella
pneumoniae, Neisseria meningitidis, Bacillus anthracis, Haemophilus influenzae,
Escherichia coli, and Salmonella.

6. Molluscum contagiosum is caused by

a) Bacteria
b) Fungus
c) Protozoa
d) Virus

Molluscum contagiosum is an infection caused by a poxvirus (molluscum contagiosum


virus). The result of the infection is usually a benign, mild skin disease characterized by
lesions (growths) that may appear anywhere on the body.
7. SSPE is a complication of

a) Measles
b) Mumps
c) Rubella
d) Rabies

Subacute sclerosing pan encephalitis (SSPE) is a very rare, but fatal disease of the central
nervous system that results from a measles virus infection acquired earlier in life. SSPE
generally develops 7 to 10 years after a person has measles, even though the person seems
to have fully recovered from the illness.

8. Which subtype of human papilloma virus has maximum chances of causing


carcinoma cervix:

a) HPV 16&18
b) HPV 6&11
c) HPV 31&32
d) HPV 1&2

 Maximum chance of developing the carcinoma cervix ( 16 & 18 ) HPV


 HPV 6 & 11 cause genital warts ( condyloma acuminata )
 HPV 1 & 2 cause common warts
 Rx - Podophyllin resin + Cryosurgery

9. Stain used in staining fungal elements

a) Acid fast stain


b) Gram stain
c) Methenamine silver
d) All of the above

Fungal stains

 10% KOH
 LPCB (lactophenol cotton blue)
 GMS (Gomori Methenamine Silver)
 Calcofluor white (Fluorescent stain)
 Mucicarmine stain (used for capsule of cryptococcus neoformans )
 Indian ink staining ( used for capsule of cryptococcus neoformans )
 H&E (hematoxylin/eosin stain)
 PAS -Periodic acid Schiff.

DERMA JUNE 2018

1. Spot diagnosis based on the clinical image:

a) Secondary syphilis
b) Lupus vulgaris
c) Acanthosis nigricans
d) Scrofuloderma

Acanthosis nigricans is a skin condition that causes a dark discoloration in body folds
and creases. It typically affects the armpits, groin and neck. Acanthosis nigricans is a skin
condition characterized by areas of dark, velvety discoloration in body folds and creases.
The affected skin can become thickened
In the given image:

 Neck folds are involved


 Hyperpigmented lesions
 Thickened skin
 Velvety appearance.

2. Drug of choice for syphilis:

a) Ceftriaxone
b) Erythromycin
c) Tetracycline
d) Benzathine penicillin

 DOC for syphilis is Benzathine penicillin except for Neurosyphilis - Procaine


Penicillin (Aqueous / Crystalline Penicillin)
3. Dermatophytes can affect:

a) Hair
b) Nail
c) Scalp
d) All of the above

A dermatophyte infection, also known as dermatophytosis or tinea, refers to a group of


fungal infections that can affect the skin, hair, and nails. Tinea infections are one of the
most common causes of superficial fungal infections around the world, and are distinguished
by the area of the body affected.

4. Wavelength of light produced by wood's lamp:

a) 320nm
b) 360nm
c) 400nm
d) 480nm

A traditional Wood lamp is a low-output mercury arc covered by a Wood filter (barium silicate
and 9% nickel oxide) and emits wavelength 320–450 nm (peak 365 nm). The lamp was
invented in 1903 by a Baltimore physicist, Robert W. Wood.

5. Acne vulgaris involves:

a) Sweat glands
b) Pilosebaceous glands
c) Sebaceous glands
d) Apocrine glands

Acne vulgaris is a common chronic skin disease involving blockage and/or inflammation
of pilosebaceous units (hair follicles and their accompanying sebaceous gland). Acne
can present as noninflammatory lesions, inflammatory lesions, or a mixture of both, affecting
mostly the face but also the back and chest.
OPTHAL JUNE 2018
1. Cornea derives its nutrition chiefly from:

a) Aqueous humour
b) Mucous layer
c) Corneal vessels
d) Perilimbal vessels

Unlike most tissues in the body, the cornea contains no blood vessels to nourish or protect it
against infection. Instead, the cornea receives its nourishment from the tears and aqueous
humor (a fluid in the anterior portion of the eye) that fills the chamber behind it.

2. Maximum cones are seen in:

a) Limbus
b) Fovea centralis
c) Macula lutea
d) Blind spot

The fovea centralis is located in the center of the macula lutea, a small, flat spot located
exactly in the center of the posterior portion of the retina. As the fovea is responsible for
high-acuity vision it is densely saturated with cone photoreceptors.

3. A 27-year-old, 38 weeks pregnant woman with height of 130 cm and a weight of 51


kg diagnosed case of cephalopelvic disproportion with fetal distress was posted
for emergency cesarean section. A lower segment cesarean section was
performed and a live male infant 1.8 kg with an APGAR of 8/9 was delivered. The
eye of this newborn would have ?

a) Hypermetropic with regular astigmatism


b) Hypermetropia
c) Hypermetropic with irregular astigmatism
d) Myopia

 At birth, the eye is hypermetropic by +2 to +3D and usually becomes emmetropic by


the age of 5 to years
 This is because of the smaller eyeball.
 Eyeball length at birth is 16mm.
4. Campimetry is used to measure ?

a) Field of Vision
b) Pattern of retina
c) Malignant melanoma
d) Squint

Perimetry or campimetry is one way to systematically test the visual field. It is


the systematic measurement of differential light sensitivity in the visual field by the
detection of the presence of test targets on a defined background.

5. Drug of choice in eye in trachoma endemic areas?

a) Erythromycin
b) Ganciclovir
c) Clindamycin
d) Azithromycin

Azithromycin is a macrolide antibiotic and is the drug of choice for trachoma. Plasma
concentrations are low, but tissue concentrations are higher, giving it value in treating
intracellular organisms. It has a long tissue half-life. A single dose is recommended.

6. Herbert's pits are seen in:

a) Vernal conjunctivitis
b) Atopic conjunctivitis
c) Gonococcal conjunctivitis
d) Chlamydial conjunctivitis

Herbert's pits are a characteristic and pathognomonic finding of trachoma, and are caused
by Chlamydia trachomatis.

7. As per WHO classification of Xerophthalmia, X2 is:

a) Night blindness
b) Conjunctival xerosis
c) Bitot's spots
d) Corneal xerosis
World Health Organization Re- Classification of Xerophthalmia Signs

Classification Ocular Signs


XN Night blindness
X1A Conjunctival xerosis
X1B Bitot's spots
X2 Corneal xerosis
X3A Corneal ulceration Keratomalacia involving one- third or less of the cornea
X3B Corneal ulceration -Keratomalacia involving one-half or more of the cornea
XS Corneal scar
XF Xerophthalmic fundus

8. Which of the following is true about given condition ?

a) Painless
b) Affects cornea
c) Sudden loss of vision
d) Bilateral in majority of cases

The given condition is Mooren’s ulcer.

Mooren's ulcer:

 Affects cornea
 Painful condition
 Slow loss of vision
 Mostly unilateral
 Peripheral corneal ulceration
 Clear cut overhanging edge seems to be melting from periphery
9. Argon laser trabeculoplasty is done in:

a) Angle closure glaucoma


b) Angle recession glaucoma
c) Secondary glaucoma
d) Open angle glaucoma

The most common type of laser surgery performed for open-angle glaucoma is called
Argon Laser Trabeculoplasty (ALT). The objective of the surgery is to help fluids drain out of
the eye, reducing intra-ocular pressure that can cause damage to the optic nerve and loss of
vision.

10. Pupil in acute iritis is:

a) Normal
b) Dilated
c) Constricted
d) Vertically oval

 In acute iritis (inflammation of iris) iris is irritated and pupil constricts due to
stimulus of irritation.
 Vertically oval, mid-dilated, non-reacting pupil is seen in acute angle closure glaucoma.

11. Cause of sudden loss of vision in a diabetic is due to:

a) Neovascular glaucoma
b) Central retinal artery occlusion
c) Vitreous hemorrhage
d) None of the above

 Commonest cause of sudden loss of vision in diabetic is vitreous hemorrhage


 Slow loss of vision in diabetic is due to -
 Cystoid macular edema
 Neovascular glaucoma

12. Optic disc changes of retinitis pigmentosa:

a) Consecutive optic atrophy


b) Blurring of disc margins
c) Hyperemia of disc
d) No significant change
Retinitis pigmentosa (RP) is a group of hereditary disorders of the photoreceptors and retinal
pigment epithelium (RPE) which gradually causes night blindness and progressive
constriction of the visual field. Waxy optic disc pallor, arteriolar narrowing and hyalinization
are found in almost all cases. The optic disc in RP show optic atrophy, classically
described as a 'waxy pallor' of the disc, and is a consequence of photoreceptor
degeneration.

PHYSIO JUNE 2018


1. Surfactant is secreted by which of the following:

a) Type I pneumocyte
b) Type II pneumocyte
c) Sertoli cell
d) Leydig cell

Type I pneumocyte: Involved in gas exchange makes up 80-90% of alveolar cells.


Type II pneumocyte: Secrete surfactant makes up 5-10 % area alveolar cells.
Sertoli cell: Present in seminiferous tubules, responsible for nourishment spermatozoa
during spermatogenesis, has FSH receptors & secrete inhibin.
Leydig cell: Produces testosterone in response to LH.

2. Which of the following receptors mediate stretch reflex:

a) Golgi tendon organ


b) Merkel's disc
c) Muscle spindle
d) Meissner's corpuscles

The classic stretch reflex involves the stretch receptor in the skeletal muscle spindle and
its afferent Ia fiber in the posterior nerve root, which directly or indirectly excites the alpha
motor neuron in the spinal cord.
3. Resting membrane potential of nerve fibre is close to isoelectric potential of:

a) Potassium ions
b) Sodium ions
c) Chlorideions
d) Magnesium ions

4. Which of the following hormones regulates blood levels of 1,25-OH-cholecalciferol


positively:

a) Thyroxine
b) Parathormone
c) Insulin
d) Calcitonin

Serum levels of 1,25-dihyroxyvitamin D have little or no relationship to vitamin D


stores but rather are regulated primarily by parathyroid hormone levels, which in turn
are regulated by calcium and/or vitamin D. In vitamin D deficiency, 1,25-dihydroxyvitamin D
levels go up, not down.

Parathyroid hormone (PTH), also called parathormone or parathyrin, is a hormone


secreted by the parathyroid glands that regulates the serum calcium concentration through
its effects on bone, kidney, and intestine.

5. Enteropeptidase enzyme is secreted by:

a) Stomach
b) Duodenum
c) Jejunum
d) Ileum

Enterokinase, also called Enteropeptidase, proteolytic enzyme (q.v.), secreted from the
duodenal mucosa, that changes the inactive pancreatic secretion trypsinogen into trypsin,
one of the enzymes that digest proteins.
6. Which of the following is the best stimulus for the release of vasopressin?

a) Hypotonicity of extracellular fluid


b) Hypertension
c) Hypotension
d) Hypo-osmolality of extracellular fluid

The main physiological stimulus to vasopressin secretion is rising plasma osmolality,


though significant reductions in arterial blood pressure and blood volume can also
stimulate vasopressin secretion, by unloading of arterial baroreceptors.

7. Electrical impulses originates in:

a) SA node
b) AV node
c) Endocardium
d) Epicardium

The electrical signal starts in a group of cells at the top of your heart called the
sinoatrial (SA) node. The signal then travels down through your heart, triggering first your
two atria and then your two ventricles.

8. Inhibin is secreted by:

a) Leydig cells
b) Sertoli cells
c) Interstitial cells
d) Peg cells

Inhibin is a protein secreted by the Sertoli cells in men and by the granulosa cells in
women. It inhibits the synthesis and release of the follicle-stimulating hormone in the
pituitary gland and reduces the hypothalamic LH - releasing hormone content.

9. Centre controlling vomiting :

a) Apneustic centre
b) Pneumotaxic centre
c) Area Postrema
d) Hypothalamus
Area postrema is the center responsible for controlling vomiting it is located outside the
blood brain barrier. and part of the chemoreceptor trigger zone, hence chemicals in blood
can directly stimulate this area and induce vomiting.

10. Renin is secreted by:

a) Tubular cells
b) Macula densa
c) JG cells
d) All of the above

Renin is an enzyme secreted by the juxtaglomerular cells of the kidney. It interacts with
aldosterone in a negative-feedback loop.

11. The rate at which the stomach empties into the duodenum depends on the type of
food ingested. Food rich in carbohydrates leaves the stomach in a few hours.
Protein-rich food leaves more slowly, and emptying is slowest after a meal
containing fat. The rate of emptying also depends on the osmotic pressure of the
material entering the duodenum. Hyperosmolality of the duodenal contents is
sensed by "duodenal osmoreceptors" that initiate a decrease in gastric emptying,
which is probably neural in origin. Which factor is responsible for increasing
gastric motility?

a) Presence D1 fatty food in the stomach


b) Distension of stomach
c) Presence of tryptophan in stomach
d) Presence of acid in stomach

Promoters of gastric emptying :

 Chemical → Gastrin
 Mechanical → Gastric Distension.

12. The adrenal cortex is derived from mesoderm and begins posteromedial to the
urogenital ridge. The adrenal medulla is derived from neural crest cells. These
neural crest cells originate from the dorsal aorta. Clusters of chromaffin cells
become the distinct medulla after birth. The adrenal gland first appears at 28-30
days post-conception. Adrenal medulla secretes?

a) Sex hormones
b) Thyroid stimulating hormones
c) Epinephrine
d) Glucocorticoid
The adrenal medulla, the inner part of an adrenal gland, controls hormones that initiate the
flight or fight response. The main hormones secreted by the adrenal medulla include
epinephrine (adrenaline) and norepinephrine (noradrenaline), which have similar
functions.

FORENSIC JUNE 2018


1. Rupture of the skin and subcutaneous tissue is seen in :-

a) Incised wound
b) Abrasions
c) Lacerated wound
d) Contusion

A laceration is the result of a shearing force and causes deeper skin tearing, through the
epidermis and sometimes through the dermis and subcutaneous tissues.

2. In case of death in lock-up, the inquest is held by:-

a) A police officer
b) Magistrate
c) Panchayat officer
d) District attorney

Magistrate inquest (176 CrPC) is Conducted in these cases :


A. Asylum death
B. Borstal (juvenile home)
C. Custodial death
D. Dowry death
E. Exhumation / Encounter death

3. The time limit for ordering an exhumation in India is:-

a) 1 year
b) 10 years
c) 20 years
d) No limit
In India: No time limit - for exhumation [176 (3) CRPC]
Exhumation :

 It is the digging out of the body out of earth


 Authorization given only by magistrate
 Done in presence of -
 Magistrate
 Police
 Doctor
 Preferred to start in early morning
 No time limit for exhumation
 500 gms of soil sample taken.

4. First Post Morterm change in a dead body:-

a) Maggot formation
b) Putrefaction
c) Greenish discoloration of right iliac fossa
d) Mummification

 First postmortem change in dead body is greenish discoloration of right iliac


fossa
 It's because of presence of caecum
 Its first external site of putrefaction (Right Iliac Fossa)
 1st internal site → Intima of aortic lumen (Reddish brown discoloration)
 Most imp. bacteria involved in putrefaction → Clostridium welchi

5. Superimposition technique is used for:-

a) Skull
b) Pelvis
c) Femur
d) Ribs

Craniofacial superimposition is a technique used in the field of forensic anthropology to


assist in the analysis of an unknown skull. The process involves superimposing an image
of the recovered skull over an ante mortem image of the suspected individual.
6. Pure Aryans have which type of skull:-

a) Brachycephalic
b) Mesaticephalic
c) Dolicocephalic
d) All of the above
 Dolichocephalic → Aryans, Negroids Cephalic index (70. 75)
 Mesaticephalic → Europeans, Chinese -Cephalic index (75 - 80)
 Brachy cephalic → Mongoloids Cephalic index (80 - 85)

7. Gettler's test is positive in:-

a) Hanging
b) Poisoning
c) Strangulation
d) Drowning

A test that traditionally was used is the Gettler Chloride Test, where the amounts of
chloride on each side of the heart were compared to determine if the person drowned
in fresh or salt water.
Gettler test not useful in-

 Dry drowning
 Hydrocution
 Patent foramen oval
 Putrefaction.

8. Rain drop pigmentation Is seen in:-

a) Arsenic poisoning
b) Phosphorous poisoning
c) Mercury poisoning
d) All of the above

Arsenic poisoning :
A - Anemia / Aldrich mee's line / Arsenophagist (Tolerate up to 300mg)
R - Rain drop pigmentation / Reinsch test / Red velvety mucosa
S - Sub endocardial hemorrhages / Sensory neuropathy
E - Eruptions
N - NAA (neutron activation analysis)
I - Imbibition of arsenic (Arsenic imbibed from surrounding soils after death)
C - Cumulative poison / Cholera like symptom / Chelation for Rx
9. A 40-year-old man has committed suicide by consuming some chemical he was
able to procure from the paint-making factory he works at. The doctor notices the
garlicky odor from the tissues and decided to preserve the hair for further
investigation. Poisoning with which one of the following requires preservation
of the hair?

a) Arsenic
b) Manganese
c) Phosphorous
d) Alcohol

Poison Organ preserved


Nux vomica Spinal cord
Aconite Heart
Alcohol / any cerebral poison Brain
Metal (chronic) - arsenic, antimony, thallium Bone, hair, nail
Volatile poisoning gas) Lungs

PATHO JUNE 2018

1. Heart failure cells are:-

a) Lipofuscin granules in cardiac cells


b) Pigmented hepatocytes
c) Pigmented alveolar marcophages
d) Pigmented cells in pancreas
A siderophage is a hemosiderin-containing macrophage. Heart failure cells are
siderophages generated in the alveoli of the lungs of people with left heart failure or
chronic pulmonary edema, when the high pulmonary blood pressure causes red blood cells
to pass through the vascular wall.

2. Virchows triad includes all except:-

a) Endothelial injury
b) Stasis of blood flow
c) Hypercoagulability
d) Platelet thrombus

3. Mark correct statements regarding surfactant

a) Surfactant production starts at 16-18 wks of fetal life


b) Maximum secretion occurs at 34-35 wks
c) Increase surfactant production by insulin.
d) 30% Dipalmitoyl phosphatidyl choline in composition

 Surfactant is the fluid secreted by type 2 pneumocyte.


 Physiological function of surfactant is [ ↓ surface tension].
 Composition of surfactant - 70% phospholipids [DPPC]-Dipalmitoyl phosphatidyl
choline AKA "lecithin" 30% fibrin, mucopolysaccharides etc.
 Surfactant covers water surface and prevent air - fluid interaction so, [ ↓ surface
tension]
KEY POINTS:

 Surfactant production starts at 18-20 weeks of fetal life


 Secretion and action into alveoli occurs at 28 wks.
 Maximum secretion occurs at 34-35 weeks
 Increase surfactant production by thyroid, sex steroids, glucocorticoids [most imp.
For lung maturation]
 Decrease surfactant production by insulin.

4. PTT is prolonged in which of the following


A. Hemophilia A
B. Christmas disease
C. Von Willebrand disease
D. ITP

a) A, B, C
b) A, B, D
c) A, C, D
d) B, C, D

aPTT is used to monitor the activity of Intrinsic pathway.

Hemophilia :

 Most common type of hemophilia is Hemophilia A


 Hemophilia A - Factor VIII deficiency - ↑aPTT

Von Willebrand disease :

 Associated with reduction in the half-life of Factor VIII - ↑ aPTT

Christmas disease :

 Factor IX deficiency - ↑ aPTT

ITP - Idiopathic Thrombocytopenic Purpura

 Disease associated with ↓ platelets, not a clotting factor problem.


 Parameters affected in ITP -
 Increase bleeding time*
 Decrease platelet count

5. AFP (alpha feta protein) is raised in:-

a) Renal carcinoma
b) Pancreatic carcinoma
c) Prostatic carcinoma
d) Hepatic carcinoma
Serum alpha fetoprotein (AFP) is the most widely used tumor marker in detecting patients
with hepatocellular carcinoma. When elevated, the AFP is 75-91% specific, and values
greater than 400 ng/mL are generally considered diagnostic of HCC in the proper clinical
context, including appropriate radiologic findings.

6. Negri bodies are characteristic of:-

a) Tetanus
b) Rabies
c) Polio
d) AIDS

Negri bodies are seen in the Pyramidal neurons within Ammon's horn of the Hippocampus
and Purkinje cells in the cerebellar cortex on postmortem of the rabid person.

7. Earliest feature of TB:-

a) Granuloma
b) Langerhan's Giant cells
c) Lymphocytosis
d) Caseation

Ingestion of mycobacterium tuberculosis inside the lung will cause stimulation in the no. of
lymphocytes, that will cause lymphocytosis [earliest feature of TB]
First cell in the lung which will be affected by myco. TB infection is alveolar macrophage
Transmission of myco. TB - cough & sneeze (droplet infection)

8. Which of the following is not a component of Jones criteria for diagnosing acute
rheumatic fever?

a) Pancarditis
b) Migratory polyrthritis
c) Erythema nodosum
d) Sydenham chorea

Jones criteria :
Major criteria

 Migratory polyarthritis
 Carditis
 Erythema marginatum
 Syndenham chorea
 Subcutaneous nodules

Minor criteria

 Arthralgia
 Fever
 First degree heart block
 Elevated inflammatory markers (ESR, CRP).

9. MC site for amoebiasis:-

a) Liver
b) Caecum
c) Transverse colon
d) Sigmoid colon

Amebiasis may involve any part of the bowel, but the cecum and the ascending colon are
predilection sites. The most common extra intestinal manifestation is liver abscess
caused by hematogenous spread from the GI tract.

10. Crescentic glomerulonephritis is:-

a) FSGS
b) MGN
c) RPGN
d) All of the above

Crescentric glomerulonephritis, also known as rapidly progressive glomerulonephritis,


is a condition of the kidney characterized by a rapid loss of renal function. On
histological examination, it has the presence of numerous glomerular crescents (usually
greater than 50%).

11. HLA is located on :

a) Short arm of chromosome -6


b) Long arm of chromosome -6
c) Short arm of chromosome -3
d) Long arm of chromosome -3
The human major histocompatibility complex HLA is located on the short arm of
chromosome 6. It is known to be the most polymorphic genetic system in humans. The
biological role of the HLA class I and class II molecules is to present processed peptide
antigens.

12. Most common tumor of anterior mediastinum:-

a) Neurogenic cyst
b) Thymoma
c) Neurofibroma
d) Sarcoma

 MC tumor of anterior mediastinum is THYMOMA


 Thymoma has a definitive association with a particular type of muscle weakness
disorder- Myasthenia gravis.
 Patients suffering from thymoma also going to have a relationship between this tumor
and hematological condition which is given the name PRCA [pure red cell aplasia]

13. A 9 Yr old male child came with complaints of an Upper respiratory tract infection.
On General examination, he was found to be anemic. The Pediatrician explained to
his parents about the causes of Anemia in children. Which of the following is the
most sensitive and specific initial laboratory test tc diagnose iron deficiency?

a) Serum iron levels


b) Serum ferritin levels
c) Serum transferrin receptor population
d) Transferrin saturation

In iron deficiency anemia, the earliest parameter that is effected is → decrease in stored iron
[ferritin] → measured by serum ferritin levels.

Hence, estimation of Serum ferritin levels is most sensitive and specific test to diagnose iron
deficiency anemia in the given options.

14. In beta thalassemia trait, electrophoresis shows:-

a) Increased HbF & normal HbA2


b) Normal HbF & normal HbA2
c) Increased HbF & increased HbA2
d) Decreased HbF & normal HbA2
If thalassemia is suspected, a hemoglobin electrophoresis will help diagnose the condition.
The hemoglobin electrophoresis with beta thalassemia trait usually has reduced or absent
HbA, elevated levels of HbA2, and increased HbF.

15. MC cause of nephrotic syndrome in adults:-

a) Good-pasture syndrome
b) Membranous GN
c) Minimal change disease
d) Focal segmental glomerulosclerosis

Primary causes of nephrotic syndrome are diseases that affect only the kidneys. The most
common primary cause of nephrotic syndrome in adults is a disease called focal segmental
glomerulosclerosis (FSGS).

16. Preferred biochemical markers in patients presenting with myocardial infarction:-

a) Myoglobin
b) Cardiac specific Troponins
c) CK-MB
d) All of the above

Cardiac troponins T and I are the preferred markers for myocardial injury as they have the
highest sensitivities and specificities for the diagnosis of acute myocardial infarction.

17. A known case of peptic ulcer disease presented with repeated episodes of
vomiting’s. The condition is associated with hypochloremic hypokalemic
metabolic acidotic state. What is the most likely part involved in the given
condition?

a) Antrum
b) Duodenum
c) Pylorus
d) Fundus

 Peptic ulcer disease , Carcinoma of the stomach → can result in Gastric outlet
obstruction
 Peptic ulcer disease, MC site of obstruction - Duodenum [1st part]
 Carcinoma of stomach, MC site of obstruction - Pyloric region of stomach
 Most common cause of Gastric Outlet Obstruction in adult patient - Carcinoma of the
stomach.

18. Gold standard method of diagnosing celiac disease is:-

a) Small bowel biopsy


b) Anti-endomysial antibodies
c) Blood picture
d) Biochemical test

Endoscopy with duodenal biopsy showing villous atrophy is the current gold standard
for diagnosing celiac disease

19. Most common site of intracranial metastasis is from primary carcinoma of:-

a) Breast
b) Lungs
c) Stomach
d) Testes

Some differences are seen in the types of primary malignancies responsible for the brain
metastasis in the two genders. Lung cancer is the most common source of brain metastasis
in males, whereas breast cancer is the most common source in females.

20. Nephrotic range proteinuria is seen in:-

a) Arteriitis
b) Polycystic kidney disease
c) Pyelonephritis
d) Glomerulonephritis

 Healthy glomerulus is responsible for repulsion of proteins like Albumin


 Any kind of inflammation affecting the glomerulus will lead to damage to
glomerulus, thus glomerulus will not function properly, leading to proteinuria. This
type of proteinuria is given the name "Sub-nephrotic proteinuria"

Additional findings in patient with glomerulonephritis

 HTN
 Hematuria
 Oliguria
21. Deficiency of which vitamin causes subacute combined degeneration of spinal
cord?

a) Vitamin B1
b) Vitamin B6
c) Vitamin B2
d) Vitamin B12

Subacute combined degeneration of the spinal cord can be caused by inadequate oral
intake of vitamin B12, poor absorption of vitamin B12, or by the use of medications such as
metformin, proton pump inhibitors, and nitrous oxide.

22. Transplant of kidney from mother to son is an example of:-

a) Autograft
b) Allograft
c) Isograft
d) Xenograft

graft taken from one part of the body of an individual and


Autograft transplanted onto another site in the same individual, e.g., skin
graft.
graft taken from one individual and placed on another individual
Isograft of the same genetic constitution, e.g., grafts between identical
twins.
Allograft graft taken from one individual placed on genetically non-
identical member of the same species.
graft taken from one individual placed on an individual belonging
Xenograft to another species, e.g., animal to man.

23. Patients with Polycystic disease of kidney presents with all except:-

a) Hematuria
b) Hypertension
c) Renal failure
d) Erythrocytosis

Polycystic kidney disease patients presents with -

 Hematuria
 Due to activation of Renin - Angiotensin - Aldosterone System (RAAS) the patient will
have HYPERTENSION.
 Over period of time, renal mass will be reduced so patient will develop Renal
failure, as the renal mass is reduced. There will be reduction in the secretion of
Erythropoietin, leading to anemia NOT Erythrocytosis.
 Adult PCKD has Autosomal dominant inheritance
 Child PCKD has Autosomal Recessive inheritance

ANESTHESIA JUNE 2018

1. False statement about post-dural puncture headache (PDPH):

a) Breach of dura
b) Onset of headache is usually 12-72 hours following procedure
c) Commonly occipito - frontal in location
d) Headache is relieved in sitting standing position

Post Dural puncture Headache

 Pathophysiology → CSF leak → ICP ↓es → Traction on nerve fibres originating from
pia mater
 Site → Occipital > Frontal > Retrorbital
 Duration → 7-10 days
 Pathognomic sign → Change in posture aggravates headache
 Predisposing conditions -
 Dura cutting needle (spinal needle)
 Most commonly seen with 22G Quincke needle.
 Wide bore needle Multiple attempts.
 Pregnancy (In pregnancy dura mater is very fragile venous engorgement fragility
increases chances of PDPH is more).
 Note- Early ambulation never predispose to PDPH ***

2. In status asthmaticus, anesthetic agent used as bronchodilator:-

a) Morphine
b) Thiopentone sodium
c) Ketamine
d) Halothane

Ketamine is a short-acting pentachlorophenol derivative that exerts


bronchodilatory effects because it leads to an increase in endogenous
catecholamine levels, which may bind to beta receptors and cause smooth
muscle relaxation and bronchodilation. Ketamine was used in the management
of status asthmaticus in patients with respiratory failure who did not respond
adequately to mechanical ventilation. This agent improved airway resistance,
particularly the lower airways, as well as improve lung compliance.

3. Which of the following inhalational agent is the induction agent of choice in


children?

a) Methoxyflurane
b) Sevoflurane
c) Desflurane
d) Isoflurane

Sevoflurane is less soluble than isoflurane and is very pleasant to breathe, which makes it
an excellent choice for inhalational induction of anesthesia, particularly in children. The
respiratory and cardiovascular effects of sevoflurane are very similar to isoflurane, but
sevoflurane does not cause 'coronary steal'.

4. Compression ventilation ratio in case of Neonatal CPR:-

a) 3:1
b) 15:2
c) 30:2
d) 100:8

There should be a 3:1 ratio of compressions to ventilations, with 90 compressions and 30


breaths to achieve approximately 120 events per minute. Thus, each event will be allotted
approximately 1/2 second, with exhalation occurring during the first compression following
each ventilation.

5. Massive transfusion of stored blood result in:-

a) Citric acid toxicity


b) Hypokalemia
c) Metabolic acidosis
d) Hypercalcemia

Citric acid toxicity: Citrate is used as Preservative in stored blood.


Massive transfusion results in:

 Hyperkalemia: Potassium gets released from stored blood after day 5 from the date
of collection.
 Metabolic Alkalosis: Citrate in blood is converted to bicarbonate in liver ,which
causes metabolic alkalosis.
 Hypocalcemia: Calcium in patient's blood binds to the chelating agent contained
within the stored blood.

PSYCHIATRY JUNE 2018


1. Jet black pigmentation of tongue with tactile hallucinations is feature of:-

a) Heroin
b) Opium
c) Alcohol
d) Cocaine

COCAINE -

 Derived from erythroxylum coca


 Has Na blocking properties
 Seizures
 FREE BASING
 Crack (smoked)
 Jet black pigmentation of tongue

Intoxication-

 Tactile hallucination - cocaine bugs formication or magnan phenomenon


 Auditory hallucination
 Delusion of persecution

2. Not a fundamental symptom of schizophrenia:-

a) Autism
b) Automatism
c) Association Disturbances
d) Ambivalence

The fundamental symptoms, which are virtually present through all the course of the
disorder, are also known as the famous Bleuler's four A's

1. Autism (autistic thinking and behavior) – excessive fantasy thinking which is


irrational and withdrawn behavior.
2. Ambivalence- inability to take a decision
3. Affect disturbances -inappropriate affect/emotions
4. Association disturbances- of thoughts such as formal thought disorders.
3. Neurotransmitter involved in depression:-

a) GABA and dopamine


b) Serotonin and GABA
c) Dopamine and serotonin
d) GABA and norepinephrine

The three neurotransmitters implicated in Depression


are: Dopamine. Norepinephrine. Serotonin.

4. A girl normal language milestones spends her time seeing hel own hands and
does not interact with others. Likely diagnosis is:-

a) ADHD
b) Autism
c) Asperger's syndrome
d) Rett's syndrome

Asperger's Syndrome, a form of Autism Spectrum Disorder, is a developmental disorder.


Young people with Asperger's Syndrome have a difficult time relating to others socially and
their behavior and thinking patterns can be rigid and repetitive.

PHARMA JUNE 2018

1. Pharmacodynamics deals with:-

a) Mode of excretion of drug


b) Mechanism of action of a drug
c) Transport of drug across the biological membranes
d) Latency of onset

Pharmacodynamics is the branch of pharmacology dealing with the mechanisms of action


of drugs. Pharmacodynamics involves the study of the biochemical and physiological
changes produced by drugs in the body during the prevention and treatment of disease.
2. Drug of choice for paroxysmal supraventricular tachycardia is:-

a) Digoxin
b) Propranolol
c) Adenosine
d) Dilitiazem

Adenosine is the first-line medical treatment for the termination of paroxysmal SVT.

3. Malignant neuroleptic syndrome is caused by:-

a) Antidepressants
b) Anxiolytics
c) Antipsychotics
d) Anti-epileptics

Neuroleptic malignant syndrome comes about, most likely, as a result of “dopamine D2


receptor antagonism”. The first-generation antipsychotics work by inhibiting
dopaminergic neurotransmission. Their effectiveness is best when they block about 72%
of the D2 dopamine receptors in the brain. Dopamine is a chemical substance
(neurotransmitter) found in the brain and elsewhere in the central nervous system that acts
to convey messages from one cell to another.

4. Treatment for Zollinger-Ellison syndrome is:-

a) Climetidine
b) Omeprazole
c) Misoprostol
d) Aluminium hydroxide

Zollinger-Ellison syndrome managed or treated:

 Medication: Drugs called proton pump inhibitors(Omeprazole) reduce the


production of stomach acid and help heal ulcers.
 Surgery to remove gastrinomas.
 Chemotherapy: Anticancer drugs to shrink gastrinomas and lower the levels of
gastrin in the blood.

5. Proton pump inhibitors suppress gastric acid secretion by acting on:-

a) H+/CI- ATPase pump


b) H+/K+ ATPase pump
c) Na+/K+ ATPase pump
d) Na+/K+/CI- ATPase pump

Proton pump inhibitors act by irreversibly blocking the hydrogen/potassium adenosine


triphosphatase enzyme system (the H+/K+ ATPase, or, more commonly, the gastric proton
pump) of the gastric parietal cells.

6. The drug aspirin has been used for secondary prevention of ischemic heart
disease because, it:-

a) Inhibits TXA2 synthesis by platelets


b) Inhibit prostacyclin synthesis
c) Inhibits release of EDRF
d) Inhibit synthesis of endothelins

Aspirin produces its effects through inhibition of thromboxane A2 (TXA2) production, while
P2Y12 antagonists attenuate the secondary responses to ADP released by activated
platelets. The anti-platelet effects of aspirin and a P2Y12 antagonist are often considered to
be separately additive.

7. Drug of choice for central diabetes insipidus is:-

a) Insulin
b) Desmopressin
c) Diazoxide
d) Chlorpropamide

In patients with central DI, desmopressin is the drug of choice. A synthetic analogue of
antidiuretic hormone (ADH), desmopressin is available in subcutaneous, IV, intranasal, and
oral preparations.

8. A pediatric patient was prescribed erythromycin for prevention of recurrent


rheumatic fever in a rural health center. the patient complained of a history of
constipation, which medication should be avoided to prevent development of any
arrhythmia?

a) Lubiprostone
b) Streptomycin
c) Ebastine
d) Cisapride
 Erythromycin is inhibitor of microsomal enzyme - CYP3A4
 CAT drugs increase QT interval (metabolized by CYP3A4).
 Cisapride
 Astemizole
 Terfenadine
 If administered with Erythromycin, metabolism is inhibited, drug concentration rises.
 At high plasma concentration, these drugs block K* channel in the heart leading to QT
prolongation →Torsades de pointes.

9. Which of the following is not an alkylating agent:-

a) Melphalan
b) Cyclophosphamide
c) Busulfan
d) 5-Fluorouracil

Alkylating agents in the classic family include nitrogen mustard, cyclophosphamide,


chlorambucil, melphalan, busulfan, and ifosfamide.

10. Which of the following drug is not metabolized by acetylation?

a) Isoniazid
b) Hydralazine
c) Phenytoin
d) Procainamide

Drugs metabolized by Acetylation are SHIP


Sulfonamides
Hydralazine
Isoniazid
Procainamide

11. Therapeutic index of a drug is an indicator of:-

a) Potency
b) Safety
c) Efficacy
d) All of these
The therapeutic index (TI, also referred to as therapeutic ratio) is a quantitative
measurement of the relative safety of a drug. It is a comparison of the amount of a
therapeutic agent that causes the therapeutic effect to the amount that causes toxicity.

12. Beta-blockers are used in all of the following except:

a) Essential tremors
b) AV block
c) Angina pectoris
d) Migraine prophylaxis

Contraindications for beta blockers include:


 Abrupt discontinuations
 Acute bronchospasm
 Acute heart failure
 Asthma
 AV block
 Bradycardia
 Bronchitis
 Cardiogenic shock
 Cerebrovascular disease
 Chronic obstructive pulmonary disease (COPD)
 Diabetes mellitus
 Emphysema
 Hypersensitivity to beta blockers
 Hypotension
 Kidney failure
 Hepatic disease
 Myopathy
 Pheochromocytoma
 Psoriasis
 Stroke
 Vasospastic angina
 Wolff-Parkinson-White syndrome.

13. A 45 year old male was admitted to the ICU after being diagnosed with Sub
arachnoid hemorrhage secondary to a ruptured aneurysm. The patient is a known
case of stage 2 hypertension and was taking Ramipril for the same. Which of the
following calcium channel blockers can be used in this case, which might help
relieve the cerebral vasospasm?

a) Nimodipine
b) Felodipine
c) Amlodipine
d) Nitrendipine
Nimodipine (DHPs)- Blocks Ca2+ channel only in cerebral blood vessels which leads
to Cerebral vasodilation is it used in subarachnoid hemorrhage (SAH).

14. A hypertensive patient was started on some anti-hypertensive drug but he


developed dry cough. Which of the following drug can be responsible for this
effect?

a) Calcium channel blockers


b) Beta blockers
c) ACE inhibitors
d) Diuretics

Side effects of ACE inhibitors may include:

 Dry cough
 Increased potassium levels in the blood (hyperkalemia)
 Fatigue
 Dizziness from blood pressure going too low
 Headaches
 Loss of taste

15. Mechanism of action of zileuton:-

a) Cyclooxygenase inhibitor
b) 5-lipooxygenase inhibitor
c) 5-phosphodiesterase inhibitor
d) Prevents mast cell degranulation

Zileuton is a 5-lipoxygenase inhibitor, which in turn, inhibits the formation of leukotrienes


B4, C4, D4, E4. Limiting these leukotrienes, in turn, helps to reduce inflammation, edema,
mucus secretion, and bronchoconstriction in the airways.

16. In the management of diabetes mellitus, lactic acidosis is caused by which of the
following?

a) Tolbutamide
b) Glipizide
c) Pioglitazone
d) Metformin
A specific class of oral diabetes medication, called biguanides, can cause a buildup of
lactic acid levels. Metformin (Glucophage) is one of these drugs. It's used to treat
diabetes and may also be prescribed for other conditions, such as renal insufficiency.

17. Antidote for alprazolam is:-


a) Protamine sulphate
b) Flumazenil
c) EDTA
d) BAL

Flumazenil, a specific benzodiazepine antagonist, is useful in reversing the sedation and


respiratory depression that often occur when benzodiazepines are administered to
patients undergoing anesthesia or when patients have taken an intentional
benzodiazepine overdose. Alprazolam belongs to a class of medications
called benzodiazepines. Benzodiazepines are medications that have anti-anxiety-producing
properties.

RADIO JUN 2018

1. What is marked by arrow?

a) Right atrium
b) Pulmonary trunk
c) Aortic knuckle
d) Superior vena cava

The aortic knob or knuckle refers to the frontal chest x-ray appearance of the distal
aortic arch as it curves posterolaterally to continue as the descending thoracic aorta. It
appears as a laterally-projecting bulge, as the medial aspect of the aorta cannot be seen
separate from the mediastinum.
2. Floating water lily sign is seen in:

a) Lung hydatid
b) Bronchial adenoma
c) Lung abscess
d) Aspergilloma

The water-lily sign, also known as the camalote sign, is seen in hydatid infections when
there is detachment of the endocyst membrane which results in floating membranes within
the pericyst that mimic the appearance of a water lily.

3. String sign is seen in:

a) Crohn's disease
b) TB of ileocecal region
c) Idiopathic hypertrophic pyloric stenosis
d) All of the above

 The gastrointestinal string sign (also known as the string sign of Kantor) refers
to the string-like appearance of a contrast-filled bowel loop caused by its severe
narrowing.
 Originally used to describe the reversible narrowing caused by spasms in Crohn
disease, it is now used for any severe narrowing of the bowel lumen, including that
seen in hypertrophic pyloric stenosis, gastrointestinal tuberculosis, carcinoid
tumor and colon cancer.

4. Dead bone on an X-ray looks ?

a) More radio-opaque
b) Radiolucent
c) Less radio-opaque
d) Not seen at all

In chronic osteomyelitis there is dead dense bone in center called sequestrum , which
is more radio opaque.

5. Multiple punched out lesions in skull is seen in:

a) Acute leukemia
b) Thalassemia
c) Multiple myeloma
d) Ca prostate

A SKULL ROENTGENOGRAM demonstrating “punched-out” lesions of the calvarium is


usually associated with the diagnosis of a number of malignant or systemic disorders. The
disorders that most frequently come to mind are metastatic carcinoma or multiple
myeloma.
PEDIA JUNE 2018

1. Earliest indication of sexual maturation in females:

a) Maturation
b) Thelarche
c) Pubarche
d) Menarche

In most girls, breast budding is the first visible sign of sexual maturation –“Thelarche”
means "the beginning of breast development, followed closely by the initiation of the
growth spurt. Shortly thereafter, pubic and axillary hair appears. Menarche generally occurs
about 2 years after onset of breast development and when growth in height slows after
reaching its peak.

2. A child of 4 years can do all of the following EXCEPT:

a) Copies Cross
b) Bridge with blocks
c) Copies Circle
d) Copies triangle

KEY FINE MOTOR MILESTONES


Age Milestone
4 months Bidextrous reach (reaching out for objects with both hands)
6 months Unidextrous reach (reaching out for objects one hand); transfers objects
9 months Immature pincer grasp; probes with forefinger
12 months Pincer grasp mature
15 months Irritates scribbling; tower of 2 blocks
18 months Scribbles tower of 3 blocks
2 years Tower of 6 blocks; vertical and circular stroke
3 years Tower of 9 blocks; copies circle
4 years Copies cross; bridge with blocks
5 years Copies triangle ; gate with blocks

3. Specific sign of kwashiorkor is:

a) Pitting edema
b) Weight loss
c) Flag sign
d) Muscle wasting
Kwashiorkor Marasmus
 Because of protein deficiency  Because of both protein and calorie deficiency
 Mc in 6 months to 3 yrs. of age  Mc in infancy
 Subcutaneous fat preserved  Not preserved
 Edema present  Edema Absent
 Ribs are not very prominent  Ribs are very prominent
 Lethargic  Alert and irritable
 Mild or absent muscle wasting  Sever muscle wasting
 Poor appetite  Voracious feeder
 Fatty liver is common  Fatty liver is Uncommon
 Skin and hair changes are more common  Skin and hair changes are Less common

4. Low osmolarity ORS has (as compared to WHO-ORS):

a) Low glucose and high sodium


b) Low glucose and low sodium
c) Low glucose and low potassium
d) Low glucose and high potassium

Composition ReSoMal (mmol/L) Standard ORS Reduced osmolarity


(mmol/L) ORS
Glucose 125 111 75
Sodium 45 90 75
Potassium 40 20 20
Chloride 70 80 65
Citrate 7 10 10
Magnesium 3 - -
Zinc 0.3 - -
Copper 0.045 - -
Osmolarity 300 311 245
(mOsm/L)

5. Jaundice at birth or within 24 hours of birth is due to:

a) Erythroblastosis fetalis
b) Congenital hyperbilirubinemia
c) Biliary atresia
d) Physiological jaundice of newborn
 Erythroblastosis fetalis is hemolytic anemia in the fetus (or neonate, as
erythroblastosis neonatorum) caused by transplacental transmission of maternal
antibodies to fetal red blood cells. The disorder usually results from incompatibility
between maternal and fetal blood groups, often Rho(D) antigens.
 Destruction of the red blood cells (hemolysis) can be rapid in a fetus. As a result, the
fetus will not receive enough oxygen, which may lead to anemia, other illnesses, or even
death.
 As hemolysis continues, the fetus will rapidly attempt to produce more red blood cells.
However, these cells new red blood cells are often immature and are unable to function
fully.
 Because the body produces red blood cells in the liver and spleen, this overproduction
can sometimes cause these organs to enlarge.
 When a newborn has this condition, it is known as hemolytic disease of the newborn.
 As the immature red blood cells continue to break down, bilirubin, which is a by-product
of the breakdown of red blood cells, builds up. The excess amounts of bilirubin
circulating in the newborn’s body will lead to jaundice, where the skin and eye whites of
the infant turn yellow.

6. 1 month old newborn presents with conjugated hyperbilirubinemia, intrahepatic


cholestasis and high alkaline phosphatase. Eosinophilic, PAS positive
cytoplasmic granules were seen in liver biopsy specimen. These are suggestive
Alpha 1 AT deficiency and usually presents as neonatal cholestasis. What is the
MC cause of jaundice in this newborn?

a) Hypoplasia of biliary tract


b) Neonatal hepatitis
c) Choledochal cyst
d) Physiological

 MC cause of cholestatic jaundice in newborn is Biliary atresia > Neonatal hepatitis.


 MCC of jaundice in 1st 24 hrs of life - Hemolytic disease of newborn.
 MCC of jaundice in 24 hrs to 72 hrs of life - Physiological jaundice.

7. MC congenital defect in Down's syndrome:

a) PDA
b) ASD
c) Atrioventricular septal defect
d) VSD

 MC congenital heart defect in Down's syndrome is Atrioventricular septal defect/


Endocardial cushion defect.
 MC congenital heart defect in turner syndrome is Bicuspid aortic valve followed by
coarctation of aorta.
 MC congenital heart defect in DM is transposition of great arteries. Most characteristics
in DM is sacral agenesis.
 MC congenital heart defect in Noonan syndrome is pulmonary stenosis.
 MC congenital heart defect in child born to mothers with lithium intake is Ebstein
anomaly.
 MC congenital heart defect in Rubella is Patent ductus arteriosus.
 Overall MC congenital heart defect is VSD
 MC ASD is foramen secundum.
 MC ASD in Down's syndrome is foremen primum.

8. MC malignancy in children is:

a) Retinoblastoma
b) Leukemia
c) Wilms tumor
d) Neuroblastoma

 Leukemia is the most common malignant neoplasm in childhood


 MC malignancy in children is ALL > Brain tumor > neuroblastoma > Wilms tumor.
 B cell ALL is more common than T- Cell ALL

9. A malignant tumour of childhood that frequently metastasizes to bone marrow


most often is:

a) Wilms tumor
b) Neuroblastoma
c) Adrenal gland tumors
d) Granulosa cell tumor of ovary

 Neuroblastoma is the most common intra-abdominal and extracranial solid tumor in


children, accounting for 7-8% of all cancers.
 Metastasis present in 60-70% children, are usually to the skeleton (facial bones, skull),
bone marrow and lymph nodes.

10. A 2 year old male child was brought by the parents to the pediatric department
with loose stools from last 3 days. On examination she has signs of severe
dehydration. The intern on duty was asked by the JR to calculate the volume of
fluid to be given in the first 3 hrs if her weight is 10 kg.

a) 1000 ml
b) 100 ml
c) 200 ml
d) 500 ml
In a 2 year old child with severe dehydration, fluid given over 1st 30 minutes is 30mL/kg,
over next 2.5 hrs is 70mL/kg: So fluid given over 3 hrs is 100mL/kg. In this case, as weight of
child is 10 kg, amount of fluid to be given is 10x100=1000mL.

11. A 1-year-old male child present to the hospital with a fever associated with neck
stiffness. You are suspecting meningitis. Which of the following is the most
common cause of meningitis in this age group?

a) Klebsiella
b) Streptococcus pneumoniae
c) Staphylococcus aureus
d) Mycobacterium tuberculosis

IN INDIA :

Age group Organism


Neonate Acinetobacter, E.coli
1 month to 2 years Strep. pneumoniae
Older children Strep. Pneumoniae, N. meningitidis

IN WORLD :

Neonate Group B strep, E.coli, Listeria


Infant and children Strep. Pneumoniae, N. meningitidis, H. influenza
Adolescent and young adults Strep Pneumoniae, N. meningitidis

12. Best method to diagnose HIV in infancy is:

a) Western blot
b) ELISA
c) PCR
d) All

WHO recommendation for HIV testing:

<18 months HIV DNA or RNA PCR or HIV culture


>18 months IgG antibody to HIV by ELISA and western blot test.
OBG JUNE 2018

1. Size of uterus (in inch):

a) 4x2x1
b) 3x2x1
c) 4x3x1
d) 5x4x2

UTERUS :

 8 cm x 6 cm wide x 3-4 cm, weight 50-80 g.


 It Consists of:
 body
 isthmus
 cervix

2. Pearl index is used to calculate:

a) Accidental pregnancies
b) Population
c) Fertility rate
d) Abortions

The Pearl Index is defined as the number of contraceptive failures(accidental


pregnancies) per 100 women-years of exposure, and uses as the denominator the total
months or cycles of exposure from the initiation of the product to the end of the study or the
discontinuation of the product.

3. Progesterone of choice in emergency contraception:

a) Micronized progesterone
b) Norgesterone
c) Levonorgesterol
d) DMPA

Only the progestin levonorgestrel has been studied for use as emergency contraception. It
is marketed as Plan B.
4. Most likely to be associated with vaginal pH of 4:-

a) Atrophic vaginitis
b) Trichomanas vaginatis
c) Candidal vaginitis
d) Gardenella vaginitis

Vaginal pH can be tested using a narrow-range pH paper. A pH above 4.5 suggests


infections such as bacterial vaginosis or trichomoniasis (pH 5-6) and helps to exclude
vulvovaginal candidiasis (pH 4-4.5).

5. Which one of the following in NOT a characteristic feature of bacterial vaginosis?

a) Presence of clue cells


b) Thick white discharge
c) Vaginal pH > 4.5
d) Fishy odour

Bacterial Vaginosis :

 Characterized by a pungent vaginal "fish-like" odor and a thin, off-white/gray,


homogenous, discharge that coats the vaginal wall. Does NOT usually cause dysuria,
dyspareunia, pruritus, or vaginal/vulvar inflammation.
 The best initial test is microscopic examination. Diagnosis is supported by the presence
of clue cells on wet mount.
 Other supportive findings include a vaginal pH >4.5 and a positive whiff-amine test.
 Treated with either metronidazole or clindamycin for seven days.
 Routine screening and treatment of all pregnant women with asymptomatic bacterial
vaginosis is NOT recommended.

6. Increased LH: FSH ratio is seen in

a) Premature menopause
b) Sheehan syndrome
c) PCOD
d) Turner's syndrome

 Premature menopause → LH↓ & FSH ↓


 Sheehan syndrome → < 1 LH & FSH because of pituitary necrosis
 PCOD → LH: FSH ratio is ↑↑ (i.e 2:1) or (3:1)
 Turner's syndrome → ovarian dysgenesis LH ↑ & FSH ↑

7. Which is not classified as primary amenorrhea:

a) Mullerian agenesis
b) Turner syndrome
c) Sheehan's syndrome
d) Turner's syndrome

Sheehan's syndrome causes secondary amenorrhea.

CAUSES OF PRIMARY AMENORRHEA :

 Mullerian agenesis/hypoplasia
 Imperforate hymen
 Vaginal agenesis
 Cervico vaginal atresia
 Endocrinological causes
 Hypogonadotropic hypogonadism
 Primary ovarian failure
 Congenital adrenal hyperplasia
 Constitutional delay
 Hypothyroidism
 Genetic causes
 Turner syndrome
 Pure gonadal agenesis
 Androgen insensitivity syndrome

8. A 23 year old woman presents with irregular cycles. Her cycles come at intervals
of 35-45 days, last for 3 to 4 days with mild to moderate flow, no passage of clots
and no dysmenorrhea. Which of the following describes this condition?

a) Amenorrhoea
b) Oligomenorrhoea
c) Polymenorrhoea
d) Menorrhagia

Oligomenorrhea is a condition in which you have infrequent menstrual periods. It


occurs in women of childbearing age. Some variation in menstruation is normal, but a
woman who regularly goes more than 35 days without menstruating may be diagnosed
with oligomenorrhea.
9. A 50 year old woman diagnosed with an ovarian malignancy most likely has?

a) Granulosa cell tumour


b) Dermoid cyst
c) Mucinous cystadenoma
d) Serous cystadenoma

Ovarian tumors:

Epithelial cell tumor Germ cell tumor Sex cord tumor


M/C tumor (90%) 5-8% 3%
Types:  Teratoma  Estrogen Secreting tumors
 Serous (75-80%) (M/C)  Yolk sac tumor  Granulosa cell
 Endometrial (8-10%)  Endodermal Sinus  Fibroma
 Mucinous (5%) tumor  Thecoma
 Clear cell  Choriocarcinoma  Androgen secreting Tumor
 Brenner tumors  Dysgerminoma  Sertoli cell tumor
 Dermoid cyst  Leydig cell
 Hilus cell tumor

10. Which is not associated with fibroid:

a) Pelvic mass
b) Amenorrhoea
c) Infertility
d) Menstrual irregularity

Symptoms of fibroid:

 M/C presentation in a fibroid is asymptomatic


 M/C symptom is menstrual disturbances i.e progressive menorrhagia
 Pressure symptom
 Infertility
 Pain
 Polycythemia
 Hypoglycemia & hypokalemia.

11. Condyloma acuminate is caused by:

a) HSV
b) HBV
c) HPV
d) HIV
 Condyloma acuminate (genital warts ) → HPV 6,11
 Cancer cervix HPV 16, 18
 Prevention → Gardasil vaccine prevents HPV types 6,11,16,18 (Quadrivalent)
 RX → Podophyllin resin + Cryotherapy

12. Ferning pattern of drying cervical mucus suggests the action of:

a) Estrogen
b) Progesterone
c) Prolactin
d) 17-ketosteroids

Ferning occurs due to the presence of sodium chloride in mucus under estrogen effect.
When high levels of estrogen are present, just before ovulation, the cervical mucus forms
fern-like patterns due to crystallization of sodium chloride on mucus fibers. This pattern is
known as arborization or 'ferning'.

13. Which of the following is treatment of choice for Trichomonas vaginalis?

a) Amikacin
b) Metronidazole
c) Chloramphenicol
d) Ampicillin

Infections with the sexually transmitted protozoan Trichomonas vaginalis are usually treated
with metronidazole, a 5-nitroimidazole drug derived from the antibiotic azomycin.
Metronidazole treatment is generally efficient in eliminating T. vaginalis infection and has a
low risk of serious side effects.

14. A liquid media used to suppress commensal bacteria while allowing the pathogen
to remain viable and grow. Which of the following is an enrichment media?

a) Alkaline peptone water


b) Monsour's taurocholate Tellurite peptone water
c) Selenite F broth
d) All of the above

Enrichment media

 LIQUID culture media


 These media are used to suppress commensal bacteria while allowing the pathogen to
remain viable and grow.
 example-
 Alkaline peptone water - V. cholerae
 Selenite F-broth - Salmonella, shigella
 Monsour's taurocholate tellurite peptone water - V. cholerae

BIO CHEM JUNE 2018

1. Km increases, but Vmax remains same. This is what Inhibition:

a) Competitive
b) Non-competitive
c) Irreversible
d) Uncompetitive

Effect on Km and Vmax for different type of Inhibitors:


Type of inhibitor Km Vmax
Competitive Increased Same
Non Competitive Same Decreased
Uncompetitive Decreased Decreased

2. Cofactor for xanthine oxidase?

a) Zinc
b) Copper
c) Molybdenum
d) Selenium

The most important function of Molybdenum is as a cofactor for xanthine


dehydrogenase/oxidase, aldehyde oxidase, and sulfite oxidase.

3. Which of the following correctly defines Isoenzymes?

a) Forms of the same enzyme that catalyze different reaction


b) Forms of the same enzyme that catalyze same reaction
c) Forms of the different enzymes that catalyze different reaction
d) Forms of the different enzymes that catalyze same reaction
 Isozymes/Isoenzymes are different forms of same enzyme, catalyzing same reaction
in same species.
 Example:- There are five isozymes of Lactate Dehydrogenase (LDH-1 to LDH-5), All
convert Pyruvate to Lactate.

4. Ammonia is detoxified in brain by:

a) Creatinine
b) Uric acid
c) Glutamine
d) Urea

 The transport form of Ammonia in blood and from Brain is Glutamine.


 The answer is not glutamate because free ammonia is trapped by glutamine. Glutamate
is formed via transamination of amino acids, not from free ammonia.

5. Atherosclerosis involves the formation of lipid-rich plaques in the intima of


arteries which can narrow the coronary arteries, leading to decreased blood flow
and hypoxia of cardiac cells. Which of the following amino acid is known to be
associated with Atherosclerosis?

a) Methionine
b) Cysteine
c) Homocysteine
d) Leucine

HOMOCYSTINURIA :
Homocysteine accumulation lead to stroke, atherosclerosis, MI, Pulmonary embolism.
Homocysteine binds with the endothelium of the blood vessels and activates signaling
pathways which leads to the release of pro-inflammatory molecules. This could result in the
following:

 Extensive atheroma formation at a young age affecting many arteries but not the
coronary arteries
 Intravascular thrombosis

6. Blood glucose levels cannot be augmented by mobilization of muscle glycogen


due to lack of:

a) G-6-P dehydrogenase
b) G-6-phosphatase
c) Aldolase
d) Glucokinase
 Muscles lack the enzyme Glucose-6-phosphatase, that's why muscles cannot maintain
blood glucose levels.
 Liver cells have this enzyme. And therefore liver can only maintain blood glucose.

7. Glucose can be synthesized from all except

a) Amino acids
b) Glycerol
c) Acetoacetate
d) Lactic acid

Gluconeogenesis (literally, “formation of new sugar”) is the metabolic process by which


glucose is formed from noncarbohydrate sources, such as lactate, amino acids, and
glycerol.

8. Which is branching enzyme?

a) Glycogen synthetase
b) Glucose-6 phosphatase
c) Amylo-1, 4-1, 6-transglycolase
d) Glycogen Phosphorylase

 Branching enzyme - alpha (1-6) extend with alpha (1-4)


 Another name amylo (1-4) (1-6) transglycosylase.

9. Most Common enzyme deficient in galactosemia:

a) Galactose-1-phosphate uridyl transferase/GALT


b) Galactosidase
c) UDP galactose epimerase
d) Galactokinase

Classic galactosemia (type 1) - the most common and severe type, caused by mutations in
the GALT gene, and characterized by a complete deficiency of an enzyme called galactose-
1-phosphate uridyl transferase (GALT).
10. Essential fatty acid:

a) Citric acid
b) Linoleic acid
c) Stearic acid
d) Palmitic acid

Essential Fatty Acid :

 Omega-3 Fatty Acid.


 Alpha-Linolenic Acid.
 Linoleic Acid.
 Eicosapentaenoic Acid.
 Docosahexaenoic Acid.
 Polyunsaturated Fatty Acid.
 Lipids.
 Fatty Acids.

11. Myocardium normally utilizes:

a) Glucose
b) Lactose
c) Fatty acid
d) Glycogen

FED FASTING STARVATION


BRAIN GLUCOSE GLUCOSE KETONE BODIES
HEART FATTY ACIDS FATTY ACIDS KETONE BODIES
LIVER GLUCOSE FATTY ACIDS AMINO ACIDS
MUSCLE GLUCOSE FATTY ACIDS FATTY ACIDS &
KETONE BODIES
ADIPOSE TISSUE GLUCOSE FATTY ACIDS FATTY ACIDS
RBC GLUCOSE GLUCOSE GLUCOSE

12. Apoprotein for chylomicron remnant:-

a) Apo E
b) Apo C1
c) Apo A2
d) Apo A1
APOPROTEIN - PROTEIN PRESENT IN LIPOPROTEIN
Lipoprotein Protein
Chylomicron APO B48
Chylomicron remnant APO B48 + APO E
VLDL APO B100
IDL APO B100 APO E
LDL APO B 100 + APO
HDL APO A, C, E

13. Energy reserve of the body is:

a) Carbohydrate
b) Triglyceride
c) Proteins
d) Fibres

The major energy reserve in the body is found stored as fat (triglyceride) in the adipose
tissue.

14. Chaperones are:

a) Purine metabolism mediators


b) Antigen presenting cells
c) Mediators of post-transitional assembly of protein complexes
d) None of the above

CHAPERONES-FOLDING /SHAPE /ASSEMBLY OF PROTEIN

 POST-TRANSITIONAL CHANGE
 They facilitate and favour the interactions on the polypeptide surfaces to finally give the
specific conformation of a protein.

15. Vitamin K is required for:

a) Hydroxylation
b) Chelation
c) Transamination
d) Carboxylation
Vitamin K :

 Act as coenzyme in Carboxylation of clotting factor 2,7,9,10


 Only fat-soluble vitamin with coenzyme function.

ENT JUNE 2018


1. Most common manifestation of nasopharyngeal carcinoma

a) Epistaxis
b) Headache
c) Nasal obs
d) Cervical lymphadenopathy

 Nasopharyngeal Carcinoma m/c site is Fossa of Rosenmuller


 Ca nasopharynx m/c presentation - neck mass (cervical lymph node metastasis)
75% of patients.
 It causes U/L serous otitis media - U/L conductive hearing loss.

2. A 14 year old Male presented with history of fever since 2 days, unable to swallow
the food with muffled voice. On examination it is noted right tonsil is shifted to
midline. What is the diagnosis:

a) Quincy
b) Acute tonsillitis
c) Para pharyngeal abscess
d) Acute retropharyngeal abscess

If tonsil is enlarged / hypertrophied and reaching midline and soft palate is moved on other
side it is a case of quincy but, if tonsil itself is pushed to the midline it is a case of
parapharyngeal abscess.

3. Adam's apple seen in boys is because of:

a) Hyoid bone
b) Tracheal rings
c) Thyroid cartilage
d) Cricoid cartilage

 Prominence of Thyroid cartilage (because of sharp angle) in males is called


Adam's apple.
 Thyroid cartilage with right and left lamina both meet al midline and form an angle of
90 in males, 120 in females.
 At rest it lies at level of C3.

4. All of the following are seen in Meniere's Disease except:

a) Tinnitus
b) Vertigo
c) Fullness of ear
d) Ear Pain

 Meniere's disease also called endolymphatic hydrops is a disorder of the inner ear where
the endolymphatic system is distended with endolymph.
 It is characterized by
 Vertigo
 Sensorineural hearing loss
 Tinnitus
 Aural fullness

5. 34 yrs old female presents with tinnitus, vertigo and fullness in ear. Likely
Diagnosis:

a) Meniere's Disease
b) Otosclerosis
c) Ototoxicity
d) Noise Induced Hearing Loss

 Meniere's disease also called endolymphatic hydrops is a disorder of the inner ear where
the endolymphatic system is distended with endolymph.
 It is characterized by
 Vertigo
 Sensorineural hearing loss
 Tinnitus
 Aural fullness

6. Most common cause of SNHL

a) Labyrinthitis
b) Presbycusis
c) Meniere's disease
d) Vestibular Schwannoma
Presbycusis :

 Most common cause of SNHL.


 Associated with physiological aging process in the ear
 Bilateral slopping curve
 Usually manifests at the age of 65 years, (old age) but may do so early if there is a
hereditary predisposition, chronic noise exposure or generalized vascular disease.

7. Most common site for laryngeal web is

a) Supraglottic
b) Subglottic
c) Anterior Glottis
d) Posterior Glottis

Laryngeal web

 It is due to incomplete recanalization of larynx -m/c site is anterior glottis


 Rx- Excision or Co2 laser excision. (depends upon thickness of the web).

8. Lifesaving muscle of vocal cords is:

a) Posterior cricoarytenoid
b) Lateral cricoarytenoid
c) Transverse arytenoid
d) Thyroarytenoid

 Safety muscle/lifesaving muscle of vocal cord is Posterior cricoarytenoid


because this muscle is the only abductor.
 Tensor of vocal cord (1 - cricothyroid, 2-vocalis).

9. All of the following is seen in superior laryngeal nerve palsy EXCEPT:

a) Aspiration
b) Stridor
c) Bowed vocal cords
d) Loss of pitch

Superior laryngeal nerve palsy causes paralysis of cricothyroid muscle, and


ipsilateral anesthesia of the larynx above the vocal cords.
Clinical features-

 Voice is weak and pitch cannot be raised with decreased ability to sing.
 Anaesthesia of the larynx on one side may pass unnoticed or cause occasional
aspiration.
 Presence of both paralysis and bilateral anaesthesia. Causes inhalation of food and
pharyngeal secretions gives rise to cough and choking fits. Voice is weak and husky.

 Stridor is not seen In superior laryngeal nerve palsy is seen in B/L recurrent
laryngeal nerve palsy.

10. Most common cause of facial nerve palsy:

a) Idiopathic Bell's palsy


b) Herpes zoster oticus
c) Mastoid surgery
d) Chronic suppurative Otitis media

Bell's palsy is the most common cause of facial paralysis, although its exact cause is
unknown. Generally, Bell's palsy affects only one side of the face; however, in rare cases, it
can affect both sides.

11. Rhinolalia clausa occurs in all except

a) Nasopharyngeal polyp
b) Nasopharyngeal carcinoma
c) Cleft palate
d) Nasopharyngeal angiofibroma

Rhinolalia clausa-

 Abnormal speech due to obstruction in nose


 Hyponasal speech
 Causes:
 Nasopharyngeal polyp
 Nasopharyngeal cancer
 Nasopharyngeal angiofibroma, adenoids

12. Cold water is not used for ear cleaning because ?

a) It will make the wax hard


b) Damage to tympanic membrane
c) Caloric stimulation caused by cold water
d) It will cause infection
Temperature for ear syringing is 37°c , because if you decrease or increase the temperature
we can stimulate lateral semicircular canal in the middle ear and patient will have vertigo and
nystagmus (caloric stimulation ).

13. Otomycosis is most commonly caused by:

a) Actinomycetes
b) Aspergillus niger
c) Mucor
d) Candida albicans

Otomycosis / Singapore ear, fungal common causative organism Aspergillus niger


Other cause :

 Aspergillus fumigatus
 Candida albicans

14. A 3 yrs old child presents with c/o fever, barking cough and stridor for 2days, what
is the diagnosis

a) Acute tonsillitis
b) Acute Epiglottitis
c) Croup
d) Adenoiditis

Croup - Acute laryngo tracheo bronchitis.


Causative - Parainfluenza virus type 1 and 2
Child - 3 month to 3 year
C/F - fever, cough (barking cough - Croup)
Stridor - expiratory
X-ray - steeple sign Rx symptomatic

15. CSF Otorrhea is due to involvement / trauma of ?

a) Cribriform plate
b) Petrous temporal bone
c) Parietal bone
d) Tympanic membrane
 CSF otorrhea - CSF coming from ear due to trauma to petrous temporal bone.
 CSF Rhinorrhea - CSF watery discharge from nose

16. Eustachian tube develops from:

a) 2nd and 3rd pharyngeal pouch


b) 1st pharyngeal pouch
c) 2nd pharyngeal pouch
d) 3rd pharyngeal pouch

Pharyngeal Pouch Derivatives


1 Tympanic (middle car) cavity, auditory (eustachian) tube
2 Palatine tonsils, tonsillar fossa
3 Inferior parathyroid gland, thymus
4 Superior parathyroid gland, ultimobranchial body (parafollicular
C cells of thyroid)

17. Recruitment phenomenon is seen in:

a) Otitis media with effusion


b) Acoustic nerve schwannoma
c) Meniere's disease
d) Otosclerosis

 Recruitment is phenomenon of abnormal growth of loudness.


 Recruitment is typically seen in lesions of the cochlea (e.g. Méniere's disease and
presbycusis) and thus helps to differentiate a cochlear from a retrocochlear
sensorineural hearing loss.

18. Tobey-Ayer test is done for:

a) Acantholysis
b) Hemoglobinuria
c) Ketosis
d) Lateral sinus thrombosis

The Tobey–Ayer test is used for lateral sinus thrombosis by monitoring cerebrospinal
fluid pressure during a lumbar puncture.
19. Laryngocele arises from ?

a) Subglottis
b) Anterior commissure
c) Saccule
d) True glottis

Laryngocele is basically a herniation of the mucosa of the laryngeal ventricle (Morgagni's


ventricle) arising usually from the saccular region.

20. Angiofibroma bleeds profusely because:

a) It has multiple sites of origin


b) It lacks a capsule
c) Vessels lack a contractile component
d) None of the above

 Angiofibroma is made of vascular and fibrous tissues


 Vessel are endothelial lined spaces with no elastic or muscle coat
 Vessels lose ability to contract , thus bleed profusely.

21. Young's operation is done for:

a) Atrophic rhinitis
b) Vasomotor rhinitis
c) Antrachonal polyp
d) Allergic rhinitis

Young's operation is a surgery designed for the treatment of atrophic rhinitis, first
described by Austen Young in 1967.

22. Mikulicz cells are seen in:

a) Rhinoscleroma
b) Rhinosporidiosis
c) Rhinophyma
d) Rhinitis

The Mikulicz cell is a large macrophage with clear cytoplasm that contains the bacilli; this
cell is specific to the lesions in rhinoscleroma.
23. MC malignancy of maxillary antrum:

a) Muco-epidermoid Carcinoma
b) Adeno-cystic Carcinoma
c) Adeno Carcinoma
d) Squamous cell Carcinoma

M/c malignancy of paranasal sinus and nasal cavity → Squamous cell Carcinoma

Most common Paranasal sinuses involved → maxillary sinus

Associated with occupation:


Furniture industry → Adeno ca.

Nickel industry → Squamous cell ca.

MEDICINE JUNE 2018

1. A 44 year old female presenting to med opd with complaints of fatigue, weakness,
and shortness of breath with minimal activity. Her friends and family have told her
she appears pale, and combined with her recent symptoms she has decided to get
checked out. She also states that she has noticed her hair and fingernails
becoming extremely thin and brittle, causing even more concern. The patient first
started noticing these symptoms a few months ago and they have been getting
progressively worse. Upon initial assessment, her mucosal membranes and
conjunctivae are pale. Considering the above scenario the most sensitive and
specific test to diagnose this particular type of deficiency anemia is:

a. Serum iron levels


b. Serum ferritin levels
c. Serum transferring receptor population
d. Transferring saturation

 This is the classic presentation of Iron deficiency anemia


 Serum ferritin levels is the most sensitive and specific test to diagnose iron deficiency
anemia
 Under steady-state conditions, the serum ferritin level correlates with total body iron
stores; thus, the serum ferritin level is the most convenient laboratory test to estimate
iron stores.
2. True for von-willebrand disease

a. Normal partial thromboplastin time


b. Decreased platelets
c. Normal prothrombin time
d. Normal bleeding time

The prothrombin time (PT) is normal in VWD (von-willebrand disease)

3. Which Is NOT true about iron deficiency anemia?

a. Hyper-segmented neutrophils
b. Microcytosis precedes hypochromia
c. MCHC < 50%
d. Commonest cause of anemia in India

 Hyper segmented neutrophils (more than 5 lobes of neutrophils) is seen in vitamin B12
deficiency.
 Iron deficiency anemia is the most common cause of anemia in India.
 It is characterized by microcytic hypochromic peripheral smear.
 MCHC < 50% Low MCHC values occur if you have anemia due to iron deficiency.

4. Severity of mitral stenosis is determined by:

a. Intensity of S1 heart sound


b. Diastolic murmur duration
c. Opening snap
d. Intensity of diastolic murmur

 Since there is mitral stenosis, more time for blood to enter LV.
 Therefore, increased transient time, so increase duration of murmur
 ***Intensity of S1 sound can be loud or soft. If sound is soft it tells us about calcified
mitral valves but not of severity.
5. Malignant hypertension can lead to all of the following EXCEPT

a. Hypertensive retinopathy
b. Respiratory failure
c. Renal failure
d. Stroke

Emergencies include

 CNS INVOLVEMENT hypertensive encephalopathy (headache, irritability, confusion,


and altered mental status due to cerebrovascular spasm), intracranial hemorrhage.
 RENAL INVOLVEMENT hypertensive nephropathy (hematuria, proteinuria, and acute
kidney injury due to arteriolar necrosis and intimal hyperplasia of the interlobular
arteries),
 MALIGNANT HYPERTENSION Encephalopathy or nephropathy accompanying
hypertensive retinopathy has historically been termed malignant hypertension.
 CARDIOVASCULAR aortic dissection, preeclampsia-eclampsia, pulmonary edema (
acute LVF) unstable angina, or myocardial infarction.

6. Which is the most common arrhythmia:

a. Inappropriate sinus tachycardia


b. Junctional premature complexes
c. Atrial fibrillation
d. Atrial flutter

Atrial fibrillation is the most common sustained arrhythmia, increases with age, and
presents with a wide spectrum of symptoms and severity Paroxysmal, persistent, and
permanent forms require very individualized approaches to management.

7. Preferred biochemical marker(s) in patients presenting with myocardial infarction:

a. Myoglobin
b. Cardiac specific Troponins
c. CK-MB
d. All of the above

Cardiac troponin I and T is the preferred markers of myocardial injury. It is highly specific to
cardiac tissue and accurately diagnoses myocardial infarction with a history of ischemic pain
or ECG changes reflecting ischaemia.
8. On ECG, ST segment elevation is seen in all of the following conditions EXCEPT:

a. Left ventricular aneurysm


b. Acute pericarditis
c. Myocardial infarction
d. Hypocalcemia

ST Elevation Myocardial Infarction (STEMI)


Pericarditis
Cardiac Ventricular Aneurysm
Brugada Syndrome
Left Bundle Branch Block
Stroke
Intracranial Raised Intracranial Pressure
Intracranial Hemorrhage
Abdominal Peritonitis
Digoxin
Drugs Isoprenaline
Quinidine
Procainamide
Hypothermia
Metabolic Hyperventilation
Hyperkalaemia
Other Spinal Cord injury
Pulmonary Embolism

9. Most common cause of death in patient on chronic hemodialysis:-

a. Uremia
b. Infection
c. Cardiovascular disease
d. Malnutrition

Cardiovascular disease is the leading cause of death in dialysis patients and sudden death
(SD) represents a significant proportion of overall mortality in both hemodialysis (HD) and
peritoneal dialysis (PD) patients.
10. True statements regarding Gluten sensitive enteropathy is/ are -

1. Associated with HLA B27


2. Due to auto antibody against brush border epithelium
3. Anti-tissue transglutaminase antibody is Screening test
4. Large intestinal biopsy is Confirmatory test

a. 2, 3 and 4 are true


b. 1, 2 , 3 and 4 are true
c. 2 and 3 are true
d. 1, 2 and 3 are true

Gluten enteropathy/Celiac Sprue

 It is associated with HLA-DQ2


 Due to auto Ab against brush border epithelium which is present in the gut mucosa
 Clinical Feature - Osmotic diarrhea
 Auto ab: Anti tissue transglutaminase Ab - Screening test
 Small intestinal mucosal Biopsy - Confirmatory test
 Treatment:
 No 'BROW (Barley, Rye, Oats, Wheat)
 Only maize is allowed.

11. In gastric outlet obstruction in a peptic ulcer patient, the site of obstruction is
most likely to be:

a. Antrum
b. Duodenum
c. Pylorus
d. Fundus

 Gastric ulcer leads to gastric outlet obstruction causing pyloric stenosis showing HOUR
GLASS STOMACH.

12. Gold standard method of diagnosing celiac disease is

a. Small bowel biopsy


b. Anti-endomysial antibodies
c. Blood picture
d. Biochemical test
 Best screening test: Anti TTG antibody
 Confirmatory / Gold standard: Small Bowel biopsy.

13. In plasmodium vivax malaria, relapse in caused by:

a. Sporozoite
b. Schizont
c. Hypnozoite
d. Gametocyte

Plasmodium vivax malaria is characterized by the presence of dormant liver-stage


parasites, called hypnozoites, which can cause malaria relapses after an initial attack.

14. A 39 year old male presents with fever, cough and anorexia. He is sputum positive
for AFB. In order to consider this as a case of XDR-TB, which of the following
should he be resistant to ?

a. Resistance to at least INH & rifampicin +/- other drugs


b. Resistance to any of the 3 first line drugs
c. Resistance to all 1st line drugs & any 3 second line injectable drugs
d. Resistance to at least INH & rifampicin + any quinolones + at least 1 injectable 2nd
line drug

XDR -TB - resistant to :

 INH & rifampicin


 Any fluoroquinolone
 1 of 3 injectable drug (Amikacin, kanamycin or Capreomycin)
 MDR-TB - Resistance to INH & Rifampicin.

15. Acromegaly is associated with all of the following EXCEPT:

a. Protruding supraorbital ridges


b. Increased heel pad thickness
c. Diabetes mellitus
d. Muscle hypertrophy
16. Vitamin B12 deficiency can lead to all EXCEPT:

a. Myopathy
b. Peripheral neuropathy
c. Optic atrophy
d. Myelopathy

Vitamin B12 deficiency causes :

 Peripheral neuropathy
 Optic atrophy
 Myelopathy But Not Myopathy
 SACD spinal cord Sub-acute combine degeneration of spinal cord).
17. You are an intern, posted in the department of Internal Medicine ward. You are
seeing the reports of a 47-year-old female who had history of AGE & having
neurological manifestations. Investigations revealed significant elevation of
protein & other normal values in CSF. This characteristic dissociation in CSF is a
feature of ?

a. Multiple sclerosis
b. GB syndrome
c. TB meningitis
d. Multiple myeloma

 Guillain-Barré syndrome, sometimes known as GBS, is a rare but serious autoimmune


disorder in which the immune system attacks healthy nerve cells in the peripheral
nervous system (PNS). This leads to weakness, numbness, and tingling. It can
eventually result in paralysis.
 Patients with GBS have an elevated CSF protein level (>400 mg/L), with normal CSF
cell counts. Elevated or rising protein levels on serial lumbar punctures and 10 or fewer
mononuclear cells/mm3 strongly support the diagnosis.

18. A 34-year-old woman presented with complaints of sudden onset headache which
she described as a characteristic 'thunderclap headache'. She has no significant
medical history. On examination, her pupils were dilated bilaterally. These findings
are consistent with the diagnosis of?

a. Meningitis
b. Brain stem encephalitis
c. Acute ischemia of midbrain
d. Acute aneurismal hemorrhage

Thunderclap headache (TCH) is a term initially introduced to describe the apoplectic onset
of a headache that begins suddenly, without warning, and peaks with severe intensity
within seconds.

 SAH → Thunder clap headache


 Cause → Rupture of berry's aneurysm
 If post. communicating artery rupture → Involves oculomotor Nerve → So Dilated
pupils.
19. A 44 year old man presents to emergency room with Progressive atrophy and
weakness of hands. On further examination he is found to have brisk jaw jerk,
Fasciculations, spasticity of lower leg and Bulbar palsy. In the given clinical
scenario Upper motor neuron lesion is characterized by which of the following?

a. Weakness and spasticity


b. Fasciculation's
c. Rigidity
d. Muscle atrophy

 The most likely diagnosis in the given clinical scenario isAmyotrophic lateral sclerosis
(ALS).
 Amyotrophic lateral sclerosis (ALS)is the Most common form of motor neuron disease.
 The diagnosis is based on clinical criteria in which there is bothUpper and lower motor
neuron signs with progressive disease.
 Spasticity is the velocity-dependent increase in muscle tone due to the exaggeration
of stretch reflex. It is only one of the several components of the upper motor neuron
syndrome.

20. In restrictive lung disease:

a. FVC is high
b. FEV1 is high
c. FEV1/FVC is high
d. All of the above

The FEV1/FVC ratio can be higher than normal, This is because it is easy for a person with
a restricted lung (e.g fibrosis) to breathe out quickly, because of the high elastic recoil of the
stiff lungs.

21. A 49 year old man, presented with 10 days history of progressive worsening
dyspnea on exertion, A chest radiograph confirmed a large left pleural effusion.
Which of the following feature and finding you won't see in this case ?

a. Horizontal fluid level


b. Low lung volume
c. Muffled heart sound
d. Decreased chest movements

Horizontal fluid level is characteristic feature of Hydropneumothorax.


Features of Pleural Effusion -

 Blunting of costophrenic angle


 Low lung volumes
 ↓ chest movement
 Dullness on percussion
 Ellis curve
 Muffled Heart sound is characteristic feature of → Pericardial effusion but can be
associated with pleural effusion also.

22. Deficiency of which vitamin causes subacute combined degeneration of spinal


cord?

a. Vitamin B1
b. Vitamin B6
c. Vitamin B2
d. Vitamin B12

Subacute combined degeneration of the spinal cord is a neurological complication of


vitamin B12 (cobalamin) deficiency. A deficiency of vitamin B12 can occur as a result of
nutritional deficiency, reduced absorption due to altered gastrointestinal anatomy or function,
or due to the intake of certain drugs.

23. Obstructive lung function is seen in which of the following condition?

a. Obesity
b. Kyphoscoliosis
c. Pleural effusion
d. Asthma

Obstructive lung function is seen in

 COPD
 Asthma
 Bronchiectasis
 α1-antitrypsin deficiency.
24. In which of the following conditions there is an increase in lung diffusion
capacity?

a. Emphysema
b. Idiopathic pulmonary fibrosis
c. Alveolar haemorrhage
d. Pulmonary oedema

DLCO(diffusing capacity of the lungs for carbon monoxide )is increased in alveolar
hemorrhage whereas it decreases in pulmonary edema, idiopathic pulmonary fibrosis,
emphysema.

25. In the given ECG widespread concave ST elevation and PR depression is present
throughout the precordial (V2-6) and limb leads (I, II, aVL, aVF).There is reciprocal
ST depression and PR elevation in aVR. What is the diagnosis ?

a. Acute myocardial infarction


b. Pericarditis
c. Digoxin toxicity
d. Hypertrophic obstructive cardiomyopathy

 A concave ST segment elevation is the classical pattern of ECG pericarditis.


 A convex ST segment elevation is the classical pattern of ECG MI.
26. A 59-year-old woman presented with a six-week history of left facial weakness, left
hearing loss, and left facial numbness, and one-week history of headaches, gait
disturbance, and dizziness. She has no significant past medical history. MRI
revealed contrast-enhancing dural-based left middle cranial fossa mass with
extension into the internal auditory canal and cerebral edema which leads to the
suspicion of intracranial metastasis The most common site of intracranial
metastasis is from primary carcinoma of ?

a. Breast
b. Lungs
c. Stomach
d. Testes

MC brain tumor metastasis, is arising from

 Oat cell Ca lung/small cell Ca lung


 Ca Breast
 M. Melanoma
MC primary Brain tumor → Glioma

MC benign brain tumor → Meningioma

SURGERY JUNE 2018

1. Which of the following is not seen in massive blood transfusion?

a. DIC
b. Hypothermia
c. Hypercalcemia
d. Thrombocytopenia

 Massive blood transfusion can cause DIC, Hypothermia, Thrombocytopenia


 Because of the use of chelating agents in blood that is transfused hypocalcemia is
caused and not Hypercalcemia.

2. In traumatic cases, shock is most likely due to:-

a. Injury to intra-abdominal solid organ


b. Head injury
c. Septicemia
d. Cardiac failure
5 Causes of massive bleeding or shock:

 Injury on the floor (external bleed)


 Injury to Thorax
 Injury to Abdomen
 Injury to Retroperitoneal organs
 Fracture Of long bone
Out of these most common cause is injury to intra-abdominal solid organ.

3. Most common site of peripheral aneurysm?

a. Brachial artery
b.Popliteal artery
c. Radial artery
d. Femoral artery

 MC site of peripheral aneurysm: Popliteal aneurysm


 Most common site of splanchnic aneurysm is splenic artery
 Most common site of visceral arterial aneurysm is splenic artery
 MC vessel involved in aneurysm: Circle of willis
 MC location of extra - cranial aneurysm: Aorta >Iliac > Popliteal > Femoral (AIPE)
 MC site of extra- cranial arterial aneurysm is infrarenal aorta
 Degenerative aneurysms (caused by atherosclerosis) are MC AAA (abdominal aortic
aneurysm) (90%) .

4. Best way to localize extra-adrenal pheochromocytoma:-

a. X-ray
b. Nucleotide scan
c. VMA excretion
d. Clinical examination

IOC for extra adrenal pheochromocytoma is MRI > MIBG scan(aka-nucleotide scan)

5. Grey-Turner's sign in seen in:-

a. Acute appendicitis
b. Acute pancreatitis
c. Acute cholecystitis
d. Acute hepatitis
In acute pancreatitis, bleeding into fascial planes can produce bluish discoloration of
the flanks (known as Grey turner's sign) or umbilicus (Cullen sign)

6. All of the following are features of Zollinger-Ellison syndrome except:

a. Intractable peptic ulcers


b. Severe diarrhea
c. Arise from beta cell tumors of pancreas
d. Very high acid output

 ZES( Zollinger-Ellison syndrome ) aka Gastrinoma - arise from G- cells/ Gastrin


producing cells
 Has increased gastrin production causing increased acid production- leads to peptic
ulcer disease
 Causes of diarrhea:
 Acid produced reaches duodenum and inactivate pancreatic enzymes(works only in
alkaline ph) causing Malabsorption
 Acid reaches proximal jejunum and causes irritation- leads to jejunal secretion.

7. Most common type of gastric sarcoma:-

a. Lipoma
b. Glomus tumor
c. Leiomyosarcoma
d. Leioblastoma

 Leiomyosarcoma is the most common type of gastric sarcoma

8. Most common type of intussusception:-

a. Colocolic
b. Ileoileal
c. Ileocolic
d. Ileal

 Most common type of intussusception is ileocolic


 Most common type of intussusception in adults is colocolic.
9. A patient after heavy drinking of alcohol presents with multiple episodes of
vomiting and hematemesis. Most likely diagnosis could be:-

a. Mallory-Weiss syndrome
b. Esophageal carcinoma
c. Achalasia cardia
d. Boerhaave's syndrome

 Mallory- Weiss syndrome characterized by longitudinal partial tear


(mucosa+submucosa) Most common site is cardia and source of bleeding for
hematemesis is left gastric artery.
 Seen in alcoholic males due to forceful and repeated vomiting or Retching

10. The treatment of choice for inguinal hernia in infants is:-

a. Herniotomy
b. Herniorrhaphy
c. Truss
d. Hernioplasty

 TOC for congenital hydrocele and inguinal hernia in infant is Herniotomy


 Mesh is not used in infants because there will be growth in the body of infants while
the mesh wont grow.

11. Serum acid phosphate is raised in:-

a. Osteosarcoma
b. Prostatic carcinoma
c. Paget's disease
d. Hyperparathyroidism

Elevated serum levels of acid phosphatase are seen in patients with carcinoma of the
prostate that has extended beyond the prostatic capsule. Patients with prostatic
carcinoma still confined within the capsule usually have a normal serum acid phosphatase
level.
12. A 61 year old patient undergoes TURP, after 3 days patient develops altered
sensitiveness and drowsiness. Most probable diagnosis is:-

a. Hypernatremia
b. Hyponatremia
c. Stroke
d. Meningitis due to spinal anesthesia

TURP syndrome (Dilutional hyponatremia or water intoxication)

 TURP syndrome (Dilutional hyponatremia or water intoxication) resulting from a


hypervolemic, hyponatremic state due to absorption of the hypotonic irrigating
solution.
 Clinical Features: Nausea, Vomiting, confusion, hypertension, bradycardia, & visual
disturbances.

13. 54 year old female known case of multinodular goitre now complaints of sudden
increase in size of swelling and mild pain over the swelling. There is no history of
dysphagia, dyspnea. The patient complaints of mild hoarseness of voice. No
lymph nodes palpable. Malignancy in this patient is most often:-

a. Follicular carcinoma
b. Papillary carcinoma
c. Anaplastic carcinoma
d. Medullary carcinoma

Follicular ca: -

 Seen in iodine deficient areas


 M/C malignancy in long standing goiter
 M/C malignancy in multinodular goiter.

14. Paget's disease of the nipple is:-

a. Infection
b. Dermatitis
c. Neoplasia
d. Hypopigmentation

Paget disease (PD) is defined as the presence of neoplastic cells of glandular


differentiation, interspersed between keratinocytes of nipple epidermis, and constitutes
approximately 1% of breast cancer cases.
15. Most common parotid tumor:

a. Metastasis
b. Adenoid cystic carcinoma
c. Warthin's tumor
d. Pleomorphic tumor

 The most common tumor of the parotid gland is the pleomorphic adenoma,
which represents about 60% of all parotid neoplasms.
 MC neoplasm of salivary gland in children: Hemangioma
 MC neoplasm of salivary gland: Pleomorphic adenoma
 MC malignant tumor of salivary gland: Mucoepidermoid carcinoma
 MC malignant tumor of salivary gland in children: Mucoepidermoid carcinoma
 MC malignant tumor of minor salivary glands: Adenoid cystic carcinoma
 Best diagnostic modality for parotid swelling: FNAC
 Open incisional biopsy is contraindicated due to tumor cell implantation and formation
of parotid fistula
 Best imaging investigation for salivary gland neoplasms: MRI

16. Commonest site of branchial cyst:-

a. Lower 1/3 sternomastoid on anterior border


b. Lower 1/3rd sternomastoid on posterior border
c. Upper 1/3rd sternomastoid on anterior border
d. Upper 1/3rd sternomastoid on posterior border

A branchial cleft cyst (BCC) commonly presents as a solitary, painless mass in the neck of a
child or young adult. They are most commonly located along the anterior border and the
upper third of the sternocleidomastoid muscle in the anterior triangle of the neck.

17. Lucid interval is classically seen in:-

a. Intracerebral hematoma
b. Acute subdural hemorrhage
c. Chronic subdural hemorrhage
d. Epidural hemorrhage

THE LUCID interval following head trauma and unconsciousness is described classically
in epidural hematomas. The historic emphasis placed on the lucid interval in cases of
extradural hematoma has made this one of the best-remembered signs of the syndrome.
18. Marjolin's ulcer is:-

a. Squamous cell carcinoma from scar


b. Adenoma of scar
c. Tubercular ulcer
d. Amoebic ulcer

A Marjolin ulcer is a rare and aggressive type of skin cancer that grows from burns, scars,
or poorly healing wounds. It grows slowly, but over time it can spread to other parts of your
body, including your brain, liver, lungs, or kidneys. Most Marjolin ulcers are cancerous
and form squamous cancer cells in the upper layers of your skin. Some Marjolin ulcers
may also form as basal cell tumors which form in deeper layers of your skin.

19. In treatment of hand injuries, the greatest priority is:-

a. Repair to tendons
b. Repair of skin cover
c. Repair of nerves
d. All

Sequence of Repair in Hand Injuries (BE FAN OF VEINS)

 Bone shortening of stabilization/ fixation


 Extensor tendon repair
 Flexor tendon repair
 Arterial anastomoses
 Nerve repair
 Venous anastomosis
 Skin/ wound closure

20. A boil is due to staphylococcal infection of:-

a. Hair follicle
b. Sweat gland
c. Subcutaneous tissue
d. Epidermis

The most common type of staph infection is the boil, a pocket of pus that develops in
a hair follicle or oil gland. The skin over the infected area usually becomes red and
swollen.
21. Pendred syndrome is due to a defect in:-

a. Chromosome 7p
b. Chromosome 7q
c. Chromosome 8p
d. Chromosome 8q

Pendred syndrome can be caused by changes, or mutations, in a gene called SLC26A4


(formerly known as the PDS gene) on chromosome 7. Because it is a recessive trait, a
child needs to inherit two mutated SLC26A4 genes—one from each parent—to have
Pendred syndrome.

22. Cock's peculiar tumor is:-

a. Papilloma
b. Infected sebaceous cyst of scalp
c. Cylindroma
d. Squamous cell carcinoma

Cocks Peculiar Tumour

 Infected or ulcerated sebaceous cyst of the scalp


 Resembles fungating epithelioma.

23. Mondor's disease is:-

a. Lymphangitis of mammary lymphatic's


b. Multiple breast cysts
c. Eczema by nipple and areola
d. Thrombophlebitis of superficial veins of breast

Mondor's disease is a condition characterized by thrombophlebitis of the superficial


veins of the breast and anterior chest wall. Initially this condition presents with rapid
development of a subcutaneous red cord-like lesion that causes pain at an early stage and
subsequently becomes a painless fibrous band.
24. What is the procedure shown in the image?

a. Mile's procedure
b. Frey's procedure
c. Scanlon's modification surgery
d. Anderson-Hynes operation

The Anderson-Hynes pyeloplasty (AHP) is a surgical technique used in the management


of pelvi-ureteric junction obstruction. This operation has been performed for over six
decades and has etched its name in the annals of urology.

25. Oliguria is defined as:-

a. Absence of urine production


b. More than 900 ml of urine excreted in a day
c. 600 ml to 700 ml of urine excreted in a day
d. Less than 300 ml of urine excreted in a day

Oliguria is defined as urinary output less than 400 ml per day or less than 20 ml per
hour and is one of the earliest signs of impaired renal function.
FMGE DEC 2018
ANATOMY
1. A 55-year-old male is brought to the Neurology OPD by his wife. She complains
that something is wrong with her husband and they were referred to this center by
a local doctor. The doctor's note read:- Patient unable to repeat words spoken,
though is able to understand what is told & nods head accordingly. When asked to
name people and write, the patient found difficulty. Referred higher center for CT
brain. You being the intern on duty, would inform the CMO it could be a lesion in
which area of the brain?

a) Arcuate fasciculus
b) Anterior commissure
c) Corpus callosum
d) Fornix

 Arcuate fasciculus connects Wernicke's area present in the posterior superior temporal
gyrus of the dominant hemisphere (sensory area of speech) to Broca's speech area
present in the prefrontal & premotor facial region of the cerebral cortex to integrate
language understanding & skilled motor function.
 Damage to Wernicke's area = Wernicke's aphasia (lack meaning)
 Damage to Broca's area = motor aphasia (vocal system couldn't emit words)
 Damage to arcuate fasciculus = conduction aphasia

2. Mammillary body is a structure on the undersurface of the brain, developing from


the diencephalon, that forms a part of the limbic system. They receive afferent
fibers via the:-

a) Corpus callosum
b) Thalamus
c) Pituitary gland
d) Fornix

 Mammillary body receives afferent fibers via Fornix


 C-shaped bundle of nerve fibers which acts as major output tract of the hippocampus
 They are association fibers(connects one brain center to another)
 It is a part of limbic system (Papez circuit)
 Papez circuit for memory consolidation, emotions & behavior

3. The parotid gland is a paired salivary gland contributing serous secretions. Its
duct opens at the level of the:.

a) First mandibular molar


b) Second mandibular molar
c) First maxillary pre molar
d) Second maxillary molar
Parotid duct/Stenson's duct passes through the buccinator muscle and pierces buccal fat,
opening into the vestibule of the mouth at the parotid papilla, which lies opposite to the 2nd
maxillary (upper) molar tooth.

4. Trismus is due to spasm of which of the following muscle (FMGE Dec 2018)

a) Buccinator
b) Lateral pterygoid
c) Medial pterygoid
d) Mentalis

 Trismus → spasm of muscles of mastication especially medial pterygoid muscle.


[classical example of tetanus]
 Lateral pterygoid + depressor of mandible pull mandible antero-inferiorly and open the
mouth
 Antagonize (MTM muscles)
 Tetanus neonatorum → spasm of masseter
 MTM- elevators of mandible
 M - Medial pterygoid → inserted to ramus of mandible from inner side
 T Temporalis → inserted into coronoid process of mandible
 M -Masseter → inserted to ramus of mandible from outside

5. Which branch of facial nerve supplies muscles of lower lip (FMGE Dec 2018)

a) Temporal
b) Cervical
c) Buccal
d) Mandibular

The marginal mandibular branch of the facial nerve passes forward beneath the platysma
and depressor anguli oris, supplying the muscles of the lower lip and chin, and
communicating with the mental branch of the inferior alveolar nerve.

 TEMPORAL BRANCH-Supply upper part of orbicularis oculi and frontalis


 ZYGOMATIC BRANCH-Supply lower part of orbicularis oculi [ helps to shut eyelid
tightly]
 BUCCAL BRANCH-Supply upper orbicularis oris, zygomaticus minor/major

6. Mediastinal surface of the right lung is related to:-

a) The Azygos vein


b) The Right Lymphatic Duct
c) The Aorta
d) The Trachea
Azygos vein > > Trachea (because heart & its blood vessels should be preferred first as an
answer as they cause direct impression on lungs).

The Mediastinal surface of

Right lung contains Left lung contains


 Heart (right atrium)  Heart (left ventricle)
 Superior vena cava  Aortic arch
 Inferior vena cava  Thoracic aorta
 Azygos vein  Esophagus
 Esophagus  Left subclavian artery & vein arch over related
 Right subclavian artery & vein arch over and to the superior lobe of the left lung.
are related to the superior lobe of the right
lung.

7. Sternum is attached to scapula via (FMGE Dec 2018)

a) Manubrium
b) Clavicle
c) First rib
d) Second rib

 By forming stenoclavicular joint & Acromioclavcular joint clavicle connects


sternum (Axial skeleton) to scapula (Appendicular skeleton).
 Clavicle transmits the weight / force of appendicular skeleton to axial skeleton.
acromioclavicular ligament is important in this.
 That's why clavicle s most commonly fractured bone in body [mc = middle third]

8. Retraction of scapula at sternoclavicular joint is done by (FMGE Dec 2018)

a) Serratus anterior
b) Trapezius
c) Subscapularis
d) Supraspinatus

Retractors of scapula:

 Trapezius
 Rhomboid major
 Rhomboid minor
Protraction of scapula:

 Pectoralis minor
 Serratus anterior (damage causes winging of scapula)

9. Winging of scapula is due to paralysis of (FMGE Dec 2018)

a) Trapezius
b) Serratus anterior
c) Latissimus dorsi
d) Pectroralis major

The most common cause of scapular winging is paralysis of the serratus anterior muscle
due to the injury of the long thoracic nerve.

10. During the embryological development, the midgut rotates a total of _____in the
intrauterine life.

a) 270 degree clockwise


b) 270 degree anticlockwise
c) 360 degree clock wise
d) 360 degree anticlockwise
During development, midgut undergoes rapid phase of growth in which loops of midgut
herniate outside the abdominal cavity of fetus & protrude into umbilical cord
This is PHYSIOLOGICAL UMBILICAL HERNIA (occurs at 6 weeks of IUL)
At week 10, it comes back to abdominal cavity.
When midgut s Herniating into umbilicus, it follows superior mesenteric artery Thus, superior
mesenteric artery becomes axis of rotation.
While entering umbilical cord there is:

 High point: duodenojejunal junction (small intestine).


 Low point: ileocecal junction (large intestine)
 Which rotates 1st 90 degree anti- clock wise resulting in small intestine to right side &
large intestine to left side.
 While coming back to abdominal cavity, there would be another 180 degree anti -
clock wise rotation.
 Total of 270 degree anti-clock wise rotation occurs during development of
midgut in IUL
 Duodenum become C shaped and have 4 parts and colon will also have 4 parts
(Ascending, transverse & descending & sigmoid colon).

11. Vasa brevia is the other name for which of the following arteries carried by the
gastrosplenic ligament?

a) Long gastric arteries


b) Short gastric arteries
c) Iliea vasa recta
d) Jejunal vasa recta

 Splenic artery (branch of celiac trunk) supply greater curvature of stomach by it following
branches:
 Short gastric arteries which are called as vasa brevia. (carried by Splenogastric
ligament)
 Long gastric artery called left gastroepiploic artery.
 Jejunal vasa recta (long arteries) & ileal vasa recta (short) are straight arteries coming
from arcades in mesentery of jejunum & ileum.

12. In a healthy adult female, the tail of pancreas lies in lienorenal ligament derived
from peritoneum. This is another name for which of the following ligaments?

a) Hapato-gastric
b) Hepato- duodenal
c) Gastro- splenic
d) Spleno- renal
 Splenorenal ligament, also known as the lienorenal ligament is a peritoneal ligament. It
represents the dorsal most part of dorsal mesentery and forms part of the lateral border
of the lesser sac. It is continuous with the gastrosplenic and phrenicocolic ligaments
 Tail of pancreas lies in Spleno renal ligament.
 Gastro splenic short gastro and left gastro-omental vessels.

13. Bleeding vessels in hemorrhoids is (FMGE Dec 2018)

a) Superior rectal artery


b) Superior rectal vein
c) Middle rectal artery
d) Middle rectal vein

External hemorrhoids:

 Hemorrhoids occurring at anal verge (distal boundary of anal canal below pectinate
line) Normally have low tendency to bleed on straining at stool.
 Bleeding comes from inferior rectal vein.

Internal hemorrhoids:

 Occur inside the rectum (above pectinate line) have high tendency to bleed.
 Bleeding from "superior rectal vein"
 Prolapsed hemorrhoids are internal hemorrhoids that pass outside anal canal & form
lump, which may undergo thrombosis & become painful.

14. Foot eversion is caused by (FMGE Dec 2018)

a) Extensor digitorum
b) Peroneus longus
c) Tibialis anterior
d) Tibialis posterior

 In human anatomy, the peroneus longus (also known as fibularis longus) is a


superficial muscle in the lateral compartment of the leg, and acts to evert and
plantarflex the ankle.
 Eversion of the Foot (tilting of the sole of the foot away from the midline):
Performed by the fibularis brevis and fibularis longus.
 Inversion of the Foot (tilting of the sole of the foot inwards towards the midline):
Performed by the tibialis posterior and tibialis anterior.
15. Tibial and common peroneal nerve supplies the muscle (FMGE Dec 2018)

a) Gracilis
b) Biceps femoris
c) Adductor longus
d) Adductor magnus

Biceps femoris is a muscle of the posterior compartment of the thigh, and lies in the
posterolateral aspect. It arises proximally by two 'heads', termed the 'long head'
(superficial) and the 'short head' (deep).
Nerve Supply

 Long head: tibial division of the sciatic nerve.


 Short head: common peroneal division of the sciatic nerve

PHYSIO DEC 2018

1. The ICF volume represents the fluid contained within the body's cells. This volume
cannot be measured directly but is calculated as the difference between the
measured TBW and the measured ECF volume. Potassium provides the osmotic
skeleton for the ICF and because water is freely diffusible into and out of the cell,
changes in the tonicity of the ECF are rapidly reflected by similar changes in ICF
tonicity. Which of the following has the least concentration in intracellular fluid?

a) Calcium
b) Magnesium
c) Potassium
d) Protein

 Calcium is kept very LOW in cytoplasm, hence intracellular fluid doesn't contain much
Ca2+
 Calcium is mostly stored in organelles inside cell
E.g. In sarcoplasmic reticulum of muscle cell etc.

2. Resistance against passive stretch is called?

a) Tone
b) Spasticity
c) Rigidity
d) Paratonia
Muscle tone is defined as the continuous and passive-partial contraction of the muscle or
the muscle's resistance to passive stretch during the resting state.

Spasticity:
Condition in which certain muscles are continuously contracted
This contraction causes stiffness/tightness of muscle & interfere with movement
Occurs in lesions of upper motor neurons.

Rigidity:
increase in muscle tone leading to resistance to passive movement throughout range of
motion
E.g. Cogwheel rigidity in parkinsonism

Paratonia:
Inability to relax muscles during muscle tone assessment.

3. The formation of the active endopeptidases from their inactive precursors occurs
only when they have reached their site of action, secondary to the action of the
brush border hydrolase. enterokinase. The powerful protein-splitting enzymes of
the pancreatic juice are secreted as inactive proenzymes. Enterokinase is an
activator of?

a) Trypsinogen
b) Trypsin
c) Chymotrypsin
d) Antitrypsin

 When first synthesized in pancreatic cells, the proteolytic digestive enzymes like
trypsinogen, chymotrypinogen & pro-carboxypeptia are in inactive forms( to prevent auto
proteolysis of pancreas)
 After secreted into intestine, they are activated:
 Trypsinogen is activated by enterokinase (secreted by intestinal mucosa)
 Trypsinogen → can also be autocatalytically activated by trypsin
 Chymotrypsinogen is activated by trypsin to form chymotrypsin.
4. Which structure connects the Broca's area and Wernicke area?

a) Arcuate fasciculus
b) Anterior commissure
c) Corpus callosum
d) Fornix

 Arcuate fasciculus connects Wernicke's area present in the posterior superior


temporal gyrus of the dominant hemisphere (sensory area of speech) to Broca's
speech area present in the prefrontal & premotor facial region of the cerebral cortex to
integrate language understanding & skilled motor function.
 Damage to Wernicke's area = Wernicke's aphasia (lack meaning)
 Damage to Broca's area = motor aphasia (vocal system couldn't emit words)
 Damage to arcuate fasciculus = conduction aphasia

5. Calcium binding proteins are proteins that participate in calcium cell signaling
pathways by binding to Ca2+, which plays an important role in many cellular
processes. Calcium-binding proteins have specific domains that bind to Ca2+ and
are known to be heterogeneous. One of the functions of calcium-binding proteins
is to regulate the amount of free (unbound) Ca2+ in the cytosol of the cell. The
cellular regulation of calcium is known as Calcium homeostasis. Which of the
following is not a calcium-binding protein?

a) Calbindin
b) Calmodulin
c) Troponin
d) Clathrin

Clathrin (does not bind Ca2+)

 On inside of cell membrane beneath coated pits, latticework of fibrillar protein is present
called clathrin
 Helps in pinocytosis

Calbindin

 Present in brush border of intestinal epithelial cells & transports Cat2into cell cytoplasm
 Amount of calcium absorption is directly proportional to quantity of calcium-binding
proteins

Troponin

 Troponin C - binds to Ca+2 ions = Strong affinity of troponin to calcium initiate


contraction process.
Calmodulin

 In place of troponin, smooth muscles have calmodulin


 Ca2+ inside the cell binds reversibly with calmodulin & activates myosin light chain
kinase resulting in a chain of events that cause muscle contraction

6. Histamine is secreted by:

a) Enterochromaffin cell
b) Parietal cell
c) Oxyntic cell
d) Chief cell

“Enterochromaffin-like (ECL) cells” are a population of cells that are found in the gastric pits
of the stomach luminal epithelium and secrete histamine. In response to gastrin released
by neighbouring G-cells, secreted histamine from ECL cells acts on parietal cells to stimulate
the release of gastric acid.

7. Reward centre in the brain is?

a) Amygdala
b) Thalamus
c) Hippocampus
d) Ventral tegmental area

 Reward center contains ventral tegmental area connected with nucleus accumbens
(through dopamine NT)
 Previously major reward centers are thought to be located along course of "medial
forebrain bundle"

8. Glucose is absorbed by?

a) Upper part of PCT


b) Ascending limb of LOH
c) Cortical Collecting duct
d) Medullary collecting duct

 Substances like glucose, AA, Na+2, H,0 is absorbed maximum from upper part of PCT
 As proximal tubule epithelial cells have:
 Large number of mitochondria to support powerful active transport processes (highly
metabolic)
 Have brush borders & provide extensive surface area for absorption
 As glucose is reabsorbed by secondary active transport, it is absorbed almost
all by proximal convoluted tubule.

9. Sperm remains in epididymis for_____hours?

a) 24
b) 48
c) 72
d) 96

 Sperms after forming in seminiferous tubules enter epididymis


 Sperms in seminiferous tubules & early portions of epididymis → non-motile & cannot
fertilize ovum
 After sperm had been for (18-24 hrs) in epididymis, sperms attain capability of motility.

10. What mediates Inverse stretch reflex

a) Golgi tendon
b) Muscle spindle
c) Unmyelinated C fibres
d) Dorsal Column

The Inverse Myotatic Reflex Involves Sensors of Muscle Force in the Tendon. Stretch
receptors called Golgi tendon organs are found within the collagen fibers of tendons and
within joint capsules. They are generally located in series with the muscle rather than the
parallel arrangement of the intrafusal muscle fibers.

11. Organelle having DNA is__________?

a) Mitochondria
b) Golgi complex
c) SER
d) RER

 The three organelles that contain DNA are the nucleus, mitochondria and
chloroplasts.
 Organelles are membrane-bound subunits within a cell -- analogous to organs in
the body -- that perform specific functions.
 The nucleus is the control center of the cell, and houses genetic information.
 The mitochondria and chloroplasts both produce energy, in animal and plant cells,
respectively.
12. All are Glucogenic hormones except?

a) ADH
b) Glucagon
c) Thyroxine
d) Glucocorticoids

Anti - diuretic hormone (ADH) / vasopressin :

 It is secreted from posterior pituitary


 Controls rate of water excretion into urine thus controlling concentration of water in body
fluids
 No effect on glucose

Glucogenic hormones are those which are capable of increasing blood glucose levels
1.Glucagon
2. Thyroxine
3. Glucocorticoids
4. Catecholamines,
5. Growth hormone etc.

13. Delivery of stimulus above threshold intensity leads to a constant amplitude of AP


and is known as:

a) All or none law


b) Electronic potential
c) Absolute refractory period
d) Relative refractory period

All or none principle -

The all-or-none law is a principle that states that the strength of a response of a nerve cell
or muscle fiber is not dependent upon the strength of the stimulus. If a stimulus is
above a certain threshold, a nerve or muscle fiber will fire.
14. Which part of thalamus is related to motor control?

a) Ventrolateral thalamus
b) Ventral posteromedial
c) Ventral posterolateral
d) Superior and inferior colliculi

Thalamus is sensory relay station for all sensations except olfaction


It is related to motor control also by :

 Ventrolateral thalamus
 Ventroanterior thalamus
 Centro median thalamus

15. Active form of Vitamin D is?

a) Calcitriol
b) Calciferol
c) Calcidiol
d) Ergocalciferol

Calcitriol is the active form of vitamin D, normally made in the kidney. It is also known
as 1,25-dihydroxycholecalciferol. It is a hormone which binds to and activates the vitamin
D receptor in the nucleus of the cell, which then increases the expression of many genes.
Calcitriol increases blood calcium (Ca2+) mainly by increasing the uptake of calcium from
the intestines.

16. Tyrosine kinase receptors are membrane-spanning proteins with large amino-
terminal extracellular domains bearing the ligand binding site, a juxtamembrane
domain, a protein kinase catalytic domain, and carboxyl-terminal tail. Which of the
hormone has 4 subunits and 2 units for tyrosine kinase receptor
binding?

a) Insulin
b) Glucagon
c) T3
d) ADH
The insulin receptor is an enzyme-linked receptor.

Insulin receptor
combination of 4 subunits
(held by disulfide linkages)

2 α subunits 2 β subunits

Lie entirely outside cell


membrane into cytoplasm Penetrate & protrude

Insulin binds to α-subunits → due to linkage, portions of α-subunits become auto-


phosphorylated → activates [ Tyrosine kinase & IRS]

17. In case a small stimulus causes more pain, it is called:

a) Hypersensitivity
b) Causalgia
c) Hyperalgesia
d) Allodynia

yperalgesia:
An increased sensitivity to feeling pain and an extreme response to pain. Hyperalgesia may
occur when there is damage to the nerves or chemical changes to the nerve pathways
involved in sensing pain.

 Hypersensitivity (also called hypersensitivity reaction or intolerance) refers to


undesirable reactions produced by the normal immune system, including allergies
and autoimmunity.
 Allodynia is a type of neuropathic pain (nerve pain). People with allodynia are
extremely sensitive to touch. Things that don't usually cause pain can be very painful.
These may include cold temperatures, brushing hair or wearing a cotton t-shirt.
 Causalgia is burning of type pain caused crushing, by injuries to nerves. (neuropathic
pain)
BIO-CHEM DEC 2018
1. Urea cycle occurs in:

a) Mitochondria
b) Cytoplasm
c) Both
d) None

In both ( first in mitochondria then in cytoplasm)- HUG

 Heme synthesis
 Urea cycle
 Gluconeogenesis

2. Start codon is:

a) UAA
b) UAG
c) UGA
d) AUG

 Start codon - AUG - synthesize methionine in eukaryote and formyl methionine in


prokaryote
 Stop codon -UAA ,UAG , UGA

3. Melanin is synthesized from:

a) Tyrosine
b) Tryptophan
c) Phenyl alanine
d) Threonine

Melanin is a highly irregular heteropolymer consisting of monomeric units derived from the
enzymatic oxidation of the amino acid tyrosine. The process of melanin formation takes
place in specialized acidic organelles (melanosomes) in melanocytes.

4. Which of the following helps in wound healing?

a) Keratan sulfate
b) Dermatan sulfate
c) Hyaluronic acid
d) Chondroitin sulfate
Hyaluronic acid (hyaluronan) is a key component of the extracellular matrix and is known to
be involved in several mechanisms of the wound healing process. It has been shown to
improve and accelerate the healing process of chronic wounds.

5. Oxidative phosphorylation is inhibited by all EXCEPT:

a) CO
b) Antimycin A
c) Malonate
d) Thermogenin

Inhibitor of ETC - inhibit oxidation phosphorylation

 Complex 1- Rotenone, phenobarbitone, amobarbitate, piericidine A


 Complex 2- Malonate, TIFA (Thionyl tri fluoroacetate)
 Complex 3 - Antimycin A
 Complex 4 - CO, CN, H2S, sodium azide

Natural uncouplers:

 Thermogenin - responsible for non- shivering thermogenesis


 Thyroxin
Synthetic uncouplers - 2,4 DNP
6. Use of zinc in diarrhea is because it:

a) Reduces the risk, duration and severity of diarrheal episodes


b) Enhances immune response
c) Regulates intestinal transport & absorption of water and electrolytes
d) All

Zinc is required for:

 Protein synthesis
 Immune response
 Cell growth & differentiation
 Regulation intestinal transport & absorption of water and electrolytes
 Improve intestinal epithelium lining and increase enzyme in brush border epithelium
 Cell membrane transport and function
 For polyribosome
 More than 100 metalloenzyme require zinc example carbonic anhydrase alkaline
phosphates , lactate dehydrogenase
 Deficiency lead to acrodermatitis enteropathica - skin affected around cheek elbow
knee anus and mouth

 In diarrhea zinc is given along with ORS to reduce the risk , duration and severity.
7. Which is cardio protective?

a) HDL
b) LDL
c) IDL
d) VLDL

High-Density Lipoprotein :

 HDL is synthesized and secreted from both liver and intestine


 HDL transport cholesterol from peripheral tissue to liver this is know as reverse
cholesterol transport that why HDL is cardio protective.
 HDL concentrations are inversely related to the incidence of atherosclerosis
 HDL have APO A, CAND E
 HDL have LCAT (Lecithin cholesterol acyl transferase ) which convert cholesterol into
cholesterol ester and then it’s taken up by HDL
 LCAT is activate by APOA1
 HDL IS smallest in size max density , max phospholipid (lecithin),min triglyceride.

8. Adenosine receptor stability is because of extensive disulfide bonds formed


between:

a) Cysteine
b) Methionine
c) Arginine
d) Alanine

 Cysteine is unique among coded amino acids because it contains a reactive sulph-
hydryl group. Therefore, two cysteine residues may form a cystine (disulfide link)
between various parts of the same protein or between two separate polypeptide chains.
 No other amino acid given as options can form disulfide bond due to absence of -SH
group

9. Steps of PCR in sequence are?


a) Extend DNA, anneal primers, Denture DNA
b) Anneal primers, extend DNA, Denature DNA
c) Denature DNA, anneal primers, Extends DNA
d) Denature DNA, Extend DNA, Anneal primers

PCR is based on three simple steps required for any DNA synthesis reaction:
(1) denaturation of the template into single strands;
(2) annealing of primers to each original strand for new strand synthesis; and
(3) extension of the new DNA strands from the primers.
10. Genetic material is transferred from one bacterial to another by all EXCEPT:

a) Conjugation
b) Transduction
c) Transformation
d) Transfection

Transfection - DNA transfer into eukaryotic cell

 Conjugation -a natural microbial recombination process. During conjugation a donor


and a recipient come together, join by cytoplasmic bridges and transfer single stranded
DNA. Inside the recipient the DNA will combine with chromosome which is rare or it will
be remain free in case of plasmid
 Transduction - transfer of DNA from one bacteria to another bacteria via virus like
bacteriophage
 Transformation - transfer of DNA from dead bacteria or environment to another live
bacteria
 Lipofection - liposome mediated gene transfer. liposome is lipid molecule that can carry
nucleic acids

11. Glucose is reabsorbed in which part?

a) Early PCT
b) Henle loop
c) Collecting duct
d) Distal convoluted tubule

Part Reabsorption Secretion


 Glucose 100%  Bile salts
 Amino acid 100%  Hydrogen ion
PCT  Na along with glucose by  Creatinine
SGLT 2  Uric acid
 Chloride (manly in late  Drugs
part of PCT)
 Bicarbonate

 Descending limb - water  Hydrogen ion by Na H


 Ascending limb - Na k, PUMP
CI by 1NA 2CI 1k
LOOP OF HENLE  Cotransporter - Slight
reabsorption of cation
such as Mg Ca also
 5% sodium and chloride
DISTAL TUBULE by sodium chloride
cotransport

 Principle cell - reabsorb  Principle cell secrete


LATE DISTAL TUBE & sodium and water. potassium
COLLECTING DUCT  Intercalated cell - ions Intercalated cell-
reabsorb potassium ions hydrogen ions

12. Low glycemic index food is:

a) Easily digestible
b) Increases plasma glucose
c) Has slower absorption
d) Increase glycogen deposits

GLYCEMIC INDEX -glycemic index which measures the time course of postprandial glucose
concentrations from a graph

 Glycemic index of complex carbohydrate is low because of slow digestion and


slow absorption
 Glycemic index of carbohydrate is low when its combine with fat , protein and fiber
 Foods that have a low glycemic index are considered to be more beneficial since
they cause less fluctuation in insulin secretion.

PHARMA DEC 2018

1. Which of the following drugs is considered as first line therapy for the prophylaxis
of motion sickness in adults?

a) Promethazine
b) Ondansetron
c) Metoclopramide
d) Domperidone

As hyoscine is not given in the options, the answer is first generation Antihistaminics
Best Drug/ DOC for prophylaxis of motion sickness - Hyoscine (scopolamine).

First generation Antihistamines can cross the blood brain barrier and also have strong Anti-
cholinergic properties.
Example of a first generation Antihistamines – Promethazine.
2. Exenatide is a new drug used in diabetes mellitus. Mechanism of action of this
drug is:

a) Inhibition of DPP - 4
b) Release of insulin via acting as agonist of GLP-1 receptors
c) Inhibiting intestinal absorption of carbohydrates
d) Stimulation of PPAR-gamma

Mechanism of Action:
Exenatide is a GLP-1 receptor agonist released from the gut and acts to increase glucose-
dependent insulin secretion from pancreatic beta cells, suppress glucagon secretion,
delay gastric emptying, and reduce food intake. The binding of the drug to pancreatic GLP-1
receptors mediates these actions.

3. The compound prostaglandin has been indicated in multiple conditions is actually


a derivative of which of the following?

a) Stearic acid
b) Arachidonic acid
c) Linoleic acid
d) Linolenic acid

The prostaglandins (PG) are a group of physiologically active lipid compounds


called eicosanoids having diverse hormone-like effects in animals. Prostaglandins have been
found in almost every tissue in humans and other animals. They are
derived enzymatically from the fatty acid arachidonic acid.

4. Which of the following drug is contraindicated in a pregnant female?

a) Erythromycin
b) Amoxicillin
c) Ceftriaxone
d) Streptomycin

Some antibiotics are known to be teratogenic and should be avoided entirely during
pregnancy. These include streptomycin and kanamycin (which may cause hearing loss)
and tetracycline (which can lead to weakening, hypoplasia, and discoloration of long bones
and teeth).
5. Which of the following is the mechanism of action of methyl dopa?

a) Blockade of beta adrenergic receptors


b) Agonism of beta 2 receptors
c) Agonism of alpha 2 receptors
d) Blockade of alpha 2 receptors

Mechanism of Action
Alpha-methyldopa is converted to methyl norepinephrine centrally to decrease the
adrenergic outflow by alpha-2 agonistic action from the central nervous system, leading
to reduced total peripheral resistance and decreased systemic blood pressure.

6. A 12 months old baby is brought to your clinic. If you have to give a cyclopegic
agent for this pediatric patient, which of the following drugs would you prescribe?

a) Atropine oral
b) Atropine eye ointment
c) Tropicamide eye drops
d) Phenylephrine eye drops

In children the tone of ciliary muscle is very high, so we have to give drug which is a strong
cycloplegic drug-Atropine.

Atropine is given in the form of eye ointment because it would then have only
localised effect in the eyes. If we give it in form of eye drops, it can be absorbed through
nasolacrimal duct, and there will be risk of hyperthermia.

Cycloplegic DOC in 1yr old child - Atropine (longest acting cycloplegic)


Cycloplegic DOC in Adults- Tropicamide (shortest acting cycloplegic)

7. Pilocarpine produces:-

a) Active miosis
b) Active mydriasis
c) Passive miosis
d) Passive mydriasis
Pilocarpine is a drug that acts as a muscarinic receptor agonist. It acts on a subtype of
muscarinic receptor (M3) found on the iris sphincter muscle, causing the muscle to contract -
resulting in pupil constriction (miosis). Pilocarpine also acts on the ciliary muscle and causes
it to contract.

8. Which of the following drugs is a direct inhibitor of clotting factor Xa?

a) Apixaban
b) Argatroban
c) Fondaparinux
d) Aspirin

The factor Xa inhibitors that are currently commercially available include rivaroxaban,
apixaban, betrixaban, and edoxaban. These drugs bind to factor Xa and prevent the
formation of thrombin by interrupting the extrinsic and intrinsic coagulation cascades.

9. Which of the following drug is a protein synthesis inhibitor?

a) Linezolide
b) Nevirapine
c) Indinavir
d) Ceftriaxone

Following category of drugs are protein synthesis inhibitors (in Bacteria):-

 Tetracyclines - doxycycline, minocycline


 Chloramphenicol
 Macrolides - Erythromycin, Azithromycin, etc.
 Linezolid
 Clindamycin
 Aminoglycoside - streptomycin, Gentamicin, Kanamycin, etc.
 Ceftriaxone → cephalosporin → inhibits cell wall synthesis
 Nevirapine → NNRTI → inhibits reverse transcriptase enzyme
 Indinavir → Protease inhibitor used in highly Active anti- retroviral therapy to treat
HIV/AIDS.

10. Drug of choice for prophylaxis of meningococcal meningitis in a pregnant female


after contact with a case is:-

a) Ciprofloxacin
b) Ceftriaxone
c) Cefalexin
d) Rifabutin
Important points to remember about Meningococcal Meningitis

 Doc for Empirical Treatment → ceftriaxone


 Doc for Definitive Treatment → Penicillin G
 Doc for Prophylaxis → ciprofloxacin
 Doc for Prophylaxis in pregnancy → ceftriaxone

11. Dose of oseltamivir for chemoprophylaxis in an infant is:-

a) 3 mg / kg once daily
b) 20 mg once a day
c) 20 mg twice day
d) 20 mg thrice day

Age Weight Dose Frequency Duration


<1yr 3mg/kg Twice daily 5 days
Oral oseltamivir for
>1yr <15kg 30mg
treatment of influenza
15-23kg 45mg
23-40kg 60mg
>40kg 75mg
3month-1yr 3mg/kg Once daily 7 days

Oral oseltamivir for >1yr <15kg 30mg


Chemoprophylaxis 15-23kg 45mg
23-40kg 60mg
>40kg 75mg
12. Purpose of adding zinc to insulin in commercial preparations for treatment of
diabetes mellitus is:-

a) To make it long acting


b) To make it short acting
c) To make is fast acting
d) Addition of zinc make the insulin effective orally

Insulin exists in two forms:-


1. Hexamer
2. Monomer
 Monomer is the active form of Insulin, so it will start producing action - > decreasing
blood sugar immediately and in no time the action will be finished.
 We want Insulin to stay as Hexamer and gradually it is released into the monomer forms
so that duration of action can be increased.
 This is achieved by adding zinc to the Insulin (on adding zinc, the insulin starts getting
crystallized and those crystals are deposited and act as depot preparation and slowly-
slowly from that site, monomers are released and insulin keep on acting) Hence, zinc is
added to make insulin long acting.

13. Which of the following is the shortest acting muscle relaxant and is considered for
rapid sequence induction?

a) Tubocurarine
b) Succinylcholine
c) Pancuronium
d) Atracurium

Muscle Relaxants are of two types:-

MR
- Depolarizing
- Non Depolarizing

 Shortest Acting Depolarizing muscle relaxant → succinyl choline.


 Overall shortest acting muscle relaxant →succinyl choline.
 Action of succinyl choline last for < 5 mins.
 Shortest Acting Non-Depolarizing Muscle relaxant → Mivacurium.

14. The drug that causes fall in elderly patients with postural hypotension is:

a) Metformin
b) Prazosin
c) Acarbose
d) Nor-adrenaline

 The side effect postural hypotension AkA "First Dose Hypotension" is observed with drug
ending with zosin - Prazosin, Terazosin, Doxazosin.
 These drugs cause blocking of a1 receptors, leading to vasodilation, that can Cause
Postural Hypotension.
15. A 23 year old female presented with burning pain during micturition. On
investigations, she is found to be suffering from co-infection with chlamydia and
gonococcus. Drug of choice for treatment of this female is:.

a) Azithromycin
b) Ceftriaxone
c) Fluconazole
d) Clindamycin

Doc for gonococcal urethritis → ceftriaxone

Doc for gonococcal and non-gonococal urethritis → Azithromycin

Doc for candidiasis (fungal infection) → fluconazole

Anaerobic bacterial infections → Clindamycin

16. A patient was given anti-cancer therapy and developed neutropenia. Which of the
following drug can be used to prevent neutropenia in next cycle of chemotherapy?

a) Filgrastim
b) Prednisolone
c) Vitamin B12
d) Folic acid

CSFs help your body make more white blood cells. This lowers your risk for febrile
neutropenia. CSFs include Neupogen (filgrastim), Neulasta (pegfilgrastim), and Leukine
and Prokine (sargramostim). They are usually given as shots 24 hours after a chemotherapy
treatment.

There are 3 types of cells in the body.


(1) RBC
(2) WBC
(3) Platelets

On bone marrow suppression due to anticancer therapy it will lead to


↓ RBC → Anemia

↓ WBC → Neutropenia

↓ Platelets → thrombocytopenia
17. Bacillus Anthrax is:

a) Gram positive cocci in cluster


b) Gram positive rods with square ends
c) Gram positive bacilli with spherical ends
d) Gram negative cocci in cluster

Bamboo Sticks appearance is seen in Anthrax


Gram positive cocci in cluster → staph aureus

Gram positive bacilli with spherical ends → Drumstick app seen in cl. Tetani.

18. Mark the incorrect statement about Corona virus :

a) Crown shaped peplomer like spike giving appearance of solar corona


b) Linear segmented positive sense single stranded RNA
c) Mostly infect animals except gamma coronavirus
d) Human corona virus 229E is alpha type

Corona virus

 Enveloped, carrying petal , club, crown shaped peplomer like spike giving appearance
of solar corona
 120-160 nm helical symmetry
 Linear positive sense single stranded RNA (Non segmented genome)
 Mostly infect animals except gamma coronavirus
 There are 6 types of coronavirus to infect humans
 Human corona virus 229E alpha
 Human corona virus NL 63 alpha
 Human corona virus OC 43- beta
 Human corona virus HK U1- beta
 SARS - Cov- beta
 MERS- beta
 SARS CoV-2 (COVID-19)

19. DNA transfer in bacteria via phage is:

a) Conjugation
b) Transduction
c) Transformation
d) Translation
Transduction is the process by which a virus transfers genetic material from one bacterium
to another. ... Later, when one of these bacteriophages infects a new host cell, this piece of
bacterial DNA may be incorporated into the genome of the new host. There are two types of
transduction: generalized and specialized.
20. Sporulation occurs in this phase of bacterial growth curve:
a) Stationary phase
b) Lag phase
c) Log phase
d) Decline phase

1.Lag phase- Adaptation phase to culture media decreased Accumulate nutrient size
2. Log Phase - Exponential increases in number
3. Stationary phase - Start of Accumulation of toxic metabolites = Dead
Live -Sporulation occur in stationary phase (S-S)
4. Decline - Complete accumulation of Toxin metabolites
21. Which of the following is the most likely cause in a case of granuloma?
a) Cat scratch disease
b) Trench fever
c) Leprosy
d) Syphilis
Cat scratch disease is the most likely cause in a case of granuloma.
Various granulomatous diseases and their causative agents.
Cat-scratch disease - Bartonella henselae (Gram-negative bacillus): The necrotic foci
enlarge and coalesce to form necrotizing granulomas called stellate micro abscesses.

22. HUS in children is caused by:


a) E. coli O57:H7
b) Malaria
c) Parvovirus B19
d) Bartonella henselae

EHEC

 Entero hemorrhagic E.coli


-Cause HUS (Hemolytic Uremic Syndrome)
-Shiga like toxin or Verotoxin (VT-1& VT-2)
Aka verotoxigenic E.coli
Culture media is Sorbitol Mac-Conkey agar (SMAC) and Rainbow agar used

 Diarrhea causing E coli


1. EPEC -Entero Pathogenic Attaching/Effacing lesions to villi
2. EIEC - Entero Invasive resemble shigellosis & sereny test +ve 3.
3. ETEC-> Traveler’s diarrhea CFA (colonization factor antigen 4.
4. EAEC -Aggregative persistent type of diarrhea (EAST-1 Toxin)
Parvo Virus B19: Single standard DNA Virus cause Aplastic Crisis
23. N. meningitis can be due to deficiency of this complement system:

a) C1-C4 deficiency
b) C5-C9 deficiency
c) C3 deficiency
d) C2 deficiency

 C5-C9, MAC (membrane attack complex) deficiency cause Recurrent Neisseria


infection.
 C3 deficiency is also associated with pyogenic infections and rheumatologic disorders.
24. Which of the following activates classical complement?

a) C1
b) C3 Convertase
c) lgA
d) Ag - Ab complex

The complement pathway. Complement can be activated through three pathways: classical,
lectin, and alternative. The classical pathway is activated when C1q binds to antibody
attached to antigen, activating C1r and C1s, which cleave C4 and C2.

FORENSIC DEC 2018


1. Gastric lavage, also commonly called stomach pumping or gastric irrigation, is the
process of cleaning out the contents of the stomach. Since its first recorded use
in the early 19th century, it has become one of the most routine means of
eliminating poisons from the stomach. Such devices are normally used on a
person who has ingested a poison or overdosed on a drug such as ethanol. They
may also be used before surgery, to clear the contents of the digestive tract before
it is opened. In which one of the following conditions is gastric lavage
contraindicated?

a) Organo-Phosphosphate poisoning
b) Hydrocarbons
c) Bicarbonate
d) PCM toxicity

Contraindications of gastric lavage:


Absolute: Corrosives to prevent further damage from perforation (except carbolic acid -
causes leathery mucosa of stomach - no risk of perforation).
Relative:
1. Convulsant - may induce more convulsions
2. Comatose - ↑ change of aspiration.

3. Varices - can cause bleeding


4. Volatile - hydrocarbons / kerosene: risk of aspiration reading to chemical pneumonitis.

2. Forensic experts investigated the crime scene. what confirmatory test will be used
to prove a bloodstain?
a) Spectroscopic test
b) Kastle meyer test
c) Benzidine test
d) Orthotolidine test

SCREENING TEST FOR BLOOD STAINS


I. Color test:  Benzidine test / Adler's test: BLUE +
 Phenolphthalein test : PINK +
 Orthotolidine test: BLUE +

II. Luminescent test:  Luminol spray


 UV Light

CONFIRMATORY TEST FOR BLOOD STAINS

I.MICROSCOPY:  To identify human RBC


II. ABSORPTION spectroscopy:  Definitive confirmatory test
 Most reliable
 Used for recent / old stains.
III. Microchemical test  Teichmann test / haemin crystal test
[brown rhombic]
 Takayama test / hemochromogen
crystal [pink feathery]

3. The charred body has been recovered from the dense vegetation close to the
highway. The police were led to the spot by a sniffer dog that was tracking a 25-
year-old IT employee who had been missing since the previous evening. The team
has to collect sample to identify the victim. A sample of which tissue from a burnt
body, is best for identification?

a) Hair
b) Bone
c) Teeth
d) Blood

Identification of burnt bodies:

Hair: singed
Blood: burned
Bone: difficult to extract DNA from charred bones
Teeth: DNA can be extracted from pulp of the teeth even in a burnt body.
4. Death registration in India done within?

a) 21 days
b) 28 days
c) 30 days
d) 40 days

According to 'registration of birth & death act 1969 every birth / still birth / death to be
registered with the covered state / UT within 21 days.

Registration of vital events

 Birth - 21 days
 Deaths - 21 days
 Marriages - 30/60/90 days
 Birth registration is the responsibility of hospital
 Child born to NRI couple abroad , birth registration done within 60 days of arrival

5. Estimation of age at death and determination of sex of the victim or remains are
important factors in the identification of an individual in forensic odontology.
Teeth are among the most reliable tools in the process of identification of age.
Teeth are one of the most durable parts of our body, which can withstand more
assaults than any other part of the body. This is particularly useful in the
identification of bodies in mass disasters and natural calamities. The first
scientific technique for age estimation in adults was presented by Gustafson
{1950}. It was based on longitudinal sections of teeth cut through the central area.
The technique consisted of attributing scores from 0-3 for the presence and
amount of age-related changes such as attrition, periodontal ligament retractions,
secondary dentin formations, root translucency, and root resorption. The most
reliable criteria among Gustafson's criteria is?

a) Root resorption
b) Paradentosis
c) Translucency of root
d) Attrition

Gustafson's criteria: age determination using secondary changes in teeth.

 Attrition
 P. Paradentosis
 S. Secondary dentin 2nd most reliable
 R. Root resorption
 T. Transparency of root most reliable
 Cementum opposition
SPM DEC 2018

1. Mosquito having spotted wings is

a) Anopheles
b) Culex
c) Aedes
d) Mansonia

Anopheles Aedes Culex

 Hunchback posture  Hunchback posture


Adult  Sit at 45° spotted wings  White stripes on body
(Tiger Mosquito)
 Boat shaped  Single  Small
Egg  Lateral floats  Cigar shape  Clusters

Rest parallel to water surface (No Rest at an angle to water Rest at an angle to water surface
Larvae siphon tube) surface (Slender Siphon tube present)
(Short Siphon tube present)

2. Population covered by one PHC in plains is


a) 3000
b) 5000
c) 20000
d) 30000
Population covered by PLAINS HILLY / TRIBAL
Subcenter 1/5000 1/3000 (MPW)

PHC 1/30000 1/20000 (Health Assistant)

CHC 1/12000 1/80000

AWC 1/400 - 800 1/300 - 800

3. Treatment of AIDS (ART) is started at CD4 cut off

a) 200
b) 350
c) 500
d) No cut-off

Treatment of AIDS is ART


ART initiation -irrespective of CD4 count

 Any age
 Any stage
 Any group

Start ART
Normally CD4 count is 900 - 1400 & if it starts going below 500 opportunistic infections start.
Most common opportunistic infection HIV world → Pneumocystis carinii Pneumonia

Most common opportunistic infection HIV India → TB

4. Normal Waist to hip-ratio

a) 0.8 in males
b) 0.85 in females
c) 0.90 males
d) < 0.90 males

Waist to hip ratio used as indirect marker of obesity and direct marker of cardiovascular risk
to the body.
Cutoffs :
Male ≤ 0.90
Female ≤ 0.85
According to new WHO guidelines waist hip ratio has been replaced by a new indicator that
is "Waist Height Ratio"
WHO SAYS, this is now the best indicator for cardiovascular risk.
Age and sex independent
Single cutoff for al age and sex → 0.5

5. Rapid diagnosis of Rabies in a Rabid living dog is by?

a) Fluorescent antibody test


b) Corneal impression
c) Inoculation in mouse
d) None of the above

 Flourescent Antibody Test : This is a highly reliable and the best single test currently
available for the Rapid diagnosis of Rabies viral Antigen in Infected specimen.
 Corneal impression & Mouse inoculations are also used for diagnosis.

6. 1 year old with Keratomalacia, treatment is

a) 1 Lac IU once
b) 1 Lac IU three times
c) 2 Lac IU once
d) 2 Lac IU three times

Keratomalacia is manifestation of vit. A deficiency syndrome known as Xerophthalmia

Xerophthalmia Treatment (oral vit. A)


≤ 1 year age >1 year age
Day 0 1 Lac IU 2 Lac IU
Day 1 1 Lac IU 2 Lac IU
Day 14 1 Lac IU 2 Lac IU

I/M vit. A dose will be exactly half of oral dose.


7. A 3-5 months old child with fever, nasal congestion, sore throat and chills
presented to the OPD with his mother. Nasopharyngeal swab confirmed H1N1 as
the causative agent. Which of the following is the correct treatment dose of
Oseltamivir?

a) 20 mg OD X 5 days
b) 12 mg BD X 5 days
c) 25 mg BD X 5 days
d) 20 mg BD X 5 days

Oseltamivir (Tamiflu ) is the drug of choice for swine flu H1N1, But it's dose is given
according to the age and weight of the person.

>3months to 12 months → mg/kg BD x 5 Days


Age >1 month to 3 months → 2.5 mg/kg BD x 5 Days
0 to 1 month → 2 mg / kg BD x 5 Days

<15 kg →g 30 mg BD x 5 Days
Weight 15-23 kg → 45 mg BD x 5 Days
24-40 kg → 60 mg BD x 5 Days
>40 kg → 75 mg BD x 5 Days

8. Beri-Beri seen in

a) Sorghum eaters
b) Wheat esters
c) Polished rice eaters
d) Maize eaters

Vitamin B1 Deficiency (Thiamine)

 Beri-Beri → Seen in Polished rice eaters


 Wernickes korsakoff psychosis → seen in Alcoholics

9. Most common cause of LBW in India

a) Smoking during pregnancy


b) Maternal tetanus
c) Intra-uterine growth retardation
d) Premature birth
The primary cause is premature birth, being born before 37 weeks gestation; a baby born
early has less time in the mother's uterus to grow and gain weight, and much of a fetus's
weight is gained during the latter part of the mother's pregnancy. Another cause of low
birthweight is intrauterine growth restriction.

10. The proposal to control malaria in towns named as Urban Malaria Scheme.
According to Urban Malaria Scheme, Which of the following is not true?

a) Utilization of anti-larva measures


b) Slide positivity rate more than 10%
c) It is done in town/city with minimum 50,000 population
d) Introduction of active surveillance

Urban malaria has three specific criteria-

 Utilization of Anti-Larval measures.


 It is done in town/city with minimum 50,000 population.
 Slide positivity rate more than 5% [active surveillance].

11. A 30 year old male truck driver visited to the clinic with fever, skin rash, swollen
glands and fatigue. The physician referred him to suraksha clinic. Which among
the following is included in provision of services a Suraksha Clinic ?

a) Immunization services
b) Blood transfusion safety services
c) RTI/STI services
d) Diarrhea control services

Services expected at suraksha clinic (STD clinic) are related to RTI/STI. It includes :

 Diagnostic services
 Treatment care
 Preventive services
12. Ujjawala scheme is for prevention of

a) Child abuse
b) Child trafficking
c) Child labour
d) Child marriage

Providing LPG connections to women from Below Poverty Line (BPL)


UJJWALA households.
Prevention of trafficking and Rescue, Rehabilitation and Re-integration of
UJJAWALA Victims of Trafficking and Commercial Sexual Exploitation
Unnat Jyoti by Affordable LEDs for All
UJALA Promote efficient lighting, enhance awareness on using efficient
equipment which reduce electricity bills and help preserve environment.

13. Peripheral blood smear sample for malaria is done by

a) Female MPW
b) ANM
c) Medical office
d) Male MPW

Multi-Purpose Worker (MPW)


MPW Male MPW Female
 Field work  Reproductive and Child Health
 Peripheral blood smear (RCH)
 Family planning
 Record
 Located at subcenter
 Education - 12th pass
 1st introduced by Kartar singh committee
 Training - 12 months

14. A government school in rural village was checking all students for their health
conditions. Under Vision 2020, to check visual acuity, teacher will refer school
children's to which of the following centre?

a) Service centre
b) Training centre
c) Centre for excellence
d) Vision centre

The goal of Vision 2020 is to eliminate needless blindness by the year 2020. The role played
by Vision Centre in achieving this target by providing primary eye care in the community
therefore assumes significant importance.
Vision2020 has developed certain policies for Vision Centre:

15. A school has a small outbreak were children are showing symptoms like runny
nose, cough, fever and skin rash, The doctors identified it as measles outbreak.
Which of the following is the correct time to give post exposure measles vaccine?

a) 3-4 days
b) 7-10
c) 10-14 days
d) 1-6 months

 IP of measles →10 days


 IP of vaccine induced measles → 7 days
 For providing protection vaccine requires only 6-7 days.
 So, vaccine should be used within 3-4 days of exposure.

Cholera period of Isolation

3 days after starting Tetracycline till 48 hours of antibiotics


7 days after starting Tetracycline
7-14 days
Until 3 weeks of effective antibiotic therapy
Cholera:

 Caused by - vibrio cholera


 Field microscopy test - hanging drop
 IP - 1-2 days
 Rice watery stool

16. Additional energy requirement during Pregnancy (Kcalories/day)

a) +300 kcal
b) +600 kcal
c) +350 kcal
d) +520 kcal

Additional Energy Requirements


nd rd
2 and 3 Trimester 1st Trimester → 150
2 & 3rd Trimester → 350
nd

Lactation 0 – 6 months → + 600


6 – 12 months → + 520

17. Highest Contraceptive failure is reported in

a) Spermicidal methods
b) Implant
c) Oral contraceptive pills
d) IUD

Spermicidal methods has pearl index of 10-25 / HWY( Highest ).


Contain chemical - Non oxynol 9
MOA - By rupture of plasma membrane of Acrosomal cap.

18. While launching a new national health programme and policy population
attributable risk is calculated. Which of the following best defines population
attributable risk?

a) Incidence of disease among exposed is so many times higher as compared to that


among non-exposed
b) So much disease can be attributed to exposure
c) If risk factor is modified or eliminated, there will be so much annual reduction in
incidence disease in the given population.
d) None of the above
Population Attributable Risk (PAR)

 It is the incidence of disease (or death) in total population minus incidence of disease
(or death) among those who were not exposed to suspected causal factor.
 It is useful as it provides an estimation of amount by which incidence of disease could
be reduced in that population if the suspected factor was eliminated or modified.
 PAR is useful for National health programmes and policies.

19. Human anatomical wastes are disposed in ___ BMW (Bio Medical Waste) category

a) Red
b) Yellow
c) White
d) Blue

 Yellow category
 Human anatomical waste
 Animal waste
 Soiled waste
 Chemical waste
 Discarded medicine
 Liquid chemical waste
 Microbiological waste
 Red category: Solid contaminated (Recyclable) (plastic/rubber)
 White category: Wasted sharp
 Blue category:
 Glassware
 Metallic body implants

20. A disaster is a sudden, calamitous event that seriously disrupts the functioning of
a community or society and causes human, material, and economic or
environmental losses. Which among the following covers Disaster management?

a) Ministry of Health
b) Ministry of HRD
c) Ministry of Home
d) Ministry of Agriculture

A disaster is a sudden, calamitous event that seriously disrupts the functioning of a


community or society and causes human, material, and economic or environmental losses.
Disaster management

 Nodal ministry - ministry of home affairs


 Nodal centre – District

21. Phase(s) of Disaster cycle before disaster impact include

a)Disaster response
b)Medial relief and rehabilitation
c)Mitigation and preparedness
d)Rehabilitation

22. At a PHC, vaccine storage is done in

a) Vaccine carrier
b) ILR
c) Walk in Cold rooms
d) Cold box

At the PHC (Primary health centre) level, all vaccines are kept in the ILR for a period of one
month at temperature of +2oC to +8oC
23. A specific package of intervention and services which are given to population
those who have either high risk of HIV or those who have reported high risk of HIV
IS known as targeted intervention. Which of the following group is not involved in
this intervention?

a) Migrant labourer’s
b) Street children
c) Long distance truckers
d) Doctors and nursing professionals

Targeted intervention :
It is a specific package of intervention & services which are
given to population those who have either high risk of HIV or those who have reported high
risk of HIV.
Groups with high risk of HIV are:

 Commercial sex workers


 Injecting Drug Users
 Truck Drivers
 Migrant Labour
 Street children
 Transgender
 Adolescents

24. A patient from rural village was given ORS + Zinc. What does ORS plus Zinc do?

a) Reduces infection
b) Antispasmodic
c) Reduces duration of diarrhea
d) Enhance sodium absorption

Zinc supplementation with ORS therapy reduce

 Duration of diarrhea
 Severity of acute and persistent diarrhea
 Volume of diarrhea
ENT DEC 2018
1. Mainstay of treatment of glue ear:-

a) Radical Mastoidectomy
b) Myringotomy + aeration to middle ear
c) Temporal bone resection
d) Tonsillectomy & adenoidectomy

Mainstay treatment of glue ear should be treat the cause, like if its adenoids or tumor so
removing it would be the treatment. But the cause is not mentioned in the question. So the
mainstay treatment should be myringotomy in the anterio-inferior quadrant with
grommet tube insertion.

2. A patient presented with 2 days history of fever. On examination there was


swelling in the neck and one side tonsil was pushed to midline. What is the most
likely diagnosis:-

a) Quinsy
b) Parapharyngeal abscess
c) Retropharyngeal abscess
d) Tonsillitis

In the history of the patient it's given that the tonsil is pushed to the midline and there is a
swelling in the neck, this is the typical clinical feature of parapharyngeal abscess.

3. Most common symptom of nasopharyngeal carcinoma:-

a) Airway obstruction
b) Headache
c) Neck swelling
d) Epistaxis

Symptoms of nasopharyngeal cancer may include:

 Lump in neck (most common)


 Blurry or double vision.
 Ear infections that recur.
 Face pain or numbness.
 Headache.
 Hearing loss, ringing in the ears, or a feeling of fullness in the ear.
 Difficulty opening mouth.
 Nosebleeds.

4. Picket fence fever is seen in:-

a) Lateral sinus thrombophlebitis


b) Quincy
c) Laryngitis sicca
d) Croup

 Picket fence fever:- fever never comes back to normal baseline. Also known as remittent
fever. (intermittent fever is malaria).
 This is the typical presentation of lateral sinus thrombosis/sigmoidal sinus thrombosis.

5. Cold water is not used for syringing because:-

a) Cold water leads to wax impaction


b) Cold water leads to foreign body impaction
c) Can cause tympanic membrane rupture
d) It causes vertigo by caloric stimulation

Syringing

 Simpsons aural syringe (50ml) is used


 Syringing is done in the posterior superior quadrant.
 The temperature of the water should be ideally 37o C.
 If we use cold water it stimulates labyrinth and the patient has vertigo.
 Cold water 300 C & 440 C is warm water used in caloric testing to check if labyrinth is
working.

6. Which of the following muscles originates from the first pharyngeal arch:-

a) Stapedius muscle
b) Anterior belly of digastric muscle
c) Posterior belly of digastric muscle
d) Stylopharyngeus

The first arch gives rise to the muscles of mastication and the mylohyoid, anterior belly of
digastric, tensor veli palatani and tensor tympani – all of which are innervated by the
branches of the trigeminal nerve.
7. Dip at 4000hz in pure Tone audiometry indicates:-

a) Otosclerosis
b) Meniere's disease
c) Noise induced hearing loss
d) Age related hearing loss

2000Hz / Carhart's Notch: seen in otosclerosis (CHL)


4000Hz Dip/boiler's notch :seen in noise induced hearing loss (SNHL)
Both are seen in bone conduction.

8. Which of the following structures is used as a landmark for facial nerve in parotid
gland surgery?

a) Tragal pointer
b) Posterior border of mylohyoid
c) Tip of helix
d) Styloid process

Surgical landmarks:

 The tympanomastoid suture is one of the landmarks that's used.


 We also use the tragus cartilage/tragal pointer. It's the most commonly used landmark
in identifying the facial Nerve.
 Styloid process can also be used but there are high chances of nerve damage
 Posterior belly of digastric is also a landmark.

9. Continuous watery discharge from nose after trauma is most likely a feature of::

a) CSF otorrhea
b) Common colds
c) CSF rhinorrhea
d) Anterior epistaxis

Watery discharge from nose always implies CSF.


CSF Rhinorrhea:
MC cause Head injury/trauma due to fracture of cribriform plate of ethmoid.
10. Which of the following is not a premalignant lesion / condition:-

a) Chronic hypertrophic candidiasis


b) Oral submucosal fibrosis
c) Lichen planus
d) Leukoplakia

Chronic hypertrophic candidiasis is a long term infection of candida seen in the


middle aged male patients, smokers. There is a membrane formation, But its not a
premalignant condition.

Premalignant lesion:

 Oral submucous fibrosis


 Lichen planus
 Leukoplakia (MC premalignant lesion seen in smokers & tobacco chewers)

11. Hypoglossal nerve supplies all of these muscle except:-

a) Palatoglossus
b) Styloglossus
c) Genioglossus
d) Hyoglossus

The hypoglossal nerve supplies:

 All of the intrinsic muscles of the tongue.


 Three extrinsic muscles of the tongue: the genioglossus, hyoglossus and
styloglossus muscles.
 As well as the geniohyoid muscle (a suprahyoid muscle)

12. After head trauma a patient in unable to smell coffee but can smell ammonia.
Which of the following statements is true about the patient?

a) Ammonia is used for testing olfactory nerve


b) The olfaction system is not damaged
c) Ammonia irritates trigeminal nerve
d) All of the above
Ammonia has pungent smell so, there is orthonasal olfaction & the odour goes directly to
cribriform plate and directly stimulates the CN - IX, V & X. so it can curse smell even when
the olfactory Nerve is not working.

13. Which of the following is an accessory masticatory muscle?

a) Buccinator
b) Masseter
c) Medial pterygoid
d) Lateral pterygoid

Masticatory muscles are

 Masseter
 Temporalis
 Pterygoids (Lateral & Medial)
These are Supplied by the mandibular nerve

 Buccinator is a secondary masticatory muscle,


 Its main function is facial expression.
 Hence, it's called an accessory masticatory muscle.

OPTHAL DEC 2018

1. True about pterygium

a) Inflammation of cornea
b) Fatty deposition of sclera
c) Conjunctival invasion making a flap over cornea
d) Dead epithelial debris accumulation

Pterygium:
 Wing shapes fold of conjunctiva encroaching upon the cornea from the either side
Etiology-

 Hot climates
 Degenerative & hyperplastic condition of conjunctiva
Symptoms

 Visual disturbance (astigmatism)


 Cosmetic intolerance
Sign -

 Triangular fold of conjunctiva mainly on nasal side


 Stocker line - Iron deposition on corneal epithelium
Rx-

 Surgical excision-recurrence chance is 30-50 %


 Best RX- Surgical excision with free conjunctival autograft taken from the same eye Of
other

2. For laser iridotomy which laser is used?

a) Argon laser
b) Nd YAG laser
c) Excimer laser
d) Co2 Laser

LASER EFFECTS CLINICAL USE


ARGON LASER Photocoagulation Diabetic Retinopathy
ND YAG LASER (1064nm) Photodisruption CAPSULOTOMY,
IRIDOTOMY
EXCIMER (193nm) Photoablation LASIK, Phototherapeutic
Keratectomy
CO2 LASER Incisional device Resurfacing Oculoplasty
device

3. A newborn is hypermetropic by

a) + 2.5 D
b) +5D
c) + 10 D
d) +1D

NORMAL - REFRACTIVE error at birth 2-3 D hypermetropic


 By 5-7 years of age the eye is emmetropic and remains so till the age of about 50
years
 After 50 years of age there is tendency to develop hypermetropia again which
gradually increase the extreme of life by which the eye has the same +2 TO +3 with
which it started. this senile hypermetropia is due to change in the lens.

4. Corneal endothelium is derived from:

a) Neuroectoderm
b) Neural crest cells
c) Mesoderm
d) Surface ectoderm

SURFACE ECTODERM NEURAL ECTODERM MESODERM NEURAL CREST

 LENS  Epithelium of ciliary  Extraocular Muscles  Stroma of cornea &


 Epithelium of the all body & iris  Endothelium of all the choroid
structures except  Retinal pigment orbit & stroma of iris &  Corneal endothelium
ciliary body & iris epithelium ciliary body  Sclera (except
temporal part)
 Glands  Optic nerve  Sclera (temporal part)
 Melanocytes
 Sphincter and dilator  Corneal keratocytes
pupillae muscles  Ciliary muscles
 Smooth muscles of iris  Ciliary ganglion
 Stroma of iris
 Choroid Trabecular
meshwork

5. True about retinoscopy with a plane mirror

a) In hypermetropia the red glow moves in opposite direction


b) In myopia the red glow moves in same direction
c) In emmetropia the red glow moves in opposite direction
d) Done at 1 meter away from patient

Retinoscopy

 Differentiate b/w type of refractive errors


 Optical error & organic error
 Performed at 1 meter away
6. Rosette cataract is seen after

a) Blunt trauma to eye


b) Copper foreign body in eye
c) Penetrating injury to eye
d) Infection

Cataracts caused by blunt trauma classically form stellate- or rosette-shaped


posterior axial opacities that may be stable or progressive, whereas penetrating trauma
with disruption of the lens capsule forms cortical changes that may remain focal if small or
may progress rapidly to total cortical opacification.

7. Patient with history of fall on symmetrical surface presents with enophthalmos


diplopia on upward gaze and loss of sensitivity over cheek. True statement about
this is :

a) Maxillary fracture
b) It is a blowout fracture
c) Zygomatic bone is most likely injured
d) Frontal bone fracture

Blow out fracture -

 MC fracture of orbit
 Inferior wall M/C involved
 Entrapment of inferior rectus in inferior wall fracture - not able to look up
Features

 Enophthalmos
 Diplopia
 Infra orbital nerve anesthesia
Hanging drop or tear drop sign on imaging

8. Blow out fracture can be due to:

a) Tennis ball injury


b) Chisel and hammer injury
c) Sudden fall
d) Punch at the chin from below

 Blow out orbital fracture result from trauma to orbit by a relatively large often rounded
object such as tennis ball, cricket ball, human fist, or part of automobile.
 The force of blow causes a backward displacement of the eyeball and increase in the
intraorbital pressure.
9. Sarcoidosis is associated with

Cataract
Ectopia lentis
Anterior uveitis
Keratitis

Sarcoidosis -Can cause

 Anterior uveitis (M /C)


 Posterior uveitis
 Intermediate uveitis
 Pan uveitis
 M/C presentation - B/L Lymphadenopathy

10. 10 years old child brought to your clinic with developmental delay associated with
extrapyramidal symptoms. On further examination, dislocation of the lens towards
the inferonasal area is revealed. which of the following is the most probable
diagnosis?

a) Homocystinuria
b) Refsum's disease
c) Wilson's disease
d) Sarcoidosis

Congenital ectopia lentis is seen in :

 Marfan's syndrome - supratemporal


 Homocystinuria - inferonasal
 WEILL -MARCHESANI SYNDROME -Forward subluxation of lens
 Ehlers Danlos syndrome -subluxation of lens blue sclera.
11. Obstacles in the condition shown in the image is:

a) Motor obstacles
b) Sensory obstacles
c) Central obstacles
d) All the above

The given image shows a Concomitant squint

CONCOMITANT SQUINT - a type of manifest squint which the amount of deviation in the
squinting eye remains constant in all directions of gaze, with no limitation of ocular
movements. Any obstacles to the development of these processes result in concomitant
squint.

Sensory obstacles-hinder the formation of a clear image in one eye.


E.g.

 Refractive errors
 Prolonged use of incorrect spectacles
 Anisometropia
 Media opacities - Corneal/Lenticular opacities

Motor obstacles - hinder the maintenance of eyes in the correct positional relationship in
primary gaze or during ocular movements.
E.g.-

 Congenital abnormalities of shape/size of the orbit


 Abnormalities of extraocular muscles

Central obstacles - Deficient development of fusion faculty.


Abnormalities of cortical control of ocular movements as occurs in mental Trauma and
hyperexcitability of CNS during teething.

OBG DEC 2018


1. A woman at 26 weeks gets ultrasound done for fetal well-being and is diagnosed
with hydrops fetalis. Which of the following fetal infections could have led to this
condition?

a) Ebstein barr virus


b) Human papilloma virus
c) Parvovirus B19
d) Influenza Virus

Causes of hydrops fetalis -

 Cardio vascular cause → congenital heart block ( Most common)


 Chromosomal cause → turner syndrome (45 XO ), triploidy , trisomy’s 21,18 and 13
Hematological → α 4 - thalassemia (Hemoglobin Bart's), fetomaternal hemorrhage

 Infections → Parvovirus B19(mc) , syphilis , cytomegalovirus, toxoplasmosis Rubella,


HSV.

Hydrops fetalis -
 Excessive destruction of the fetal red cells leads to severe anemia, tissue anoxemia
and metabolic acidosis
 It is accumulation of fluid in the body of the fetus in two out of three compartments:
ascites, pleural effusion, pericardial effusion
 Most common cause of immune hydrops fetalis - Rh incompatibility.

2. Childbirth is the single most common cause of uterovaginal prolapse in our


country. All the events during child birth may account for risk of uterine prolapse
in future, EXCEPT?

a) Precipitated labour
b) Prolonged 2nd stage
c) Premature bearing down
d) Episiotomy

Episiotomy will not cause prolapse

Main cause of prolapse is abnormal conduct of labor***


Abnormal conduct of labor is by :

 Prolonged 2nd stage


 Faulty instrumentation
 Early bearing down
 Cervical lacerations & muscle lacerations

3. Conservative treatment for pelvic organ prolapse is

a) Perineorrhaphy
b) Shirodkar Procedure
c) Pessary
d) Le Fort Repair

 Shirodkar procedure
Surgical treatment of prolapse  Perineorrhaphy
 temporary Rx
 Le fort repair

Conservative treatment of prolapse  Pessary ( should be replaced every 3


months )

Management of Genital Prolapse


Nulliparous Abdominal sling operations
Pregnancy Ring pessary up to 16 weeks
Postnatal  Ring pessary and pelvic floor exercises for 3-
6 months.
 Surgery if required thereafter.
Young woman <40 years Conservative vaginal surgery (fertility sparing
surgery)
 Cystocele, rectocele repair
 Manchester repair
 Sling operation

Woman beyond 40 years and multipara Vaginal hysterectomy and pelvic floor repair

4. Cut off point for 50 gram glucose challenge test at 1 hour mg%

a) 100
b) 140
c) 180
d) 200
Glucose challenge test :
50 grams of glucose to non-fasting women :
After 1 hour

 After 1 hour → Value: < 140 normal


 >140 → May be gestational diabetes mellitus.
 We do the diagnostic test i.e. glucose tolerance test
 >200 - gestational diabetes mellitus

Glucose tolerance test


100 grams of glucose to fasting women

 Fasting value → < 105

Value:

1 hr - < 190 any '2' abnormal values then it is


2 hr - < 165 diagnostic of gestation diabetes mellitus.

3 hr - < 145

5. 28 year old lady (G2 P1) who does not want to conceive after abortion, which is the
preferred mode of contraception:

a) OCP
b) IUD
c) Barrier
d) Implant

Contraception → IUD > OCP > Barrier

6. A 30 year old primigravida delivers a full term baby. However she chooses to
formula feed her newborn. After how much duration can she start using combined
oral contraceptives for avoiding pregnancy?

a) 2 weeks
b) 6 weeks
c) 12 weeks
d) Immediately after delivery

Post-partum use of combined oral contraceptives


 COC are contraindicated in breast feeding mothers (causes breast milk suppression)
and can be started after 6 weeks post-partum.
 COC can be used in non-breast feeding mothers after 3 weeks if there are no persisting
risk factors for VTE such as smoking, history of DVT, known thrombogenic mutations,
peripartum cardiomyopathy
 COC can be safely started after 6 weeks in non-breast feeding mothers
 Here in this question, since there is no history suggestive of any risk factors for VTE, the
better answer would have been 3 weeks, but since it is not in the options, the answer
would be 6 weeks.

Please note that COC should never be started immediately post-partum in any scenario
since post-partum period in itself is high risk state for thromboembolism, maximum risk
being in the first week. Progesterone only pills can be safely used in lactation, and can be
started as soon as contraception is required.

7. A 43 year old woman, from low socio-economic status, a chronic smoker,


hypertensive, P3L3, tests positive for type 16 HPV on cervical cancer screening.
She now uses IUCD for contraception. All of the following factors in her history
predispose to her acquiring cervical cancer, EXCEPT?

a) HPV 16, smoking


b) Hypertension and IUCD use
c) Multiparity and low socioeconomic status
d) Multiparity and smoking

Risk factors for development of cervical cancer

 HPV
 16: Most common
 18: most malignant
 HSV 1 and.
 HIV 1 and 2
 Commercial sex worker
 Women with many sex partners
 Partner with STD
 Early intercourse (< 16 yrs)
 Low Socio Economic status
 Smokers

8. EDD ( Expected Date of Delivery) is calculated by:

a) Young's rule
b) Naegele's formula
c) Cardiff Formula
d) Hadlock Formula
Expected date of delivery is calculated by Naegele's formula
From the 1st day of last menstrual cycle → 9 months + 7 days.

9. If a zygote divides 2-3 days after fertilization which of the following type of twining
is seen:

a) Monochorionic, Monoamniotic
b) Monochorionic, Diamniotic
c) Diamniotic, Dichorionic
d) Dichorionic, Monoamniotic

If a zygote divides : Type of twining

Before 3rd day of life  Dichorionic Diamniotic

4th to 8th day of life  Monochorionic Diamniotic

After 8th day of life  Monochorionic Monoamniotic

After 14th day of life  Conjoined twins / Siamese twins

10. Chromosome pattern in complete mole:

a) 46 XY both maternal origin


b) 46 XX both maternal origin
c) 46 XX both paternal origin
d) 46 YY both paternal origin

Complete mole
A complete mole may be formed if.
23 , - X bearing haploid sperm
penetrates a empty ova paternal
chromosome then duplicate to create
46 XX diploid complement Solely of
paternal origin
11. A woman is diagnosed with an unruptured tubal ectopic. She is admitted for
medical management with methotrexate while a strict watch is kept for signs of
tubal rupture. An ectopic sac in which of the following parts of the tube is likely to
rupture the earliest in case of failed medical management?

a) Interstitial
b) Isthmus
c) Ampulla
d) Infundibulum

Fate of Ectopic pregnancy -Tubal abortion

 M/C site s ampulla


 2nd M/C site S infundibulum

Rupture of the tube:

 M/C site is Isthmus (4-6 weeks)


 Longest survival → interstitium (12 weeks)

12. A pregnant woman with a known history of fibroid uterus is being planned for an
elective cesarean section for breech presentation. She insists her fibroid be
removed with the surgery, however the gynaecologist counsels her the regarding
disadvantages of myomectomy at the time of cesarean and tells her that he would
remove it only if it is ?

a) Pedunculated subserosal
b) In the broad ligament
c) Posterior in location
d) Small in size

A myomectomy should not be performed during a caesarean section as the uterus is


extremely vascular at this time and any attempt at a myomectomy can lead to torrential
bleeding. However pedunculated fibroid may be removed by clamping and ligating the
pedicle, if easily accessible.

13. After 28 weeks POG major amount of amniotic fluid is secreted from:

a) Fetal skin
b) Fetal urine
c) Placental cell membrane
d) Plasma
In early pregnancy Ultrafiltrate of maternal plasma
Beginning of 2nd trimester Extra cellular fluid which diffuses through
fetal skin
After 20 weeks Fetal urine > fetal lung

14. Which of the following infections can commonly lead to infertility:

a) Chlamydia
b) Trichomoniasis
c) Candidiasis
d) Bacterial vaginosis

Major infectious cause of infertility is chlamydia


Chlamydia is an indolent PID (Quit / symptomless)

15. A young couple presents to fertility clinic with inability to conceive since years.
They are living together,have been married for 4 years and not using any
contraception at present. The initial tests that should be advised to this couple
are?

a) Semen analysis, CXR, montoux


b) Semen analysis, Ovulation test, Tubal patency test
c) Ovulation, tubal patency, Montoux test
d) Testicular biopsy, USG, Sperm penetration test

Order for initial work- up for infertility


1. Semen analysis
2. Ovulation test
3. Tubal patency
Note: Tubal patency test is done by 2 ways
1 . HSG (initial test)
2. Laparoscopy with Chromopertubation (Confirmatory)

16. A G4P2 lady presented with history of two abortions at 16 weeks and 20 weeks
POG. Which of the following could be the most likely reason for these abortions?
a) Chromosomal abnormality
b) Cervical incompetence
c) Placenta previa
d) Thyroid abnormality

Abortions- MC causes

 1st trimester - chromosomal causes


 2nd trimester - Anatomical cause

17. True about endometriosis:

a) Presence of endometrial gland in deep myometrium


b) Presence of endometrium at ectopic locations
c) Treated preferably with hysterectomy
d) Seen in multiparous women

Endometriosis

 Presence of endometrial tissue at ectopic location


 Etiology : Retrograde menstruation
 Proposed by Sampson ( Sampson's Implantation theory
 70-80% of all women have retrograde menses
 Among them 5-10% have poor immunity and ↑ estrogenicity and develops
Endometriosis

NOTE: Option A, C, D. are related to Adenomyosis.

18. A 29 year old female presented with infertility. There is history of abdominal pain,
dyspareunia, dysmenorrhea, menorrhagia. Most likely cause:

a) Adenomyomatosis
b) Endometriosis
c) Myomas
d) Cervicitis

Symptoms of Endometriosis :

 Dysmenorrhea
 Abdominal pain
 Dyspareunia
 Infertility
 Menorrhagia
 Chronic pelvic pain
19. A 32 year old pregnant woman presents with 36 week pregnancy with complaints
01 pain abdomen and decreased fetal movements. Upon examination PR= 96/min,
BP = 156/100 mm Hg, FHR = 128 bpm. On per-vaginum examination there is altered
blood seen and cervix is soft 1 cm dilated. What is the preferred management?

a) Tocolytics
b) Induce labour
c) Immediate LSCS
d) Wait and watch

This appears to be a case of abruption with a normal fetal heart rate.


The patient should be induced to expedite delivery with a strict watch over maternal vitals
and fetal heart rate.

20. A 30 year old woman presents to the clinician with excessive growth of facial hair
associated with hairgrowth on her upper back, arms and thighs. Which of the
following is the most likely cause?

a) Drug induced
b) PCOS
c) Endometriosis
d) Adenomyosis

Excessive hair growth in male pattern in women is known as hirsutism


Hirsutism

 M/C cause : PCOS


 Due to Hyperandrogenism
 Increase Androgen → increase Serum Testosterone & Androstenedione →decrease
Sex hormone binding globulin → increase Free Androgen → Hirsutism.

21. 52 year old lady presents to the physician with complains of episodes of feeling of
intense warmth followed by profuse sweating. She stopped having her periods
since the past 3 years. Which of the following is correct regarding her condition?

a) Increased FSH, increased estrogen


b) Decreased FSH, decreased estrogen
c) Decreased FSH, increased estrogen
d) Increased FSH, decreased estrogen

The patient has presented with post-menopausal hot flashes.


Due to exhaustion of ovarian function after menopause, the estrogen levels decline. As a
result of low estrogen, the feedback inhibition of gonadotropins IS lost, thus increasing levels
of FSH and LH.

 FSH > 10 is suggestive of menopause


 FSH > 40 is diagnostic of menopause

22. A primigravida presents at 9 weeks of amenorrhea with a positive pregnancy test.


She complains of nausea and vomiting which gets better by evening. The doctor
advices her folic acid and dietary modifications to help with her vomiting. He
explains that the cause of such presentation is a hormone. The source of this
hormone is?

a) Yolk sac
b) Liver
c) Syncytiotrophoblast
d) Umbilical cord

 Pregnancy nausea is directly linked to the pregnancy hormone hCG, Early in


pregnancy, when morning sickness is typically at its worst, hCG levels rise rapidly to
help grow and nourish the fertilized egg.
 Human chorionic gonadotrophin (hCG) is a glycoprotein hormone synthesized in
the rough ER of syncytiotrophoblast immediately after implantation of the
blastocyst into the uterine wall.

PEDIA DEC 2018


1. Continuous murmur is heard in?

a) Patent ductus arteriosus


b) Ventricular septal defect
c) Atrial septal defect
d) Tetralogy of fallot

PDA → Continuous murmur

VSD → Pan systolic murmur

ASD → Silent, wide fixed split of S2 is seen

TOE → Ejection Systolic Murmur


2. Menke's disease is due to defect in the metabolism of:-

a) Zinc
b) Copper
c) Selenium
d) Iron

Menke's disease: aka kinky hair disease

 X- linked recessive disorder caused by mutations in genes coding for the copper
transport protein ATP7A.
 Copper level is low in liver & brain, high in enterocytes & fibroblasts

Clinical features:-

 Characterized by sparse hypopigmented kinky hairs


 On Microscopic examination of the hair: Trichorrhexis nodosa (fractures along the
hair shaft) and pili torti (twisted hair) seen
 Growth retardation
 CNS involvement- Intellectual disability, delayed milestones, seizures, hypotonia
 Hypothermia
 Patients with classic form of Menke's disease usually die before 3 years of age.

3. Pellagra like rash is seen in?

a) Gaucher disease
b) Alkaptonuria
c) Hartnup disease
d) Phenylketonuria

Hartnup disorder:

 Defect in transport of neutral amino acids by intestinal mucosa & renal tubules:
 Autosomal recessive inheritance
 Defect in SLC6A19 gene on chr 5p15
 Most children remain asymptomatic
 Cutaneous photosensitivity and pellagra like rash.

4. A pediatrics patient present to the hospital with


complaints of multiple pathological fractures and the
X-ray reveals the below-depicted image. Blood
picture presents Low phosphorus and calcium levels
with normal Vitamin D levels and elevated PTH.
Which of the following is the most probable
diagnosis?
a) Osteopetrosis
b) Osteomalacia
c) Osteosclerosis
d) Histiocytosis X

Osteopetrosis/ marble bone disease:

 Bone within bone appearance on X-ray (increased bone density)


 Due to mutation in CLCN7 gene → Leads to defective bone
 C/F: Neurological problems present, Deafness, Macrocephaly, hepatosplenomegaly.
Pancytopenia
 Investigation: Low phosphorus and calcium levels with normal Vitamin D levels and
elevated PTH.

5. All of the following are manifestations of congenital syphilis except:-

a) Gumma
b) Hutchinson's teeth
c) Olympian brow
d) Interstitial keratitis

 GUMMA is seen in TERTIARY SYPHILIS.

Manifestations of early congenital syphilis include:

 Syphilitic rhinitis
 Maculopapular rash
 Condyloma lata

Late signs of Congenital syphilis:

Symptom/sign Descriptions
Olympian brow Bony prominence of the forehead caused by persistent or
recurrent periostitis.
Higoumenakis sign Unilateral or bilateral thickening of the sternoclavicular third of
the clavicle
Saber shins Anterior bowing of the midportion of tibia
Hutchinson teeth Peg-shaped upper central incisors with a notch along the biting
surface
Mulberry molars Abnormal 1st lower molars with excessive number of cusps

Saddle nose Depression of the nasal root


Rhagades Linear scars in a spoke-like pattern from mucocutaneous
fissures of mouth, genitalia.
Hutchinson triad Hutchinson teeth, interstitial keratitis, and 8th nerve deafness.

Clutton joint Unilateral or bilateral painless joint swelling

6. Abnormality involving which chromosome is seen in hunter disease?

a) X chromosome
b) Y chromosome
c) Chromosome 21
d) Chromosome 22

All mucopolysaccharidoses are Autosomal Recessive Except Hunter's disease which


is inherited as X-linked Recessive

Mucopolysaccharidoses:

Type Name Enzyme deficient Clinical Features


I Hurler alpha-L-iduronidase Coarse face+ Corneal clouding +
MR + HSM + bony changes
II Hunter iduronate-2-sulfatase (I2S) Same as above but NO corneal
clouding.
IV Morquio N-acetylgalactosamine-6- Bony abnormalities severe &
sulfate sulfatase (GALNS) corneal clouding may be present

7. Mental retardation is seen in?

a) Phenylketonuria
b) Alkaptonuria
c) Albinism
d) Von Gierke disease

Mental retardation is feature of PKU due to elevated level of phenylalanine


hydroxylase in brain.

8. A 6 year child presents with hepatosplenomegaly & generalized lymphadenopathy


along with fever The child developed a rash, after being administered ampicillin.
What could be the possible diagnosis?
a) Infectious mononucleosis
b) Scarlet fever
c) Kawasaki disease
d) HIV infection

Infectious mononucleosis:

 Aka glandular fever


 Etiological agents: Epstein Barr virus (EBV)
 Mode of transmission: Blood transfusion, Direct contact
 C/F: Fever, headache, sore throat, generalized lymphadenopathy, splenomegaly,
hepatomegaly
Patients with infectious mononucleosis treated with ampicillin or amoxicillin may experience
"Ampicillin rash" which may occur with other α- lactam antibiotics.

 This morbilliform, vasculitis "ampicillin rash" is probably immune mediated and


resolves without specific treatment.

9. In an eye camp doctor's try to make people aware of the causes of blindness.
Vitamin A is the leading cause for preventable childhood blindness. A parent
brings her 18 month old child to the doctor for routine eye check up. Doctor
notices thinned out cornea and from old records it was seen that the child also
has iron deficiency anemia. Weight of the child is 10kg, what should be the dose
of vit. A to be given in this case if doctor marks it as keratomalacia.

a) 50,000 IU
b) 1,00,000 IU
c) 2,00,000 IU
d) 5,00,000 IU

Vitamin A DEFICIENCY
It is characterized by night blindness, conjunctival xerosis. Bitot spots, Keratomalacia and
fundus changes in severe cases

Treatment:

 <6 months of age Vitamin A 50,000 IU orally/dose


 6-12 months of age Vitamin A 100,000 IU orally/dose
 ≥12 months of age Vitamin A 2,00,000 1U orally/dose
 The same dose is repeated next day & weeks later.
 Clouding of cornea in a child with Vit A deficiency is emergency → requires parenteral
administration of 50,000 - 100,000 IU Retinol
 In keratomalacia → Local treatment with antibiotic drops & ointment, padding of eye →
enhances healing
10. A new born loses maximum heat from:-

a) Head
b) Abdomen
c) Palms & soles
d) Neck

A new born loses maximum heat from ‘Head’ because of greater surface area.

11. Recommended daily dose of lodine in child is:-

a) 120-200 μg
b) 90-120 αg
c) 30-60 μg
d) 500 μg

Recommended dietary allowance of iodine:.

 For Infants- 30 μg /kg/day


 For children → 90-120 μg /day
 For adolescents and adults →150 μg /day
 Additional 25 μg & 50 μg during pregnancy & lactation respectively.

PATHO DEC 2018


1. Most characteristic feature of diabetic nephropathy which is show below is:

a) Kimmelstiel Wilson change


b) Armani Ebstein change
c) Focal segmental glomerulosclerosis
d) Membranoproliferative glomerulonephritis
Features of diabetic nephropathy are:
Glomerular lesions like :-

 Capillary basement membrane


thickening → Earliest & most common
change in DM
 Diffuse mesangial sclerosis → presents
as nephrotic syndrome in severe cases
 Nodular glomerulosclerosi aka
Kimmelstiel-Wilson lesion →
Pathognomonic of diabetes
Renal vascular lesions, principally
arteriolosclerosis
Pyelonephritis, including necrotizing papillitis.

2. Xenograft is:

a) Graft across species


b) Graft from same species
c) Graft from same individual
d) Graft from same organ

graft taken from one part of the body of an individual and


Autograft transplanted onto another site in the same individual, e.g., skin
graft.
graft taken from one individual and placed on another individual
Isograft of the same genetic constitution, e.g., grafts between identical
twins.
Allograft graft taken from one individual placed on genetically non-
identical member of the same species.
graft taken from one individual placed on an individual belonging
Xenograft to another species, e.g., animal to man.

3. Which of the following is most abundant glycoprotein in basement membranes?

a) Fibronectin
b) Laminin
c) Integrin
d) Syndecan

 Laminin is the most abundant glycoprotein in basement membranes.


 Connects cells to underlying ECM components such as type IV collagen and heparan
sulfate.
 Laminin can also modulate cell proliferation, differentiation, and motility.

4. Which of the following is true about Anaplasia?

a) Loss of cohesion between cells


b) Loss of differentiation
c) Change of epithelium type
d) Benign and fully reversible

Lack of cellular differentiation or Anaplasia is considered a hallmark of malignant


transformation. The term anaplasia means “to form backward,” which implies
dedifferentiation (or loss of the structural and functional differentiation) of normal cells during
tumorigenesis.

5. Cell that can form the maximum number of other cells in the body is called?

a) Totipotent
b) Multipotent
c) Pluripotent
d) Lineage stem cells

Totipotent cells- can form maximum no of other cells in body


e.g. cells of zygote [MCQ]
Note: Totipotent cells give rise to teratoma.

6. Which of the following disease has X linked recessive pattern of inheritance?

a) Hurler Syndrome
b) Thalassemia
c) Hereditary spherocytosis
d) G6PD deficiency

X-LINKED RECESSIVE DISORDERS: Oblivious Female Will Give Her Boys Her X-Linked
Disorders"

 Ocular albinism
 Fabry disease
 Wiskott-Aldrich syndrome
 G6PD deficiency
 Hunter syndrome
 Bruton agammaglobulinemia
 Hemophilia A & B
 Lesch - Nyhan syndrome
 Duchenne muscular dystrophy

7. Which patter of inheritance of disease is associated with consanguinity?

a) Autosomal recessive
b) Autosomal dominant
c) X - linked recessive
d) X - linked dominant

 CONSANGUINITY condition in which closely related individuals are going to have the
marriage.
 This can lead to expression of even those genes which are recessive in nature.
 Therefore, it is a risk factor for expression of Autosomal Recessive disorder.

8. Which organ is involved in Goodpasture syndrome?

a) Liver
b) Adrenals
c) Kidney
d) Brain

Good pasture syndrome

 Type II Hypersensitivity reaction.


 Auto Ab against glomerular basement membrane type IV collagen.
 Affects kidneys & lung causing hematuria & hemoptysis.
 Characterized by linear deposits.

9. A 57-year-old male came with complaints of swelling over the scalp region which
gradually developed over 6 months. Now complaints of pus discharge several
times in the past month associated with pain. On examination, swelling sized 3x2
cm, Firm, Fixed; Tenderness(+), Warmth(+), Punctum(+), Discharge(+). Incision &
Drainage was done, the wound was left open to heal. Which of the following is a
mechanism for healing in this case?

a) Granuloma formation
b) Scab formation
c) Granulation tissue
d) Neovascularization

 Healing intention: Wounds by secondary are left open and allowed to heal by
granulation, contraction and epithelialization.
 In healing by second intention, large amounts tissue are formed of granulation to fill a
bigger gap caused by the larger area of deficit.
10. A 47 years old male, active smoker with a known case of Bronchial Asthma now
presents with difficulty in breathing with the breath sounds being audible
externally. Which of the following is not a feature you will expect to find in this
patient?

a) Thickening of bronchial wall


b) Increase in number of airway goblet cells
c) Fibroblastic foci
d) Sub-basement membrane fibrosis

Fibroblastic foci (exuberant proliferation of fibroblasts) is a feature of


idiopathic pulmonary fibrosis (IPF). Bronchial asthma is an example of Type I
Hypersensitivity reaction and is characterized by airway remodeling which includes:

 Thickening of bronchial wall with hypertrophy and/or hyperplasia of bronchial smooth


muscles
 Increase in size of submucosal glands and number of goblet cells
 Increased vascularity
 Sub-basement membrane fibrosis (due to deposition of type I and III collagens)

Key points - Sputum findings in bronchial asthma:

 Curshmann spirals - impacted mucin plugs.


 Charcot Leyden crystals - Eosinophil crushing artefact.
 Creola bodies - damaged respiratory epithelial cells.

Mneumonic Type 2 HSR is seen in following condition:


My : Myasthenia gravis
Blood : Blood transfusion (mismatch)
Group : Graves, good pasture
Is: Immune hemolytic anemia
Rh: Rheumatic heart disease
Positive: Pernicious anemia/ pemphigous vulgaris

11. Acute graft rejection occurs within?

a) Seconds to minutes
b) Days to weeks
c) Weeks to months
d) Months to years
Hyperacute rejection: type 2 HSR, happens over span of few minutes to hours. Eg: kidney
transplant
Acute rejection: Few weeks to months(< 6 months)
Chronic rejection: After 6 months.
Graft vs host disease (Runt Disease):

 Acute GVH: < 100 days


 Chronic GVH: > 100 days

12. All are reduced in iron deficiency anemia EXCEPT:

a) Total iron binding capacity


b) Percentage saturation of Transferrin
c) Hemoglobin level
d) Serum ferritin

Iron deficiency typically causes low serum iron, high iron-binding capacity, and low serum
ferritin levels. Anemia occurs when you have a decreased level of hemoglobin in your red
blood cells (RBCs).

13. Pheochromocytoma produces all except?

a) Nor-epinephrine
b) Secretin
c) Vasoactive intestinal polypeptide
d) Somatostatin

Pheochromocytoma - Tumor of adrenal medulla

Follows rule of 10 -
10% are extra adrenal
10% are bilateral
10% are children
10% are w/o hypertension
Tx: Surgical Resection
α + β blocker
SURGERY DEC 2018

1. Most common immediate complication of splenctomy:

a) Hemorrhage
b) Fistula
c) Bleeing from gastric mucosa
d) Pancreatitis

Complications of splenectomy:
 Early:
 Haemorrhage
 Injury to stomach, splenic flexure, pancreas

 Delayed:
 Fistula from stomach, pancreas
 Subdiaphragmatic collection
 Left basal atelectasis and pleural effusion
 Thrombocytosis- thrombotic complications

2. All are premalignant conditions of oral cavity except:

a) Chronic hyperplastic canidiasis


b) Oral submucosal fibrosis
c) Oral lichen planus
d) Leucoplakia

All the given options increase the risk of carcinoma of oral cavity but, oral Lichen
planus has the least risk as it is Low-risk or Equivocal lesion.

3. A 6 year old child is brought with high fever with rigors for 5 days with pain
examination patient s anicteric and tenderness is noted in right upper quadrant.
What is best investigation for this case?

a) USG
b) Serology
c) SGOT/LFT
d) CECT

 The patient is having pain in the right hypochondrium which suggests the involvement of
the liver.
 An abscess can be suspected as the patient is suffering from fever accompanied with
rigors & pain and tenderness in the right upper outer quadrant
 Abscess be of two types:
 Pyogenic liver abscess
 Amoebic liver abscess
 On examination the patient is anicteric. So, the patient could be suffering from Amoebic
liver abscess.
 In developing countries like India, Amoebic liver abscess is more common because of
the feco-oral and poor sanitation barrier
 Initial investigation for the diagnosis of amoebic liver abscess can be done by USG and
CT-Scan.
 Investigation of choice or the best investigation for this patient is Serology i.e.
ELISA which detects the anti-bodies formed by the parasite Entamoebahistolytica.
 The amoebic liver abscess can be treated by metronidazole empirically.

4. Comment the diagnosis of this 60 years of man:

a) Basal cell carcinoma


b) Plunging Ranula
c) Epulis
d) SCC of lip

 Ulcero proliferative growth or Cauliflower like growth on the lower lip suggestive of
squamous cell carcinoma of the lower lip
 In total 95%of carcinomas of lip arise the lower lip and 5% arise in central one-third
and commissures.
 Aetiological feature include UV radiation exposure.
 Tumors tend to spread laterally over the mucosal surface.
 Lymph node metastases, usually to submental or submandibular nodes which reflect
aggressive disease behavior then approach.
5. Best treatment strategy for carcinoma anal canal:

a) Chemoradiation
b) Surgery
c) Radiation
d) Chemotherapy

 Nowadays the primary treatment of choice is chemoradiotherapy (Combined modality


therapy (CMT) Nigro Regimen), which usually includes 5-FU, Mitomycin-C and cisplatin
 Radical surgery is indicated in those with persistent or recurrent disease following CMT
or Nigro Regimen.

6. 32 year old female presented to the OPD with complaints of abdominal pain,
diarrhea and weight loss. On further evaluation, patient was found to have multiple
strictures of small bowel with interspersed areas of normal mucosa in between.
Which of the following is false regarding the given condition?

a) No recurrence after surgery


b) Aphthous ulcer
c) Skip lesions
d) Fistula formation

Recurrences occur after surgery in Crohn's disease. Fistula formation, skip lesions and
aphthous ulcers all seen in Crohn's disease.

7. Most common subtype of Rodent ulcer is:

a) Superficial
b) Cystic
c) Nodular
d) Pigmented

 Rodent ulcer is also known as basal cell carcinoma of the skin, or tear cancer which is a
slow-growing type of cancer.
Types of Rodent ulcers:

 Nodular (90%) (MC)


 Pigmented
 Cystic
 Superficial
 The most common type being nodular which has characteristic pearly white nodules in
the periphery with central umbilication and telangiectatic vessels.
8. Which of these is the best for management of a 3 cm stone in renal pelvis without
evidence of hydronephrosis?

a) ESWL
b) PCNL
c) Antegrae pyeloplasty
d) Retrograde pyeloplasty

 Generally, ESWL is a non-invasive method used for removal of renal calculi but the
maximum size of stone that can be removed by ESWL is up to 1.5 cm
 If the size of the stone is more than 2 cm then the preferred treatment option is
Percutaneous nephrolithotomy (PCNL)
 Other contraindications to ESWL are obese patient, pregnancy, & patients taking oral
anti-coagulants.

9. AFP is a tumour marker for which of the following?

a) HCC
b) RCC
c) Oncocytoma
d) Chordoma

Markers of Hepatocellular carcinoma are:


AFP (α -fetoprotein)
PIVKA-2 (Protein induced by vitamin K absence 2)

10. Germ cell tumour not seen in males:

a) Choriocarcinoma
b) Seminoma
c) Sertoli cell tumour
d) Teratoma

WHO classification of testicular tumor:


Germ cell tumors:-

 Seminoma
 Non-Seminomatous GCT
 Yolk sac tumor
 Teratoma
 Embryonal carcinoma
 Choriocarcinoma
11. Dye for Senitnel Lymph Node Biopsy is injected in which of the following sites?

a) Axilla
b) Tail of spence
c) Nipple
d) Areola

 Sentinel lymph node biopsy (SLNB) is done by two techniques:


1. Blue dye technique - 1% lymphazurin (ISO sulfan blue)
2.Radioactive colloid technique-technetium99 labelled Sulphur colloid.

 Maximum accuracy is seen with both techniques combined.


 A radioactive dye is injected around the areola subcutaneously in the subdermal
plexus.

12. Correct description abut Paget's disease of breast:

a) Eczema of skin of nipple


b) Eczema of skin of areola
c) Mastitis carcinomatosis
d) Atrophic scirrhous carcinoma

 Paget's disease of the nipple is a superficial manifestation of an underlying breast


carcinoma.
 It presents as eczema-like condition initially around the nipple and then spreads to
areola.
 In Paget's disease of nipple underlying manifestations are:
 Ductal carcinoma in situ (DCIS).
 Invasive ductal carcinoma.
 Paget's is not associated with lobular breast cancer.
 It is Estrogen and Progesterone Receptor negative.
13. Cut off duration for diagnosis of Priapism is:

a) 1 hour
b) 2 hours
c) 3 hours
d) 4 hours

Priapism: It is defined as a persistent painful erection lasting longer than 4 hours irrespective
of any sexual stimulation & beyond reaching of orgasm.
It is an emergency situation
It is of two types:
1.Ischemic priapism
2.Non-ischemic priapism

14. The image given below indicates which of the following procedures?

a) Percutaneous transhepatic cholangiogram


b) T- tube cholangiogram
c) ERCP
d) HIDA scan

Endoscopic retrograde cholangiopancreatography, or ERCP, is a procedure to diagnose


and treat problems in the liver, gallbladder, bile ducts, and pancreas. It combines X-ray
and the use of an endoscope—a long, flexible, lighted tube.
15. Which of the following is not done in carcinoma esophagus?

a) Biopsy
b) pH - metry/monitoring
c) CT chest
d) PET scan

INVESTIGATIONS of Carcinoma Esophagus :


 1st investigation → Barium Swallow
 IOC for Dx → Endoscopy with biopsy
 IOC for staging → EUS
 IOC for staging in cases of obstructed lumen/ advanced stage: CECT
 IOC for distant metastasis → PET Scan

16. Bell clapper testis predisposes to:

a) Torsion testis
b) Varicocele
c) Cancer of testis
d) Hydrocele

A bell clapper deformity is a predisposing factor in testicular torsion in which the tunica
vaginalis has an abnormally high attachment to the spermatic cord, leaving the testis free to
rotate. Bell clapper deformity predisposes to intravaginal torsion of the testis.
17. A 29 yr old man with a history of varicose veins since the age of 12 presented with
c/o pain, heaviness, and swelling on the right leg. The patient underwent below-
knee stripping of the Long saphenous vein after which the symptoms completely
resolved along with visible varicose veins. The most common complication that
can occur to this patient is:

a) Hemorrhage
b) Thromboembolism
c) Neuralgia
d) Infection

 Ideally, the long saphenous vein should not be stripped below the knee as it causes
injury to the saphenous nerve which is the most common serious complication.
 The incidence of saphenous nerve neuralgia up to 7 % following long saphenous vein
stripping to the knee.
 And the, most commonly injured nerve while stripping the short saphenous vein is sural
nerve whose incidence of injury may be as high as 20%.

18. Most common benign breast tumour:

a) Fibroadenoma
b) Fibroadenosis
c) DCIS
d) Phyllodes tumour

 Fibroadenoma is the most common benign breast tumor which is most commonly
seen in young females.
 Fibroadenoma is also known as "Breast Mouse" due to its characteristic high mobility.
 On the other hand, fibro adenosis is a benign breast diseaseso it is excluded
 On Mammography (if performed on older female) there is popcorn calcification.
 The name DCIS(Ductal carcinoma in-situ) suggests its malignancy and hence can be
excluded.
 Phylloides tumor or Cystosarcoma Phylloides is a malignant tumor.

19. Sebaceous cyst is not seen in:

a) Back
b) Soles
c) Scalp
d) Scrotum

Sebaceous cyst arises from sebaceous glands, commonly related to hair follicles
They usually occur on the scalp, face, ear, back & scrotum
A Sebaceous cyst consists of a punctum usually which discharges on pressing
The punctum is absent in the sebaceous cyst o1 scalp & scrotum.

20. Which of the following is not a Hospital-Acquired infection?

a) Surgical site infection


b) STD
c) UTI
d) Pneumonia

Infection acquired from the environment or the staff following surgery or after admission to
the hospital is termed as Hospital-acquired infection (HAI) which is also referred to as
Nosocomial infections.
There are four main groups of HAI’s
1. Respiratory infections → including Ventilator-associated pneumonia (10-15%)

2. Bacteremia → related to indwelling vascular catheters(10-15% )

3. Urinary tract infections (UTI) Related to urinary catheters(30-40%-MC)


4. Surgical site infection (SSI) (2ndMC)

21. Amount of blood loss in class III circulatory failure/Haemorrhagic shock?

a) <15%
b) 15-30%
c) 30-40%
d) >40%
Class 3: Volume loss from 30% to 40% of total blood volume, from 1500 mL to 2000
mL. A significant drop in blood pressure and changes in mental status occurs. Heart
rate and respiratory rate are significantly elevated (more than 120 BPM). Urine
output declines. Capillary refill is delayed.
 Class 1: Volume loss up to 15% of total blood volume, approximately 750 mL.
Heart rate is minimally elevated or normal. Typically, there is no change in
blood pressure, pulse pressure, or respiratory rate.
 Class 2: Volume loss from 15% to 30% of total blood volume, from 750 mL to
1500 mL. Heart rate and respiratory rate become elevated (100 BPM to 120
BPM, 20 RR to 24 RR). Pulse pressure begins to narrow, but systolic blood
pressure may be unchanged to slightly decreased.
 Class 4: Volume loss over 40% of total blood volume. Hypotension with
narrow pulse pressure (less than 25 mmHg). Tachycardia becomes more
 pronounced (more than 120 BPM), and mental status becomes increasingly
altered. Urine output is minimal or absent. Capillary refill is delayed.
22. Comment on the diagnosis of al film shown of a 65-year -old man with acute
abdomen:

a) Ileocolic intussusception
b) Sigmoid volvulus
c) Toxic megacolon
d) Colocolic intussusception

Notice the significant Clawsign in the given film of a Ba. Enema which is suggestive of
intussusception.

INTUSSUSCEPTION:
M/C type → "ileocolic

M/C type in adults → Colocolic

Since in the question the age of the patient was clearly mentioned as 65 years old. The
diagnosis will be colocolic intussusception.

23. Breast triple assessment contains all except:

a) Clinical examination
b) Axillary sampling
c) USG
d) FNAC and biopsy
Triple assessment, as the name indicates, includes three modalities, physical examination,
imaging (mammography and/or ultrasound), and biopsy (FNAC and core biopsy).

24. Identify the lesion shown below:

a) Wet gangrene
b) Dry gengrene
c) Frost bite
d) Ainhum

From the given image we can figure out the lesion is a gangrene due to the presence of
black necrotic areas.

Feature Dry gangrene Wet gangrene


Site Commonly limbs More common in bowel
Mechanism Arterial occlusion Venous obstruction
Macroscopy Dry, shrunken & black Moist, soft, swollen, rotten and dark

Putrefaction Limited due to less blood supply Marked due to congestion of organ
with blood
Line of demarcation Present No clear-cut line
Bacteria Bacteria fail to survive Numerous bacteria present
Prognosis Better due to septicemia Poor due to profound toxemia
25. 30 year old male presents to the OPD with complaints of lower abdominal pain and
bleeding per rectum. On further evaluation , patient was found to have multiple
superficial ulcers in the rectum and extending upward to involve entire colon. On
histology, only mucosa and submucosa were found to be involved. Surgery is
indicated for the above disease in all of the following conditions except:

a) Toxic megacolon
b) Colonic polyp
c) Dysplasia
d) Massive colonic bleeding

The given clinical condition is ulcerative colitis

Indication for surgery in a patient of Ulcerative colitis:

 Intractability → refractory to medical management


 Dysplasia
 Carcinoma
 Massive colonic bleeding
 Toxic megacolon

26. Patient complains of painless swelling over the face with difficulty in swallowing.
The appearance of face is shown. The probable diagnosis is:

a) Acute parotitis
b) Cancer of parotid gland
c) Angioedema of face
d) Acute sialadenitis

 Painless swelling usually points towards malignancy.


 Among the given options Acute parotitis and Acute sialadenitis can be excluded
because they both are painful condition.
 Angioedema of the face is characterized by the presence of swelling around the lips
and around the oral cavity.
 Hence the answer is cancer of the parotid gland
 Among malignant tumors of the parotid gland, mucoepidermoid carcinoma is the most
common followed by adenocystic carcinoma.

27. A 35-year-old lady the presented with a 6-month painless fluctuant, non-
transilluminant swelling with a thin watery discharge. Clinical diagnosis is?

a) Brachial cyst
b) Secondaries
c) TB
d) Lymphoma

Among the given options discharge can only be seen with the TB which has
characteristic undermined edges, which can be identified easily from the given image.
The branchial fistula may, however have discharge but around the anterior border of SCM
and doesn't have undermined edges.
Lymphoma has a lump, anorexia, weight loss but no discharge

28. The following patient has presented after chest trauma. On examination crepitus is
felt. The clinical diagnosis is?

a) Subcutaneous emphysema
b) Gas gangrene
c) Acute tubular necrosis
d) SVC syndrome
 From the given image, you can notice that the patient has puffy eyes and
swelling around the face and crepitus is felt when palpated.
 This type of appearance is usually noticed in Sub-cutaneous emphysema.
 It is advised not to confuse between gas gangrene and subcutaneous emphysema as
both the conditions have crepitus.
 Gas gangrene is a condition that is caused by CI. Perfiringens which has a
gangrenous type of appearance with crepitus due to clostridial myonecrosis and
takes more than 24 hours to appear.

MEDICINE DEC 2018

1. Which of the following is not done in W.P.W?

a) Treadmill test
b) Electrophysiological studies
c) Oral beta blocker
d) Procainamide

C/I of treadmill test:


1) severe Aortic Stenosis
2) DVT
3) Charcot's joint
4) WPW
5) MI in first 48 hours
6) Unstable angina during attack
7) Buerger's disease
8) Bilateral osteoarthritis of knee

2. What is the impact on fetus in case of use of Indomethacin in utero in third


trimester?

a) Patent ductus arteriosus


b) Early closure of ductus arteriosus
c) Ventricular septal defect
d) Atrial septal defect

Prostaglandins helps to keep ductus arteriosus open.


Indomethacin inhibits prostaglandins levels → So, closure of ductus arteriosus →
DOC for PDA

3. In fever of unknown origin, blood sample should be drawn how many times?

a) 2
b) 3
c) 4
d) 5

Criteria for fever of unknown origin-


1. Fever 101.30 F on 2 separate occasions
2. For > 3 weeks
3. Absence of immunocompromised state
4. Adequate workup-

 Blood culture > 3 times should be normal


 Urine analysis- sterile
 CRP normal
 Antinuclear antibody negative
 LDH normal

4. Which of the following is correct about cancer developing in Ulcerative colitis?

a) Bad prognosis if it involved only rectum


b) Good prognosis if it involves only rectum and is low grade
c) 2% chance of malignancy if present for 10 years
d) Young age onset has low chances as compared to old age onset

 Cancer developing in ulcerative colitis is directly proportional to duration and extent of


disease.
 More the duration and extent more chances of cancer to develop
 Incidence of malignancy is 2% if disease persist for 10 years
 Incidence of malignancy is 18% if disease persist for 30 years

5. Refractory Septic shock is defined as?

a) Shock persisting > 1 hour after IVF administration


b) Shock persisting > 1 hour after IVF administration and high dose vasopressors
c) Shock that is refractory to medical treatment
d) Shock that does not improve despite treatment for 24 hours
 Refractory Septic Shock :
Septic shock that last for 1 hour and does not respond to fluid or pressor administration.

 Septic Shock :
Sepsis with hypotension despite adequate fluid resuscitation for at least 1 hr, with perfusion
abnormalities. or Need of vasopressors to maintain the blood pressure.

6. Waist to hip ratio that increases risk of heart disease is?

a) 0.80 in male
b) 0.80 in female
c) 0.85 in male
d) 0.85 in female

Waist to hip ratio that increases risk of heart disease for male is > 1
Waist to hip ratio that increases risk of heart disease for female is > 0.85

7. A 65-year-old, male with hypertension develops sudden onset weakness on the


right side of the face & arm. His attender recalls history of headache, blurred
vision in the patient which leads to the suspicion of increased ICP. Which is the
most common cranial nerve involved in raised ICP?

a) Abducens nerve
b) Trochlear nerve
c) Trigeminal nerve
d) Facial nerve

 Abducens= Longest subarachnoid course (so whenever ICP will raise abducens nerve
will affect first )
 Trochlear=Longest intracranial route, thinnest, only nerve has dorsal origin
 Trigeminal=Thickest nerve
 Facial= Longest intraosseous route

8. After trauma a patient is unable to smell coffee and asafetida but can smell
ammonia. Which of the following statement is true about the patient?

a) Ammonia is used for testing olfactory nerve


b) The olfaction system is not damaged
c) Ammonia activates trigeminal nerve
d) All of the above
Ammonia cannot be used as test to check olfactory because it is an irritant not smell, it
stimulates trigeminal nerve not olfactory.

9. Deepika a 26 years old female , labourer by occupation presented to you with


complain of pain in abdomen, nausea and restlessness since 4 hours. She gives
h/o 2 episodes of hematuria. USG showed a 3*3 cm stone in Right renal pelvis.
There is no evidence of hydronephrosis . Which of following is the best procedure
for management of this patient ?

a) ESWL
b) PCNL
c) Antegrade Pyeloplasty
d) Retrograde pyeloplasty

 ESWL- done for stone less than 2.5 cm size


 PCNL-if stone is > 2.5 cm will treat by PCNL
 Most common kidney stone → calcium oxalate
 Etiology=idiopathic hypercalciuria

10. A new vaccine which was launched in 2018 in Thailand with the name CYD-TDV. In
which of the following condition it is used?

a) Dengue
b) Malaria
c) Yellow fever
d) Japanese encephalitis

 CYD-TDV vaccine introduced in Thailand in 2018 (DENGVAXIA) is used for


Dengue.
 Vaccine is tetravalent
 Live attenuated vaccine
 Dengue is a mosquito (Aedes aegypti) borne viral (Arbovirus) infection.

11. A 45 year old male who had a history of domestic animal bite has come for follow
up post 90 days after receiving rabies post-exposure prophylaxis. Now what will
you use for Rapid diagnosis of rabies in this case ?

a) Skin biopsy with fluorescent antibody testing


b) Rabies virus specific antibodies
c) Inoculation in mouse
d) Corneal impression smear
 Fluorescent Antibody Test : This is a highly reliable and the best single test currently
available for the Rapid diagnosis of Rabies viral Antigen in Infected specimen.
 Corneal impression & Mouse inoculations are also used for diagnosis.

12. Common neurological tumour in NF-2?

a) Acoustic neuroma
b) Optic glioma
c) Café-Au-Lait macules
d) Meningioma

ACOUSTIC NEUROMA: It is an autosomal dominant condition it is an eighth nerve tumor. It


compresses adjacent cranial nerves therefore multiple palsies develop.
OPTIC GLIOMA: occurs in NF1
NF 1 Characterized by presence of cafe au lait macule or small potato like Lesion all over
the Skin.

13. A 7-year-old with fever for days with pain in right hypochondrium. On examination
tenderness is noted in Right upper quadrant. What is the next best investigation?

a) USG
b) Serology
c) SGOT/LFT
d) CT abdomen

The next best investigation in this case is to do an USG.


Causes of Right upper quadrant pain in pediatric-
1. Acute viral hepatitis - In viral hepatitis on USG= starry sky appearance seen***
2. Acalculous cholecystitis
3. Budd Chiari syndrome
4. Acute heart failure

14. Which of the following is not seen in Horner syndrome?

a) Mydriasis
b) Ptosis
c) Anhidrosis
d) Enophthalmos
Horner syndrome is characterized by damage to sympathetic chain
Features are – “SAMPLE”
Sympathetic Nerve Fiber Injury
Anhidrosis
Miosis
Ptosis
loss of ciliospinal reflex
Enophthalmos

15. Which of the following is not correct about Defibrillation?

a) Easy for untrained person


b) Decrease in success rate with delay in initiation
c) Improve prognosis
d) minute gap between 2 shocks

Yes, Defibrillation can be given by non-medico also


Basic life support protocol (BLS protocol) = 30:2 in 2-minute gap

16. The following image shows?

a) Achalasia cardia
b) Diffuse oesophageal spasm
c) Carcinoma Oesophagus spasm
d) Schatzki's Ring

Diffuse esophageal spasm - Cork screw appearance


Achalasia cardia - Bird peak appearance
Carcinoma esophagus - Apple core appearance.

17. Which of the following features are true about Allergic Broncho Pulmonary
Aspergilloma?
1)Distal bronchiectasis
2)Serum precipitins to Aspergillus
3) Increased IgE Levels
4)Seen in asthmatics

a) 1,2,3 only
b) 2,3,4 only
c) 1,3,4 only
d) All of the above

18. Mark the true statements -


1. MEN 2B is also known as Sipple syndrome
2. MEN 1 is also known as Werner syndrome
3. Marfanoid Habitus is a feature of MEN 2B
4. MEN 4 is associated with CDKN1B

a) 1 and 2 are true


b) 3 and 4 are true
c) 1,2 and 4 are true
d) 2,3 and 4 are true

MEN 2A - SIPPLE Syndrome


MEN1 was originally known as Wermer syndrome

19. Which of the following is not a component of child pugh Classification

a) SGOT
b) Bilirubin
c) Albumin
d) Prothrombin time
The Child-Pugh (CP) classification is the standard to assess preoperative liver function and
is determined by five factors: serum bilirubin and albumin levels, prothrombin time,
ascites, and encephalopathy.

20. Fulminant diabetes mellitus is seen in?

a) Diabetic ketoacidosis
b) Coxsackie B virus
c) Non ketonic hyperosmolar coma
d) Autoimmune pancreatitis

 Fulminant diabetes mellitus - caused by Coxsackie B virus (viral myocarditis)


 Coxsackie B virus:
 Herpangina
 Hand foot mouth virus.

21. Which is correct about rheumatoid nodules?

Tender, located on extensor surface and seen with arthritis


Non tender, located on extensor surface and seen with arthritis
Non tender, located on flexor surface and seen with arthritis
Tender, located on flexor surface and seen with arthritis

 Seen in extensors such as olecranon process, occiput, tips of spinous process of


vertebrae.
 Rheumatoid nodules are MC extra articular manifestation of Rheumatoid arthritis

22. Modified duke criteria are used for diagnosis of?

a) Infective endarteritis
b) Infectious mononucleosis
c) Inflammatory myopathy
d) Infective endocarditis

 The Modified Duke Criteria for the Clinical Diagnosis of Infective Endocarditis
 Duke's staging - Cancer of rectum
 Duke's score - Chronic stable angina
23. Which of the following shall be seen with use of a small size BP cuff?

a) False elevation of BP
b) Falsely low value of BP
c) Cancels the effects of calcified arteries
d) Increases trans arm impendance

 SMALL SIZED CUFF: It leads to false elevation of BP as Blood vessels are squeezed
so there is an increase in the peripheral resistance
 BIG SIZED CUFF: It leads to falsely low measure of BP

24. Which is correct about larval stage of taenia solium?

a) Cysticercosis cellulose
b) Larva currens
c) larva migrans
d) Visceral larva migrans

Cysticercosis is a common helminthic infection caused by the larval stage of pork tapeworm,
Taenia solium. The larvae, also called as cysticercosis cellulose, reside within the
muscles and other tissues of pigs that act as their intermediate hosts.

25. On putting an internal jugular vein catheter, a patient has developed sudden onset
severe respiratory distress. Clinical diagnosis is?

a) Pneumothorax
b) Sepsis
c) ARDS
d) Infective endarteritis

The catheter penetrated the jugular vein and traumatized the lung parenchyma
damaging the lung tissues leading to PNEUMOTHORAX.

26. You are an intern, posted in the neurology ward in the Department of Internal
Medicine. A 48-year-old woman was admitted with a history of muscle weakness.
Investigations & examination revealed normal nerve conduction but flaccidity is
seen. Which of the following would be the possible diagnosis?

a) Myasthenia gravis
b) GBS
c) Transverse myelitis
d) Traumatic neuritis
Myasthenia Gravis- Problem in Neuromuscular junction due to improper functioning of Ach
receptor, so nerves are not damaged. neuromuscular junction is affected leading to muscle
weakness.
In option “b,c,d” the nerve conduction is always abnormal .

27. Which of the following is the most sensitive test for the diagnosis of
Organophosphate poisoing ?

a) Plasma acetylcholinesterase
b) Organophosphate level in blood
c) Organophosphate level in plasma
d) RBC transaminase levels

The most commonly used test to confirm acute organophosphate poisoning


is measurement of plasma cholinesterase activity. Plasma cholinesterase levels usually
decline to less than 50% of the normal value before any symptoms of poisoning are
observed.

28. Extensor Reflex on pinching gastrocnemius muscle is called?

a) Gower Sign
b) Homan Sign
c) Oppenheim Sign
d) Gordon Sign

On squeezing or pinching the calf muscles (gastrocnemius), the Big toe goes away -
GORDON SIGN

29. Reversible dementia is seen in?

a) SACD
b) AD
c) CJD
d) Pick's disease

Vit B12 deficiency Reversible dementia is seen


SACD Subacute combined demyelination of spinal cord.
Reversible Dementia Irreversible Dementia
 Hypothyroidism  Alzheimer disease (MC cause of senile
dementia
 Normal pressure Hydrocephalus  CJD (Creutzfeldt-jakob disease)
o Caused by Prion Disease
o Prion mediated damage.

 Whipple Disease (Bacteria: T. Whippli)  Pick's Disease


(Frontal part of brain is affected)

 Vit B12 ↓

RADIO DEC 2018


1. Identify the pathology in the shown CT:

a) Intraventricular bleed
b) Massive Epidural hemorrhage
c) Subdural hemorrhage
d) Subarachnoid hemorrhage

Extradural Hemorrhage (EDH):

 Lentiform shaped, Biconvex , Lemon shape


 Middle meningeal artery
 #pterion
2. A 45-year-old female presents with a complaint of abdominal pain and fever for the
past 3 days. She localizes the pain to her epigastric area and states that it radiates
to her right upper quadrant. . She notes that it became markedly worse after eating
dinner last night. Her past medical history is significant for hypertension and
hypercholesterolemia. On Abdominal examination, Inspiratory arrest is noted on
deep palpation of the right hypochondrium. Which of the following is the
Investigation of choice for this condition?

a) H.I.D.A Scan
b) USG
c) CT Scan
d) Biopsy

 IOC for Acute Cholecystitis: USG


 USG is the IOC for most GB pathologies
 Most Accurate Investigation for Acute Cholecystitis: H.I.D.A Scan (H.I.D.A is used for
equivocal cases where USG is not conclusive )
 IOC for Abdominal trauma :CECT

3. Which of the following investigation work on the same principle?

a) CT and MRI
b) CT and X-ray
c) USG and HIDA Scan
d) MRI and PET Scan

Principle of:

 CT : Linear attenuation of X-rays


 X-ray: X-rays (Fluoroscopy also works on the principal of X rays)
 MRI: Gyromagnetic property of H+
 USG: Pulse-Echo Principle with Piezoelectric effect
 PET scan: Annihilation and coincidence imagine, gamma rays -511 key
 HIDA Scan: Radioisotope scintigraphy, gamma rays.

4. Comment on the diagnosis of the image shown:

a) Pneumoperitoneum
b) Pneumothorax
c) Eventration of diaphrgram
d) Pneumomediastinum
Pneumoperitoneum - "Air below Diaphragm" is the characteristic feature

ANAESTHESIA DEC 2018


1. Preoperative assessment of oral cavity for endotracheal intubation in an adult is
done by:

a) Mallampati score
b) Cormack lehane
c) ASA grading
d) AHA grading

 Mallampati score - preanesthetic evaluation of airway in which we are evaluating


size of tongue with respect to oral cavity.
 Cormack lehane - laryngoscopic view of glottic opening.
 ASA grading - preanesthetic evaluation by which we assess risk associated with
surgery.
 AHA grading- done in cardiac patients.

2. Which of the following Anaesthesia agent is hepatotoxic:

a) Sevoflurane
b) Isoflurane
c) Halothane
d) Procaine

Drugs causing Hepatotoxicity


ANAESTHETIC AGENTS :

 Halothane
 Chloroform
 Isoflurane, Enflurane & Desflurane
 Nitrous oxide.

3. Anaesthesia given to dog:

a) Ketamine
b) Medetomidine hydrochloride
c) Midazolam
d) All of the above

Any I/M injectable drug which we are using in human can be used in canines as well.
4. Shortest action local anaesthetic:

a) Procaine
b) Cocaine
c) Dibucaine
d) Chloroprocaine

Shortest Acting LA → Chloroprocaine

Longest acting LA → Dibucaine

5. During laryngoscopy, best method for checking whether the intubation is correct
or not:

a) Capnography
b) Air entry on auscultation
c) Ultrasonography
d) Chest movement

 Best method to check correct placement of endotracheal tube (ETT) is capnograph


 Put the tube, inflate the cuff, attach a device (capnograph) if tube has Co2, the
graphical representation of Co2 comes in the monitor then we confirm tube is rightly
placed.

ORTHO DEC 2018


1. Blount Disease is involvement of

a) Proximal tibia
b) Distal tibia
c) Proximal femur
d) Distal femur

Blount's disease is a progressive bow-leg deformity associated with abnormal growth of


posterior-medial part of proximal tibia.
2. Involvement of DIP PIP and Sparing of MCP and wrist is seen in:

a) Osteoarthritis
b) Rheumatoid arthritis
c) Psoriasis
d) Hyperparathyroidism

 Sparing of MCP is a feature of OA


 DIP is spared in RA
 In psoriasis all joints are involved but most specifically DIP is involved
 In Hyperparathyroidism all the bones are involved.

3. Tennis player can spontaneously reduce a shoulder dislocation. He can do it again


and again himself. He is suffering from?

a) Acute shoulder dislocation


b) Recurrent shoulder dislocation
c) Inferior shoulder dislocation
d) Fracture upper end humerus

 Most common shoulder dislocation - Anterior type of shoulder dislocation which


accounts for 95%. Since, it was mentioned that the player can reduce the dislocation
"again and again", it could be a habitual type of dislocation which is recurrent.
 Acute shoulder dislocations are very painful which cannot be reduced or relocated
again and again.
 Inferior type of shoulder dislocation usually presents with a characteristic "salute
position" hence car be excluded.

4. Adamantinoma affects which of the following ?

a) Radius
b) Humerus
c) Femur
d) Tibia

 Epithelial tumors are classified into Ameloblastoma and adamantinoma, though both the
tumors show same type of histopathology, they are not interchangeable.
Ameloblastoma Adamantinoma
Site Mandible Tibia

 Most common site for Ameloblastoma is mandible


 Most common type of cancer of mandible is squamous cell carcinoma.

5. A lady had a trauma to the neck. X-ray is attached. What is the diagnosis?

a) Hangman's fracture
b) Jefferson's fracture
c) Clay shoveler fracture
d) Sciworal

 Hangman's fracture is fracture of pars interarticularis of C2 and slip oi C2 over C3 as


shown in the image.
 Jefferson's fracture involves Atlas (C1).
 Clay shoveler fracture involves the spinous process of lower cervical spine.
 Sciwora usually occurs in children <8yrs old. It is a spina cord injury without obvious
radiological abnormality.
6. Patient had an injury to thumb causing thumb abduction. Which of the following
can happen ?

a) Game keepers thumb


b) Bowler's thumb
c) Kaplan injury
d) Mallet finger

 Thumb abduction leads to ulnar collateral ligament damage. It is the most common injury
of metacarpal phalangeal joint. Game keeper's thumb occurs due to forceful abduction of
thumb as seen in gamekeepers who twist necks of small animals.

7. In fracture of upper 1/3 of forearm, it is immobilized in:

a) Pronation
b) Supination
c) Mid prone
d) Any position

Immobilization of forearm at various fracture sites: 'SMP"

 Upper 1/3rd - Supination


 Middle 1/3rd - Midprone
 Lower 1/3rd – Pronation

8. A 80 yr old lady had slip and fall, the following X-ray was taken. Which of the
following is treatment?

a) Hemiarthroplasty
b) Hip spica
c) Total Hip Replacement
d) Nailing
X-ray shows fracture in the neck of Femur. The treatment for neck of femur fracture is
based on age:

 < 65 years old → Fixation


 > 65 years old → Hemiarthroplasty
 Total hip replacement is done for arthritis of any age group. X- ray shows no arthritis.
 Nailing is used for shaft of femur fracture.

9. A child comes with limp and limb shortening. X-ray is below, what is given the
diagnosis?

a) Perthes
b) SCFE
c) DDH
d) Fracture neck of femur

X-ray show normal right side of hip joint. In left side of hip joint, the epiphysis is smaller,
shallow acetabulum and the hip joint is dislocated- indicates Developmental Dysplasia of Hip
(DDH).

PSYCHIATRY DEC 2018

1. Management of violent patient in psychiatry is treated with all except:-

a) BZD
b) CBT
c) ECT
d) Haloperidol

Usually a violent patient in Psychiatry comes with a condition of mania or schizophrenia.


 BZD can be used to decrease agitation
 ECT(Electro convulsive therapy) can bring the symptoms down immediately.
 Haloperidol inj. can be used in the treatment of acute agitation and it is
considered as the best treatment for acute agitation

 CBT (cognitive behavioral therapy)takes longer i.e. months of therapy SO can't be used
in emergency management of a violent patient. Cognitive behavioral therapy (CBT) is
usually short-term approach, typically with 12 to 20 sessions. CBT is collaborative,
structured, and goal-oriented intervention that targets core components of a given
disorder.

2. Double depression is:-

a) Major depressive disorder from 2 years


b) Major depression with OCD
c) Depression with dysthymia
d) Depression with anxiety attack

HOW TO REMEMBER: double = D+D = Depression + Dysthymia

Double depression: Patient has symptoms of Dysthymia (mild depression symptoms


continuing for 2 years) the patient develops superimposed major depressive episode.
Major depressive disorder from 2 years is called chronic depression.

3. Most commonly psychiatric illness seen in medico-surgical patient -

a) Depression
b) Schizophrenia
c) Delirium
d) Dementia

Patients who have medical comorbidities, surgical comorbidities or the postoperative


patients are vulnerable to delirium. Especially if they are elderly.

4. Negative symptoms of schizophrenia are all except.-

a) Over activity
b) Anhedonia
c) Alogia
d) Apathy
Negative symptoms of Schizophrenia :

 Avolition: loss of drive for meaningful activities.


 Apathy: lack of concern.
 Anhedonia: lack of pleasure.
 Asociality: lack of social interaction.
 Affective flattening/Emotional blunting.
 Alogia: decreased verbal communication.
 Do not respond well to medications.**
 Poor prognostic factor.**

5. Most common acute organic mental disorder:-

a) Delirium
b) Dementia
c) Amnesia
d) Anxiety disorder

Delirium: Always has an acute onset, there's disturbance of consciousness, has fluctuating
course.
Dementia: Always has an insidious onset, and there's no disturbance of consciousness, has
a progressing course.

DERMA DEC 2018


1. Identify condition in the shown image:-

a) Impetigo
b) Chickenpox
c) Herpes Simplex
d) Molluscum contagiousm
Impetigo is a common and highly contagious skin infection that mainly affects infants
and young children. It usually appears as reddish sores on the face, especially around the
nose and mouth and on the hands and feet. Over about a week, the sores burst and develop
honey-colored crusts.

2. What are these horizontal lines called?

a) Lymphatic development
b) Dermatomes
c) Blaschko lines
d) Vascular development

Blaschko lines are consistently V-shaped on the upper spine, S-shaped on the abdomen,
inverted U-shaped from the breast area to the upper arm, and perpendicular down the front
and back of the lower extremities. They never cross the anterior truncal midline but run along
it.

3. Which of the following is used for treatment of vitiligo?

a) Retinoids
b) Systemic steroids
c) Psoralen PUVA
d) Nd-YAG laser
Vitiligo is an autoimmune disease, can only be treated with immunosuppressants.
Treatment of choice PHOTOTHERAPY
Target Phototherapy is used in vitiligo with Excimer laser.
PUVA Given with UV radiation.

4. Tzanck Smear shown in the image shows:

a) Acanthosis
b) Acantholytic cells
c) Smudge cells
d) Anaplasia

On Tzanck smear if we see

 Acanthocytes - Pemphigus
 Multi nucleated giant cells - Herpes

Acanthocytes/Acantholytic cells:

 Oval
 Single cell with nucleus
 Perinuclear halo
ANATOMY JUN 2019

1. Failure of closure of rostral neuropore at 25th day leads to ?

a) Hydranencephaly
b) Rachischisis
c) Anencephaly
d) Spina bifida

Anencephaly is the absence of a major portion of the brain, skull, and scalp that
occurs during embryonic
development. It is a cephalic disorder that results from a neural tube defect that
occurs when the rostral (head) end of the neural tube fails to close, usually between
the 23rd and 26th day following conception.

2. Killian dehiscence is present in ?

a) Superior constrictor muscle


b) Inferior constrictor muscle
c) Middle constrictor muscle
d) Esophageal fibres

Killian's dehiscence is a potential triangular gap between the oblique fibers of


thyropharyngeus and transverse fibers of cricopharyngeus (Thyropharyngeus and
cricopharyngeus are 2 parts of the inferior constrictor of pharynx)
3. Which of the following gland is supplied by glossopharyngeal nerve ?

a) Submandibular salivary gland


b) Lacrimal gland
c) Parotid salivary gland
d) Sublingual salivary gland

4. Inferior scapular angle is at which level ?

a) T2
b) T4
c) T6
d) T8
5. What is the structure highlighted ?

a) Supraspinatus
b) Infraspinatus
c) Teres major
d) Teres minor
Muscle highlighted in this picture is TERES MINOR

Origin Lateral border of scapula

Insertion Greater tubercle of humerus

Action Shoulder joint: Arm external rotation, arm


adduction;
Stabilizes humeral head in glenoid cavity

Innervation Axillary nerve (C5, C6)


Blood supply Suprascapular artery, dorsal scapular
artery

6. Which single muscle is involved in the actions of arm's flexion, adduction


and medial rotation?

a) Serratus Anterior
b) Pectoralis major
c) Pectoralis minor
d) Subclavius

Muscle Action
Pectoralis major Adducts and medial rotation of arm
Pectoralis minor Stabilizes the scapula
Subclavius anchors and depresses the scapula
Serratus Anterior  protracts the scapula
 laterally rotates the scapula
 overhead abduction

7. One of the nerves supplying the upper limb, the upper lateral cutaneous
nerve of arm, supplying the skin overlying the posterior border of the
deltoid muscle as well as covering the long head of the triceps, is a branch
of:-

a) Musculocutaneous nerve
b) Radial nerve
c) Ulnar nerve
d) Axillary nerve

Axillary Nerve
This nerve originates from the posterior cord of the brachial plexus with contributions
from the C5 and C6 ventral rami. The axillary nerve travels through the inferior-
lateral axilla, and around the surgical neck of the humerus with the posterior
circumflex humeral artery. After providing motor innervation to the deltoid, the
axillary nerve gives off the lateral superior cutaneous nerve branch which
innervates the lateral skin of the shoulder

8. Which muscle originates from medial epicondyle of humerus (FMGE June


2019)

a) Supinator
b) Pronator quadratus
c) Pronator teres
d) Brachioradialis

PRONATOR TERES-Humeral head: medial supracondylar ridge of humerus Ulnar


head: Coronoid process of ulna

SUPINATOR- Lateral epicondyle of humerus, radial collateral ligament, annular


ligament, supinator crest of ulna

PRONATOR QUADRATUS- anterior surface of ulna.

BRACHIORADIALIS- muscle originates from the upper two-thirds of the lateral


supracondylar ridge of humerus and the anterior surface of the lateral inter-muscular
septum of the arm

9. An injury to the shown area can lead to fracture of which bone ?

a) Lunate
b) Scaphoid
c) Hamate
d) Trapezium
Anatomical snuff box: The given picture shows anatomical snuff box which Is a
depression formed at the
posterolateral side of the wrist and metacarpal by the extensor tendons passing into
the thumb. Base of the triangle is at the wrist and apex is directed into thumb. It is
most apparent when thumb is extended.

Boundaries:
Lateral: Abductor pollicis longus and extensor pollicis brevis tendons.
Medial :Extensor pollicis longus tendon
Floor: Scaphoid and trapezium the distal ends of the tendons of the extensor carpi
radialis longus and extensor carpi radialis brevis

Scaphoid Fracture:
The commonest carpal injury is a fracture across the waist of the scaphoid bone It is
uncommon to see other injuries. In approximately 10% of individuals, the scaphoid
bone has a sole blood supply from the radial artery, which enters through the distal
portion of the bone to supply the proximal portion. When fracture occurs across the
waist of the scaphoid, the proximal portion therefore undergoes avascular necrosis.

10. A patient presents with hypothenar muscle wasting and loss of sensation
of medial one and a half digits. Which nerve is involved ?

a) Musculocutaneous
b) Radial
c) Ulnar
d) Median
11. Testicular artery is a branch of -

a) Abdominal aorta
b) Common iliac artery
c) External iliac artery
d) Internal iliac artery

Branches of Abdominal aorta:

Segment Branches

 Ventral branches:
 Celiac trunk -T12
 Superior mesenteric artery - L1
 Inferior mesenteric artery - L3

 Dorsal branches
 4 pairs of lumbar artery
 Median sacral artery

 Lateral branches
 Inferior phrenic artery (1t branch)
 Middle suprarenal artery
 Renal artery
 Gonadal artery
 Terminal branches Common iliac arteries

12. Deep inguinal ring is formed in

a) External oblique aponeurosis


b) Internal oblique aponeurosis
c) Transversus abdominis
d) Transversalis fascia

The deep (internal) ring is found above the midpoint of the inguinal ligament. which is
lateral to the epigastric vessels, The ring is created by the transversalis fascia, which
invaginates to form covering of the contents of the inguinal canal.

13. Inguinal ligament is formed due to

a) Cremaster muscle
b) External oblique aponeurosis
c) Internal oblique aponeurosis
d) Transversus abdominis
The inguinal ligament(also known as Poupart's ligament or the groin ligament) is a
fibrous band extending from the anterior superior iliac spine to the pubic tubercle.
It is formed by the external oblique aponeurosis and is continuous with the fascia lata
of the thigh.

14. Abduction at the hip joint is done by

a) Gluteus maximus
b) Gluteus medius
c) Obturator internus
d) Quadratus femoris

Gluteus medius is the prime mover of abduction at hip joint. Anterior portion of
Gluteus medius abduct, assist in flexion and medial rotation of hip. Posterior portion
of Gluteus medius abduct, assist in external and lateral rotation of hip.

15. Identify the marked tendon in the following image ?

a) Extensor hallucis longus


b) Flexor hallucis longus
c) Extensor digitorum longus
d) Tibialis anterior

Extensor Hallucis Longus(EHL):


Origin : Middle one third of anterior surface of Fibula and adjacent interosseous
membrane.
Innervation: Deep fibular nerve
Action: Extension of greater toe and dorsiflexion of foot.

ANASTHESIA JUN-2019

1. In CPR, number of chest compression per minute in an adult:

a) 30-50 per minute


b) 50-72 per minute
c) 100-120 per minute
d) 120-200 per minute

Rate : 100-120/ minute

Depth:
Adult - 5-6 cm
Pediatric - 4-5 cm
Infants - At least 4 cm

Allow complete recoil for proper filling of heart


Adequate ventilation avoid excessive ventilation

2. Spinal anaesthesia in an adult is given at this level:

a) T12-L1
b) L1-L2
c) L3-L4
d) L5-S1

Spinal cord ends at lower border of L, in adults .Hence spinal anesthesia can be
given at levels below L1. L2-L3 or L4-L5 are considered best in adults.

3. Colour of nitrous oxide cylinder is?

a) Blue
b) Blue body with white shoulder
c) White
d) Black
Gas State in cylinder International USA
Air Gas Black body and Yellow
shoulders with
black and white.
Oxygen Gas Black body and Green
white shoulders
Nitrous oxide Gas + liquid(below Blue Blue
98oF)
Carbon oxide Gas + liquid(below Grey Grey
88oF)
Helium Gas Brown Brown
Entonox Blue body and
shoulders with blue
and white quarters
Cyclopropane Orange Orange
Nitrogen Gas Black Black

4. The duration of spinal anaesthesia is based directly on:

a) Dose
b) Height
c) Age
d) Total body fat

The duration of spinal anesthesia primarily depends upon the dose, intrinsic
properties of local anesthetic
used, use of additives
The level and duration is directly based on dose of the drug

5. On repeated use, which of the following inhalational anaesthetic agent can


cause hepatitis:

a) Isoflurane
b) Halothane
c) Sevoflurane
d) Ether

Halothane causes auto immune hepatitis


Halothane produces byproduct during metabolism which causes autoimmune
hepatitis
Auto immune hepatitis d/t repeated use of halothane, preexisting liver dysfunction
More seen in female patients, obese patients & middle age patients
6. Which of the following is used for day care surgery?

a) Ketamine
b) Thiopentone
c) Propofol
d) Etomidate

 Propofol (Single Best Drugs for Day


Care Surgery)
 Desflurane / sevoflurane
 Fentanyl/ Alfentanil / Sufentanil /
Remifentanil
 Midazolam
 Intermediate or short acting
neuromuscular blocker (vecuronium
, rocuronium, atracurium ,
cisatracurium)
 Beta blocker – Esmolol
Drugs Used in Day Care Surgery  Vasodilator - Nitroglycerine, sodium
nitroprusside

Anesthetic agents to be avoided in


Day Care Surgery :

1. Thiopentone
2. Halothane
3. Morphine
4. Long acting neuromuscular blockers
5. Diazepam / Lorazepam

MICROBIOLOGY JUNE- 2019

1. Which is correct about Diphtheria membrane?

a) Grey membrane that bleeds on removal


b) Grey membrane that does not bleed on removal
c) Grey pseudo-membrane that bleeds on removal
d) Grey pseudo-membrane that does not bleed on removal

Diphtheria presents with grey pseudo-membrane that bleeds on removal.


Pseudo membrane on posterior pharynx is a tough leathery coat composed of inner
band of fibrin surrounded by neutrophils, RBCs and bacteria.
Pseudo membrane:

 It is adherent to basal mucosa and bleeds on removal.


 It can be localized ( Tonsillar, pharyngeal etc) or widely distributed i.e.in the entire
tracheobronchial tree
 Most common site - faucial pillars (Tonsillar).
 Most dangerous site laryngeal as it causes respiratory obstruction.

Bull's neck appearance seen in diphtheria is due to cervical edema and cervical
lymphadenopathy.

Bull's neck appearance:


2. DNA transfer in bacteria via phage is:

a) Conjugation
b) Transduction
c) Transformation
d) Translation

Transformation
 There is direct uptake of donor DNA by the recipient cell, which may be natural
or forced.
 Transduction - donor DNA is carried in a bacteriophage and is transferred
to the recipient by the mechanism used for phage infection , two type
lytic and lysogenic
 Lysogenic conversion, bacteriophages exhibit two types of cycles, virulent or
lytic cycle where the host bacterium is lysed and the temperate or non-lytic
cycle where host bacterium is unharmed. Eg diphtheria
 Conjugation- The donor cell contributes energy and building blocks to the
synthesis of a new DNA strand, which is physically transferred into the
recipient cell, usually by tube called sex pilus.
 Transferable drug resistance mediated by the factor is the most important
method of drug resistance
 Mutational resistance is of great importance in tuberculosis. If a patient is
treated with streptomycin alone, initially the appear and multiply unchecked. If
two or more anti-re-population by resistant mutants does not occur, as a
mutant resistant to one drug will be destroyed by the other drug.

3. Sporulation occurs in which phase of bacterial growth curve:

a) Stationary phase
b) Lag phase
c) Log phase
d) Decline phase
1.Lag phase- Adaptation phase to culture media decreased Accumulate nutrient
size
2. Log Phase - Exponential increases in number
3. Stationary phase - Start of Accumulation of toxic metabolites = Dead
Live -Sporulation occur in stationary phase (S-S)
4. Decline - Complete accumulation of Toxin metabolites

4. Bacillus Anthrax is:

a) Gram positive cocci in cluster


b) Gram positive rods with square ends
c) Gram positive bacili with spherical ends
d) Gram negative cocci in cluster

Gram Positive bacilli

Bacillus Anthrax-
 Gram positive rods with square ends ( non bulging spore) (bulging spore seen in
cl. tetani)
 GPB, Non Motile, capsulated.
 Mc fadyeans reaction

M - Mc'fadyean reaction
Medusa Head appearance
I- Inverted fir tree appearance
S - String of pearl reaction
Bamboo - Bamboo stick appearance ( on Gram stain)

5. Which of the following is the most likely cause in a case of granuloma with
positive AFB?

a) Cat scratch disease


b) Trench fever
c) Leprosy
d) Syphilis

 Granuloma with positive AFB Is Leprosy caused by M. leprae. M leprae is acid


fast bacilli.
 Syphilis caused by Treponema pallidum (spirochaete), which is not stained by
acid fast stain. Diagnosed by dark ground microscopy and other serological tests.
 Trench fever caused by Bartonella quintana. Bartonella species are Gram-
negative pleomorphic rods that stain poorly with the Gram stain but better with
the Gimenez stain.
 Cat scratch disease caused by Bartonella henselae leads to bacillary
angiomatosis.

6. Legionnaire disease is implicated in causation of which among the


following?

a) Respiratory disease
b) U.T.I
c) Retroperitoneal fibrosis
d) Acute gastroenteritis

Legionellosis ( Atypical pneumonia)


 Fever, muscle pain, headache
 Bacterium found in AC fresh water
 Spread by breathing
Diagnosis By urinary Antigen test & Sputum culture
Treatment - Fluoroquinolones, Azithromycin or doxycycline
Culture for legionella → BCYE (Buffered charcoal yeast Extract with Fe & Cysteine)

7. When should you perform Widal test in a case of Typhoid?

a) 1st week
b) 2nd week
c) 3rd week
d) 4th week

Widal test should be performed in 2nd week.

Lab diagnosis for typhoid:


B - Blood culture - 1st week
A - Agglutination Test 2nd week
S- Stool culture - 3rd week
U - Urine culture - 4th week

Widal test
Highly sensitive but poorly specific.
Also biological false positive in -
Infectious mononucleosis
Malaria
SLE [ auto immune Disease]

Antibody titre against O → 1:100 and H → 1:200 is positive.

Flagellar antigen is more immunogenic.


Paired testing should be done, after 2 weeks of initial testing.

8. Antemortem diagnosis of rabies is made with:

a) Rabies virus specific antibodies


b) Inoculation in culture media
c) Negri bodies in hippocampus
d) Corneal impression smear

Ante mortem diagnosis


 Cornual impression smear
 Hair follicle from Nape of neck and of DFAT (direct fluorescent antigen test)

9. Hemolytic uremic syndrome associate with

a) E. coli
b) Malaria
c) Parvovirus B19
d) Bartonella henselae

EHEC
 Entero hemorrhagic E.coli
-Cause HUS (Hemolytic Uremic Syndrome)
-Shiga like toxin or Verotoxin (VT-1& VT-2)
Aka verotoxigenic E.coli
Culture media is Sorbitol Mac-Conkey agar (SMAC) and Rainbow agar used
 Diarrhoea causing E coli
1. EPEC -Entero Pathogenic Attaching/Effacing lesions to villi
2. EIEC - Entero Invasive resemble shigellosis & sereny test +ve 3.
3. ETEC-> Traveler’s diarrhea CFA (colonization factor antigen 4.
4. EAEC -Aggregative persistent type of diarrhea (EAST-1 Toxin)

Parvo Virus B19: Single standard DNA Virus cause Aplastic Crisis

10. Which of the following virus is from Herpes virus family?

a) Rubella
b) Measles
c) Rabies
d) EBV

HSV - 1 - Herpes simplex virus 1


HSV - 2 - Herpes simplex virus 2
HHV - 3 -varicella zoster [chicken pox virus]
HHV - 4 - EBV
HHV - 5 - Cytomegalovirus
HHV - 8 - Kaposi saroma causing virus

11. Molluscum contagiosum is caused by ?

a) Adenovirus
b) Flavi virus
c) Rubi virus
d) Pox virus

Molluscum contagiosum is an infection caused by a poxvirus


Pearly white umblicated nodule over the skin / genitals
Caused by MCV 1,2,3,4
On biopsy, Henderson Peterson [HP] bodies seen [ intra cytoplasmic]
Rx - podophyllin resin + cryosurgery

12. A patient presented with some unknown fungal infection. Microscopic


examination revealed brown colored spherical fungi with septate hyphae.
Possible condition:

a) Histoplasmosis
b) Chromoblastomycosis
c) Coccidioidomycosis
d) Candida albicans

 Dematiaceous Fungi
 Involves skin & sub cutaneous tissues
 Sclerotic bodies seen in tissue biopsy Brown colored globose bodies Aka medlar
bodies Show copper penny appearance

Examples -
1) Phialophora
2) Cladosporium

13. Which is correct about larval stage of Taenia solium?

a) Larva currens
b) Cysticercosis cellulose
c) Cutaneous larva migrans
d) Visceral larva migrans

 Larva stage of taenia solium


 Cysticercosis cellulose
 Taenia saginata
 Cyst cercus bovis
 Visceral larva migrans
 Toxocara canis
 Toxocara catis
 Angio stronglus cantonesis
 Cutaneous larva migrans
 Ancylostoma
 Brasilencs

14. N. meningitidis can be due to deficiency of this complement system:

a) C1-C4 deficiency
b) C5-C9 deficiency
c) C3 deficiency
d) C2 deficiency

 C5 - C9-MAC (membrane attack complex) deficiency can lead to increased risk


of meningococcal disease.
 C1. C2, and C4 deficiencies are associated with pyogenic infections and
rheumatologic disorders, particularly systemic lupus erythematosus.
 C3 deficiency is also associated with pyogenic infections and rheumatologic
disorders

15. Classical complement activated by:

a) C1
b) C3 convertase
c) IgA
d) Ag-Ab complex
The complement pathway. Complement can be activated through three pathways:
classical, lectin, and alternative. The classical pathway is activated when C1q
binds to antibody attached to antigen, activating C1r and C1s, which cleave C4
and C2.

PSYCHIATRY JUNE 2019

1. A 25-year-old lady presented with sudden onset chest pain , palpitations


lasting for about 20 minutes. She says there were 3 similar episodes in the
past. All the investigations were normal. What is the likely diagnosis?

a) Acute psychosis
b) Panic attack
c) Depression
d) Mania

Panic Attack: It is an acute attack of intense anxiety with a "feeling of impending


Doom" and Patient feels he is about die or he feels he is having a heart attack or he
feels he is about to go crazy or he feels he will lose control.

Panic Disorders: -
Recurrent & unexpected panic attacks
Anticipatory anxiety in between the attacks

2. All are stages of death, except:

a) Denial
b) Anger
c) Bargaining
d) Agitation

Five Stages of Dying: Denial, Anger, Bargaining, Depression, and


Acceptance (DABDA).

PHARMACOLOGY JUNE-2019

1. Shortest acting corticosteroid he following is?

a) Dexamethasone
b) Hydrocortisone
c) Triamcinolone
d) Deflazacort
 Short acting steroids → Cortisone
Hydrocortisone
 Intermediate acting → Prednisone
Prednisolone
Triamcinolone
 Long acting → Dexamethasone
Betamethasone
Paramethasone

2. Which of the following anti-epileptic drug has the highest teratogenic


potential and is not prescribed to pregnant women?

a) Valproate
b) Carbamazepine
c) Phenytoin
d) Lamotrigine

 Anti-epileptic with highest risk of teratogenicity → Valproate - causes neural tube


defects
 Anti-risk epileptic with lowest of teratogenicity → Lamotrigine Levetiracetam

3. You, being an ardent researcher in the field of vaccine production, are the
in-charge of preparing samples to make a vaccine against the new variant
of SARS-CoV 2. After getting satisfactory results in pre-clinical studies, you
seek approval to do clinical studies, which requires healthy volunteers.
Which of the following phase does this refer to?

a) Phase 1
b) Phase 2
c) Phase 3
d) Phase 4

 Phase 1→ Done on healthy people. Maximum tolerable dose of drug is tested.


 Phase 2 → Done on fewer patients (20-200). Efficacy and safety of drug is
tested here.
 Phase 3 → Done on more patients ( up-to 5000). Efficacy is confirmed in this
phase.
 Phase 4 → Post marketing surveillance. Long term and rare side effects are
monitored in this phase.
4. Long term use of which of the following drug is most likely associated with
development of tremours

a) Salbutamol
b) Propofol
c) Betaxolol
d) Timolol

Tremors occur with β2-agonists like salbutamol


Side effects of salbutamol : Tremor- Most common side effect Tachycardia (in
overdose) Tolerance ( on long term use) Hypokalemia

5. Which of the following is a long acting bronchodilator?

a) Salbutamol
b) Terbutaline
c) Adrenaline
d) Formoterol

Bronchodilators are β2-agonists

SABA( short acting β2 agonist) Used in acute attacks -Salbutamol ,Terbutaline


LABA ( Long acting β2 agonist) Used for prophylaxis -Salmeterol, Formoterol- fast
acting , hence can be used in acute attacks.

6. Drug of choice for neurogenic diabetes insipidus is:

a) Desmopressin
b) Vasopressin
c) Terlipressin
d) Conivaptan

 Diabetes Insipidus is due to deficient ADH, ADH function is to reabsorb water


from kidneys
 Deficient ADH ( ADH not released ) →Neurogenic DI
 Resistance to ADH in kidneys → Nephrogenic DI
 Desmopressin is DOC for Neurogenic DI
 Thiazides are DOC for nephrogenic DI

7. Which of the following inhalational anaesthetic agent can cause hepatitis


on repeated use?
a) Halothane
b) Isoflurane
c) Sevoflurane
d) Ether

Halothane causes hepatitis .It can also cause arrhythmia by making adrenaline more
adrenogenic, i.e, in conditions Adrenaline Shows more adrenogenic action when
halothane is used. Hence it should not be used where adrenaline is high ,i.e,
pheochromocytoma

8. Preferred drug for the treatment of ventricular tachycardia is

a) Digoxin
b) Propranolol
c) Diltiazem
d) Lignocaine

Doc for ventricular tachycardia – Lignocaine

9. Which of the following drugs mentioned below is the first choice for
chronic gout?

a) Allopurinol
b) Febuxostat
c) Probenecid
d) Sulfinpyrazone

Doc for acute gout : NSAID except paracetamol, aspirin


Most effective drug for acute gout : Colchicine
DOC for chronic gout: Allopurinol --- Inhibits Xanthine Oxidase and thus reduces
production of uric acid
Febuxostat is also Xanthine Oxidase inhibitor - used in patients allergic to uric acid

10. Which of the following is used for day care surgery?

a) Ketamine
b) Thiopentone
c) Propofol
d) Etomidate

Drugs used in day care surgery:

Dr -Desflurane
Manmohan- Midazolam
Singh -Sevoflurane
Is -Isoflurane
A –Alfentanyl
Prime -Propofol- mc drug used in day care surgery
Minister- Mivacurium

Ketamine causes dissociative anesthesia


Thiopentone is shortest acting due to redistribution
Etomidate causes adrenal suppression

11. A female with 20 weeks of pregnancy presents with fever and dysuria. A
preliminary diagnosis of cystitis was made. Which of the following drugs
will be safe to use for this patient?

a) Ciprofloxacin
b) Gentamicin
c) Cotrimoxazole
d) Amoxicillin

*Amoxicillin is safe in pregnancy.

Antibiotics safe in pregnancy


P - Penicillin
C - Cephalosporin
M - Macrolides

Ciprofloxacin causes cartilage and bone destruction


Gentamycin causes hearing loss in the fetus
Cotrimoxazole contains sulfonamides which can cause kernicterus in the child

12. A 66 year old known diabetic, on poor compliance, was referred to your
institution as her had symptoms of DKA. Which of the following insulin
would you prefer for the treatment of this condition?

a) Regular Insulin
b) NPH insulin
c) Insulin glargine
d) Insulin lispro

 Diabetic ketoacidosis is a medical emergency that requires immediate treatment


with insulin by I/V route.
 Insulin available in W/V route is regular insulin (short-acting, soluble, crystalline
zinc insulin).
 Regular insulin is DOC for diabetic ketoacidosis.
13. A 45 year old walks in to your clinic with complaints of increased hunger
and thirst. On testing, was noted to levels and HbA1c of 11. If you decide to
start insulin therapy, which of the have elevated blood sugar attain
following long-acting insulin would you pick, knowing it does not peak
concentration in plasma?

a) Insulin lispro
b) Insulin aspart
c) Insulin glulisine
d) Insulin glargine

Insulin glargine:
 Long acting insulin.
 Subcutaneous glargine given, most of it stay in subcutaneous area, very small
amounts comes into the blood, gradually it keeps coming into the blood
(maintains basal level)
 Never become high in plasma
 Single injection work for 24 hrs
 Glargine is called as peakless insulin.
 Rapid acting insulins (insulin Lispro, insulin aspart, insulin glulsine)

14. Praziquantel is used for the treatment of

a) Strongyloidiasis
b) Trichomoniasis
c) Schistosomiasis
d) Rhinosporidiosis

Platyhelminthes (flukes & tapeworms)


 DOC for all the platyhelminths is praziquantel
 DOC for Fasciola hepatica (liver fluke) is triclabendazole
 DOC for dog tapeworm is albendazole
 DOC for trichomoniasis is metronidazole
 Surgery is done for rhinosporidiosis
 Nematodes
 DOC for nematodes is albendazole
 DOC for filaria is diethyl carbamazine
 DOC for strongyloidiasis & onchocerca volvulus is Ivermectin

15. A young male presents with reduced sleep, hyperactivity and elevated
mood. He has a family history of mania. Which of the following drug should
be used for long term treatment of this patient?
a) Sodium valproate
b) Lithium carbonate
c) Carbamazepine
d) Barbiturates

DOC for long term treatment of mania is lithium


L-leucocytes
I-Increase
T-Tremors (M/C side effect)
H-Hypothyroidism
I-Increase
U-Urine
M-Mothers (teratogenic), C/I in pregnancy, baby may develop Epstein anomaly

16. Drug of choice for nasal carriers of MRSA is:

a) Vancomycin
b) Teicoplanin
c) Mupirocin
d) Linezolid

Vancomycin is the DOC for the treatment of MRSA


DOC for nasal carriers of MRSA is mupirocin
DOC for VRSA is daptomycin

17. Anti-depressant drug which is used for smoking cessation is?

a) Venlafaxine
b) Topiramate
c) Bupropion
d) Amitriptyline

 Bupropion is antismoking drug


 Buspirone is a drug which is used for anxiety

18. Which of the following drug is a selective COX-2 inhibitor?

a) Ketorolac
b) Etoricoxib
c) Piroxicam
d) Nimesulide

 Selective COX 2 inhibitor, COX1B (COX 2 inhibitor)


 Etoricoxib > longest acting COX1B
 Major side effects of COX1B (long term use causes myocardial infarction)
 Other drugs like :
 Ketorolac
 Piroxicam inhibit both COX & COX 2 (nonselective COX inhibitors)
 Nimesulide

19. Aerosol drug used for treatment of RSV infection in a child is?

a) Indinavir
b) Amantadine
c) Ribavirin
d) Tenofovir

DOC for treatment of RSV infection is ribavirin


Indinavir → protease inhibitor used in HIV
Tenofovir → NRTI used in HIV
Amantadine → used for influenza virus

20. Drug of choice for treatment of Alzhiemer's disease is:

a) Donepezil
b) Atropine
c) Physostigmine
d) Fluoxetine

Drug of choice for treatment of Alzhiemer's disease is: Donepezil


 Atropine is DOC for organophosphate poisoning
 Physostigmine is DOC for atropine poisoning
 Fluoxetine is DOC for depression
 AcHE inhibiting drugs
 Donepezil
 Rivastigmine
 Galantamine

21. Mechanism of action of atropine in treatment of organophophate poisoning


is?

a) It inhibits secretion of acetylcholine


b) It is reactivator of acetylcholine esterase enzyme
c) It has antimuscarinic activity
d) It is agonist of acetylcholine receptors

 Organophosphates
 ACHE inhibitors (acetylcholine cannot be broken)
 Level of acetyl choline increases
 Stimulate nicotinic & muscarinic receptors
 Over activity of M1- acidity
M2-- bradycardia
M3-- bronchoconstriction, secretion, diarrhea & urination

 Atropine is DOC for organophosphate poisoning


 But atropine doesn't block the nicotinic receptors (muscle weakness)

22. Secretion of prolactin is inhibited by?

a) Dopamine
b) Nor - adrenaline
c) Adrenaline
d) Serotonin

 PIH is secreted from hypothalamus


 inhibiting hormone PIH (dopamine)
 Dopamine in the pituitary it stimulates D2 receptors
 Stimulated D2 receptors decrease prolactin
 D2 receptors are stimulated by drug called as cabergoline
 DOC for hyperprolactinemia is cabergoline
 DA stimulates D2 receptors — decrease prolactin

OBG JUNE 2019

1. 15-year-old girl presented with primary amenorrhea with orderly


appearance of secondary sexual characteristics like breast and pubic hair.
What is the next best step for this patient:

a) Reassure
b) USG
c) HSG
d) Hormonal study

In the question the girl is 15 yrs with normal secondary sexual characteristics, she is
a case of primary amenorrhea and needs evaluation (reassurance could have been
given had the patient been younger than 15 years)
The next important step would be an USG to check whether she has normal
mullerian structures (uterus and tubes) and ovaries or not
Hormonal studies and karyotyping are done subsequently to find but the cause.

2. A 19-year-old primigravida, presents with 8 weeks amenorrhea, light


bleeding and pain. O/E Uterine size corresponds to POG and Os is closed.
USG reveals intra uterine pregnancy. What is preferred management in this
case?
a) Estrogen plus Progesterone therapy
b) Dilatation and Curettage
c) Bed rest and Progesterone
d) Beta hCG

The above case history is suggestive of Threatened abortion.

Management -
1. Bed Rest
2. Avoid Coitus
3. Avoid Lifting weights
4. Progesterone is given to compliment the placental function

3. A 35 year female presented with complaints of infertility. She has previous


history of PID. Preliminary investigations like USG showed normal organs
and hormone levels were also normal. What is the next best investigation?

a) Repeat USG
b) Hysterosalpingography
c) Endometrial biopsy
d) Urine culture and sensitivity

A past history of PID and normal pelvic USG and hormonal assays is most likely
suggestive of tubal block (secondary to adhesions due to PID) as a cause of the
patient's infertility.

HSG

It is the procedure to see the uterus and the tubes


Done using leech Wilkinson cannula
Done in 1st half of the menstrual cycle preferably
Drawbacks of hsg : 1) exterior of the tubes is not visible
NOTE : the best test and a better answer would have been laparoscopic
chromopertubation, which would be able to visualize inside and outside of uterus and
tubes and also look for extrauterine adhesions and endometriotis implants which is
another important differential in this case.

4. Couvelaire uterus is seen in:

a) Placenta previa
b) Vasa previa
c) Abruptio placenta
d) Placenta percreta

Couvelaire uterus: widespread extravasation of blood into the uterine musculature


and beneath the serosa. Effusions of blood are also seen beneath the tubal serosa,
between the leaves of the broad ligaments, in the substance of the ovaries, and free
in the peritoneal cavity.

Type Ill Abruption


 Most severe type of Abruptio placenta
 FHR problem, Fetal distress
 II A - without DIC
 Ill B - with DIC

Presentation
 Painful bleeding
 Protraction of labour
 PPH
5. Risk of endometrial cancer is least in:

a) A positive family history


b) Obesity
c) Late menopause
d) Multigravida

Risk factors for endometrial cancer-

1) Late menopause
2) Nulliparity
3) Unopposed estrogen therapy
4) Obesity, diabetes, hypertension
5) History of persistent anovulation (PCOS)
6) History of irregular & excessive premenopausal bleeding
7) Atypical endometrial hyperplasia
8) Tamoxifen therapy
9) Radiation menopause.

6. A grand multipara is a women who has given birth _ times

a) >2
b) >3
c) >4
d) ≥5

Grand multigravida - ≥ 5 times pregnant


Problems related:
 Malpresentation
 Meconium stained liquor
 Placenta previa
 Abruption'
 PPH
 Umbilical cord prolapse
 Low apgar score

7. A 26 weeks pregnant female presented with HTN for the first time. There is
no proteinuria. Diagnosis of such condition

a) Chronic hypertension
b) Eclampsia
c) Gestational Hypertension
d) Preeclampsia
Gestational hypertension - bp > 140/90, > 20weeks of gestation
Pre eclamptic toxemia - gestational hypertension + proteinuria >300mg
Eclampsia - above 2 + convulsions
Chronic hypertension - hypertension before she is pregnant

8. Minimum sperm count for normal semen analysis according to WHO:

a) 5 million/mL
b) 10 million/mL
c) 15 million/mL
d) 20 million/mL

Parameters (WHO Criteria 2010)


Parameter Lower Reference Limit
Semen volume (ml) 1.5
Sperm concentration (106/ml) 15
Total sperm number (106/ejaculate) 39
Progressive motility (PR %) 32
Total motility (PR + NP. %) 40
Vitality (live sperms, %) 58
Sperm morphology (NF,%) 4
pH* >/=7.2
Leucocyte* (106/ml) <14
MAR/Immunobead test* (%) <50

9. A 26 year old woman presents with infertility. She has regular cycles of 28
days with no dysmenorrhea. Her LMP was on 8th of May. When should her
endometrial biopsy be planned ?

a) 22nd May
b) 8th-12th May
c) Anytime during cycle
d) 29th May

Endometrial biopsy

Best done preferably around 7 days before the onset of menstruation (that is around
21st day of a 28 day cycle)
Secretory changes prove that the cycle has been ovulatory
Can predict the functional integrity of the corpus luteum
Can detect luteal phase defect

10. In this normal menstrual cycle graph, the mark X represents levels of which
hormone?

a) LH
b) FSH
c) Estrogen
d) Progesterone

Green line: LH surges and peaks before ovulation


Blue line: FSH : responsible for follicular development in the early phase of the
cycle, increases gradually up-to ovulation
Red line: Estrogen : increases steadily in follicular phase secondary to follicular
development as estrogen is produced by the follicle and is responsible for growth of
the endometrium in follicular phase
Dotted line (marked X ): Progesterone: starts to rise after ovulation (after formation
of corpus luteum), responsible for stabilization of endometrium. Withdrawal of
progesterone leads to endometrial shedding (menstrual bleed)

11. A patient who was using CuT for contraception, presented with a 20 weeks
pregnancy. The IUCD is placed at fundus, tail visible at os and she wants to
continue the pregnancy. What is your next step this patient:
a) Leave IUD in-situ, continue pregnancy
b) Do medical termination of Pregnancy
c) Remove IUCD and continue pregnancy
d) Remove IUCD and do MTP

 With intrauterine pregnancy, if the tail is seen, it should be grasped and the IUD
removed by gentle outward traction This action reduces complications such as
abortion, chorioamnionitis and preterm birth.
 Specifically, in one cohort, a 54% abortion rate and 17% preterm delivery rate
was noted if the device remain insitu

12. Which of the following can be a cause of Oligohydramnios?

a) Multiparity
b) Twins
c) Renal agenesis
d) Macrosomia

Oligohydramnios causes:
1) Fetal chromosomal or structural anomalies
2)Renal agenesis
3) Obstructed uropathy
4) Spontaneous rupture of membranes
5) Intrauterine infections
6) Postmaturity
7) lugr
8) Amnion nodosum

PEDIA JUN 2019

1. A 3-year-old child with weight of 12 kg is having loose motions. He is


thirsty, drinks eagerly, tears are absent and skin pinch goes back slowly.
What is the best treatment plan for this child?

a) 1200 ml RL over 12 hours


b) 600ml RL over 6 hours
c) 900ml ORS over 4 hours
d) 300 ml ORS per episode of loose stool
Assessment of severity of dehydration in children
Parameters No Dehydration Some dehydration Severe dehydration

Condition Well, alert "Restless, irritable" "Lethargic"


Eyes Normal Sunken Very sunken
Tears Present Absent Absent
Mucosa Moist Dry Very dry
Thirst Normal "Thirsty drinks eagerly" "Drinks poorly"
Skin pinch Goes back quickly Goes back slowly Goes back slowly

Management of dehydration in children


Plan A Plan B Plan C
No dehydration Some dehydration Severe dehydration
WHO ORS 5-10 ml/kg/ WHO ORS 75 ml/kg IV RL 100ml/kg
loose stool (Replacement over 4 hours + Over 6 hours(< 1year
of ongoing losses) replacement age)
3 hours (>1year age)

2. Wide open posterior fontanelle large tongue ,rough dry skin with
constipation is seen in?

a) Pellagra
b) Down syndrome
c) Hypothyroidism
d) Nutritional rickets

Clinical features Hypothyroidism

 Wide open Anterior & Posterior fontanelle


 Prolonged physiological jaundice (earliest sign sometimes)
 Myxedematous facies
 Large, protruded tongue
 Skin - dry & scaly
 Hypotonia, hypothermia, hoarse cry
 Constipation
 Abdominal distension
 Umbilical hernia

3. Chronic malnutrition in child is best evaluated by?


a) Weight for height
b) Weight for age
c) Height for age
d) Ponderal index

Acute malnutrition → wasting ( ↓ in Weight for Height)


Chronic malnutrition → stunting ( ↓ in Height for age)

Neonatal hyperbilirubinemia, defined as a total serum bilirubin level above 5 mg per


dL (86 βmol per L), is a frequently encountered problem. Although up to 60 percent
of term newborns have clinical jaundice in the first week of life, few have significant
underlying disease.

4. Neonatal jaundice becomes detectable at serum bilirubin exceeding?

a) 2 mg%
b) 3 mg%
c) 4 mg%
d) 5 mg%

Neonatal hyperbilirubinemia, defined as a total serum bilirubin level above 5 mg per


dL (86 βmol per L), is a frequently encountered problem. Although up to 60 percent
of term newborns have clinical jaundice in the first week of life, few have significant
underlying disease.

5. The pathophysiology of a tet spell is a decrease in systemic vascular


resistance, increasing right to left shunt which leads to a bunch more
things that all just keep increasing the right to left shunt. The severity of
cyanosis best decided by?

a) Size of V.S.D
b) Overriding of aorta
c) Degree of RVH
d) Degree of pulmonary stenosis

Severity of TOF is:


 Directly proportional to the severity of cyanosis &
 Inversely proportional to the intensity & duration of the murmur
 which depends on the degree of pulmonary stenosis

The more severe the pulmonic stenosis, the less flow into the pulmonary artery and
bigger the right to left shunt. Thus, the severity of cyanosis is directly proportional to
the severity of pulmonic stenosis, but the intensity of the systolic murmur is inversely
related to the severity of pulmonic stenosis.
6. Infant is admitted with respiratory distress and prolonged expiration with
rhonchi in chest. CXR shows hyperinflation. What is the diagnosis?

a) Pneumonia
b) Croup
c) Asthma
d) Bronchiolitis

Given scenario suggests diagnosis of Bronchiolitis: MC LRTI in infants.

Acute Bronchiolitis:
 Age group 6 months to 2 years.
 MC agent is RSV
 Clinical features: Viral prodrome followed by tachypnea, retractions and
hypoxemia, wheeze and crepitation heard on examination
 Chest x-ray: Hyperinflated chest
Treatment:
 Moist Oxygen
 In immunocompromised children and those on ventilator: Nebulized Ribavirin is
indicated

Acute Laryngotracheobronchitis (Croup):


 MC organism: Parainfluenza virus
 C/F's: Starts as viral prodrome and progresses to stridor, barking cough and
respiratory distress
 x ray shows 'Steeple sign' due to subglottic narrowing
Treatment:
 Oxygen support
 Mild cases: Single dose dexamethasone
 Moderate to severe cases: Supportive care Single dose dexamethasone +
Nebulized epinephrine

Pneumonia: h/o Fever, cough, tachypnea, retractions, sick looking child, CXR
shows consolidation
Asthma: recurrent h/o wheezing present, cough which worsens at night or early
morning, family h/o atopy present.

7. Protein in Cow's milk in comparison to human milk is?

a) Double
b) Triple
c) Quadruple
d) Same
Breast Milk Cow Milk
Proteins 1 g/dl 3.5 g/dl
Carbohydrates 7 g/dl 4.5 g/dl
Lipids Richer in PUFA Poorer

Composition of Breast milk (BM):

 Carbohydrates:
BM contains more lactose (7g/dI) compared to cow's milk (CM), which contains 4.5
g/dl

 Proteins:
BM contains lesser protein (1g/dl vs 3.5 g/dl) as compared to CM → lesser solute
load on
BM contains adequate cysteine, taurine → help in brain development
BM is richer in whey proteins (lactalbumin), which are easily digestible as compared
CM, which is richer in casein.

 Lipids:
Breast milk is richer in PUFA (poly unsaturated fatty acids)
It contains adequate amounts of DHA (Docosa Hexaenoic Acid) = helps in brain &
eye development
It is more easily digestible due to presence of lipase in breast milk.

 Minerals:
Calcium : Phosphate ratio in breast milk favours calcium absorption
Iron present in breast milk is more easily absorbable than in cow's milk

 Vitamins:
Breast milk contains adequate amounts of all vitamins except vitamin D. K(& B12 in
strictly vegan mothers).

8. 6th day disease is?

a) Erythema Infectiosum
b) Exanthema subitum
c) Erythema marginatum
d) Erythema nodosum
Terminology Other names for the disease Etiology

First disease Rubeola/ measles Measles virus


Second disease Scarlet fever Streptococcus pyogenes
Third disease Rubella/German measles/3- day Rubella virus
measles
Fourth disease Staphylococcal scalded skin Staphylococcus aureus
syndrome/Ritter's disease
Fifth disease Erythema infectiosum Parvovirus B19
Sixth disease Erythema infectiosum Human Herpes Virus 6 or 7

MEDICINE—JUNE 2019

1. Which of the following statements are true about tremors?

1)PD is characterized by resting tremor


2)Tremor consist of alternate contraction of agonist and antagonist muscle in an
oscillating rhythmic manner
3)Essential tremor is an uncommon movement disorder affecting 5%of the
population
4)Normal individuals can have physiologic tremors that manifest as mild high
frequency, postural or action tremors.

a) 1,2,3 only
b) 2,3,4 only
c) 1,2,4 only
d) All of the above

Options 1, 2, and 4 are true

Essential tremor is most common movement disorder.


Tremors may be most prominent:
 At rest-Resting tremor
 On assuming posture- postural tremor
 On actively reaching for target- Kinetic tremor
 On carrying out movement- Action tremor
 Physiologic tremor- Normal individual and they affect upper extremities.
 Enhanced physiologic tremor - anxiety, fatigue, metabolic disturbance, drugs,
toxins( alcohol, smoking and caffiene).
2. MC joint involved in diabetes is?

a) Ankle
b) Knee
c) Shoulder
d) Foot

Most common joint involved in diabetes is Foot Joint

Charcot's joint –
Painless, Paralyzed joint
Neuropathy
Delayed wound repair
Angiopathy
Non-heading ulcer on soul of test

Ankle Joint - in tables dorsalis


Knee - OA
Shoulder- Syringomyelia - Dilation of central canal of Spinal cord. MC Cervical spine
involved

3. All are true about steps followed in management of ventricular fibrillation


except?

a) CPR cycle duration is 2 minutes


b) Chest compressions at 100-120/min
c) Start CPR followed by immediate defibrillation
d) Intravenous access followed by Immediate defibrillation

CPR DC shock 200 J Biphasic *3 *2 min


Asystole - IV adrenaline

4. A 70 yrs old male patient was diagnosed of having heart failure.His ejection
fraction was found to be <40%. Which of the following drug does not
reduce the mortality for this patient?

a) Metaprolol
b) Digoxin
c) Captopril
d) K*sparing diuretics

Digoxin is not mortality reducing drug in HFrER


Mortality reducing drug in patients with HFrEF:

 ACE inhibitors
 Angiotensin receptor blockers (ARBS)
 Angiotensin receptor blocker with Neprilysin Inhibitor (ARNI)
 Beta blockers
 Mineralocorticoid receptor antagonists
—Spironolactone
—Eplerenone
 Isosorbide dinitrate + Hydralazine

5. Which of following leads to a continuous murmur ?

a) Peripheral Pulmonic stenosis


b) Severe Pulmonary artery hypertension
c) Type A aortic dissection
d) Rupture of cardiac chamber

Continuous murmurs begin in systole, peak near the second heart sound, and
continue into all or part of diastole. Their presence throughout the cardiac cycle
implies a pressure gradient between two chambers or vessels during both systole
and diastole

Continues Murmur seen in


1. COA
2, PDA a/s with congenital rubella syndrome
3. Mammary souffle of pregnancy is created by enhanced arterial flow through
engorged breasts and usually appears during the late third trimester or early
puerperium.
4. Venous hump =dilated vein Internal mammary artery
5. Rupture of sinus of Valsalva
6. Peripheral pulmonic stenosis

6. Which is true about Postural Hypotension?

a) Decrease in systolic blood pressure 20 mm Hg within 6 mins of postural change


b) Decrease in systolic blood pressure 20 mm Hg within 3 mins of postural
change
c) Decrease in diastolic blood pressure 20 mm Hg within mins of postural change
d) Decrease in diastolic blood pressure 20 mm Hg within 3mins of postural change

Postural Hypotension – Supine 3 min stand


SBP↓ > 20 mmHg
DBP↓ > 10 mmHg

7. A 45-year patient working in a factory for past 20 years presents with


breathlessness. HRCT chest shows pleural thickening and fibrosis.
Diagnosis is?
a) Asbestosis
b) Coal worker pneumoconiosis
c) Silicosis
d) Berylliosis

ASBESTOSIS Pleural thickening + Fibrosis - Mesothelioma


Silicosis B/L Hilar LN + / miliary picture

8. Most commonly observed autosomal aneuploidy leading to spontaneous


abortions is?

a) Trisomy 16
b) Trisomy 21
c) Monosomy
d) Trisomy 18

Trisomy 16: Never seen postnatally 100% abortion Rate

9. Which of these is a Nephritic syndrome?

a) Minimal change disease


b) Membranous Glomerulopathy
c) Post infectious Glomerulonephritis
d) Focal segmental glomerulosclerosis

Nephrotic Syndrome Nephritic Syndrome


Minimal changes PSGN/Post infectious Nephritic
syndrome
Membranous IgA Nephropathy
FSGS

10. The following serological status is noted in a patient: HbsAg positive and
HbeAg positive. Diagnosis is?

a) Carrier
b) Chronic viral hepatitis
c) Acute viral hepatitis with infectivity
d) Remote infection
Hbs Ag positive
IgM anti Hbc Ag Acute Hepatitis + Infective
Hbe Ag positive

Carrier - Hbs Ag positive > 6 month


Chronic Hepatitis Hbs Ag positive

IgG positive
Remote infection . Hb Ag negative
IgG anti Hbc Ag positive

11. Chronic viral hepatitis is seen with?

a) HBV
b) HCV
c) HDV
d) HEV

Chronic Viral Hepatitis-


HCV - Cirrhosis/ HCC
DOC – Lepsidavir

12. Acute auto-graft rejection occurs within?

a) Few hours
b) <1 month
c) < 6 months
d) 6-12 months

Hypersensitivity Time
reaction
1.Hyperacute II Minutes
2.Acute II <6 months
3.Chronic IV >6 months

13. Mark the true statements .

I. Pernicious anemia is an example of Type II Hypersensitivity


2. Serum Sickness is example of Type III Hypersensitivity
3. Pathergy test is an example of Type IV Hypersensitivity
4. Pathergy test is done for Reiter's disease
a) 1,2 and 3 are true
b) 2, 3 and 4 are true
c) 1, 2, 3 and 4 are true
d) 3 and 4 are true

Pernicious anemia is an example of Type II Hypersensitivity

Pernicious anemia
 Characterized by antibodies against parietal cells of stomach
 Results in vitamin B12 deficiency ---- Anemia
Serum Sickness/ Arthus reaction are example of Type III HSN
Pathergy phenomenon
 For Behcet disease
 Type IV Hypersensitivity

14. Which of the following is a causative agent of Farmer's Lung?

a) Thermophilic Actinomycetes
b) Aspergillus Fumigatus
c) Actinobacter
d) Aspergillus Flavus

Farmer's lung-T. Actinomyecetes


ABPA
 Black sputum plugs in Asthmatic Patients
 Aspergillus Fumigatus
HCC Aspergillus flavus

15. What is the diagnosis ?

a) Malaria
b) Hereditary spherocytosis
c) G6PD Deficiency
d) Thalassemia
The above image shows Banana shaped gametocytes which are characteristic of
Plasmodium falciparum malaria.
Hereditary spherocytosis --- Spherocytes (absence of central pallor)
G6PD --- Bite cells
Thalassemia Target cells

16. True statement regarding Whipple's disease is/ are:

1. Caused by a Protozoa Tropheryma Whipplei


2. PAS positive macrophages in the lamina propria of the small intestine
3. Culture-negative endocarditis is a common cardiac presentation
4. Treated with IV Ceftriaxone

a) 1,2 and 4 are true


b) 2, 3 and 4 are true
c) 1, 2 and 3 are true
d) 1 and 2 are true

Whipple's disease

By Bacteria Tropheryma Whipplei


PAS positive (Schiff reagent- Intra cellular bacteria found in macrophage in gut)
PAS positive macrophages in the lamina propria of the small intestine in Whipple's
disease:

Rx

IV - Ceftriaxone
Oral - Cotrimoxazole
Whipple's triad -seen in Insulinoma
Whipple's Sx- is performed in Ca of head pancreas.
Extra Edge: The most common type of cardiovascular presentation in Whipple's
disease is culture-negative endocarditis.
17. Most common site of Brain metastasis is?

a) Lung cancer
b) Head and neck cancer
c) Prostate cancer
d) Breast cancer

Oat Cells Ca > Breast Ca > Malignant Melanoma

18. Which is not seen in Tumour lysis Syndrome?

a) Hypophosphatemia
b) Hypocalcemia
c) Hyperuricemia
d) Hyperkalemia

Tumor lysis syndrome is characterized by high blood potassium (hyperkalemia),


high blood phosphate (hyperphosphatemia), low blood calcium (hypocalcemia), high
blood uric acid (hyperuricemia), and higher than normal levels of blood urea nitrogen
(BUN) and other nitrogen-containing compounds (azotemia).

19. Comment on the diagnosis for SAP normal, PTH normal, Vitamin D3 normal
with elevated serum Calcium values?

a) Vitamin D intoxication
b) Hyperparathyroidism
c) Multiple myeloma
d) Nutritional rickets

Multiple myeloma:
 SAP Negative
 PTH Negative
 D3 Negative
 ↑ Sr Cal

20. Which of the following interfere with iron absorption?

a) Vitamin
b) Phytates
c) Oxalate
d) Myoglobin

Phytates - compete with iron absorption


21. Most sensitive and specific marker for MI is?

a) Troponin
b) Cytokeratin
c) Myoglobin
d) CPK- MM

 Most sensitive & specific marker for MI = Troponin


 1st Investigation - ECG
 Best Investigation - Troponin I * 2 times
 Myoglobin 1 - Skeletal Muscle damages also
 Cytokeratin - Cancer
 CPK MM - Only right at Skeletal Muscle

PATHO- JUNE 2019

1. Berry aneurysm most commonly occurs due to?

a) Endothelial injury of vessel due to HTN


b) Muscle intimal elastic lamina layer defect
c) Muscle and adventitial layer defect
d) Adventitia defect

Berry aneurysm due to defect in tunica media and intimal elastic lamina layer leading
to weakening of vessel wall and out-pouching in anterior portion of circle of willis.
Saccular Aneurysms: is a 2nd most common cause of clinically significant
subarachnoid hemorrhage is rupture of a saccular ("berry") aneurysm in cerebral
artery following trauma. Patient will describe the pain as worst headache of his life.
Bloody tap on lumbar puncture is seen. Nimodipine is a cerebro-selective
vasodilatory drug helpful for treatment of symptoms associated with subarachnoid
hemorrhage. Berry aneurysm is associated with adult polycystic kidney disease
(ADPKD).
2. A 45-year patient working in a factory for past 20 years presents with
breathlessness. HRCT chest shows pleural thickening and fibrosis, What is
the person suffering from

a) Asbestosis
b) Coal worker pneumoconiosis
c) Silicosis
d) Berylliosis

Occupational lung disease/pneumoconiosis:

 Caused by dust particle with range of 1-5 micron and it mainly involves the upper
lobe of lung.
 HRCT will show pleural thickening and fibrosis.

Asbestosis: In contrast to coal workers' pneumoconiosis and silicosis, asbestosis


begins in the lower lobes and sub pleurally. The middle and upper lobes of the lungs
become affected as fibrosis progresses. The scarring may trap and narrow
pulmonary arteries and arterioles, causing pulmonary hypertension and cor
pulmonale. Pleural plaques, the most common manifestation of asbestos exposure,
are well-circumscribed plaques of dense collagen that are often calcified. They
develop most frequently on the anterior and posterolateral aspects of the parietal
pleura and over the domes of the diaphragm.
Asbestos is responsible for causing mesothelioma and bronchogenic carcinoma.
Most common lung cancer associated with asbestosis bronchogenic carcinoma.
Most specific lung cancer associated with asbestosis is mesothelioma. Most
common pneumoconiosis: silicosis/sand blaster disease.
Coal worker pneumoconiosis: anthracosis (black lung) Berylliosis: associated with
aerospace industry.

3. Correct about of Anaemia of chronic disease

a) High ferritin
b) Increased percentage of saturation of transferrin
c) High transferrin
d) High serum iron

In patient with Anemia of Chronic disease, there is increase in blood level of


hepcidin protein which interferes with metabolism of iron. Hepcidin is a negative iron
regulator. It inhibit the usage of iron from storage form of iron and GI tract. Therefore
these patient develops microcytic anemia. Lab findings:
 Serum iron decrease
 Transferrin saturation is decrease
 Increase in the concentration of serum ferritin
 Decrease in total iron binding capacity

Microcytic hypochromic anemia:

 S: Sideroblastic
 I: Iron deficiency
 T: Thalassemia
 A: Anemia of chronic disease

4. Which of these is a Nephritic syndrome?

a) Minimal change disease


b) Membranous Glomerulopathy
c) Post infectious Glomerulonephritis
d) Focal segmental glomerulosclerosis

The nephritic syndrome is characterized by hematuria, oliguria with azotemia,


proteinuria, and hypertension.
The most common causes are immunologically mediated glomerular injury; lesions
are characterized by proliferative changes and leukocyte infiltration.

 Acute postinfectious glomerulonephritis typically occurs after streptococcal


infection in children and young adults but may occur following infection with many
other organisms; it is caused by deposition of immune complexes, mainly in the
subepithelial spaces, with abundant neutrophils and proliferation of glomerular
cells. Most affected children recover; the prognosis is worse in adults.
 Rapidly progressive glomerulonephritis (RPGN) is a clinical entity with features of
the nephritic syndrome and rapid loss of renal function. Nephrotic syndrome is
caused by derangement in glomerular capillary walls resulting in increased
permeability to plasma proteins.
The manifestations of the syndrome include:
 Massive proteinuria, with the daily loss of 3.5 gm or more of protein less in
children)
 Hypoalbuminemia, with plasma albumin levels less than 3 gm/dl
 Generalized edema
 Hyperlipidemia and lipiduria

5. The following serological status is noted in a patient: HbsAg positive and


HbeAg positive. Diagnosis is?

a) Acute viral hepatitis


b) Chronic viral hepatitis
c) Acute viral hepatitis with infectivity
d) Remote infection

Acute viral hepatitis: HBsAg +/lgM antiHBc


Chronic viral hepatitis: HBsAg +/IgG anti HBc
Acute viral hepatitis with infectivity: HbeAg+
Remote infection: anti HBS

6. Chronic viral hepatitis is seen with all of the following viruses, except?

a) HBV
b) HCV
c) HDV
d) HEV

 Chronic hepatitis is defined as symptomatic, biochemical, or serologic evidence


of continuing or relapsing hepatic disease for more than 6 months.
 Etiology rather than the histologic pattern is the most important determinant of
the probability of developing progressive chronic hepatitis.
 Chronic viral hepatitis is seen in HBV, HCV, HDV.
 Hepatitis A virus (HAV) infection is a self-limited disease and does not lead to
chronic hepatitis or carrier state.
 HAV and HEV (in immunocompetent hosts) do not cause chronic hepatitis.

7. Acute graft rejection occurs within?

a) Few minutes
b) Few hours
c) < 6 months
d) 6-12 months

Hyper acute rejection: type 2 HSR, happens over span of few minutes to hours. Eg:
kidney transplant
Acute rejection: Few weeks to months(< 6 months)
Chronic rejection: After 6 months.
Graft vs host disease (Runt Disease):
Acute GVH: 100 days
Chronic GVH: > 100 days

8. Type II Hypersensitivity is seen in?

a) Pernicious anaemia
b) Serum sickness
c) Arthus phenomenon
d) Pathergy phenomenon

Type 2 HSR is seen in following condition:

My : Myasthenia gravis,
Blood : Blood transfusion (mismatch)
Group : Graves, good pasture
Is: Immune hemolytic anemia
Rh: Rheumatic heart disease
Positive: Pernicious anemia/ pemphigous vulgaris

Type 3 HSR: SHARP

S: Serum sickness/SLE
H: Henoch schonlein purpura
A: Arthus reaction
R: Reactive arthritis
P: Post strep glomerulonephritis
Pathergy phenomenon (Type 4 HSR) is useful in diagnosis of Behcet
syndrome(oculo-oro-genital syndrome): patient complains of recurrent oral ulcer eye
lesion, skin lesion and genital lesion.
9. A stable boy admitted with acute onset of fever. cough and dyspnea. Chest
radiograph showed findings of pneumonia for which he was treated with
antibiotics; showed good response. After month he again presented in
hypoxemia state with progressive dyspnea. CT chest showed diffuse
ground glass changes. Transbronchial biopsies showed numerous small
granulomas and marked lymphocytosis. Which of the following is a
causative agent for given disease?

a) Thermophilic Actinomycetes
b) Aspergillus Fumigatus
c) Actinobacter
d) Aspergillus Flavus

Farmer's lung is. subtype of Hypersensitivity pneumonitis resulting from exposure to


dusts generated from humid, warm, newly harvested hay that permits the rapid
proliferation of the spores of thermophilic actinomycetes.

Pathogenesis C/F
Acute Phase 1st exposure Ab formation Cough / Dyspnea /Fever
Repeat exposure - Ag-Ab immune
complex formation ~ type lll
hypersensitivity

Chronic Phase Non- caseating granuloma Interstial fibrosis –


Type IV HR > Type III HR Progressive dyspnea
Respiratory Failure
Cyanosis

10. Most sensitive indicator for Iron deficiency anaemia?

a) Serum Ferritin
b) TIBC
c) Percentage saturation of transferrin
d) Bone marrow iron

 Most specific test for ron deficiency anemia is bone marrow iron. But bone
marrow iron test is invasive procedure
 Most sensitive indicator for iron deficiency anemia: serum ferritin.
Lab finding in iron deficiency anemia:
 Serum ferritin : decrease (early indicator)
 Serum iron: decrease
 % Tf saturation: decrease
 TIBC: increase
11. Whipple's disease is caused by ?

a) Bacteria
b) Virus
c) Protozoa
d) Helminths

Whipple disease is cause by bacteria called tropheryma whipli it is responsible fot


causing infux of lot of macrophages leading to compression of lacteals and
impairment in absorption of nutrients.

Rod shaped bacillis in macrophage is seen. Patient complains of malabsorption. Pas


+ve diastase resistant is seen macrophage.
Treatment with antibiotic will show dramatic improvement.

12. Most common primary for brain metastasis is which of the following?

a) Lung cancer
b) Head and neck cancer
c) Prostate cancer
d) Breast cancer

Most common Primary for brain metastasis is - LUNG CANCER


Particular Lung cancer which is notorious for causing CNS metastasis - SMALL
CELL LUNG CARCINOMA
Most common Primary malignant CNS tumor seen in children -
MEDULLOBLASTOMA
Most common Primary malignant CNS tumor seen in adults GLIOBLASTOMA
Most common Primary benign CNS tumor seen ir adults. MENINGIOMA

13. 12-year-old Boy came with complaints of Severe Abdominal pain and
headache for the past 2 days, H/0 On & off for we the past 1 week,
associated with myalgia, and several episodes vomiting self-medicated
with Tab Paracetamol with no relief. Presents with Dengue NS -1 antigen
(+). What is the first line of defense of the body in the given scenario?

a) NK cell
b) T cell
c) Histiocyte
d) Macrophage

The major components of innate immunity are epithelial barriers that block entry of
microbes, phagocytic cells (mainly neutrophils and macrophages), dendritic cells,
natural killer cells and other innate lymphoid cells and several plasma proteins,
including the proteins of the complement system.

NK cells
 Ability to kill virus-infected cells and tumor cells
 NK cells act as an early line of defense against viral infections and some tumors.
 Two cell surface molecules, CD16 and CD56, are commonly used to identify NK
cells.
 CD16 is an Fc receptor for IgG, and it confers on NK cells the ability to lyse lgG-
coated target cells. This phenomenon is known as antibody-dependent cell-
mediated cytotoxicity (ADCC).
The functional activity of NK cells is regulated by a balance between signals
from activating and inhibitory receptors
 NK cell inhibitory receptors recognize self-class I MHC molecules, which are
expressed on all healthy cells. The inhibitory receptors prevent NK cells from
killing normal cells.
 Virus infection or neoplastic transformation often enhances expression of ligands
for activating receptors and at the same time reduces the expression of class I
MHC molecules.

14. Which of the following interfere with iron absorption?

a) Vitamin
b) Phytates
c) Oxalate
d) Myoglobin

Maximum site of Iron absorption - DUODENUM


The form in which Iron is absorbed - Fez+
Absorption of inorganic iron is enhanced by ascorbic acid, citric acid, HCL, amino
acids, and sugars the diet, and inhibited by Phytates and tannates (found in
tea), carbonates, oxalates, and phosphates.

15. Most sensitive and specific marker for myocardial infarction is?

a) Troponin
b) Cytokeratin
c) Myoglobin
d) CPK-MM

The laboratory evaluation of Mi is based on measuring the blood levels of proteins


that leak out of irreversibly damaged myocytes, the most useful of these molecules
are cardiac specific troponin T and I (cTnT and cTnI).

 Most sensitive and specific marker for MI-Troponin


 Most important enzyme or making the diagnosis of Reinfarction - Troponin.
 1st enzyme that increases after the attack of MI- Myoglobin.

16. Antecedent diagnosis of Group A streptococcal infection in Acute


rheumatic fever can be made by?

a) ASO
b) CRP
c) ESR elevation
d) Low C3 levels

Acute RF typically appears 10 days to 6 weeks after group A streptococcal infection


in about 3% of patients.
It occurs most often in children between ages 5 and 15, but first attacks can occur in
middle to later life. Although pharyngeal cultures for streptococci are negative by the
time the illness begins, antibodies to one or more streptococcal enzymes, such as
streptolysin O and DNase B can be detected in the sera of most patients with RF.

17. Most common anterior mediastinal tumour?

a) Thymoma
b) Neurofibroma
c) Pericardial cyst
d) Bronchogenic cyst

The designation thymoma" is restricted to tumors of thymic epithelial cells. Such


tumors typically also contain benign immature T cells.
The tumors usually occur in adults older than 40 years of age; thymomas are rare in
children. Males and females are affected equally. Most arise in the anterior superior
mediastinum, but sometimes they occur in the neck, thyroid, pulmonary hilum or
elsewhere

18. Most characteristic finding of diabetic nephropathy?

a) Kimmelstein Wilson disease


b) Diffuse glomerulosclerosis
c) Focal segmental glomerulosclerosis
d) Armani Ebstein change

Diabetic Nephropathy Three lesions are encountered: (1) glomerular lesions; (2)
renal vascular lesions, principally arteriolosclerosis; and (3) pyelonephritis, including
necrotizing papillits.

The most important glomerular lesions are capillary basement membrane


thickening, diffuse mesangial sclerosis, and Nodular glomerulosclerosis.
Capillary Basement Membrane Thickening. Widespread thickening of the
glomerular capillary basement membrane (GBM) occurs in virtually all cases of
diabetic nephropathy and is part and parcel of the diabetic microangiopathy Diffuse
Mesangial Sclerosis. This lesion consists of diffuse increase in mesangial matrix.
Nodular Glomerulosclerosis. This is also known inter capillary glomerulosclerosis or
Kimmelstiel-Wilson disease. It is the Most characteristic finding of Diabetic
Nephropathy.

19. A 23 year old individual with serum alkaline phosphatase normal, PTH
normal, Vitamin D3 normal with elevated serum Calcium values what will be
the diagnosis ?

a) Vitamin D intoxication
b) Hyperparathyroidism
c) Multiple myeloma
d) Nutritional rickets

Multiple myeloma is a plasma cell neoplasm commonly associated with lytic bone
lesions, hypercalcemia, renal failure, and acquired immune abnormalities.

Clinical Features.
The clinical features of multiple myeloma stem from
(1) the effects of plasma cell growth in tissues, particularly the bones
(2) the production of excessive Igs, which often have abnormal physicochemical
properties; and
(3)The suppression of normal humoral immunity.

Bone resorption often leads to pathologic fractures and chronic pain. The attendant
hypercalcemia can give rise to neurologic manifestations, such as confusion,
weakness, lethargy. constipation, and polyuria, and contributes to renal dysfunction.
Decreased production of normal lgs sets the stage for recurrent bacterial infections.
Cellular immunity is relatively unaffected. Of great significance is renal insufficiency,
which trails only infections as a cause of death. The pathogenesis of renal failure
,which occurs in up to 50% of patients, is multifactorial. However, the single most
important factor seems to be Bence Jones proteinuria, as the excreted light chains
are toxic to renal tubular epithelial cells. Certain light chains (particularly those of the
λ6 and λ3 families)are prone to cause amyloidosis of the AL type, which can
exacerbate renal dysfunction and deposit in other tissues as well.

20. MC cause of atypical pneumonia?

a) Mycoplasma pneumoniae
b) Klebsiella pneumoniae
c) Hemophilus influenzae
d) Chylamydia
MC cause of Atypical pneumonia- Mycoplasma Pneumonia aka "Eaton's agent".
This organism does not have cell wall which makes them resistant to the treatment
by Penicillin's.
The infection will be treated by the use of Macrolide antibiotic (Azithromycin).

BIOCHEM JUNE 2019

1. Enolase is inhibited by which of the following substances ?

a) NaF
b) Fluoroacetate
c) iodoacetate
d) Potassium oxalate

 NaF is added for in vitro test of blood glucose estimation.


 NaF Inhibits glycolysis
 If not added in the blood testing vial, glucose used by RBC and WBCs is not
measured, then the reported glucose value will be falsely decreased or
underestimation of real value.

Fluoroacetate--non-competitive inhibitor of Aconitase (TCA cycle)

Iodoacetate--Inhibits glycerate 3-phosphate Dehydrogenase (glycolysis)

Potassium oxalate--anti-coagulant

2. Maple syrup urine disease is due to deficiency of:

a) β keto acid decarboxylase


b) α keto acid carboxylase
c) α keto acid desulphate
d) α keto acid chain oxidase

BCAA b6 BCKAL
Transamination α-keto acid decarboxylase

accumulation of BCKA (branched chain keto acid)

Branched chain amino acid - valine, leucine, isoleucine this amino acid are
essential amino acid
Maple syrup urine disease - defect in branched chain amino acid metabolism
Enzyme - alpha keto acid dehydrogenase alpha- keto acid decarboxylase
C/F- burnt sugar like odour in urine
 Ketosis
 Mental retardation
 Abnormal muscle tone
 Coma & death

3. Collagen is rich in which amino acid:

a) Glycine
b) Arginine
c) Phenyl - alanine
d) Tyrosine

The Main Amino Acid in Collagen

Collagen is a structural protein that contains high amounts of glycine. In fact, every
third to fourth amino acid in collagen is glycine. Collagen is the most abundant
protein in your body. It provides strength for your muscles, skin, cartilage, blood,
bones and ligaments.
The triple-helical structure of collagen arises from an unusual abundance of three
amino acids: glycine, proline, and hydroxyproline.

4. Nitric oxide is derived from:

a) Arginine
b) Histidine
c) Glycine
d) Aspartate

Nitric oxide (NO) is a gaseous molecule with autocrine and paracrine effects on
many cell types. NO is synthesized from the amino acid l-arginine by NO
synthase (NOS) and is involved in a myriad of cellular functions, including muscle
relaxation, neuronal signaling, and immune function.

5. De—Novo synthesis of Fatty Acid requires which coenzyme?

a) NADPH
b) TPP
c) FAD
d) NAD

NADPH Reductive Biosynthesis Pathway

Fatty acid Synthesis


1st enzyme Acetyl CO A carboxylase
 Require biotin, ATP, CO2 and Bicarbonate

2nd enzyme -Fatty Acid Synthesis complex requires NADPH


 TPP is required for carbohydrate metabolism & Trans ketolase (HMP Pathway)
 FAD and NAD are required in beta oxidation of fatty acids and carbohydrate
metabolism (Glycolysis and TCA)

6. All are cofactors for Dehydrogenase except:

a) NADP
b) FAD
c) NAD
d) SAM

Dehydrogenases --remove H2 and which is usually taken up by their cofactors;

1. FAD → FADH2 →(ETC)


2. NAD → NADH →(ETC)
3. NADP → NADPH
 NADPH → used in anabolic pathway
→ SAM (S - Adenosyl Methionine) (Donor of methyl group) is a nucleotide but not a
cofactor.

7. After strenuous exercise, alkaline pH of skeletal muscle is in which


glycogen storage disease:-

a) Mc Ardle's disease
b) Von- Gierke's disease
c) Her's disease
d) Pompe's disease

Glycogen Phosphorylase absent an Mc Ardle's disease lactate will not formed


alkaline PH
Normal Condition

Glycogen glucose
glycogen phosphorylase
Anaerobic Glycolysis

Lactate
(PH ↓ )
8. Most potent lipid phase antioxidant:

a) Vitamin A
b) Vitamin E
c) Vitamin C
d) Vitamin K

Vit E delays the progression of atherosclerotic plague because its convert oxidased
LDH to Normal LDH
Vit E act simultaneously with selenium & selenium is part of glutathione peroxidase
which is involved in the oxidant Pathway in cell

Other anti-oxidant Vitamin - Vit A, C & D

Vit K - Fat soluble vitamin


only fat soluble vitamin which has co-enzyme role in carboxylation of glutamate in
clotting factors 2,7,9,10

9. Wernicke encephalopathy caused by?

a) Vit B1
b) Vit B2
c) Vit B7
d) Vit B12

Vit B1 deficiency seen in chronic alcoholics & lead to Wernicke's Korsakoff psychosis
and patient has memory loss / confabulation ,cerebral hemorrhage, muscles
paralysis

Vit B2 → riboflavinosis → 2C
 Corneal Vascularization
 Cheilosis

Vit B7 → alopecia, bowel inflammation, muscles pain

Vit B12 → megaloblastic anemia + Neurological manifestation

10. Pheochromocytoma is a rare, usually a benign tumor that develops in


chromaffin cell (pheochromocytes) of adrenal gland. The most specific
biomarker pheochromocytoma is which one of following?

a) VMA
b) Catecholamine
c) HIAA
d) Serotonin
Vanillylmandelic acid (VMA) and other catecholamine metabolites (homovanillic
acid: HVA and dopamine) are typically elevated in patients with catecholamine-
secreting tumors example: neuroblastoma, pheochromocytoma, and other neural
crest tumors.

Pheochromocytoma - tumor or Adrenal gland which secretes high amount of


epinephrine & Nor-epinephrine
catabolic products of catecholamine – VMA

End Products of Catecholamines

Epinephrine Dopamine Norepinephrine

Homo vanillic acid Vanillyl mandelic acid Metanephrine

11. Which cell releases histamine?

a) Mast cells
b) Eosinophils
c) T-cells
d) NK cells

Mast cell release Histamine & Heparin


Histamine is vasodilators & increase permeability for WBC in Blood vessels.

FORENSIC JUNE 2019

1. Grounds of divorce:

a) Sterility
b) Frigidity
c) Impotence developing after the marriage
d) Poverty

Impotence at the time of marriage is a ground for nullity of marriage.


Impotence developing after the marriage is a ground for divorce.
Sterility is not a ground for divorce.

Impotence: is the inability of a person to perform sexual intercourse and achieve


gratification.
Frigidity: It is the inability to initiate or maintain the sexual arousal pattern in female
(absence of desire for sexual intercourse or incapacity to achieve orgasm).

Sterility: It is the absolute inability of either a male or a female to procreate. In male,


it is inability to make a female conceive, and in females, it is inability to conceive
children.

2. IPC for causing abortion without women's consent:

a) 312
b) 313
c) 314
d) 315

IPC - Abortion

Section Defines/Punishes

312 Punishment for Criminal abortion with


consent of woman
313 Punishment for Criminal abortion without
consent of woman
314 Punishment for Criminal abortion
leading to death of mother
315 Punishment for Causing death of quick
unborn child

Criminal Abortion is dealt under Section 312-316 IPC:

Sec. 312 1.P.C. whoever voluntarily causes criminal abortion is liable for
imprisonment up to three years, and/or fine; and if the woman is quick with child the
imprisonment may extend up to seven years. It is necessary that the woman should
be pregnant and that abortion should be carried with her consent. Both the person
causing the abortion and the woman are liable for punishment.

Sec. 313 IPC: if the miscarriage is caused without the consent of the woman, the
imprisonment may be up to ten years.

Sec. 314 PC: if a pregnant woman dies from an act intended to cause miscarriage,
the offender is liable to be punished with rigorous imprisonment which shall not be
less than ten years and also fine up to two lakh rupees.
Sec. 315 IPC: a person doing an act intended to prevent the child from being born
alive or to cause to die after its birth, is liable to be punished with imprisonment up to
ten years.

Sec. 316 IPC: causing death of quick unborn child by any act amounts to culpable
homicide, and the punishment may extend up to ten years imprisonment.

3. In corrosive acid case is opened along:


a) Lesser curvature
b) Greater curvature
c)Vertical
d) Pylorus

In corrosive acid poisoning, it is open along the greater curvature.

Toxic substances may be held in high concentrations in the rugae and crypts of the
mucosa in the actual stomach wall.
The pathway of acids and alkalis in food-filled stomach starts along the lesser
curvature of the stomach and leads to the pylorus, which explains the location of
greatest damage in stomach.

4. Inhaling the cloth soaked in drug is

a) Bagging
b) Spraying
c) Huffing
d) Sniffing

Huffing: Applying the chemical to a cloth/rag and then inhaling by covering nose and
mouth with the cloth/rag.

Sniffing: Inhaling fumes from the liquid in an open container.

Bagging: Placing the chemical in a bag and then putting it over the face.

5. One of the victims of the fire which had destroyed 6 huts in a nearby slum
has decided to give a dying declaration. The doctor was called in as part of
the team to record the statement. Which among the following is the role of
the doctor ?

a) To assess compos mentis


b) To record statement even in presence of magistrate
c) Cross examine the person
d) Put person under oath before declaration
Dying declaration : written or oral statement of a person, who dying as a result of
some unlawful act, relating to the material facts of the cause of death or the
circumstances surrounding it.

The law does not provide who can record dying declaration, nor is there any
prescribed form, format or procedure for the same.
The doctor should certify that the person is conscious and his mental faculties are
normal, i.e. he is in compos mentis.

Dying Declaration VS Dying Deposition


Dying Declaration Dying Deposition
Recorded by Magistrate/doctor/lay Magistrate only
person
Oath - +
Cross examination of - +
person
Superiority Less Superior
Accepted under SEC(32 IEA) Not accepted in India

PSM JUNE - 2019

1. Inertization is:

a) Reducing organic and combustible waste to inorganic


b) Burning biomedical waste
c) Biomedical waste converted into non-harmful
d) To avoid water contamination

Inertization technique is used when large volume of toxic Bio medical waste is
converted to non- toxic waste.

It is achieved by mixing,
 15% lime and
 15% cement with
 65% biomedical waste
 5% water

2. Amplifier host of Japanese encephalitis:

a) Pig
b) Dogs
c) Cats
d) Birds
Japanese Encephalitis

 Actual host →Ardeid birds


 Natural / Amplifier host → Pig
 Vector Culex → Tritaeniorhynchus
 Live vaccine → SA - 14-14-2
 Two doses of vaccine SA-14-14-2 is given, 1st at 9 months & 2nd at 16 to 24
months of age.
 Mainly affects children under 1-15 years of age.

3. Best index for calculation of nutrient value of protein

a) Serum albumin concentration


b) Arm-muscle circumference
c) Creatinine-height index
d) Serum transferrin level

 Assessment of protein nutritional status.


 5 Major indicators

 Arm muscle circumference


 Creatinine height index
 Serum Album
 Serum Transferrin
 Total Body Nitrogen (TBN)

 Serum albumin is the best indicator


 Serum albumin concertation
 >3.5 g/ dl - Normal
 3.0-3.5 Mild to moderate malnutrition
 < 3.0 - Sever malnutrition

4. Best index for measurement health indicator in society:

a) Under 5 mortality rate


b) Infant Mortality Rate
c) Child death rate
d) Maternal mortality rate

Infant mortality is the indicator / marker of


 Health status population
 Health care infants
 Health services
Under 5 mortality is marker of → Social development of the Country.
Material mortality rate is the indicator of → Maternal care.
Perinatal mortality rate is the very good indicator of → Combined Maternal and
newborn care.

5. A community medicine student was asked to explain about natural sources


of Vit.A. Which of the following has the maximum natural source of vitamin
A?

a) Carrot
b) Halibut liver oil
c) Cod fish oil
d) Cow milk

 Halibut Liver Oil has the maximum natural content of Vitamin A & also it has the
richest source of the Vitamin D.
 Among fruits Ripe Mango & among vegetables carrot have high content of
Vitamin A.
 Vitamin A deficiency causes Xerophthalmia.

6. According to WHO, marker for obesity Index is:

a) Quetelet index
b) Chandler index
c) Pearl index
d) Sullivan's index

BMI is still referred to as the Quetelet index, for which the formula is: BMI (kg/m2) =
mass (kg) / height (m)2.

Body Mass Index Values


 Normal → 18.5 to 25
 Overweight → 25 to 29.9
 Obesity → > 30
 Underweight→ < 18.5
BMI in Percentile
 >95th Percentile → Obesity
 >85th Percentile → Overweight
 <5th Percentile → Underweight

7. According to WHO, which one is a notifiable disease?

a) Yellow fever
b) Polio
c)HIV
d) Malaria
The International Health Regulations (WHO IHR) does not cover HIVAIDS
Immediately Notifiable Diseases: Smallpox, Human Influenza, Wild Polio, SARS
Potentially Notifiable Diseases:
 Public health importance: Cholera, Plague, Yellow fever, Viral hemorrhagic
fevers, West nile ever, Dengue, Rift valley fever, Meningococcal disease
 Biological/Chemical/Radiological events
 Serious illnesses of Unknown origin

8. Gujarat recently reported an massive earthquake disaster causing loss of


life and leading to sufferings. Which of the following is the most common
reported disease in post disaster period?

a) Acute gastroenteritis
b) Pneumonia
c) Leptospirosis
d) Malnutrition

 Most Common reported disease in post disaster phase is Acute Gastroenteritis,


Typhoid & Cholera.
 Most important step during Post-disaster phase is, Chlorination of water >
Vaccination
 All vaccines are CI in post-disaster phase; except Measles

Extra edge:
Chairman of National Disaster Management Authority (NDMA) of India is Prime
Minister of India

9. Amount of sodium in ORS as per WHO:

a) 65 mmol/L
b) 75 mmol/L
c) 20 mmol/L
d) 100 mmol/L
10. As compared to cow milk, the protein content of human milk is:

a) 3-4 times more than cow milk


b) 3-4 times less than cow milk
c) Double than cow milk
d) Same as cow milk

Human Milk Cow's Milk


Total Proteins 11 33
Casein 4 28
Soluble Proteins 7 5

Human milk is higher in Lactose, Iron, Water and Ca: P ratio.


Cow's milk is higher in Proteins, Fats, Energy & absolute quantity of Calcium and
Phosphorous.

11. BCG is diluted with:

a) Normal saline
b) Distilled water
c) Ringer lactate
d) Distilled water
BCG is available in Powder form, which is diluted with Normal saline and it should be
used within 4 hours,
Otherwise Toxic shock syndrome can occur.
 Strain →Danish - 1331 •
 Dose → 0.1 ml intradermally in left deltoid in newborns within 28 days. -
 Efficacy of dose →0 to 80%
 Protective duration → 20 years

12. Nowadays which is the most common pasteurization method?

a) Holder (VAT) method


b) Ultra-high temperature method
c) High temperature and short time method
d) Low temperature and longtime method

 High temperature and short time is known as Flesh Method 72 °C X 15 seconds.


 In rural areas Holder VAT method is most popular 63-66 •C for 30 mins.
 Phosphatase test- used to test the sufficiency of pressurized milk.

13. True about Quarantine:

a) Restriction of movement of Infected patient


b) Restriction of movement of healthy contact of an infectious disease
c) Movement restricted for shortest incubation period
d) Isolation of diseased person

QUARANTINE ISOLATION
Is done for Healthy contact of a case Cases until recovery
with maximum Insulation
period
Level of prevention Primary Secondary

 Quarantine was first done for the disease plague.

14. A 26 year old pregnant female visited the sub centre for her check-up.
Which of the following test is done at Subcentre during pregnancy?

a) USG
b) Haemoglobin
c)OGTT
d) Triple test

Tests done at Sub center:


 Pregnancy detection test
 Hemoglobin level estimation
 Urine test for albumin and sugar
 Rapid malaria test

Tests done at PHC/CHC/DH:

 Blood grouping with Rh


 HIV testing
 VDRL, RPR
 Rapid Malaria test
 Blood Sugar
 HBSAg.

15. 25 year old patient with diarrheal episodes presented to the hospital. The
physician started his treatment with ORS. In preparation of ORS sugar is
added, Which of the following is the correct reason to add sugar in ORS?

a) Enhance acceptability
b) Enhance salt absorption
c)Enhance shelf life
d) Enhance taste

Adding sugar to ORS, enhances absorption of salt & water.


Sugar in the ORS has found to have sym- transporter mechanism with sodium.
Composition Weight

Sodium chloride 2.6 gm

Potassium chloride 1.5 gm

Sodium citrate 2.9 gm

Glucose anhydrous 13.5 gm

Total 20.5 gms

Osmolar (mmol/liter)
concentration
Sodium 75

Potassium 20

Chloride 65

Citrate 10
Glucose 75
anhydrous
Total 245

ReSoMAL: Rehydration solution for MALnourished


Sodium • halved to 45 mmol/L : Potassium Doubled to 40 mmol / L
SUPER ORS: Rice /starch/ Alanine based [not mono-sugars]

16. Vitamin which helps in Iron absorption is:

a) Vitamin A
b) Vitamin B
c) Vitamin C
d) Vitamin E

Decrease Iron absorption


Phytate (cereals, Pulses)
Tannates (Tea, Coffee)
Oxalates (green vegetables)
Calcium
Drugs like Tetracycline
 Iron absorption in India is < 10%
 Vitamin- C is the only facilitators of Iron absorption

17. Wernicke encephalopathy is due to deficiency of:

a) Thiamine
b) Biotin
c)Niacin
d) Hydroxycobalamin

 Thiamin (Vitamin B1) deficiency leads to Beri- Beri seen in polished rice eaters.
 In polished rice outer layer of rice is removed which contains Thiamine, it is
more commonly in Andhra Pradesh & Telangana States.
 Thiamine deficiency also causes Wernicke's encephalopathy & Wernicke's
Korakoff psychosis (seen in alcoholics)

18. Healthy volunteers are recruited in which phase of Clinical trials?

a) Phase1
b) Phase 2
c)Phase 3
d) Phase 4

Phase 1: Healthy volunteers are re elevated to establish safety & Non- toxicity of
drugs.
Phase 2: Patients are used to record the efficiency of the drug.
Phase 3: Patients are used mainly for compassion of existing drugs.
Phase 4:
 Done on patients where post marketing surveillance of rare and long term side
effects is evaluated.
 This is the longest phase of trail with minimum 10 to 25 years preferably till
lifelong.
Phase 1 → tell the Maximum Tolerated Dose of the drug.
Phase 2 → Maximum Drug Failure occurs.
Phase 3 → Is a Randomized control trail
Phase O → Used before phase with healthy volunteers with 1/10th of the dose
(micro dosing) with very less volunteers

19. A 29 year old male patient with abdominal pain, fatigue, dark colored urine,
clay colored stools and yellow sclera presented to the hospital. On
serological examination the patient is showing HBsAg positive and HBeAg
positive. Which of the following is the correct clinical diagnosis ?

a) Acute viral hepatitis


b) Chronic viral hepatitis
c) Acute viral hepatitis with infectivity
d) Remote infection

HBs Ag →Epidemiologic marker (1st antigen to appears in infection)


HBc Ag →Rarely appears in plasma.
HBe Ag →Marker of infectivity, indicates & active viral replication.

Condition Serological Pattern

Acute Hepatitis B HBsAg, HBeAg, IgM Anti-HBC

Chronic Hepatitis B, with active viral HbsAg, HBeAg, IgG Anti-HBc


replication

Chronic Hepatitis B, with low viral HbsAg, IgG Anti-HBc, Anti-HBe


replication

Recovery from Hepatitis B infection Anti-HBS, IgG Anti-HBC

Post Vaccination Anti-HBS

20. Which of the following are agents of Farmer's Lung?

a) Thermophilic Actinomycetes
b) Aspergillus fumigatus
c) Aspergillus flavus
d) Aspergillus niger
Farmers lung-exposure to mouldy hay contaminated with Thermophilic
Actinomycetes/ Micropolyspora Faeni .
Aspergillus fumnigatus and flavus are opportunistic pathogens and common causes
of infections in humans.
Pneumoconiosis occurs with particles of size 0.5- 3μ diameter.

21. Which of the following interfere with Iron absorption?

a) Vitamin C
b) Phytates
c)Hemoglobin
d) Myoglobin

Decrease Iron absorption :


 Phytate (cereals, Pulses)
 Tannates (Tea, Coffee)
 Oxalates (green vegetables)
 Calcium
 Drugs like Tetracycline
Iron absorption in India is <10%
Vitamin- C is the only facilitators of Iron absorption

22. Chronic malnutrition in child is best evaluated by?

a) Weight for height


b) Weight for age
c) Height for age
d) Ponderal index

Low weight for age → underweight →acute or chronic malnutrition


Low Height for age→ stunting → chronic malnutrition
Low weight for height → wasting → Acute malnutrition

23. A married couple visited to the gynaecology department complaining of not


able to conceive even after year of regular unprotected coitus. The
physician sent the male partner for semen analysis. According to WHO
which of the following is the correct minimum sperm count for Normal
Semen analysis

a) 2 million / mL
b) 15 million /mL
c)10 million / mL
d) 20 million ml

Latest semen analysis guidelines of 2010 says:


 Minimum sperm count- 15 million / ml
 Minimum volume - 1.5 ml
 Motility- > 40%
 Progressive motility- > 32%
 Vitality- > 58%
 PH-> 7.2
 Normal morphology- > 4%

24. Most common cause of Blindness in India:

a) Cataract
b) Refractive error
c)Trachoma
d) Glaucoma

 According to latest 2015-2019 survey in India, prevalence of blindness is about


0.32% among that most common cause contributing to this cataract (66.2%),
reflective error contributes 20%
 Cataract is also the most common cause of visual impairment in India.
 Blindness is defined as < 3/60 in better eye with best possible duration.

PHYSIO JUNE 2019

1. What is the cause of dicrotic notch?

a) Passive filling of blood in ventricles


b) Rapid ejection phase
c) Closure of semilunar valves
d) Isovolumic contraction

The dicrotic notch is a small and brief increase in arterial blood pressure that
appears when the aortic valve closes. The dicrotic notch would therefore be the
result of a short period of backward flow of blood immediately before the aortic valve
closes.

2. Which of the following is correct regarding Isovolumic relaxation?

a) AV valves are closed


b) Corresponds to QT interval
c)C wave of JVP
d) Semilunar valves open

ISOVOLUMETRIC RELAXATION:
Heart:
At the beginning of this phase the AV valves are closed.

Pressures & Volume:


Throughout this and the previous two phases, the atrium in diastole has been filling
with blood on top of the closed AV valve, causing atrial pressure to rise gradually.
The "v" wave is due to the back flow of blood after it hits the closed AV valve. It is the
second discernible wave of the jugular venous pulse.
The pressure in the ventricles continues to drop.
Ventricular volume is at a minimum and is ready to be filled again with blood.

Heart sounds:
The second heart sound (S2, "dup") occurs when the semilunar (aortic and
pulmonary) valves close. S2 Is normally split because the aortic valve closes slightly
earlier than the pulmonary valve.

3. Adequate 02 oxygen delivery at cellular level occurs in which type of


hypoxia?

a) Hypoxic
b) Anaemic
c)Stagnant
d) Histotoxic

Histotoxic hypoxia (also called histoxic hypoxia) is the inability of cells to take up
or use oxygen from the bloodstream, despite physiologically normal delivery of
oxygen to such cells and tissues but the oxygen delivery to tissue is normal.

4. If radius of a vessel is doubled, then the blood flow is increased by?

a) 8 times
b) 16 times
c)32 times
d) 256 times

Vessel resistance (R) is directly proportional to the length (L) of the vessel and the
viscosity (n) of the blood, and inversely proportional to the radius to the fourth power
(r4)
So when radius is increased by 2 times blood flow decreases by 16 times.

5. Normal anion gap is__ mmol/L?

a) 4-6
b) 6-12
c) 12-24
d) 24-50
The cation normally measured is Nat, and the anions are usually Cl® and HCO3 The
"anion gap" is the difference between unmeasured anions and unmeasured cations
and is estimated as

PLASMA ANION GAP = [Na+] – [HCO3-] – [Cl-] = 144 – 24 – 108= 12 mEq/L


Normal range = 8-16 mEq/L

6. Breathing pattern disorders occur when ventilation exceeds metabolic


demands, resulting in symptom-producing hemodynamic and chemical
changes. As well as having a marked effect on the biochemistry of the
body, BPDs can influence emotions, circulation, digestive function as well
as musculoskeletal structures involved in the respiratory process. Which of
the following have prolonged inspiratory spasms that resemble breath
holding?

1. Biot breathing
2. Apneustic breathing
3. Cheyne stokes breathing
4. Kussmaul breathing

APNEUSTIC BREATHING → long inspiratory spasms (resembles breath holding)


CHEYNE-STROKE BREATHING → apnea followed by hyperapnea
KUSSMAUL RESPIRATION → increased depth and prolonged inspiration
REGULARLY IRREGULAR→ seen in meningitis

SURGERY JUNE 2019

7. A patient presented to emergency after RTA with multiple rib fractures. He


is conscious, speaking single words. On examination, respiratory rate was
40/min and BP was 90/40 mm Hg, What is the immediate next step?

a) Urgent IV fluid administration


b) Intubate the patient
c)Chest -ray
d) Insert needle in 2nd intercostal space

 In this case, Patient having tachypnea, dyspnea, hypotension


 After RTA, there is also rib fracture
 Above all are suggestive of tension pneumothorax
 It is an emergency condition, so wide bore needle is inserted in 2nd intercostal
space
According to ATLS (2018):
In adults, wide bore needle is inserted in 5th intercostal space slightly anterior to mid
axillary line
TOC: ICD insertion in 5th intercostal space in triangle of safety
8. Correct procedure of inserting Nasogastric tube is?

a) Supine with neck flexed


b) Supine with neck extended
c) Sitting with neck flexed
d) Sitting with neck extended

 Ask the patient to sit and flex the neck


 Lubricate the tube and the nasal cavity
 And then insert the tube

9. A 7-day old infant presents with bilious vomiting and gross abdominal
distention with absent bowel sounds. X ray abdomen shows multiple gas
filled loops. Diagnosis is?

a) Hirschsprung disease
b) Congenital Hypertrophic pyloric stenosis
c)Duodenal atresia
d) Malrotation of gut

Malrotation of gut:
 Surgical emergency
 Bilious vomiting - first sign of volvulus
 Abdominal distention
 Multiple gas filled loops

The cardinal symptom of intestinal obstruction in the newborn is bilious emesis.


Proximal intestinal obstructions typically present with bilious emesis and minimal
abdominal distension. X-ray shows a paucity or absence of bowel gas, which should
normally be present throughout GIT within 24 hours
Distal obstructions typically present with bilious vomiting and abdominal distension.
Passage of black green meconium should have occurred within the first 24 to 38
hours

X-ray calcification- complicated meconium ileus


Pneumatosis or pneumoperitoneum - necrotising enterocolitis
Contrast enema - microcolon - jejunoileal atresia or meconium ileus
No microcolon - Hirschsprung's disease, small left colon syndrome or meconium
plug syndrome.

10. Newborn infant presents with abdominal distension and bilious vomiting.
There is failure to pass meconium beyond 24 hours. X-ray shows dilated
small bowel loops with no air fluid levels and soap bubble appearance in
right lower quadrant. This condition is initially treated by:
a) Paul mikulicz ileostomy
b) Bishop ileostomy
c) Contrast enema
d) Barium enema

Infant presents with signs and symptoms of small bowel obstruction (abdominal
distension, bilious vomiting and inability to pass meconium). X ray shows typical
features, suggestive of meconium ileus
Uncomplicated meconium ileus is initially treated by contrast enema failure of
which surgical management is advocated.

11. A full-term, 2.0 Kg male baby, delivered at home presented with excessive
frothing within a few hours of birth. The antenatal period was unsupervised
and the delivery, uncomplicated. At presentation, the baby had mild
respiratory distress. The first investigation to be done in this case IS:

a) Bronchoscopy with injection of methylene blue


b) NG Tube insertion and CXR to check position of tube
c) CT chest
d) Endoscopy

Two problems
 Collection of saliva in the mouth
 Air coming inside the mouth
Therefore, saliva is not going into the esophagus
Esophageal atresia with tracheoesophageal fistula

Diagnosis:
 The inability to pass a NG int the stomach is a cardinal feature for the diagnosis
of EA
 If gas is present in the GIT below the diaphragm, an associated TEF is confirmed
 Inability to pass a NG tube with absent radiographic evidence for gastrointestinal
gas is virtually diagnostic of an isolated EA without TEF
 For H-type: tracheobronchoscopy + endoscopy should be performed

12. Calculate the GCS of a patient exhibiting eye opening on pain, conscious
but confused and cannot tell time and exhibits flexion on painful noxious
stimuli to the arm.

a) 8
b) 9
c) 10
d) 11

BEHAVIOR RESPONSE SCORE


Spontaneously 4
To speech 3
Eye opening response To pain 2
No response 1
Oriented to time, place, 5
and person
Confused 4
Best verbal response Inappropriate words 3
Incomprehensible sounds 2
No response 1
Obeys commands 6
Moves to localized pain 5
Flexion withdrawal from 4
pain
Best motor response Abnormal flexion 3
(decorticate)
Abnormal extension 2
(decerebrate)
No response 1
Total score Best response 15
Comatose client 8 or less
Totally unresponsive 3

13. A 27-year-old woman presents with 26 weeks of gestation with thyroid


lesion which is found to be papillary carcinoma of thyroid (PCT). Which is
the best treatment for this patient?

a) Thyroid ablation using radioactive lodine


b) Total thyroidectomy
c)Observation
d) Hemi-thyroidectomy

In this case patient is pregnant therefore total thyroidectomy is the treatment of


choice

Treatment of PCT:
 Total thyroidectomy (TOC)
 Radioactive iodine (RAI)
Contraindications of RAI:
 Pregnancy
 Lactation

14. 36 yr old African-american male presents to plastic surgeon with recurrent


keloid lesion in the left earlobe with a history of previous injuries. The
lesion was operated on, but always grows back postoperatively. Most
appropriate management of Recurrent Keloid is?

a) Excisional surgery
b) Intramarginal excision followed by radiation
c)Cryosurgery
d) Silicone Gel Sheeting
 Keloids rarely regress with time, often refractory to medical and surgical
intervention
 First line treatment: silicone in combination with pressure therapy and
intralesional corticosteroid injection
 Refractory cases (after 12 months of therapy): excision + post op radiotherapy
(external beam or
 brachytherapy)
 New treatment modalities: internal cryotherapy and 5% imiquimod

15. Seat Belt injury leads to?

a) Splenic laceration
b) Splenic contusion
c) Mesenteric tear
d) Mesenteric adenitis

 In Seat belt injury there is sudden deceleration → Mesenteric tear


 Classical seat belt sign is abrasion involving neck, chest and abdomen
 Most commonly injured structure is mesentery
 Two types of tears are seen
 Longitudinal tear - can easily repair
 Transverse tear - resection and anastomosis

Mesenteric adenitis is a condition that affects the lymph nodes in the membrane
which connects bowel to the abdominal walll (mesentry). A viral infection is the usual
cause.

16. Which complication is seen after Varicose vein stripping procedure?

a) Neuralgia
b) Deep vein thrombosis
c)Acrocyanosis
d) Telangiectasia

 During stripping of long saphenous vein most commonly injured nerve is


saphenous nerve (saphenous neuralgia)
 During stripping of short saphenous vein most commonly injured nerve is sural
nerve
 Neuralgia is prevented by limit the stripping of long saphenous vein till knee
joint

17. Which of the following is not scanned by FAST USG?


a) Pericardium
b) Pleural cavity
c)Spleen
d) Liver

FAST- Focussed assessment with sonography for trauma


It is performed in 2-4 minutes-4 views (4 P) -
 Pericardium
 Perihepatic region
 Peri splenic region
 Pelvis

e- FAST- Extended FAST- 6 views


 Pericardium
 Perihepatic region
 Peri splenic region
 Pelvis
 Right thoracic view
 Left thoracic view

18. A 40-year-old man is suffering from heaviness in scrotum. A bag of worms


feel is observed on scrotal examination and the swelling is seen to reduce
in supine position. What is the best treatment?

a) Suction drainage
b) Varicocelectomy
c)Jaboulay procedure
d) Herniotomy

VARICOCELE
 Bag of worm like sensation
 Dilated tortuous pampiniform plexus of vein
 Treatment- Varicocelectomy/ high inguinal ligation of pampiniform plexus of veins
 Jabuloy procedure is eversion of sac performed for median sized hydrocele
 Herniotomy is preferred for Hernia

19. A 9-year-old boy presented in the urology outpatient department with


dysuria, intermittently interrupted urinary flow, and pain in the lower
abdomen for one year. USG of abdomen detected a single urinary bladder
stone of 14.6mm size. All the blood tests and routine examination of urine
were within normal limits. There was no significant growth in urine culture
preoperatively. Open cystolithotomy was done under general anesthesia.
He was catheterized and a drain was placed for two days for drainage of
retro pubic space. Post-operative dressing done and amoxicillin-clavulanic
acid (375mg) was prescribed thrice daily for ten days. He was discharged
with a urinary catheter on the seventh post-op day and was advised to apply
mupirocin ointment topically at the surgical site daily. But the surgical wound
never healed properly and after six weeks patient presented with the discharge
of pus from the incision site with lower abdominal pain and a burning
sensation in the urethra. In this patient, the occurrence of SSI could have been
reduced if Prophylactic antibiotics were given before?

a) 60 minutes before skin incision


b) 1-3 hours before skin incision
c)At time of surgical incision
d) Night before surgery for peaking of effect

Prophylactic antibiotic to minimize SSI are given 30- 60 minutes before skin incision /
at the time of induction

20. Which procedure is done in case of recurrent Ranula management?

a) Incision and drainage


b) Aspiration
c) Excision
d) Sclerosant injection

 Incision over ranula and edges are sutured- Marsupialization


 In cases of recurrence- Excision

21. Which of the following is best for diagnosis of pheochromocytoma?

a) 24-hour urinary Vaniyll Mandelic acid


b) 24-hour urinary Fractionated Metanephrine
c)24-hour Urinary Hydroxy indole acetic acid
d) 24-hour Urinary Hydroxy tryptamine

Pheochromocytoma

 Most sensitive screening test for pheochromocytoma- Urinary catecholamines


and Vanillyl mandelic acid (VMA)level.
 Best test for diagnosis of pheochromocytoma- 24 hours plasma fractionated
Metanphrine >24 hours urinary plasma fractionated Metanephrine
 24 hours urinary hydroxy indole acetic acid is performed for carcinoid tumors
22. Identify the instrument in the image below ?

a) Artery forceps
b) Kocher forceps
c)Allis forceps
d) Babcock forceps

Artery forceps are used to compress an artery to stem bleeding, because of this
they belong to a group of surgical instruments called hemostats.

23. Identify the image ?

a) Tongue depressor
b) Doyen retractor
c)Self-retaining retractor
d) Langenbeck's retractor
Tongue depressor
 Used in surgery department for evaluation of oral cavity
 Used in ENT department generally

24. What is the Diagnosis ?

a) Sebaceous cyst
b) Implantation dermoid
c)Angular dermoid
d) Lipoma

This image is a Swelling in the scalp with no punctum


Sebaceous cyst
 Generally sebaceous cyst has punctum
 But if present in scalp and scrotum it does not have punctum.
25. What is the diagnosis ?

a) Lipoma
b) Encephalocele
c)Cystic hygroma
d) Lymphadenopathy

Encephalocele-protrusion of brain and membranes covered by skin and due to


Failure of neural tube to close completely

26. 40 year old male presented with complaints of dysphagia ,regurgitation and
foul smelling mouth. On further evaluation he was diagnosed to have
Zenker's diverticulum. This diverticulum is due to herniation of esophagus
through the Killian's dehiscence which is located:

a) Below Superior constrictor


b) Between Inferior constrictor
c) Below cricopharyngeal muscle
d) Below upper 1/3rd of smooth muscle of oesophagus

Killian's dehiscence is weakness located between the inferior constrictor muscle

27. Which organ has highest chances of Graft rejection response?

a) Cornea
b) Gut
c)Liver
d) Skin
Graft vs Host disease
Maximum chance of Graft vs Host disease is GUT transplantation/small bowel
transplantation because Donor's small bowel- presence of leucocytes in the lumen

28. In hypovolemic shock which organ should be assessed for determining


under-perfusion?

a) Kidney
b) Heart
c)Lung
d) Liver

In hypovolemic shock, kidney has to be assessed for determining under perfusion


Best clinical indicator of tissue perfusion - Urine output.

ORTHO JUN 2019

1. A 6-year-old child is suspected with supracondylar fracture of right hand,


complaining of pain and swelling ,X-ray of right elbow was not significant.
What is the next best step in this case?

a) Compare with X-ray of left hand


b) Closed reduction and slab
c) Closed reduction with K wire fixation
d) Cast

Supracondylar humerus fracture:

Suspected elbow fracture in a child - Supracondylar humerus fracture


Undisplaced # of humerus → posterior fat is elevated → "Fat pad sign"
Fracture is displaced → distal fragments goes posteriorly → Extension
Common n/v involved - AIN (Anterior interosseous nerve) > Median > Radial> Ulnar
(AMRU)

2. A patient presented with following serum parameters: Normal serum ALP


normal PTH level and increased Ca+ and PO4. Most likely cause:

a) Vitamin D toxicity
b) Hyperparathyroidism
c) Osteoporosis
d) Osteomalacia
Bio-Chemical Changes
Osteoporosis Osteomalacia Hyperparathyroidism
Calcium Normal ↓/ Normal Increased
Phosphate Normal ↑ Low (phosphateuria)
ALP Normal ↑ -
PTH - ↑ -

Hyper toxicity of Vit-d: ↑Ca &Phosphate levels

3. Three point bony relationship has diagnostic value in:

a) Elbow fracture
b) Monteggia fracture
c)Galeazzi fracture
d) Colles' fracture
4. Management of olecranon fracture as seen in the image is:

a) Above elbow plaster slab


b) Below elbow plaster slab
c)Close reduction with Tension band wiring
d) Open reduction with Tension band wiring

Given X ray shows olecanon fracture (type 2)- Tension band wiring

TYPES OF OLECRANON FRACTURE:-


 Type I : Crack without displacement of fragments.
 Type I: Clean break with separation of fragments.
 Type Il. Comminuted fracture

TREATMENT:-
 Type I → above-elbow plaster slab in 30 degree of flexion
 Type II → open reduction and internal fixation using tension-band wiring
 Type III→ plating or tension-band wiring or excision of the fragments
5. The diagnosis of the given image

a) Scapho lunate instability


b) Lunate injury
c) Distal radio ulnar joint instability
d) Wrist dislocation

Given X ray shows scaphoid instability


 Scaphoid - Most common carpal bone to fracture
 Scapho-Lunate dissociation -Gap between scaphoid & lunate Terry Thomas sign
/ David letterman sign
 Terry-Thomas sign indicates scapholunate instability
 It denotes the increased gap between the scaphoid and the lunate.
 It results from tearing of the scapholunate ligament following fall on outstretched
hand.
 A scapholunate gap of >3 mm is suggestive of instability.

The given x-
ray shows
the Terry-
Thomas sign
and the
image shows
comedian
Terry-
Thomas,
after whom
this sign has
been named:
6. Diagnosis of Gout is confirmed by which test?

a) Serum Uric acid level


b) Synovial fluid analysis
c)Urine uric acid levels
d) X- ray changes

Synovial fluid test


Synovial fluid is a liquid found in your body's joints. This test looks at whether you
have urate crystals in the joint fluid, which could mean you have gout. This is
considered the most accurate test for diagnosing gout.

OPTHAL JUN 2019

1. Most common cause of Blindness in India:

a) Cataract
b) Refractive error
c)Trachoma
d) Glaucoma

 Most common cause of blindness in India is cataract


 Refractive error is not a cause of blindness, it is ocular morbidity
 Trachoma is the most common infectious cause of blindness
 Glaucoma is second commonest cause of blindness in the world

2. In patient of head injury, who presents with headache and increased


intracranial pressure, the effect on pupil is:

a) Ipsilateral mydriasis
b) Contralateral mydriasis
c) Ipsilateral miosis
d) Contralateral miosis

Effect on pupil is ipsilateral mydriasis.


 In this patient of head injury bleeding occurs, which increases Intra cranial
pressure
 This increased intra cranial pressure compresses and damages the 3rd nerve
 3rd nerve carry the parasympathetic fibers which is responsible for pupillary
constriction
 So, the unopposed sympathetic fibers mediated by the 5th nerve dilate the
pupil
 Hence patient comes with ipsilateral mydriasis

Hutchinson's pupil is an ipsilateral mydriasis due to 3rd cranial nerve


compression by an intracranial mass lesion including intracranial bleeds.
3. Foster Kennedy syndrome is characterized by all of the following except:
1. Ipsilateral optic atrophy
2. Contralateral anosmia
3. Contralateral papilledema
4.Contralateral central scotoma
5. Bilateral progressive optic neuropathy

a) 1,4,5 are correct


b) 3.4,5 are correct
c)1,3,4 are correct
d) 2,4,5 are correct

Foster-Kennedy syndrome.-
 It is associated with olfactory or sphenoidal meningioma and frontal lobe
tumors.
 In this condition, there occurs pressure optic atrophy on the side of lesion and
papilledema on the other side (due to raised intracranial pressure).
 Ipsilateral central scotoma
 Ipsilateral anosmia

4. A 42-year-old man presented to the emergency department with multiple


knife wounds, one of which penetrated the right eye creating a 207mm full
thickness laceration of the upper lid and a 12?mm superonasal full-
thickness scleral laceration. The laceration extended radially back towards
the optic nerve with prolapse of the intraocular contents. On admission, the
patient underwent Evisceration of his right eye ball .Out of the given
choices, Which layer of eyeball is removed during evisceration?

a) Middle and inner


b) Outer and middle
c)Outer and inner
d) All the lavers of eyeball

Evisceration means Removal of the viscera

Evisceration:
The contents of the eye are removed, but the outer layer of the eyeball (sclera) is left
intact.

Enucleation
The entire eye, including the globe, is removed but the orbital contents are left in
place.

Exenteration
The contents of the eye socket are removed, including the muscles, lacrimal gland,
optic nerve and various bones of the orbit.
5. The following image shows ?

a) Bacterial keratitis
b) Fungal keratitis
c) Viral keratitis
d) Syphilitic keratitis

Viral keratitis showing dendritic ulcer Caused by HSV

Fluorescein staining
 May reveal dendritic lesions (linear branching patterns with terminal bulbs)
 If the lesion enlarges, it can form a geographic ulcer

EPITHELIAL KERATITIS →DENDRITIC ULCER (Loss of corneal sensation)


→GEOGRAPHICAL ULCER

Dendritic ulcer
 Finger like projections
 Feathery margins
 Satellite lesions may be seen
 Hypopyon

6. Drug contraindicated in keratitis

a) Tear drops
b) Steroids
c)Cycloplegics
d) Timolol

Drug contraindicated in corneal ulcer is steroids because it causes


perforations.
TREATMENT IN CORNEAL ULCERS
Topical 4th generation fluoroquinolones
Bacterial (DOC)
-Moxifloxacin
-Gatifloxacin
Fungal Topical Natamycin (DOC)
Viral Topical Acyclovir / Famciclovir
/Valacyclovir
PHMB (Poly Hexa Methylene
Acanthamoeba Biguanide) (DOC)
-Popamidine
-Chlorhexidine

7. Which of the following is seen retinitis pigmentosa?

a) Arteriolar attenuation
b) Neovascularization
c)Retinal artery thrombosis
d) Papilledema

The classic clinical triad of RP is arteriolar attenuation, retinal pigmentary changes


(could be either hypopigmentation and/or hyperpigmentation in form of bone-spicule
and pigment clumpings), and waxy disc pallor.

8. Not used for color vision testing:

a) Holmgren wool test


b) Ishihara chart
c)FM 100 Hue test
d) Ames test

Ames test is not used for color vision testing

Holmgren wool test


 It is a very ancient test used for color vision testing
 Not used anymore

Ishihara chart
 Most commonly used for color vision testing

FM 100 hue test


 Used for color vision testing

9. Where is the lesion that produces the visual field defect shown in the image
?
a) Craniopharyngioma
b) Meningioma
c)Pituitary adenoma
d) Temporal lobe lesion

SUPERIOR QUADRANTANOPIA / PIE IN THE SKY


 Inferior fibres are damaged
 Inferior fibres pass through temporal lobe
 ALL PIE IN THE SKY ARE SEEN IN OPPOSITE TEMPORAL LOBE LESIONS

LESIONS

10. Which of the following field defects occurs due to a lesion at the optic
chiasma?

a) Bitemporal hemianopia
b) Unilateral blindness
c) Pie on the floor
d) Bilateral homonymous macular defect

BITEMPORAL HEMIANOPIA
 Nasal fibres gets damaged
 Caused by optic chiasmal lesion → pituitary adenoma (mc)

LESIONS

Disc/optic nerve lesions Same sided monocular blindness

Chiasmal lesions Bitemporal Hemianopia

Optic tract lesions (mc) Incongruous Homonymous hemianopia

Lt. Geniculate body lesions Opposite sided Homonymous


hemianopia
Meyer's loop (Temporal lobe) Pie in the sky

Baum's loop (parietal lobe Pie on the Floor

Occipital lobe lesion Congruous Homonymous hemianopia


with macular sparing

11. In which of the following condition there is ipsilateral 3rd nerve palsy with
contralateral hemiplegia and facial palsy of upper motor neuron?

a) Weber syndrome
b) Terson syndrome
c) Millard-Gubler syndrome
d) Foville's syndrome

Weber's syndrome
 Ipsilateral 3rd nerve Palsy
 Contralateral Hemiplegia
 Facial palsy of upper motor neuron

12. All among the following are incorrect statements regarding lens induced
glaucoma except :
a) Phacomorphic glaucoma is an open angle glaucoma
b) In phacolytic glaucoma lens capsule is intact.
c) Phacomorphic glaucoma has a deep anterior chamber
d) Mature cataract is an example of phacolytic glaucoma

In phacolytic glaucoma lens capsule is intact. There are micropores through


which lens proteins leak.

Phacolytic glaucoma is caused by an inflammatory process caused by the leakage


of lens material through the capsule of a mature cataract. The released lens material
is composed of altered lens protein, macrophages, and other inflammatory cells that
lead to trabecular meshwork obstruction and precipitate glaucoma.

13. A 28-year-old man reported within 6 weeks after sustaining penetrating


accidental injury to his left eye with wooden stick On examination there is
marked diminution of vision in the RE associated with pain, redness and
photophobia for 1?week. The attending doctor diagnosed it as case of
sympathetic ophthalmitis and the first sign he can see on a fundus
examination is ?

a) Circumcorneal congestion
b) Hypopyon
c) Retrolental flare
d) Iris nodule

First sign in sympathetic ophthalmitis : retrolental flare

Sympathetic ophthalmitis ( Pan uveitis):


 B/L granulomatous uveitis
 Caused by trauma to 1 eye
 Injured eye → Exciting eye
 other eye→ Sympathizing eye
 Accommodation failure in good eye
 Earliest sign: Retrolental flare [ vitreous flare ]
 Mutton fat KP'S
DERMA JUNE 2019

1. A 25-year-old female presents with history of fever and oral ulcers and has
developed erythematous lesions on her face. Comment on the diagnosis.

a) SLE
b) Dermatomyositis
c)Melasma
d) Rosacea

Erythematous lesions given in this image is a feature of SLE - Butterfly rashes


 Fixed erythema
 Naso labial folds are spared

Systemic Manifestation
 Fever
 Joint pain
 Oral ulcer
 Photosensitivity

Melasma
 Brownish
 Malar area and Nose

Rosacea
 Erythematous lesion + Telangiectasia
 Convex areas
Dermatomyositis → Heliotrope rash → Violaceous → eye
2. The following lesion was noticed in a patient with history of involuntary
weight loss. What is the diagnosis?

a) Acanthosis nigricans
b) Leser-Trélat sign
c)Actinic keratosis
d) Intertriginous Candida

Acanthosis nigricans is a skin condition that causes a dark discoloration in body


folds and creases. It typically affects the armpits, groin and neck. Acanthosis
nigricans is a skin condition characterized by areas of dark, velvety discoloration in
body folds and creases. The affected skin can become thickened
Causes
Benign → obesity (mc), Endocrine disorders
Malignant → Gastric adenocarcinoma

3. Which is not correct about the lesion shown?


a) The lesions are infectious to seronegative individuals
b) Can be associated with meningoencephalitis
c) Bilaterally symmetrical dermatomal vesicular eruption
d) Geniculate ganglion is involved in Ramsay Hunt syndrome

Fluid filled lesions grouped together along particular dermatome and


unilateral which is seen in Herpes Zoster

Herpes zoster
Caused by Varicella zoster virus
Meningoencephalitis can be due to varicella zoster virus
In Ramsay hunt syndrome, painful grouped fluid filled lesions →vesicles →Herpes
zoster oticus.

4. The shown instrument is:

a) Single cutting razor


b) Double cutting razor
c) Humby's knife
d) Finochietto knife

Humby's knife → In 3rd degree burn → used for skin grafting


Uses/ Indication:
1 . Burn patients
2. Vitiligo surgery

5. Most appropriate Management to reduce recurrence of keloid:

a) Intralesional steroids
b) Surgery
c)Cryotherapy
d) Electrocoagulation
SURGERY:
Treatment of keloid:
 Silicon dressing
 Mechanical compression
 Intralesional steroids [Doc- keloids]

6. An infant presents infection. with the condition as shown below after a


pharyngeal The shown condition is:

a) Staphylococcal scalded skin syndrome


b) Epidermolysis bullosa simplex
c)Impetigo contagiosa
d) Pemphigus foliaceus

Staphylococcal scalded skin syndrome (SSSS), also known as Ritter von


Ritterschein disease (in newborns), Ritter disease, and staphylococcal epidermal
necrolysis, encompasses a spectrum of superficial blistering skin disorders caused
by the exfoliative toxins of some strains of Staphylococcus aureus.

7. The shown penile lesion is:

a) Genital wart
b) Pearly penile papule
c) Balanitis
d) Sebaceous gland prominence
Pearly papules :

 Asymptomatic
 Seen in uncircumcised patients
 Etiology unknown
 Not infective
 Normal lesions
 No treatment required

8. The show lesions are ?

a) Molluscum contagiosum
b) Chicken pox
c)Pemphigus
d) Common warts

Molluscum contagiosum is a skin infection caused by the virus Molluscum


contagiosum. It produces benign raised bumps, or lesions, on the upper layers of
your skin. The small bumps are usually painless. They disappear on their own and
rarely leave scars when they're left untreated.
RADIO JUNE 2019

1. Comment on the diagnosis of CT chest shown below.

a) Aortic dissection
b) Pulmonary embolism
c)Cardiac myxoma
d) Aortic aneurysm

NORMAL CT

Aortic dissection
Aortic dissection:

 M/C site of aortic dissection is Ascending aorta


 Intimal flap with double lumen is seen
 IOC for acute aortic dissection
 CT angiogram (stable pt.)
 TEE (unstable pt.)
 In Chronic aortic dissection: IOC: MRI

2. A 55 year old male patient presented with complaints of dyspnea with


congestive heart failure . The clinician wants to know that whether its HF
with preserved EF or reduced EF. Which of the below investigation is used
for calculation of ejection fraction:

a) MUGA
b) SPECT using thallium 201
c)PET myocardial perfusion imaging
d) Sestamibi scan with pharmacological stress

Nuclear Imaging of heart:


 MUGA scan (Multi-gated acquisition scan):
 Tc 99m labeled RBC IS used
 Evaluate ventricular function / ejection fraction- Cardiac ventriculography (but
most accurate investigation for ventricular function is -MRI)
 Tc99m labeled RBC is very sensitive investigation for detecting minimal lower
GIT bleeding (as low 0.1 ml per minute)

3. Refer to the image given below and give the correct diagnosis?
a) Diaphragmatic hernia
b) Intestinal obstruction
c)Pleural effusion
d) Pneumonia

Diaphragmatic hernia is a birth defect where there is a hole in the diaphragm (the
large muscle that separates the chest from the abdomen). Organs in the abdomen
(such as intestines, stomach, and liver) can move through the hole in the diaphragm
and upwards into a baby's chest.

4. Maximum radiation exposure :

a) X-ray abdomen
b) Chest X ray
c)IV pyelography
d) Barium Enema

Barium Enema- Sequential/ multiple imaging, covers larger area. therefore


comparatively maximum radiation exposure.

X-ray abdomen- Single spot film, therefore comparatively less radiation exposure.
Chest X ray- Single spot film, therefore comparatively less radiation exposure.
IV pyelography- Sequential/ multiple imaging but covers less area, therefore.
comparatively less radiation
exposure.
PET CT - Has highest radiation exposure.

5. In the shown lateral view, the structure marked with arrow head is?

a) Esophagus
b) Pulmonary artery
c) Trachea
d) Left atrium
6. A patient presented with complaints with dyspnea. The shown X- ray is
suggestive of:-

a) Consolidation
b) Exudative Pleural Effusion
c)Pnemothorax
d) Hydropnemothorax

Chest X-ray with pleural effusion


 Obliteration of CP angle
 Distinct border
 Concave upward curve called

ELLIS CURVE
 150-200 ml on PA view
 5 ml lateral view
 15-20 ml on lateral decubitus view
 Damoiseau-Ellis line is a clinical sign, that refers to upper limit of dullness
to percussion caused by pleural effusion. It has characteristic shape of
curved line with highest point at middle axillary line

ENT JUNE 2019

1. Narrowest part of airway in infant:

a) Supraglottis
b) Glottis
c) Subglottis
d) Epiglottis

Adult VS pediatric airway


In adult larynx is cylindrical and the narrowest part is glottic (vocal cords).
In case of paediatric larynx is funnel shape. And the narrowest part is subglottic.

2. A 15 year old male patient presented with nasal mass reaching upto cheek
and causing unilateral obstruction with intermittent epistaxis. Most likely
diagnosis is:

a) Angiofibroma
b) Nasal polyp
c)Nasopharyngeal Carcinoma
d) Inverted Papilloma

Angiofibroma is a benign (noncancerous) nasal cavity tumor that almost


exclusively affects adolescent boys. It also may be referred to as juvenile
nasopharyngeal angiofibroma (JNA).
Presentation
M/C- Profuse recurrent epistaxis
Nasal obstruction, check swelling.
O/E →Reddish polypoid mass
Diagnosis - CECT-Anterior bowing of posterior maxillary wall- ANTRAL/HOLMANN
MILLER SIGN
→Biopsy is Highly contraindicated
Rx → Endoscopic excision

3. HIGH TRACHEOSTOMY is done in which of the following?


a) Laryngeal papilloma
b) Carcinoma larynx
c)Bilateral vocal folds palsy
d) Laryngomalacia

Tracheostomy → Emergency/ Planned in hospital (between- 2nd and 3rd tracheal


ring)
High tracheostomy is done in carcinoma larynx (Between 1st and 2nd tracheal Ring)

4. ALL ARE PARTS OF HYPOPHARYNX EXCEPT

a) Posterior pharyngeal wall


b) Pyriform Sinus
c)Post cricoid area
d) Vallecula

 Hypopharynx is also know laryngopharynx.


 Component
1. Posterior pharyngeal wall
2.Post cricoid area
3. Pyriform fossa
 Vallecula is part of oropharynx.it is space between epiglottis and Posterior 1/3rd
of tongue

5. A 3 yrs old child presented with profound Sensorineural deafness, not


responding to hearing aids, with intact auditory nerve function. Which of
the following procedure is preferred in this patient?

a) Hearing Aids
b) BAHA
c) Cochlear implant
d) Auditory Brainstem Implant

INDICATIONS FOR COCHLEAR IMPLANTS

 Bilateral severe to profound H.L. that is not adequately treated with standard
hearing aids.
 Congenital H.L./ prelingual deafness
 Acquired H.L./ post lingual deafness
 Severe hearing loss that can be aided and that deteriorates to profound loss in
childhood, adolescence, or adulthood (prelingual).

6. A patient presented with 2 days history of fever and difficulty in swallowing.


On examination there was a swelling seen in oropharynx pushing tonsil to
midline and swelling over upper part of neck. What is the most likely
diagnosis:
a) Quinsy
b) Parapharyngeal abscess
c)Retropharyngeal abscess
d) Tonsillitis

In case of parapharyngeal space abscess this abscess grow and form a


swelling on lateral wall of oropharynx and it will also push the tonsil to
midline.
Parapharyngeal space- lateral boundaries-
1 . Parotid gland (Deep lobe)
2. Ramus of mandible
3. Median pterygoid muscle
In case of parapharyngeal space abscess this abscess grow and form a
swelling on lateral wall of oropharynx and it will also push the tonsil to
midline.

 When question says there is swelling in oropharynx pushing tonsil to midline it


doesn't means quinsy or tonsillitis.
 Retropharyngeal abscess will be seen in midline

7. Deep lobe of parotid gland swelling presents at

a) Below the ear lobe


b) Cheek
c)Temporal lobe
d) Parapharyngeal Space

Parapharyngeal / lateral pharyngeal space Boundaries


 Superiorly - skull bone
 Lateral - deep lobe of parotid gland ,Ramus of mandible, Median pterygoid
muscle
 Medial - oropharynx with superior constrictor muscle, Palatine tonsil
 Inferior- hyoid bone.

8. Which of the following muscles is the lifesaving muscle of larynx that pulls
back vocal cords in order for breathing to take place

a) Cricothyroid
b) Posterior Cricoarytenoid
c)Lateral Cricoarytenoid
d) Transverse Arytenoid

Posterior cricoarytenoid muscle is regarded as Safety muscle of Larynx.


Posterior cricoarytenoid muscle of larynx is the only muscle which abducts true
vocal cords and opens the rima glottidis. This muscle is supplied by recurrent
laryngeal branch of vagus nerve. Injury to this nerve results in hoarsness of voice.
9. Rodent ulcers are which type of malignancy

a) Squamous cell carcinoma


b) Basal cell carcinoma
c)Malignant Melanoma
d) Adenocarcinoma

Rodent ulcers (basel cell carcinoma)- most common skin malignancy


Seen in - sun: exposed area (hand and face )(UV-B)
This is locally invasive and metastasis is rare.
Dx →wide local excision

10. What is the most common cause of deafness in a patient of


Nasopharyngeal Carcinoma:

a) Metastasis to Temporal Bone


b) Middle ear Infection
c) Serous otitis media
d) Infiltration of Tumour middle ear

Nasopharyngeal carcinoma
Due to obstruction of eustachian tube there is serous otitis media and leads to
hearing loss.
M/C site of origin → Fossa of Rosenmuller
M/C type → Squamous cell carcinoma
M/C Presentation →cervical lymphadenopathy (Neck Mass)

11. Most Common Presenting Complaint of Nasopharyngeal Carcinoma

a) Cervical lymphadenopathy
b) Hearing Loss
c) Airway Obstruction
d) Epistaxis

Nasopharyngeal carcinoma
Due to obstruction of eustachian tube there is serous otitis media and leads to
hearing loss.
M/C site of origin → Fossa of Rosenmuller
M/C type → Squamous cell carcinoma
M/C Presentation →cervical lymphadenopathy (Neck Mass)

12. Structures not seen in indirect laryngoscopy?

a) Anterior Commissure
b) Posterior Commissure
c) Epiglottis
d) False Vocal Cord
Anterior commissure can't be seen because of bulge of anterior wall of larynx
which hides the anterior commissure

13. Killian's Dehiscence is seen at the level of:

a) Superior Constrictor
b) Middle constrictor
c) Inferior Constrictor
d) Thyroepiglottis

The Killian dehiscence is a triangular-shaped area of weakness in the muscular wall


of the pharynx, between the transverse and oblique bundles of the inferior
pharyngeal constrictor. It is the most common site of pharyngeal diverticula.
 Killian dehiscence also known as "Gateway of tears"
 M/C site for esophageal tear (perforation)

14. Earliest symptoms of vocal fold growth is?

a) Cough
b) Change of voice
c) Fatigue voice
d) Pain

Vocal fold growth earliest symptom is change of voice or hoarseness.

15. Parotid gland is supplied by which nerve?

a) Facial Nerve
b) Glossopharyngeal
c) Vagus Nerve
d) Hypoglossal Nerve

Nerve supply to parotid gland- Parasympathetic innervation to the parotid comes


from the glossopharyngeal nerve (CNIX)and causes secretion of saliva.
Parasympathetic innervation to the submandibular glands is provided by the
superior salivatory nucleus via the chorda tympani, a branch of the facial nerve, that
becomes part of the trigeminal nerve's lingual
nerve prior to synapsing on the submandibular ganglion.
FMGE DEC 2019

ANATOMY DEC 2019

1. Which of the following is the safety muscle of tongue?

a) Genioglossus
b) Hyoglossus
c) Palatoglossus
d) Styloglossus

The genioglossus arises from the mandible and protrudes the tongue. It is also
known as the tongue's "safety muscle" since it is the only muscle that propels the
tongue forward. The hyoglossus, arises from the hyoid bone and retracts and
depresses the tongue.

2. Trigone of bladder is derived from;

a) Distal part of Paramesonephric duct


b) Distal part of Mesonephric duct
c) Ventral part of Mesonephric duct
d) Ventral part of Urogenital sinus

Embryologically, the trigone of the bladder is derived from the caudal end of
mesonephric ducts, which is of mesodermal origin (the rest of the bladder is
endodermal).

3. Identify the marked structure;

a) Anterior superior illac spine


b) Anterior inferior iliac spine
c) Iscial tuberosity
d) Pubic symphysis
The anterior superior iliac spine is an important bony surface landmark and is the
prominence is the most anterior part of the ilium. It can be palpated at the
lateral end of the inguinal fold. ... It is also the start point for the measurement of
true leg length (anterior superior iliac spine to medial malleolus).

4. Which muscle tendon is stretched in patellar tendon reflex?

a) Biceps femoris
b) Quadriceps femoris
c) Addactur magnus
d) Quadratus

The knee-jerk reflex, also known as the patellar reflex, is a simple reflex that
causes the contraction of the quadriceps muscle when the patellar tendon is
stretched.
5. Celiac artery is seen at which vertebral level?

a) T9
b) T10
c) T12
d) L2

The celiac trunk, also known as the celiac artery, is a short vessel that arises
from the aorta and passes below the median arcuate ligament, just as the aorta
enters the abdomen at the level of the T12 vertebra.

6. Highest point of iliac crest is seen at;

a) L3
b) L4
c) S1
d) S2

The top of the iliac crests also marks the level of the fourth lumbar vertebral
body (L4), above or below which lumbar puncture may be performed.

7. 45-year-old policeman comes from duty with complain of pain in right leg.
On examination, pulse in right popliteal fossa was absent and pain in right
buttock, whereas in left side it was normal. The block is at which level?

a) Femoral
b) Iliac
c) Aortoiliac
d) Popliteal

Aortoiliac occlusive disease is a type of vein condition in the legs. It occurs when
the iliac artery that brings blood to your legs becomes narrow or blocked by plaque.
Symptoms can include pain, numbness, or cramping in the lower limbs, gangrene in
the feet, and erectile dysfunction (ED) in men.

8. Lacrimal gland is supplied through parasympathetic system. It is supplied


by which ganglion?

a) Ciliary ganglia
b) Otic ganglia
c) Pterygopalatine
d) Submandibular
The postganglionic axons are the output of the pterygopalatine ganglion project to
the lacrimal gland and supply the parasympathetic innervation of the lacrimal gland

9. The structure which doesn’t pass through the parotid gland?

a) Facial nerve
b) ICA
c) Branch of ECA
d) Retromandibular vein

The facial nerve and its branches pass through the parotid gland, as does the
external carotid artery and retromandibular vein. The external carotid artery
forms its two terminal branches within the parotid gland: maxillary and superficial
temporal artery

10. Protraction of neck is due to which of the following muscle?

a) Serratus anterior
b) Pectoralis major
c) Pectoralis minor
d) Lateral pterygoid

11. Middle meningeal artery is branch of;

a) Maxillary artery
b) Mandibular
c) Facial artery
d) Ophthalmic artery

Middle meningeal artery primarily is derived as a branch from the maxillary


artery and then passes through the foramen spinosum into the middle cranial fossa

Five branches, each of which enters a bony foramen:


 deep auricular artery (enters squamotympanic fissure)
 anterior tympanic artery (enters squamotympanic fissure)
 middle meningeal artery (enters foramen spinosum)
 accessory meningeal artery (enters foramen ovale)
 inferior alveolar artery (enters mandibular foramen)
12. The muscle which is required to open Eustachian tube;

a) Tensor veli palatini muscle


b) Salpingo pharengeous
c) Tensor tympani
d) Levator veli palatini

The eustachian tube opens upon swallowing or yawning by contraction of the tensor
veli palatini muscle.

13. The highlighted muscle is supplied by which nerve?

a) Radial nerve
b) Ulnar nerve
c) Median nerve
d) Musculocutaneous nerve
The musculocutaneous nerve innervates the three muscles of the anterior
compartment of the arm: the coracobrachialis, biceps brachii, and brachialis. It is
also responsible for cutaneous innervation of the lateral forearm.

14. Identify the encircled structure;

a) Pterion
b) Glabella
c) Asterion
d) Nasion

The pterion is the H-shaped formation of sutures on the side of the calvarium
representing the junction of four skull bones: the greater wing of the sphenoid bone,
squamous portion of the temporal bone, frontal bone, parietal bone.

15. What is the name of marked artery?


a) Posterior Tibial Artery
b) Deep Phalangeal artery
c) Lateral planter
d) Medial planter

The posterior tibial artery (PTA) is one of the 2 branches of the tibioperoneal
trunk in the lower leg and provides oxygenated blood to the posterior
compartments of the leg and plantar surface of the foot. It is accompanied by the
posterior tibial vein, along its course.

PATHO DEC 2019

16. Most common tumour of mediastinum?

a) Thymoma
b) Lymphoma
c) Neurogenic tumor
d) Neuroblastic tumor

The most common mediastinal masses are neurogenic tumors (20% of mediastinal
tumors), usually found in the posterior mediastinum, followed by thymoma (15–20%)
located in the anterior mediastinum. Lung cancer typically spreads to the lymph
nodes in the mediastinum.

17. Most common type of Embryonic neural tumor;

a) Fibrous astrocytoma
b) Neuroglioma
c) Ependymoma
d) Medulloblastoma
Medulloblastomas. The most common type of embryonal tumor, these fast-growing
cancerous brain tumors start in the lower back part of the brain, called the
cerebellum. The cerebellum is involved in muscle coordination, balance and
movement.

18. Which of the following is not a component of SIRS (Systemic Inflammatory


Response Syndrome)?

a) Fever >38 degrees or hypothermia < 36 degree,


b) Tachycardia >90 beats /min
c) Tachypnea>25 breaths / min
d) leucocytosis >12*109/l or leucopoenia <4*109/l

Systemic Inflammatory Response Syndrome (SIRS) is the occurrence of at least two


of the following criteria:
 fever >38.0°C or hypothermia <36.0°C,
 tachycardia >90 beats/minute,
 tachypnea >20 breaths/minute,
 leucocytosis >12*109/l or leucopoenia <4*109/l

19. Which of the following has Autosomal Recessive inheritance?

a) Hereditary spherocytosis
b) Sickle cell anaemia
c) Osteogenesis imperfecta
d) VWD

Autosomal Recessive inheritance Examples:


 Albinism (oculocutaneous)
 Ataxia telangiectasia
 Crigler-Najjar syndrome type
 Congenital adrenal hyperplasia
 Cystic fibrosis
 Deafness (some forms)
 Dubin-Johnson syndrome
 Epidermolysis bullosa (some forms)
 Fanconi syndrome
 Friedreich's ataxia
 Galactosaemia
 Gaucher's disease
 Glycogen storage disease
 Haemochromatosis
 Homocystinuria
 Hurler's syndrome (mucopolysaccharidosis l)
 Infantile polycystic kidney disease
 Laurence-Moon-Biedl syndrome
 Phenylketonuria
 Sickle cell disease
 Tay-Sachs disease
 B-Thalassemia
 Wilson's disease

20. Which of the following is an Autosomal Dominant disease?

a) Von Willebrand disease


b) Wilson’s disease
c) Cystic fibrosis
d) Friedrichs ataxia

Mnemonic for Autosomal Dominant disorders: Very Powerful DOMINANT


Humans
 Von willebrand disease / Von hippel-lindau
 Pseudo-hypoparathyroidism
 Dystrophia myotonica
 Osteogenesis imperfecta / Osler-weber-rendu
 Marfan syndrome
 Intermittent porphyria
 Neurofibromatosis
 Achondroplasia / Adult polycystic kidney disease
 Noonan syndrome
 Tuberous sclerosis
 Hypercholesterolemia
 Huntington's disease
 Hypertrophic obstructive cardiomyopathy
 Hereditary spherocytosis
 Hereditary non polyposis coli
 Hereditary hemorrhagic telangiectasia

21. A patient has MCV < 80, MCH < 23. Which type of anemia shall be
classified?

a) Normocytic hypochromic
b) Microcytic hypochromic
c) Hyperchromic macrocytic
d) Normocytic normochromic
 Erythrocytes that have a normal size or volume (normal MCV) are called
normocytic,
 When the MCV is high, they are called macrocytic.
 When the MCV is low, they are termed microcytic.
 Erythrocytes containing the normal amount of hemoglobin (normal MCHC) are
called normochromic.
 When the MCHC is abnormally low they are called hypochromic, and when
the MCHC abnormally high, hyperchromic.

MCV MCHC
Female: 80-95 fl 30-34
gHb/100ml
Male: 80-95 fl 30-34
gHb/100ml

22. Which among the following ions is important in cell injury?

a) Calcium
b) Sodium
c) Potassium
d) Chloride

Calcium ions have been proposed as mediators of the functional consequences of


plasma membrane injury. An influx of extracellular calcium ions across a damaged
permeability barrier and down a steep concentration gradient may convert potentially
reversible injury into irreversible injury.

23. Which type of anaemia is seen in vitamin B12 deficiency?

a) Microcytic, hypochromic
b) Macrocytic, Hypochromic
c) Normocytic Normochromic
d) Macrocytic, Normochromic

Macrocytic - Normochromic Anemias


 Pernicious anemia
 Caused by a lack of intrinsic factor from the gastric parietal cells
 Required for vitamin B12 absorption
 Results in vitamin B12 deficiency
24. Fertilized egg cells are;

a) Totipotent
b) Pluripotent
c) Multipotent
d) None of above

Totipotent stem cells can differentiate into embryonic and extraembryonic cell types.
These cells are produced from the fusion of an egg and sperm cell, and can
construct a complete, viable organism. Cells produced by the first few divisions of
the fertilized egg are also totipotent.

25. 16-year-old girl with short height, amenorrhea with widely spaced nipple.
karyotyping is;

a) 45 X0
b) 46 XO
c) 47 XXX
d) 46XY

Turner syndrome (TS), also known as 45X , or 45XO , is a genetic condition in


which a female is partly or completely missing an X chromosome. Turner syndrome,
a condition that affects only females, can cause a variety of medical and
developmental problems, including short height, failure of the ovaries to develop and
heart defects.

Signs of Turner syndrome at birth or during infancy may include:


 Wide or web like neck
 Low-set ears
 Broad chest with widely spaced nipples
 High, narrow roof of the mouth (palate)
 Arms that turn outward at the elbows
 Fingernails and toenails that are narrow and turned upward
 Swelling of the hands and feet, especially at birth
 Slightly smaller than average height at birth
 Slowed growth
 Cardiac defects
 Low hairline at the back of the head
 Receding or small lower jaw
 Short fingers and toes
26. Which type of necrosis is seen in brain?

a) Coagulative
b) Liquefactive
c) Fat
d) Fibrinoid

Type of Necrosis Site


Coagulative Necrosis Heart, spleen & kidney
Liquefactive (or) Colliquative Brain, abscess cavity
necrosis
Fat necrosis Pancreas, breast
Fibrinoid necrosis Arterioles in hypertension, peptic
ulcer, etc.
Caseous necrosis In the centre of foci of tuberculus
infection

27. Following heart sound is almost always pathological;

a) Loud S1
b) Loud S2
c) Systolic murmur
d) S4

A S4 heart sound is almost always pathologic. Heart sounds can be described by


their intensity, pitch, location, quality and timing in the cardiac cycle. Intensity: Heart
sounds can be described as increased in intensity (loud), decreased in intensity
(soft) or absent.

SPM DEC 2019

28. Koplik spots are seen in;

a) Mumps
b) Measles
c) Rubella
d) Varicella

Koplik spots are seen with measles. They are small, white spots (often on a
reddened background) that occur on the inside of the cheeks early in the course of
measles.
29. NITI AAYOG Represents;

a) Nutritional institute for transforming India


b) National Institute for Transforming India
c) Newer Initiative transforming India
d) Nutrition Intake to India

National Institution for Transforming India, better known as NITI Aayog, was formed
via a resolution of the Union Cabinet on 1 January 2015. NITI Aayog is the premier
policy think tank of the Government of India, providing directional and policy inputs.

30. On republic day, a camp was organized and people were screened for
Hypertension by checking BP and for diabetes by checking their BMI and
Blood sugar level, which level of prevention is this?

a) Primary
b) Primordial
c) Secondary
d) Tertiary

There are 3 levels of disease prevention:


Primary Prevention - trying to prevent yourself from getting a disease.
Secondary Prevention - trying to detect a disease early and prevent it from
getting worse.
Tertiary Prevention - trying to improve your quality of life and reduce the symptoms
of a disease you already have.

31. A person comes with right eye 6/60 and left eye 3/60. He shall be
categorized into which type of blindness?

a) Manifest
b) Social
c) Economical
d) No blindness

Types of Blindness:

 Economic blindness: Inability of a person to count fingers from a distance of 6


meters or 20 feet .
 Social blindness: Vision 3/60 or diminution of field of vision to 10°
 Manifest blindness: Vision 1/60 to just perception of light
 Absolute blindness: No perception of light
 Curable blindness: That stage of blindness where the damage is reversible by
prompt management e.g. cataract
 Preventable blindness: The loss of blindness that could have been completely
prevented by institution of effective preventive or prophylactic measures e.g.
xerophthalmia, trachoma and glaucoma
 Avoidable blindness: The sum total of preventable or curable blindness is
often referred to as avoidable blindness.

32. Recycling, tubing and recyclable catheters will be disposed in which


bag according to biomedical waste management?

a) Red
b) Blue
c) Yellow
d) White

33. Main focus of UNICEF is on;

a) Child health
b) Nutritional health
c) Mental health
d) Social health

UNICEF is mandated by the United Nations General Assembly to advocate for the
protection of children's rights, to help meet their basic needs and to expand their
opportunities to reach their full potential.

34. Subacute Sclerosing Panencephalitis is a rare and dangerous complication


of;
a) Measles
b) Mumps
c) Rubella
d) Varicella

Subacute sclerosing panencephalitis (SSPE) is a very rare, but fatal disease of the
central nervous system that results from a measles virus infection acquired earlier
in life. SSPE generally develops 7 to 10 years after a person has measles, even
though the person seems to have fully recovered from the illness.

35. Spermicidal cream which is used in contraceptives;

a) Gossypol
b) Nonoxynol-9
c) Centchroman
d) Clomiphene

Nonoxynol-9 is the primary chemical in spermicides to inhibit sperm motility. Active


secondary spermicidal ingredients can include octoxynol-9, benzalkonium
chloride and menfegol. The most effective strength spermicide contains at least
100 mg of nonoxynol-9 per dose.

36. Kyoto Protocol is for;

a) Water sanitization
b) Food and nutrition
c) To reduce greenhouse gas emission
d) Water chlorination

Kyoto Protocol operationalizes the United Nations Framework Convention on


Climate Change by committing industrialized countries and economies in
transition to limit and reduce greenhouse gases (GHG) emissions in
accordance with agreed individual targets.

37. Sustainable Development Goals is Designed by;

a) UNICEF
b) United Nations
c) WHO
d) Government of India

The Sustainable Development Goals (SDGs) or Global Goals are a collection of 17


interlinked global goals designed to be a "blueprint to achieve a better and more
sustainable future for all". The SDGs were set up in 2015 by the United Nations
General Assembly and are intended to be achieved by the year 2030.

38. Which of the following is the chemoprophylaxis of choice in a person


who is on a journey to endemic malarial region?

a) Chloroquine
b) Quinine
c) Lumefantrine
d) Pyrimethamine

Atovaquone/proguanil (Malarone), doxycycline, and mefloquine are the drugs of


choice for malaria prevention in most malaria-endemic regions. Chloroquine
(Aralen) may be used safely in all trimesters of pregnancy, and mefloquine may be
used safely in the second and third trimesters of pregnancy.

39. Worms, bacteria and bugs attached in clothing of a person. This is;

a) Infection
b) Infestation
c) Contamination
d) Contagion

contamination is the act or process of contaminating; pollution; defilement; taint;


also, that which contaminate while infestation is the presence of a large number of
pest organisms in an area or field, on the surface of a host or anything that might
contact a host, or in the soil.

40. Target couple is;

a) Couple with 3 children


b) Couple using contraception
c) Currently married couple where the wife is in reproductive age (15-49 year)
d) Couple that is eligible for practicing family planning

Target couples :
The term target couples was applied to couples who have had 2-3 living children, or
even newly married couples to make them accept the idea of family planning from
the earliest possible stage.

41. Yellow fever vaccine is valid for;

a) Life long
b) 10 years
c) 5 years
d) 20 years
As of 11 July 2016, a certificate of vaccination against yellow fever (ICVP) is valid
for the lifetime of the person vaccinated. Prior to 2016, yellow fever vaccination
certificates were valid for a period of 10 years, commencing 10 days after the date of
vaccination.

42. In Africa, vaccination was given in an endemic outburst area for a


specific disease but due to shortage of vaccines, mass
chemoprophylaxis treatment was given to people who were not
diseased. Which type of prevention is this?

a) Specific protection
b) Health promotion
c) Early diagnosis and treatment
d) Rehabilitation

SPECIFIC PROTECTION:
Idea is to avoid disease altogether. Currently available interventions include:

 Immunization
 Use of specific nutrients
 Chemoprophylaxis
 Protection against occupational hazards
 Protection against accidents
 Protection from carcinogens
 Avoidance of allergens
 Better environmental conditions e.g. control of air and noise pollution
 Control of consumer product quality and safety of foods, drugs, cosmetics.

43. Patients are categorized on the basis of chances of survival in Disaster


management;

a) Triage
b) Tagging
c) Mitigation
d) Surge capacity

Triage is the process of quickly examining sick or injured people, for example
after an accident or a battle, so that those who are in the most serious condition can
be treated first.
44. Vision 2020 - which was added recently in India?

a) Cataract
b) Childhood blindness
c) Glaucoma
d) Refractive errors and low vision

Glaucoma is now regarded as a sort of “missing link” and VISION 2020 India
successfully advocated for the inclusion of glaucoma in the next phase by the Indian
government.

45. Diseases spread by the shown vector is all except;

a) Kala azar
b) Oroya Fever
c) Chagas disease
d) Cutaneous leishmania

Sandfly (Phlebotomus) Diseases Transmitted


 Leishmaniasis
 Kala Azar
 Oriental Sore (Cutaneous Leishmaniasis)
 Sandfly fever

46. SARS is caused by novel variant of;

a) Pox virus
b) Toga virus
c) Coronavirus
d) Lyssa virus

Corona viruses are a large family of viruses which may cause illness in animals or
humans. In humans, several coronaviruses are known to cause respiratory infections
ranging from the common cold to more severe diseases such as Middle East
Respiratory Syndrome (MERS) and Severe Acute Respiratory Syndrome (SARS)
.

47. N95 respirator is used for?

a) Airborne Precautions
b) Droplet Precautions
c) Contact Precautions
d) None of the Above

N95 respirators and surgical masks are examples of personal protective


equipment that are used to protect the wearer from Airborne particles or from
liquid contaminating the face.

48. Which of the following is not a measure of dispersion?

a) Mean
b) Range
c) Variance
d) Standard deviation

Measures of dispersion describe the spread of the data. They include the range,
interquartile range, standard deviation and variance. The range is given as the
smallest and largest observations.

49. Anthracosis is caused by;

a) Silica
b) Asbestos
c) Coal dust
d) Sugar cane waste

Anthracosis is defined in Bioline as, “the asymptomatic, milder type of


pneumoconiosis as caused by the accumulation of carbon in the lungs due to
repeated exposure to air pollution or inhalation of smoke or coal dust
particles”.

50. Breast feeding should be continued for minimum how many months?

a) 6 months
b) 2 months
c) 12 month
d) 5 months

The World Health Organization recommends that all babies be exclusively breastfed
for 6 months, then gradually introduced to appropriate foods after 6 months while
continuing to breastfeed for 2 years or beyond. Stopping breastfeeding is called
weaning.
51. Components of Kuppuswamy scale-

a) Housing, Livestock, land


b) Income, Occupation, Education
c) Income, Housing, Education
d) Income, Livestock, Housing

The modified Kuppuswamy scale is commonly used to measure SES in urban and
rural areas. This scale was devised by Kuppuswamy in 1976 and consists of
a composite score which includes the education and occupation of the Family
Head along with income per month of the family, which yields a score of 3–29.

52. Mode of transmission to bacteria through bacteriophage is by?

a) Transformation
b) Transduction
c) Translation
d) Conjugation

The transfer of genes from one bacterial strain to another by a bacteriophage is


called transduction and can occur in a generalized or specific manner.

53. Identify the parasite as shown in photo;

a) Fasciola hepatica
b) Schistosoma japonicum
c) Schistosoma mansoni
d) Schistosoma hematobium
54. A patient presented with following findings. what will be the media used to
diagnose this condition?

a) Loeffler’s serum slope


b) Lownstein Johnson
c) Karry blair
d) Chocolate agar

Loeffler’s serum slope:


The medium contains horse serum, beef extract, dextrose, and proteose peptones
which together supply the complex nitrogenous substances and nutrients necessary
to support the growth of Corynebacterium diphtheriae.
Diphtheria destroy healthy tissues in the respiratory system. The dead tissue
forms a thick, gray coating that can build up in the throat or nose. This thick gray
coating is called a "pseudomembrane"

55. Identify the parasite as shown in this photo;


a) Brugia Malayi
b) Wucheria bancrofti
c) Tinea solium
d) Tinea saginatum

Brugia malayi is a filarial (arthropod-borne) nematode (roundworm), one of the


three causative agents of lymphatic filariasis in humans. Lymphatic filariasis, also
known as elephantiasis, is a condition characterized by swelling of the lower limbs.
The two other filarial causes of lymphatic filariasis are Wuchereria
bancrofti and Brugia timori, which both differ from B.malayi morphologically,
symptomatically, and in geographical extent.

56. Most sensitive test for microfilariae;

a) DEC challenge test


b) Immunofluroscence test
c) Thick films
d) Membrane filter concentration technique

Membrane filtration and the Knott concentration method are more sensitive than
routine thick blood smears for microfilaria detection, because they can be used to
concentrate microfilariae from 1 mL or more of blood.

57. What is the causative agent for malignant otitis externa?

a) Pseudomonas
b) Staph aureus
c) Streptococcus
d) Influenza
Malignant external otitis (MEO) is an infection that affects the external auditory canal
and temporal bone. The causative organism is usually Pseudomonas aeruginosa,
and the disease commonly manifests in elderly patients with diabetes

58. Smallest cestode among the following;

a) H. nana
b) Schistosoma
c) T. saginata
d) T. Solium

Hymenolepis nana, the dwarf tapeworm, is the smallest and a common tapeworm
in humans worldwide. H. nana infection occurs more frequently in warm climates and
temperate zones such as Asia, Central and South America, and Eastern Europe.

59. Which antibody is seen in recent infection?

a) IgM
b) IgG
c) IgE
d) IgD

Immunoglobulin M (IgM) is one of several isotypes of antibody (also known as


immunoglobulin) that are produced by vertebrates. IgM is the largest antibody, and
it is the first antibody to appear in the response to initial exposure to an antigen.

60. A patient who was recently got dentature fixed, found with oral thrush. It
bleeds on scraping. Causative agent for this condition

a) Diphtheria
b) Candida
c) Strep Mutans
d) Staph Aureus
Oral thrush (mouth thrush)
 Also called: oral candidiasis
 An infection in which the fungus Candida albicans accumulates in the mouth.
 It's common in babies, people with immune deficiency and those who use
steroid sprays for asthma.
 Oral thrush causes white lesions on the tongue or inner cheeks.
 Unsweetened yoghurt or an anti-fungal medication can help.

61. Bacterial vaginosis is caused by all except-

a) Gardenerella
b) Mobilincus
c) Proteus
d) Coccobaccillus

Proteus mirabilis is a common pathogen responsible for complicated urinary tract


infections (UTIs) that sometimes causes bacteremia. Most cases of P. mirabilis
bacteremia originate from a UTI; however, the risk factors for bacteremia and
mortality rates from P. mirabilis UTI have not been determined. Proteus species
also produce urease, which has been shown to be associated with an increased
risk of pyelonephritis and upper UTIs. Proteus species also hydrolyze urea to
ammonia, thereby alkalinizing the urine. Through the production of urease and
ammonia, Proteus can produce an environment where it can survive
Whereas BV is linked to an imbalance of “good” and “harmful” bacteria that are
normally found in a woman's vagina. Having a new sex partner or multiple sex
partners, as well as douching, can upset the balance of bacteria in the vagina. This
places a woman at increased risk for getting BV.

62. Kala azar cannot occur at an altitude of;

a) 400 meters
b) 100 meters
c) 600 meters
d) 200 meters

A disease caused by infection with Leishmania parasites.


Visceral leishmaniasis is spread by Sandfly bites. This type of leishmaniasis affects
the internal organs, usually the spleen, liver and bone marrow.
Some people have no symptoms. For others, symptoms may include fever, weight
loss and swelling of the spleen or liver.
Medication exists to kill the parasites. If left untreated, severe cases are typically
fatal.
Kala-azar or Visceral leishmaniasis (VL) is a disease of low altitude
(approximately 500 meters mean sea level)
63. A bacterium which can sustain >100 degree comes is classified as;

a) Thermophilic
b) Mesophilic
c) Cycrophilic
d) Halophilic

A thermophile is an organism—a type of extremophile—that thrives at relatively


high temperatures, between 41 and 122 °C (106 and 252 °F). Many thermophiles are
archaea, though they can be bacteria. ... The enzymes in thermophiles function at
high temperatures.

64. The causative agent for duodenal ulcer is;

a) Shigella
b) E.Coli
c) H. Pylori
d) Lactobacilli

Helicobacter pylori infection is the most common cause of duodenal ulcer


disease, yet duodenal ulcer is an uncommon outcome of H. pylori infection

65. Identify the microorganism;

a) Influenza virus
b) Corona virus
c) Rheo virus
d) Rabies virus

Influenza is a viral infection that attacks your respiratory system — your nose, throat
and lungs. Influenza is commonly called the flu, but it's not the same as stomach "flu"
viruses that cause diarrhea and vomiting.
66. Which of the following is cardio protective?

a) HDL
b) LDL
c) VLDL
d) CHYLOMICRON

High-density lipoprotein cholesterol (HDL-C) has been identified as a powerful


independent negative predictor of cardiovascular disease.

67. Gluconeogenesis is inhibited by?

a) Glucagon
b) Insulin
c) Cholecystokinin
d) 5 alpha hydroxylase synthase

Insulin is a key hormone that inhibits gluconeogenesis, and insulin resistance is a


hallmark of type 2 diabete

68. Which structure of Protein is not denatured after heating upto 100 degree?

a) Primary
b) Secondary
c) Tertiary
d) Quaternary

Primary structure, such as the sequence of amino acids held together by covalent
peptide bonds, is not disrupted by denaturation.

69. Which of the following is present in skeletal muscle?


a) GLUT 4
b) GLUT 5
c) GLUT 7
d) GLUT 2

GLUT4 expression is highest in adipose tissue and skeletal muscle, but GLUT4 is
also found in other organs such as brain, kidney and intestine

70. Synovial fluid contains–

a) Hyaluronic acid
b) Chondroitin sulphate
c) Keratan sulphate
d) Dermatan sulphate

Synovial fluid is produced by the synovium and is composed of water, inorganic


salts and macromolecules, hyaluronic acid, lubricin and aggrecans, which
contribute to the boundary lubrication.

71. Melanin is derived from which amino acid?

a) Tyrosine
b) Tryptophan
c) Phenylalanin
d) Alanine

Melanin is a highly irregular heteropolymer consisting of monomeric units derived


from the enzymatic oxidation of the amino acid tyrosine. The process of melanin
formation takes place in specialized acidic organelles (melanosomes) in
melanocytes.

72. The bases which are present in human DNA?

a) adenine-guanine-thiamine- uracil
b) adenine-guanine- cytosine-uracil
c) adenine- guanine-cytosine-thiamine
d) None of the above
The bases in DNA are adenine (A), thymine (T), guanine (G) and cytosine (C).
RNA consists of four nitrogenous bases: adenine, cytosine, uracil, and guanine.

73. Keshan’s disease is due to deficiency of


a) Selenium
b) Zinc
c) Copper
d) Iron

Keshan disease (KD) is a dilated cardiomyopathy closely related with a diet


deficient in the mineral selenium. It is named for the northeastern Chinese county
Keshan, where the disease prevalence is high because of selenium-deficient soil.

74. Type IV complex of ETC is inhibited by;

a) Oligomycin
b) Antimycin
c) Cyanide
d) CO2

Cyanide binds to Fe3+ in heme-containing proteins. This inhibits the terminal


cytochrome complex IV of the electron transport chain. The blocklock of complex IV
by cyanide depletes ATP culminating in cell death.

75. Zinc is cofactor of which enzyme?

a) Carbonic anhydrase
b) Lysyl oxidase
c) Carboxylase
d) Kinase

Carbonic anhydrase is a metalloenzyme that requires Zinc as a cofactor and is


involved in diverse biological processes including pH regulation, CO2 transfer, ionic
exchange, respiration, CO2 photosynthetic fixation, and stomatal closure.

76. Allopurinol inhibits which enzyme?

a) Xanthine oxidase
b) Kinase
c) Lysyl oxidase
d) Carboxylase

Allopurinol was effective in inhibiting xanthine oxidase activity in vivo as measured


by the dramatic reduction of uric acid production.

77. 1g of protein yields how much energy?

a) 1 Kcal
b) 4 Kcal
c) 8 Kcal
d) 2 Kcal
1 gram of protein (1 gram times 4 calories = 4 calories from protein)

78. Ammonia formed in brain, is converted into;

a) Glutamine
b) Glycine
c) Cyteine
d) Urea

Much of the waste nitrogen produced in tissues such as skeletal muscle and brain is
packaged as glutamine. Glutamine synthase catalyzes the conversion of ammonia
and glutamate to glutamine

79. Most sensitive screening test for vitamin A deficiency;

a) Retinol < 10
b) Beta carotene < 10
c) Bitot spot
d) Night blindness

According to the World Health Organization (WHO). One of the first signs of
vitamin A deficiency is night blindness, Young children and pregnant women are
most commonly affected. Vitamin A testing is used to help diagnose a vitamin A
deficiency in people with signs and symptoms, such as night blindness, or in people
who are at risk of vitamin A deficiency.

80. Which enzyme level is tested in thiamine deficiency?

a) PDH
b) Pyruvate kinase
c) Transketolase
d) Kinase

In conjunction with whole blood or erythrocyte transketolase activity preloading and


postloading, a thiamine loading test is the best indicator of thiamine deficiency. An
increase of more than 15% in enzyme activity is a definitive marker of deficiency.

81. Most abundant amino acids found in collagen;

a) Glycine
b) Cysteine
c) Valine
d) Lysine
The triple-helical structure of collagen arises from an unusual abundance of three
amino acids: glycine, proline, and hydroxyproline. These amino acids make up the
characteristic repeating motif Gly-Pro-X, where X can be any amino acid.

82. Which is a specific test for uric acid for gout;

a) Synovial uric acid


b) Plasma uric acid
c) Uric acid in urine
d) Serum uric acid

Joint fluid test (Synovial fluid, also known as joint fluid, is a thick liquid located
between your joints). Your doctor may use a needle to draw fluid from your
affected joint. Urate crystals may be visible when the fluid is examined under a
microscope.

83. HCG is secreted by?

a) Syncytiotrophoblast
b) Yolk sac
c) Cytotrophoblast
d) Placenta

hCG is secreted by the syncytiotrophoblast which originates from fused and


differentiated cytotrophoblast cells

84. Lady presents with infertility and diagnosed with bilateral cornual block
on hysterography. What is the next step?

a) IVF
b) Tuboplasty
c) Laproscopy and hysteroscopy
d) USG

85. Amsel criteria is used in the diagnosis of ?

a) Bacterial vaginosis
b) Candidiasis
c) Trichomoniasis
d) Chlymadia infection

The Amsel criteria are in the diagnosis of bacterial vaginosis. It is considered


particularly helpful during situations in which the diagnostician's microscopic
experience, availability of microscope tools, or time may be a limiting factor. There
are four parameters used to determine the presence or absence of BV.
86. Testosterone secreted by;

a) Leydig cell
b) Sertoli cell
c) Theca lutein
d) Granulosa cell

The main hormone secreted by the testes is testosterone, an androgenic hormone.


Testosterone is secreted by cells that lie between the seminiferous tubules, known
as the Leydig cells.

87. After a normal delivery in a 27-year-old female, placenta is still


attached with uterus. Most common complication which can occur
due to forceful traction of cord?

a) Haemorrhage
b) Uterine inversion
c) Uterine rupture
d) Placenta retraction

Controlled cord traction may result in complications such as uterine inversion,


particularly if traction is applied before the uterus has contracted sufficiently, and
without applying effective counter‐ pressure to the uterine fundus

88. A patient with 37 weeks of gestation, came to hospital without


antenatal check-up and presented with onset of labour. On
examination, the mother is hep B positive, what management should
be given to neonate?

a) Hep b vaccine only


b) Hep B vaccine+ IG
c) First IG then HEP B vaccine after 1 month
d) Only IG

Infants born to HBsAg-positive mothers should receive HBIG (0.5 mL)


intramuscularly (IM) once they are physiologically stable, preferably within 12 hours
after birth. HB vaccine, either plasma-derived (10 *gmg per dose) or recombinant (5
*gmg per dose), should be administered IM in three doses of 0.5 mL each.

89. A 32-year lady with uterine fetal death after normal vaginal delivery
continues passing of urine from vagina. What is the most probable
diagnosis?

a) Bladder rupture
b) Vesicovaginal fistula
c) Urge incontinence
d) Stress incontinence
A vesicovaginal fistula is an abnormal opening that forms between the bladder
and the wall of the vagina. A fistula can be a complication after vaginal or bladder
surgery. The only way to repair this opening is through surgery, which is very often
successful.

90. A 16 years old girl with acute bleeding per vaginal was presented to
the clinic. What is the immediate management?

a) Uterine ablation
b) Uterine artery block
c) Progesterone
d) Hysteroscopy examination

400mg oral Progesterone SR (compounded) daily, and sometimes higher, has


been tried to stop bleeding. Occasionally it works well, and the main side effect is
drowsiness. After bleeding is controlled, progesterone is dosed cyclically at normal
physiologic doses.

91. A 67 years old female with HTN and diabetes comes with heavy
bleeding per vaginal. What is the next management?

a) Uterine ablation
b) Uterine artery block
c) Hysterectomy
d) Laser

Indications of Hysterectomy:
 Fibroids 30%
 Endometriosis
 Uterine prolapse
 Cancer of the uterus, cervix, or ovaries
 Vaginal bleeding, DUB 20%
 Uncontrollable PPH.
92. A lady with amenorrhea and galactorrhea;

a) Pituitary adenoma
b) Adrenal hyperplasia
c) 70 alpha hydroxylase deficiency
d) None of above

The most common pathologic cause of galactorrhea is a pituitary tumor. Other


causes include hypothalamic and pituitary stalk lesions, neurogenic stimulation,
thyroid disorders, and chronic renal failure.

93. A 35 years old with 36 week pregnancy comes with history of 5


convulsions at home. Diagnosis made by doctor is eclampsia. Next
Management;

a) Only Labetalol
b) Only MgSO4
c) MgSO4 + Labetalol
d) Clonidine

labetalol and hydralazine intravenous infusion regimens are well tolerated and
effective in controlling severe hypertension in pregnant women with severe
preeclampsia in combination with magnesium sulfate.

94. A 34 years old lady with 4 children, after 5th normal vaginal delivery,
excessive bleeding after placenta removal is due to;

a) Estrogen
b) Progesterone
c) FSH and LH
d) PPH

After the placenta is delivered, these contractions help put pressure on the bleeding
vessels in the area where the placenta was attached. If the uterus does not contract
strongly enough, these blood vessels bleed freely. This is the most common cause
of postpartum hemorrhage.

95. 32 years old lady with twin pregnancy dichorionic diamniotic, first baby
breech presentation and second baby cephalic presentation. What is the
management?

a) C – Section
b) Assisted breech
c) Instrumental delivery
d) Normal vaginal delivery
Indications of caesarean section in Twins
Obstetrics Specific for twins
 Placenta previa  Both the fetus or even the 1st
 Severe preeclampsia fetus with non-cephalic
 Post C/S presentation.
 Cord prolapse of the 1st  Conjoined twins.
baby  Collision of both heads at the brim
 Abnormal uterine preventing engagement of either
contraction head.
 Contracted pelvis  Monochorionic twins with Twin to
twin transfusion syndrome (TTTS)

96. A lady with 36-week pregnancy with previous C section comes with low
BP, tachycardia and on USG fluid present in peritoneum. What is
diagnosis and next management?

a) Uterine scar rupture with Laparotomy


b) Laparoscopy
c) Ectopic pregnancy and abortion
d) None of the above

A complete uterine rupture is a tear through the thickness of the uterine wall at
the site of a prior cesarean incision. It is a potentially life threatening condition for
both the mother and/or the baby and requires immediate surgical intervention.
Treatment of uterine rupture is immediate laparotomy with cesarean delivery and,
if necessary, hysterectomy.

97. 32 years old lady presented with pregnancy, mild bleeding and pain. On
examination uterus is tender and fetal heart sound is absent. What is
diagnosis?

a) Abruptio placenta
b) Fetal distress
c) Ectopic pregnancy
d) Placenta Previa

Placental abruption occurs when the placenta separates from the inner wall of
the uterus before birth. Placental abruption can deprive the baby of oxygen and
nutrients and cause heavy bleeding in the mother

98. A lady with 12-week pregnancy with bleeding and on examination vagina
is normal, Internal OS is closed and live birth sign on USG and fundal
height is 13 weeks. Diagnosis;
a) Threatened abortion
b) Complete abortion
c) Incomplete abortion
d) Inevitable abortion

Threatened Abortion
 It is a clinical entity where the process of abortion has started but has not
progressed to a state from which recovery is impossible.
 C/o - Slight bleeding
 O/E - Uterus - corresponds to gestational age, Cx - Internal os is
closed
 USG - live fetus with subchorionic haemorrhage

99. A newly married couple, the woman is having irregular menstruation. What
is the contraceptive of choice?

a) Barrier method
b) OCP
c) Calendar method
d) Progesterone only pills

Oral contraceptive pills -


These are hormonal birth control pills, with some amount of estrogen and progestin
to preserve the pregnancy. Low dose oral contraceptive pills are low on hormone
contraception. They have less amount of estrogen & progestin as compared to
traditional pills. It is advised by doctors to newly married women.

100. The contraceptive which is contraindicated in DVT;

a) OCP
b) Barrier method
c) Billing’s method
d) IUD

COCs, combined injectable contraceptives, and combined patch/vaginal ring


contraceptives should all be avoided in those with a history of DVT/PE, acute
DVT/PE, DVT/PE while receiving established anticoagulant therapy, major surgery
with prolonged immobilization, or known thrombophilia

101. Absolute contraindication of IUCD;


a) Acute PID
b) PCOD
c) Breast cancer
d) Anemia
Several absolute contraindications to IUDs are recognized, including: acute or
chronic pelvic inflammatory disease (PID); known or suspected pregnancy;
abnormal uterine bleeding; confirmed or suspected malignancy of the genital tract;
and congenital uterine abnormalities or fibroids distorting the cavity in a manner

102. A female came with 36 weeks of gestation, gravid 2 and parity 1,


with pain and clinical findings were hematuria, fetal HR was not
detected. what is the diagnosis and treatment?

a) Placenta previa and Suction and evacuation


b) Abruption placenta and CS
c) Obstructed labour and CS
d) Abruptio placenta and observation

 Labour is said to be obstructed when despite good uterine contractions,


there is arrest of progress due to mechanical factors causing obstruction
to delivery.
 The uterine contractions are normal (power is normal)
 There is a problem in passage or passenger

Management of obstructed labor-


• Management of dehydration- by giving i: v fluids
• Antibiotics are given to prevent infection
• Most important step is to relieve obstruction
The mode of relieving obstruction is based on fetal viability

Viable fetus
• Immediate cesarean
Dead fetus
• Perform a destructive procedure and relieve the obstruction
• If the obstetrician is not well versed with destructive procedures then
immediate cesarean
• Immediate cesarean even in a dead baby because even a dead baby
causing obstruction can lead to rupture or severe infections

103. Angle of cut in episiotomy

a) 45 degrees mediolateral right


b) 30
c) 60
d) 15

In a mediolateral episiotomy, the incision begins in the middle of the vaginal opening
and extends down toward the buttocks at a 45-degree angle.

104. Investigation of choice for endometriosis;

a) CT
b) MRI
c) USG
d) Laproscopy

Consider laparoscopy to diagnose endometriosis in women with suspected


endometriosis, even if the ultrasound was normal. ureter, consider a pelvic
ultrasound or MRI before an operative laparoscopy. laparoscopic surgery for
endometriosis should perform a systematic inspection of the pelvis

105. A multigravida woman in labour room, after delivery and placenta


removal, uncontrolled bleeding was seen. What is the most common
cause of PPH in this woman?

a) Atonic
b) Traumatic PPH
c) Clotting factor deficiency
d) None of the above

Atonic PPH

• It is the most common cause of primary PPH accounting for 90% of cases.
• The bleeding occurs as the blood vessels are not obliterated by contraction
and retraction of uterine muscle fibres

o The causes of atonicity are


• Grand multipara
• Malnutrition/Anemia
• Previous H/o atonic PPH
• Antepartum hemorrhage
• Over distended uterus due to multiple pregnancy, hydramnios and
macrosomia
• Uterine malformations or fibroid uterus
• Precipitate labour and prolonged labour
• Mismanaged third stage of labour
• Inadvertent use of oxytocin
• Use of general / epidural anesthesia especially halothane

106. What is the dose of mifepristone in emergency contraception?

a) 600 mg
b) 200 mg
c) 400 mg
d) 100 mg

Mifepristone is a highly effective and well-tolerated emergency


contraceptive when given in a dose of 600 mg within 72 h of unprotected coitus. We
assessed whether the same effectiveness can be achieved with lower doses of
mifepristone (50 mg and 10 mg) and a longer postcoital treatment period (120 h)

107. Double decidual sign is seen in;


a) Uterine gestational sac
b) Pseudo gestational sac
c) Threatened Abortion
d) Miscarriage

The double decidual sac sign (DDSS) is a useful feature on early pregnancy
ultrasound to confirm an early intrauterine pregnancy (IUP) when the yolk sac or
embryo is still not visualized.
It consists of the decidua parietalis (lining the uterine cavity) and decidua
capsularis (lining the gestational sac) and is seen as two concentric rings
surrounding an anechoic gestational sac.

108. Most common organism causing pelvic inflammatory diseases

a) Nesseria meningitis
b) GardenellaVaginalis
c) Bacteroids
d) Chalamydia

Most often, PID is caused by infection from two common STIs: gonorrhea and
chlamydia. The number of women with PID has dropped in recent years. This may
be because more women are getting tested regularly for chlamydia and gonorrhea.
109. Late rupture of tubal pregnancy is seen in;

a) Interstitium
b) Isthmus
c) Ampula
d) Fimbriae
An ectopic pregnancy occurs when a fertilized egg grows outside of the uterus.
Almost all ectopic pregnancies—more than 90%—occur in a fallopian tube. As the
pregnancy grows, it can cause the tube to burst (rupture). A rupture can cause
major internal bleeding. Very late rupture of tubal pregnancy is seen when the
implantation is at Interstitium.

110. A woman post coitus after 96 hours, best contraceptive of choice is

a) OCP
b) IUCD
c) Mifepristone
d) Progesterone only pills

Insertion of a copper IUD should be performed as soon as possible after unprotected


or inadequately protected sexual intercourse. It is effective when placed up to 5
days after sexual intercourse and, in some studies, was used up to 10 days
afterward without failure 18.

111. Delusion is;


a) Disorder of thinking
b) Disorder of memory
c) Disorder of perception
d) Disorder of affect

Delusional disorder, previously called paranoid disorder, is a type of serious mental


illness called a psychotic disorder. People who have it can't tell what's real from
what is imagined. Delusions are the main symptom of delusional disorder. They're
unshakable beliefs in something that isn't true or based on reality

112. 25-year-old male had RTA after that he is fearful of driving vehicle and
going to that place since 6 weeks and he is having flashbacks of
incidents and he is not going for his office also. What is diagnosis?

a) Post-traumatic stress disorder


b) Acute stress
c) Depression
d) Acute panic disorder

Posttraumatic stress disorder (PTSD) is a psychiatric disorder that may occur in


people who have experienced or witnessed a traumatic event such as a natural
disaster, a serious accident, a terrorist act, war/combat, or rape or who have been
threatened with death, sexual violence or serious injury.

113. 20-year-old female scared of elevator since birth. She prefers stairs
instead elevator. Diagnosis

a) Agoraphobia
b) Specific phobia
c) Social phobia
d) Acrophobia

Fear of places and situations that might cause panic, helplessness or


embarrassment.
Agoraphobia is an anxiety disorder that often develops after one or more panic
attacks.
Symptoms include fear and avoidance of places and situations that might cause
feelings of panic, entrapment, helplessness or embarrassment.
Treatments include talk therapy and medication.

114. A female was brought to psychiatry after 5 weeks of delivery she


appeared to fearfulness irritability and getting suicidal thoughts.
diagnosis is
a) Post-partum blues
b) Postpartum depression
c) Postpartum psychosis
d) Schizophrenia

Depression that occurs after childbirth.


Those who develop postpartum depression are at greater risk of developing major
depression later on in life.
Symptoms might include insomnia, loss of appetite, intense irritability and difficulty
bonding with the baby.
Untreated, the condition may last months or longer. Treatment can include
counselling, antidepressants or hormone therapy.

115. A person is unable to find pleasure in doing old activities, which he


used to enjoy earlier. Mostly he is suffering from;

a) Alexithymia
b) Depression
c) Anhedonia
d) Derailment
An inability to experience pleasure from activities usually found enjoyable.

COMMON CAUSES
Lack of pleasure can have causes that aren't due to underlying disease. Examples
include overwork, recent tragedy, financial problems, bad weather and boring
activities.

116. A clinician asked the patient “how is your mood”, patient replied
“up and down”, what could be the diagnosis?

a) Neologism
b) Tangentiality
c) Clanging
d) Derailment

Tangentiality is the tendency to speak about topics unrelated to the main topic of
discussion. While most people engage in tangentiality from time to time, constant
and extreme tangentiality may indicate an underlying mental health condition,
particularly schizophrenia

117. Identify the case;

a) Anterior uveitis
b) Glaucoma
c)Optic Neuritis
d) Keratoconus

Anterior uveitis is an inflammation of the middle layer of the eye. This middle
layer includes the iris (colored part of the eye) and adjacent tissue, known as the
ciliary body.

118. Keratometer is used to assess;


a) Curvature of cornea
b) Diameter of cornea
c) Thickness of cornea
d) Curvature of lens
A keratometer. This device measures the curvature of the anterior corneal
surface based on the power of a reflecting surface. It does this by measuring the
size of an image reflected from 2 paracentral points and utilizes doubling prisms to
stabilize the image enabling more accurate focusing.

119. Internal hordeolum is due to inflammation of-

a) Zeis gland
b) Meibomian glands
c) Mohl gland
d) Lacrimal gland

usually caused by Staphylococcus that infects the eyelash hair follicle. The external
hordeolum is caused by a blockage of the sebaceous (Zeis) glands or sweat (Moll)
glands. The blockage occurs at the lash line and presents as a painful red swollen
area that develops into a pustule. The internal hordeolum is caused by a blockage of
the Meibomian glands, and the pustule forms on the inner surface of the
eyelids. Hordeola may present on both the upper and the lower eyelids.

120. Identify the clinical condition;

a) Herpes simplex keratitis


b) Bacterial keratitis
c) Mycolic keratitis
d) Vernal keratoconjuctivitis

HSV (Herpes Simplex Virus) keratitis is an infection of the cornea—the clear dome
that covers the colored part of the eye—that is caused by HSV. The infection usually
heals without damaging the eye, but more severe infections can lead to scarring of
the cornea or blindness. The hallmark of HSV keratitis is the presence of multiple
small branching epithelial dendrites on the surface of the cornea, although
often times it first presents as a coarse, punctuate epithelial keratitis, which may be
mistaken for a viral keratitis

121. Fleischers ring is seen in;


a) Keratoconus
b) Keratomalacia
c) Keratoglobus
d) Trachoma

Fleischer rings are indicative of keratoconus, a degenerative corneal condition that


causes the cornea to thin and change to a conic shape.

122. Phlyctenular conjunctivitis seen due to


a) Post fungal infection
b) Allergic
c) Post protozoal
d) Post tuberculoid

Phlyctenular keratoconjunctivitis is an eye disorder that involves an immune


reaction(allergy) of the cornea (the clear layer in front of the iris and pupil) and
conjunctiva (the membrane that lines the eyelids and covers the white of the eye) to
bacteria. Small, yellow-gray, raised bumps form on the eye. Phlyctenular
keratoconjunctivitis is not an infection. It is an immune reaction to bacteria that are
on or around the eyes. Some of the bacteria that trigger this disorder are
staphylococci, and rarely, tuberculosis, and Chlamydia. This disorder is more
common among children.

123. Chronic granulomatous inflammation in upper lid (painless swelling);


a) Internal Hordeolum
b) External hordeolum
c) Chalazion
d) Trachoma

A chalazion is a small, slow-growing lump or cyst that develops within the


eyelid. They are not usually painful and rarely last longer than a few weeks. A
chalazion can develop when a meibomian gland at the edge of an eyelid becomes
blocked or inflamed. These glands produce oil that lubricates the surface of the eye.
124. In myopia image is formed at;
a) Anterior to retina
b) Posterior to retina
c) On retina
d) None

In a myopic eye, the image of a distant object is formed in front of the retina and
not at the retina itself. This defect may arise due to (i) excessive curvature of the eye
lens, or (ii) (ii) elongation of the eyeball. This defect can be corrected by using a
concave lens of suitable power.

125. 20-year-old male complaints of repeatedly changes in glasses, may


result into;
a) Keratoconus
b) Pathological myopia
c) Glaucoma
d) Cataract

We must keep in mind that keratoconus is a progressive condition. Unfortunately, an


eye with keratoconus is a changing eye and there is no accurate way to predict how
fast it will change and how severe it will progress. It is our hope that eventually the
condition will slow down and will not worsen beyond a certain point- but do we take
our chances as to when we hope that might happen. Even when the vision is best
corrected with glasses or contact lenses, it is important to remember that
keratoconus can still progress and worsen. It is not uncommon that a prescription
that worked wonderfully one year, will suddenly be not as sharp the next year.
Regularly scheduled eye examinations are vital to ensuring clear and comfortable
vision.

126. Find out the clinical condition as shown in photo;

a) Keratoconus
b) Limbus
c) Keratomalacia
d) None
A condition in which the clear tissue on the front of the eye (cornea) bulges outward.
With keratoconus, the clear, dome-shaped tissue that covers the eye (cornea) thins
and bulges outward into a cone shape. Its cause is unknown.
Symptoms first appear during puberty or the late teens and include blurred vision
and sensitivity to light and glare.
Vision can be corrected with glasses or contact lenses early on. Advanced cases
may require a cornea transplant.

127. Visual loss due to cerebral degeneration is related to which artery?


a) Posterior cerebral artery
b) Middle cerebral artery
c) Internal carotid artery
d) Inferior cerebral artery

Radiographic angiography of the posterior circulation. Bilateral vertebral


arteries fuse to form the basilar artery. The ultimate branches of the
basilar artery form the posterior cerebral arteries (PCAs, arrows), which
supply the visual cortex. Infarctions in the PCA territories can cause
cortical vision loss.

128. Appropriate treatment for the mild congenital ptosis;


a) LPS Resection
b) Antibiotics and hot compression
c) Tarsal fracture
d) Wedge resection of conjunctiva

Resection of LPS is the most physiological approach to the treatment of ptosis.


About 70% to 80% ptosis repairs are done by this technique. This is the operation of
choice in all cases with fair to good (more than 4 mm.) levator function.

129. Vision 2020, which disease has added recently for India?
a) Cataract
b) Childhood blindness
c) Glaucoma
d) Refractive errors and low vision

VISION2020 aims to eliminate avoidable blindness in the world by 2020 and targets
the world's leading causes of avoidable visual impairment: cataract, trachoma,
onchocerciasis, childhood blindness (including vitamin A deficiency), and refractive
error and low vision.
Glaucoma is now regarded as a sort of “missing link” and VISION 2020 India
successfully advocated for the inclusion of glaucoma in the next phase by the Indian
government.

130. Diabetic patient sudden loss of vision, 6/60 vision, fundus picture is
given here. What is the diagnosis?

a) CRVO
b) CRAO
c) Haemorrhage
d) Hypertensive retinopathy

Central retinal vein occlusion (CRVO) is a common retinal vascular disorder.


Clinically, CRVO presents with variable visual loss; the fundus may show retinal
hemorrhages, dilated tortuous retinal veins, cotton-wool spots, macular edema, and
optic disc edema

131. Advantage of contact lens over normal glasses;


a) Prismatic effect will less
b) Prevention from UV rays
c) Decrease inflammation
d) Decrease infection

Contact lenses move with your eye, allow a natural field of view, have no frames to
obstruct your vision and greatly reduce distortions. Unlike glasses, they do not fog up
or get splattered by mud or rain. Contact lenses are excellent for sports and other
physical activities. since the contact lens are extremely close to the eye. they reduce
the prismatic effect which is normal while wearing spectacles due to the distance
between the glasses and the eye

132. Which of the following drug causes postural hypotension commonly?


a) ACE inhibitor
b) Beta blocker
c) Alpha blocker
d) Angiotensin receptor blockers

Alpha blockers are a type of blood pressure medication. They lower blood
pressure by preventing a hormone called norepinephrine from tightening the muscles
in the walls of smaller arteries and veins. As a result, the blood vessels remain open
and relaxed. This improves blood flow and lowers blood pressure. When you start
taking an alpha blocker, you might develop low blood pressure and dizziness,
which can make you faint when you rise from a sitting or lying position.

133. Mechanism of action of allopurinol is

a) Xanthine oxidase inhibition


b) Recombinant uricase
c) Increase uric acid excretion
d) Decrease chemotaxis

Mechanism of Action
Allopurinol and oxypurinol both inhibit xanthine oxidase, an enzyme in the purine
catabolism pathway that converts hypoxanthine to xanthine to uric acid.

134. Which of the following drugs follow zero order kinetics?


a) Phenytoin
b) Aspirin
c) Morphine
d) Propranolol

Zero order kinetics is a way of describing how the body uses and breaks down
some medicines. While the rate at which the body eliminates most drugs is
proportional to the concentration administered, known as first order kinetics, drugs
that work by zero order kinetics work at a predictable, constant rate

List of drugs following zero-order kinetics


 Phenytoin, Phenylbutazone.
 Warfarin.
 Heparin.
 Ethanol.
 Aspirin & other salicylates.
 Theophylline, Tolbutamide.
 Salicylates.

135. Which of the following is an oral direct thrombin inhibitor?


a) Dabigatran
b) Rivaroxaban
c) Warfarin
d) Lepirudin

Dabigatran is a potent, synthetic, reversible, non-peptide thrombin inhibitor. The


inhibition of thrombin results in decreased formation of fibrin and reduces thrombin-
stimulated platelet aggregation and thus prevents the formation of thrombi.

136. Therapeutic drug monitoring is done for;

a) Phenytoin
b) Heparin
c) Aspirin
d) Metformin

Therapeutic drug monitoring of phenytoin is necessary to ensure therapeutic and


nontoxic levels. Hypoalbuminemia, renal failure, and interactions with other highly
protein-bound drugs (e.g., valproic acid) alter protein binding of phenytoin.

137. A lady has taken medication for ameiobiasis infection. She drank
alcohol in a party. She has nausea vomiting and dizziness. Which anti
amoebic drug could have led to interaction with alcohol to produce these
symptoms?
a) Metronidazole
b) Nitazoxanamide
c) Paromomycin
d) Diloxanide

Metronidazole (Flagyl) is a common and effective treatment for bacterial vaginosis. A


known adverse effect of metronidazole (Flagyl) is a drug-drug reaction that occurs
when metronidazole and alcohol are taken together. Symptoms of this disulfiram-like
reaction include severe nausea and vomiting.

138. Most potent opioid is;

a. Fentanyl
b. Morphine
c. Pentazocine
d. Pethidine

Fentanyl is a powerful synthetic opioid analgesic that is similar to morphine but is 50


to 100 times more potent. In its prescription form it is prescribed for pain, but
fentanyl is also made illegally.

139. Most cardiotoxic anti-cancer drug among the following;


a) Anthracyclines
b) Cyclophosphamide
c) Lmatinib
d) Tamoxifen

Anthracyclines, such as doxorubicin and idarubicin, remain an important class of


chemotherapeutic agents. Unfortunately, their efficacy in treating cancer is limited by
a cumulative dose-dependent cardiotoxicity, which can cause irreversible heart
failure.
This progressive cardiotoxicity usually occurs after the completion of treatment
with anthracyclines, and may become apparent within one year of the completion
of treatment (early onset chronic cardiotoxicity) or many years after chemotherapy
has been completed (late onset chronic cardiotoxicity).

140. Which of the following is a carbonic anhydrase inhibitor?


a) Acetazolamide
b) Hydrochlorthiazide
c) Furosemide
d) Mannitol

Carbonic anhydrase inhibitors are a medication used in the


management and treatment of glaucoma, idiopathic intracranial
hypertension, altitude sickness, congestive heart failure, and epilepsy,
among other diseases. Carbonic anhydrase inhibitors are considered
part of the diuretic class of medications
Acetazolamide, dichlorphenamide, and methazolamide are carbonic
anhydrase inhibitors.

141. Which of the following is the chemoprophylaxis of choice in a


person who is on a journey to endemic malarial region?
a) Chloroquine
b) Quinine
c) Lumefantrine
d) Pyrimethamine

Malaria chemoprophylaxis concerns prescribing healthy individuals medication for an


infection they have an unknown chance of getting. Sensible use of malaria
chemoprophylaxis is a balance between the risk of infection and death, and the risk
of side effects.
Chloroquine phosphate is used to prevent and treat malaria. It is also used to treat
amebiasis. Chloroquine phosphate is in a class of drugs called antimalarials and
amebicides. It works by killing the organisms that cause malaria and amebiasis.

142. All of the following drugs are used for prophylaxis of migraine except;
a) Propranolol
b) Amitriptyline
c) Flutamide
d) Flunarizine
143. Antidote for opioid poisoning;
a) Naloxone
b) Pethidine
c) Flumazenil
d) Physostigmine

Opioid use can lead to death due to the effects of opioids on the part of the brain
which regulates breathing. An opioid overdose can be identified by a combination of
three signs and symptoms:
 pinpoint pupils;
 unconsciousness; and
 difficulties with breathing.

Death following opioid overdose is preventable if the person receives basic life
support and the timely administration of the drug naloxone. Naloxone is an antidote
to opioids that will reverse the effects of an opioid overdose if administered in time.
Naloxone has virtually no effect in people who have not taken opioids.

144. Monoclonal antibody used in cancer is


a) Rituximab
b) Cisplatin
c) 5-fluorouracil
d) Methotrexate

Rituximab is a chimeric monoclonal antibody targeted against CD20 which is a


surface antigen present on B cells. Therefore, it acts by depleting normal as well
as pathogenic B cells while sparing plasma cells and hematopoietic stem cells as
they do not express the CD20 surface antigen.
145. A patient after valve replacement will require follow up treatment with
a) Warfarin
b) ACE inhibitors
c) Beta blockers
d) Thiazides

To help prevent thromboembolism, people who receive mechanical heart valves


must take anticoagulant medications (anticlotting drugs) for the rest of their lives.
All patients with mechanical prosthetic valves require lifelong VKA (eg,
warfarin) anticoagulation. Additional indications for anticoagulation, such as atrial
fibrillation (AF), may require a higher goal INR than for the mechanical valve alone

146. All of the following can result in gynecomastia except;


a) Liver failure
b) Spironolactone
c) Digoxin
d) Aromatase inhibitors

Gynecomastia can be caused by an imbalance between the sex hormones


testosterone and oestrogen. Oestrogen causes breast tissue to grow. While all
men produce some oestrogen, they usually have much higher levels of testosterone,
which stops the oestrogen from causing breast tissue to grow.
Aromatase inhibitors work by blocking the enzyme aromatase, which turns the
hormone androgen into small amounts of estrogen in the body. This means that less
estrogen is available to stimulate the growth of hormone-receptor-positive breast
cancer cells.

147. Hydrochlorothiazide works by inhibiting


a) Na+ CI pump in late DCT
b) Na+ CI pump in early DCT
c) Na+ K+ 2CT pump in descending limb of loop of henle
d) Na+K+2CT pump in ascending limb of loop of henle

Hydrochlorothiazide acts on the distal convoluted tubules and inhibits the sodium
chloride co-transporter system. This action leads to a diuretic action that lowers
blood pressure, but there is also a potassium loss in the urine.

148. The drug of choice for hyperthyroidism in third trimester of pregnancy


is;
a) Carbimazole
b) Prophylthyouracil
c) Sodium iodide
d) Radioactive iodine

PTU(propylthiouracil) Propylthiouracil is the drug of choice for hyperthyroidism in


first trimester of pregnancy. It has a side effect of hepatotoxicity and hence due to
this reason DOC for hyperthyroidism in second and third trimester is Carbimazole

149. Mechanism of action of exenatide in diabetes mellitus is


a) It is analogue of GLP released from gut and increase glucose dependant
insulin secretion
b) It is DPP-4 inhibitor and result in decreased breakdown of GLP
c) It inhibits SGLT-2 and cause glucosuria
d) It is amylin analogue and decrease glucagon

Exenatide is a GLP-1 receptor agonist released from the gut and acts to increase
glucose-dependent insulin secretion from pancreatic beta cells, suppress
glucagon secretion, delay gastric emptying, and reduce food intake. The binding of
the drug to pancreatic GLP-1 receptors mediates these actions

150. Which anti-asthma drug is avoided with erythromycin?


a) Salbutamol
b) Ipratropium
c) Theophylline
d) Terbutaline

Administration of allopurinol, oral contraceptives, cimetidine, erythromycin,


quinolone antibiotics, fluvoxamine, zileuton, and zafirlukast increase theophylline
levels.
Using erythromycin together with theophylline increase the effects of theophylline.
High serum theophylline levels increase the risk of toxicity. Toxic effects include
nausea and vomiting, headaches, gastric discomfort, diuresis, insomnia, cardiac
arrhythmias, behavioral disturbances, and epileptic seizures

151. McNaughton’s rule;


a) IPC 82
b) IPC 84
c) IPC 87
d) IPC 85

Section 84 IPC embodies McNaughton rules as follows: "Nothing is an offence


which is done by a person who, at the time of doing it, by reason of unsoundness of
mind, is incapable of knowing the nature of the act or that he is doing what is either
wrong or contrary to the law.

152. A farmer with pinpoint pupils, increased secretions and urination. What
is the most likely diagnosis?
a) Organophosphate poisoning
b) Opioid poisoning
c) Alcohol poisoning
d) Atropine poisoning

Organophosphate poisoning is poisoning due to organophosphates (OPs).


Organophosphates are used as insecticides, medications, and nerve agents.
Symptoms include increased saliva and tear production, diarrhea, vomiting, small
pupils, sweating, muscle tremors, and confusion.
The mainstays of medical therapy in organophosphate (OP) poisoning
include atropine, pralidoxime (2-PAM), and benzodiazepines (eg, diazepam).
Initial management must focus on adequate use of atropine.
153. Posthumous birth;
a) Child born after death of father
b) After death of mother during delivery
c) After death of both parents
d) None

A posthumous birth is the birth of a child after the death of a biological parent. A
person born in these circumstances is called a posthumous child or a posthumously
born person

154. Poisoning caused by bullet retained in body is;


a) Lead
b) Phosphorus
c) Nitro cellulose
d) Iron

Lead poisoning is an unusual complication of gunshot wounds that occurs when


retained lead bullet fragments are in contact with body fluids capable of solubilizing
lead. The epidemic of violence by gunfire may result in increasing numbers of lead
poisoning cases from this exposure.

155. After injury hair bulbs are seen crushed under microscopy. Probable
type of wound;
a) Abrasion
b) Laceration
c) Stab injury
d) None

Lacerations
o These wounds are commonly known as ‘gashes, tears or cuts’ of the skin
o It is an open wound.
o Breach in the epidermis, dermis or underlying tissue as a result of blunt force

Features
o Tissues not cleanly divided and is bridged across base of the wound.
o Foreign material may be found as well
o Hair and hair bulbs are crushed
o Margins are irregular, ragged and uneven and their ends are pointed or
blunt
o Lacerations produced without excessive skin crushing may have
relatively regular sharp margins.
o Bruising is seen either in the skin or subcutaneous tissue around the wound
o Hair bulbs are crushed
o Hair and epidermal tags may be driven deeply into the wound
o Less haemorrhage
o Presence of foreign matter in the wound
o Shape and size of the wound does not correspond to the weapon used

156. A patient was found to have pink coloured skin and mucosa, bitter
almond smell from breath and frothy discharge. Cause of death in this
patient;
a) Cyanide poisoning
b) CO poisoning
c) H2S poisoning
d) Dhatura poisoning

Cyanide can be a colorless gas, such as hydrogen cyanide (HCN) or cyanogen


chloride (CNCl), or a crystal form such as sodium cyanide (NaCN) or potassium
cyanide (KCN). Cyanide sometimes is described as having a “bitter almond” smell
Cyanide prevents the cells of the body from using oxygen. When this happens,
the cells die. Cyanide is more harmful to the heart and brain than to other organs
because the heart and brain use a lot of oxygen.

157. Entry wound kind of depression will be seen in


a) Inversion
b) Eversion
c) Pond’s fracture
d) Depression

Entrance wounds: point where the bullet enters the biological target
o General features:
 Abrasion ring: a reddish brown abrasion on the skin surrounding the
gunshot wound of entrance
 Greasy rim or bullet wipe is a dirt collar due to the lubricant carried on
the bullet
 Inverted edges: due to the bullet movement inside the body
 Smaller than bullet caliber (usually): due to the elastic recoil of the
affected tissues
158. A lady comes to OPD after fall from scooty. Her vitals are
stable. She is having continuous, clear watery discharge from
nose after 2 days. These are most likely the feature of;
a) CSF rhinorrhoea
b) Acute respiratory infection
c) Middle cranial fossa fracture
d) Rhinitis

Cerebrospinal fluid (CSF) rhinorrhea is a condition where the fluid that surrounds
the brain leaks into the nose and sinuses. Head trauma, surgery, or even birth
defects can make a hole in the membranes that hold this fluid. It then leaks into your
nose or ear, causing a watery, runny nose

159. Office headache is due to inflammation of which sinus?


a) Frontal
b) Maxillary
c) Ethmoidal
d) Sphenoid

Frontal sinusitis is inflammation or infection of the sinuses located just behind the
eyes and in the forehead. The sinuses are a system of connected hollow cavities in
the face that contain air and a thin layer of mucus. All sinuses produce mucus that
moisturizes the airways and drains into the nasal passages.
If the frontal sinuses are inflamed or infected, they cannot drain mucus efficiently,
and this can make breathing difficult. It can also lead to a feeling of increased
pressure around the eyes and forehead.
Since most cases of acute frontal sinusitis are caused by a viral infection, treatment
would be to recommend taking a nasal spray or decongestant to decrease
inflammation, assist with mucus drainage, and relieve pressure in the frontal sinuses

160. Adam’s apple in Male is due to;


a) Thyroid cartilage
b) Cricoid cartilage
c) Hyoid cartilage
d) Epiglottis cartilage

The Adam's apple is made up of the thyroid cartilage. Cartilage is the same tissue
that makes up your nose, ears, and windpipe (i.e., trachea). The voice box and
windpipe have several kinds of cartilage, which work together to make sure your
airways stay clear and you're able to speak.

161. Most prominent and larger air cell of ethmoidal sinus?


a) Agger nasi cell
b) Haller cell
c) Onodi cell
d) Bulla ethmoidalis

Bulla ethmoidalis

This is the largest and most prominent anterior ethmoid air cell. It is
related laterally to the lamina papyracea. It may fuse with the skull
base superiorly and with the lamella basalis posteriorly

162. Most common presentation of nasopharyngeal carcinoma;


a) Neck lymph node
b) Blockage of nose
c) Loss of smell
d) Blood tinged discharge

The four most common symptoms in order of frequencies were cervical


lymphadenopathy, epistaxis, hearing loss and diplopia. 80.8% of the patients
presented with cervical lymphadenopathy and about 85% of the patients presented
in the advanced stages.
Other signs and symptoms may include:
 Swelling of the neck.
 Persistent headaches.
 Nasal congestion (a blocked nose)
 Facial pain.
 Nosebleeds.
 Changes in hearing.
 Ringing in the ears.
 Many people have no symptoms

163. EBV is associated with;


a) Nasopharyngeal carcinoma
b) Esophageal cancer
c) Gastric cancer
d) Renal carcinoma

EBV-associated diseases
 Nasopharyngeal carcinoma (NPC) ...
 EBV-associated gastric cancer...
 Breast cancer. ...
 HL and NHL. ...
 Post-transplant lymphoproliferative carcinoma (PTLD) ...
 Infectious mononucleosis (IM) ...
 Chronic active EBV infection (CAEBV) ...
 EBV-associated hemophagocytic lymphohistiocytosis (EBV-HLH)
164. All of the following is true about Meniere’s disease except;
a) Idiopathic
b) Low frequency hear loss
c) Increase in endolymph
d) Destroying the cochlea is mandatory

An inner ear disorder that causes episodes of vertigo (spinning).


Ménière's disease usually starts in one ear, but later may involve both. Smoking,
infections or a high-salt diet may worsen the disease.
Symptoms include a spinning sensation (vertigo), hearing loss, ear ringing (tinnitus)
and ear pressure. The vertigo may cause severe nausea and imbalance. Hearing
loss may become permanent.
Drugs for motion sickness or nausea may help manage symptoms.

165. Investigation used for CSF rhinorrhoea are all except


a) CT cisternogram
b) Skull X-ray
c) beta2 transferrin
d) Nasal endoscopy

Diagnosing a CSF leak includes an analysis of the nasal fluid for a protein called
beta-2 transferrin which is most only found in cerebrospinal fluid. X-ray,CT and MRI
scans may also be requiring to determine the location and severity of the leakage.

166. Treatment of choice in nasopharyngeal carcinoma;


a) Chemotherapy
b) Radiotherapy
c) Chemo given with radiation
d) Surgery

The main treatment for NPC is radiation therapy. It is often given in combination
with chemotherapy. This approach may be called concomitant chemoradiotherapy.
Surgery for NPC is occasionally used, mainly to remove lymph nodes after
chemoradiotherapy or to treat NPC that has come back after initial treatment.

167. A 6-year-old boy came to hospital with complaints of sore throat


and difficulty in swallowing, his left tonsils were pushed medially. what
will be the diagnosis?
a) Peritonsillar abscess
b) Retropharyngeal abscess
c) Ludwig’s angina
d) Parapharyngeal abscess
A parapharyngeal abscess is a deep neck space abscess of the
parapharyngeal space, which is lateral to the superior pharyngeal
constrictor muscle and medial to the masseter muscle. This space is
divided by the styloid process into anterior and posterior compartments.
A parapharyngeal abscess is a deep neck abscess. Symptoms
include fever, sore throat, odynophagia, and swelling in the neck
down to the hyoid bone. Diagnosis is by CT. Treatment is antibiotics and
surgical drainage.

168. A young boy came to OPD with the complaints of difficulty in


breathing. On examination, bilateral polyps were found. On aspiration,
bleeding was seen. What will be the initial management?
a) I and D
b) Antihistaminic
c) Conservative
d) Septoplasty

Corticosteroids. Oral and topical nasal steroid administration is the primary


medical therapy for nasal polyposis. Antihistamines, decongestants, and cromolyn
sodium provide little benefit. Immunotherapy may be useful to treat allergic rhinitis
but, when used alone, does not usually resolve existing polyps.

169. Identify condition in the shown image?

a) Impetigo contagiosa
b) Contact dermatitis
c) Molluscum contagiosum
d) HSV

Nonbullous impetigo, or impetigo contagiosa, is caused by Staphylococcus


aureus or Streptococcus pyogenes, and is characterized by honey-colored crusts
on the face and extremities. Impetigo primarily affects the skin or secondarily infects
insect bites, eczema, or herpetic lesions.
A highly contagious skin infection that causes red sores on the face.
Impetigo mainly affects infants and children.
The main symptom is red sores that form around the nose and mouth. The sores
rupture, ooze for a few days, then form a yellow-brown crust.
Antibiotics shorten the infection and can help prevent spread to others.
170. Exclamation mark hair pattern is seen in

a) Alopecia areata
b) Androgenic alopecia
c) Anagen effluvium
d) Telogen effluvium

Sudden hair loss that starts with one or more circular bald patches that may overlap.
Alopecia areata occurs when the immune system attacks hair follicles and may be
brought on by severe stress.
The main symptom is hair loss.
Treatment may address any underlying conditions and includes topical scalp
medication.
The characteristic diagnostic finding of alopecia areata is the exclamation point hair.
These can be found in areas of hair loss and are short broken off hairs that are
narrower closer to the scalp and therefore mimic an exclamation point. In some
cases a biopsy is necessary for diagnosis.

171. What is the name of this appearance which is seen in pellagra?

a) Burn
b) Castle neck appearance
c) Shawl sign
d) Gottron's papule
The Casal collar or Casal necklace is a clinical sign in which there is an
erythematous pigmented skin rash in the distribution of a broad
collar (dermatomes C3 and C4). It is seen in patients with pellagra, as a result of
vitamin b3 deficiency. The sign is named after Gaspar Casal

172. A lady with 50% burns of dermis & subcutaneous tissue came to
emergency department. Burns will be classified as;
a) 1st degree
b) 2nd degree superficial
c) 2nd degree deep
d) 3rd degree burn
173. Identify the abnormal condition given below;

a) Testicular torsion
b) Hydrocele
c) Varicocele
d) Testicular atrophy

A twisting of the male organ that makes hormones and sperm (testicle).
When the testicle rotates (testicular torsion), it twists the cord supplying blood to the
loose bag of skin (scrotum) beneath the penis. This may occur after vigorous activity,
a minor injury to the testicles or sleep.
Sudden, severe pain and swelling in the testicle are symptoms.
Surgery is required. Treated promptly, the testicle can often be saved. A longer wait
may affect fertility.

174. 20 year old female diagnosed previously with adenocarcinoma


stomach and on examination following is seen;

a) Sister marry joseph nodule


b) Ulcer
c) Infected umbilical hernia
d) Irish node

Sister Mary Joseph nodule or Sister Mary Joseph Sign refers to a palpable nodule
bulging into the umbilicus as a result of metastasis of a malignant cancer in the
pelvis or abdomen. Gastrointestinal malignancies account for about half of the
underlying sources (gastric, colonic, pancreatic cancer), gynecologic (ovarian,
uterine cancer), unknown primary tumors and rarely bladder or respiratory
malignancies cause umbilical metastasis.

175. Rodent ulcer is


a) Basal cell cancer
b) Squamous cell ca
c) Rhinophyma
d) Adenocarcinoma

Rodent ulcers are a type of skin cancer. Also known as basal cell carcinoma,
rodent ulcers occur in the lower most layer of your skin. They usually develop in skin
areas exposed to the sun. Rodent ulcers are a very slow growing cancer that usually
does not spread to other parts of your body.

176. Most common oral cancer;


a) Squamous cell ca
b) Adenocarcinoma
c) Transition cell ca
d) Mucoepidermoid

Squamous cell carcinoma is the most common type of mouth cancer, accounting
for 9 out of 10 cases. Squamous cells are found in many areas of the body, including
the inside of the mouth and in the skin. Less common types of mouth cancer include:
adenocarcinoma, which is cancers that develop inside the salivary glands.

177. Distal ileum was removed in a 20-year-old girl. Which absorption


deficient will be seen?
a) Iron
b) Bile salts
c) Folic acid
d) Copper

The main function of the ileum is to absorb vitamin B12, bile salts, and whatever
products of digestion were not absorbed by the jejunum. ... They absorb fatty acid
and glycerol, the products of fat digestion.

178. 62-year-old female has kidney stone and treated with PCNL. After 2
days she again comes to OPD with chills and fever. What is the
complication?
a) Bacterial sepsis
b) Acute pancreatitis
c) Splenic injury
d) Ureteric stricture

Bacterial sepsis is a life-threatening condition that arises when the body's


response to an infection injures its tissues and organs. Sepsis is caused by
conditions that threaten the functional integrity of the host such as microbial invasion.
The signs and symptoms of sepsis can include a combination of any of the
following:
 confusion or disorientation,
 shortness of breath,
 high heart rate,
 fever, or shivering, or feeling very cold,
 extreme pain or discomfort, and.
 clammy or sweaty skin.

179. Cellulitis is

a) Nonsuppurative and noninvasive


b) Suppurative and noninvasive
c) Nonsuppurative and invasive
d) Suppurative and invasive

Nonpurulent cellulitis includes rapidly spreading superficial cellulitis and


erysipelas; typically involves groups A, B, C, and G beta-hemolytic streptococci and,
occasionally, methicillin-susceptible Staphylococcus aureus (MSSA); these
infections are diagnosed clinically

180. identify the condition as shown in image;

a) Port wine stain


b) Strawberry nevus
c) Erythema multiforme
d) Exanthem subitem

A port-wine stain is a type of birthmark. It got its name because it looks like maroon
wine was spilled or splashed on the skin. Though they often start out looking pink at
birth, port-wine stains tend to become darker (usually reddish-purple or dark red) as
kids grow.
Port-wine stains are caused by an abnormal formation of tiny blood vessels in
the skin. In rare cases, port-wine stains are a sign of Sturge-Weber syndrome or
Klippel-Trenaunay-Weber syndrome.

181. 28-year-old alcoholic patient walks to hospital with the complaints of


binge vomiting, chest pain, fever and pneumomediastinum. The most
probable condition;
a) Boerrhave syndrome
b) Tension pneumo thorax
c) PUD perforation
d) Mallory weis tear

Effort rupture of the esophagus, or Boerhaave syndrome, is a spontaneous


perforation of the esophagus that results from a sudden increase in
intraesophageal pressure combined with negative intrathoracic pressure (eg,
severe straining or vomiting).
It is induced by increased esophageal pressure followed by straining, which can
result from retching, vomiting, weightlifting, childbirth, or defecation

182. Abdominal pain, fever and jaundice. This triad is known as;
a) Charcot’s triad
b) Saint’s triad
c) Virchow triad
d) Renault’s triad

Charcot's cholangitis triad is the combination of jaundice; fever,


usually with rigors; and right upper quadrant abdominal pain. It
occurs as a result of ascending cholangitis (an infection of the bile duct
in the liver).

183. 40 years old patient with femur fracture with pulmonary infiltration and
respiratory distress
a) Fat embolism
b) Pulmonary embolism
c) Air embolism
d) Obstruction

A disruption to blood supply caused by fat globules in a blood vessel.


A fat embolism occurs when fat globules are released into the bloodstream. It’s most
commonly associated with a trauma, such as a bone fracture.
Symptoms, if present, typically occur 24 to 72 hours after the trauma. They include
shortness of breath, confusion and a rash.

184. Which patient has better prognosis in breast cancer?


a) Luminal A
b) Luminal B
c) Patient who has triple test negative
d) Patient who has triple test positive

Luminal A breast cancer is hormone-receptor positive (estrogen-receptor and/or


progesterone-receptor positive), HER2 negative, and has low levels of the protein Ki-
67, which helps control how fast cancer cells grow. Luminal A cancers are low-
grade, tend to grow slowly and have the best prognosis

185. 33-year man presented with fever and pain in upper right
hypochondrium after food intake. Investigation of choice;
a) CECT
b) Ultrasound
c) MRI
d) HRCT

Ultrasonography is the most common test used in the emergency department for
the diagnosis of biliary colic and acute cholecystitis This imaging modality may be
diagnostic for biliary disease, help exclude biliary disease, or may reveal alternative
causes of the patient's symptom

186. Long term diabetic patient with blisters walked barefoot few miles on
hot sand. He presented with this clinical condition. What is the most
probable diagnosis?

a) Diabetic foot
b) Burn
c) Necrotizing Fasciitis
d) Elephantiasis

A serious bacterial infection that destroys tissue under the skin.


Flesh-eating disease occurs when bacteria enter the body through a break in the
skin. People with a weakened immune system can be at greater risk of developing
this condition.
The condition spreads quickly. Symptoms include blisters, fever, fatigue and pain
worse than a person would expect based on the wound's appearance.
Treatment involves immediate delivery of IV antibiotics. Surgical removal of dead or
infected tissue from the wound is often required.

187. A man was presented to emergency department with head


injury after an accident with vehicle. Investigation of choice should
be;
a) CECT
b) MRI
c) NCCT
d) MRI

Noncontrast head CT is the most useful study for the evaluation of patients after
trauma, focal neurologic symptoms, change in mental status, and severe
headaches to rule out life-threatening pathology. Clinicians in all fields must be able
to readily identify a number of different abnormalities.

188. ALVARADO score 2 defines:


a) Temperature
b) Leukocytosis
c) Tenderness in left iliac fossa
d) Migratory pain

189. A patient of Varicose veins came to hospital; intern was on duty.


Which test he shall perform to rule out the DVT?
a) Brodie Trendelenberg test
b) Perthe’s test
c) Thomas test
d) Ober test

The Brodie–Trendelenburg percussion test is a medical test to


determine valvular incompetence in superficial veins. A finger is placed over the
lower (distal) part of the vein being examined. The upper (proximal) part of the vein
is then tapped (percussed). If the impulse is felt by the finger placed at the lower
end, it indicates incompetence of valves in that vein.
190. Early investigation of choice for genital TB
a) Early Morning Urine Sample
b) Urine sample with giemsa staining
c) Endometrial ablation
d) Incision and curettage

Early morning urine (EMU) is accepted as a useful method for improving the
concentrations of a number of antigens and analytes found in human urine. The
strategy has been shown to improve the yield of urine culture for TB diagnosis

Urine is easily obtainable and can be tested using either a lateral flow assay
(LFA) for TB-lipoarabinomannan (TB-LAM) or the Xpert MTB/RIF assay (Xpert;
Cepheid, Sunnyvale, CA). Tuberculosis-LAM is emerging as the key urine-based TB
diagnostic

191. True about crohn’s disease except


a) Recurrence is more common
b) Rectum is involved
c) Fissures are formed
d) Transmural

A chronic inflammatory bowel disease that affects the lining of the digestive tract.
Crohn's disease can sometimes cause life-threatening complications.
Crohn's disease can cause abdominal pain, diarrhoea, weight loss, anaemia and
fatigue. Some people may be symptom-free most of their lives, while others can
have severe chronic symptoms that never go away.
Crohn's disease cannot be cured. Medications such as steroids and
immunosuppressants are used to slow the progression of disease. If these aren't
effective, a patient may require surgery. Additionally, patients with Crohn's disease
may need to receive regular screening for colorectal cancer due to increased risk.

192. Patient underwent gastrectomy, after eating within 20 minutes


sweating, diarrhoea, what could be the cause?
a) Hyperglycemia
b) Early dumping syndrome
c) Late dumping syndrome
d) Hypoglycemia

Early dumping syndrome happens when the sudden influx of food into your
intestine causes a lot of fluid to move from your bloodstream into your
intestine as well. This extra fluid causes diarrhea and bloating. Your intestines also
release substances that speed your heart rate and lower your blood pressure.

193. In an accident case, after the arrival of medical team, all should be done
in early management except;
a) Stabilization of cervical vertebrae
b) Check BP and Pulse
c) Check Respiration
d) Glasgow coma scale

The Glasgow Coma Scale (GCS) is used to objectively describe the extent of
impaired consciousness in all types of acute medical and trauma patients. The scale
assesses patients according to three aspects of responsiveness: eye-opening,
motor, and verbal responses.

194. Patients are categorized on the basis of chances of survival in disaster


management is;
a) Triage
b) Mitigation
c) Tagging
d) Surge capacity
195. Risk factors of oesophageal carcinoma are all except
a) GERD
b) Betel chewing
c) Oxalate food
d) Spicy food

Multiple risk factors are associated with the development of esophageal


adenocarcinoma. These include Barrett's esophagus, acid peptic disorders, motor
disorders of the esophagus, other malignancies, medications, environmental
exposures, diet, and nutrition.

196. Left kidney transplantation will be placed in which place?


a) Iliac fossa
b) kidney fossa
c) Retroperitoneal space
d) Left lower quadrant

Traditionally the right iliac fossa is the standard fossa for a kidney transplantation
procedure and the left iliac fossa is the preferred site for simultaneous kidney-
pancreas transplantation.

197. For severe malnutrition MAC will be less than-


a) 14.5
b) 13.5
c) 12.5
d) 11

Mid-arm circumference of < 11cm indicates severe malnutrition in infants from


1-6 months of age. Physiopathological principle of arm circumference
measurement in children aged 1-5 years and the relationship with severity of
malnutrition
198. 3 months old baby with fever and respiratory rate 60, irritable,
and well feed, no stridor, no chest indrawing, no convulsions.
Diagnosis-
a) Pneumonia
b) Severe pneumonia
c) Very sever pneumonia
d) No Pneumonia

199. Number of primary teeth;


a) 20
b) 24
c) 16
d) 32

At birth people usually have 20 baby (primary) teeth, which start to come in (erupt)
at about 6 months of age. They fall out (shed) at various times throughout childhood.
By age 21, all 32 of the permanent teeth have usually erupted.

200. All of the following are types of Primary headache except


a) Migraine
b) Cluster
c) Tension
d) Temporal arteritis
201. Which is associated with alcohol related cardiomyopathy
a) Dilated
b) Restrictive
c) Hypertrophic
d) None

Alcoholic cardiomyopathy is a type of dilated cardiomyopathy typically found in


people with alcohol use disorder. Due to the direct toxic effects of alcohol on heart
muscle, the heart is unable to pump blood efficiently, leading to heart failure.

202. 40-year-old male presents with fever and abdominal pain and
diagnosed with HIV and TB. How will you give treatment?
a) ATT and AIDS treatment simultaneously.
b) First ATT and then ART
c) ATT only
d) First ART and then ATT

WHO guidelines recommend starting ART between 2 and 8 weeks of TB treatment


in patients with CD4 < 200 cells/μL and during continuation phase in those with CD4
counts 200-350 cells/μL . In patients with very low CD4 counts there is greater
urgency to start ART.

203. Patient with abdominal pain, IgA deposition, haematuria and


purpura over leg as given in image. Diagnosis;

a) IgA nephropathy
b) HSP
c) Buerger’s disease
d) PSGN

A disorder causing inflammation and bleeding in the small blood vessels.


Henoch-Schönlein purpura affects the small blood vessels of the skin, joints,
intestines and kidneys. It's most common before the age of seven but can affect
anyone.
Symptoms include reddish-purple spots on the lower extremities, swollen and sore
joints, abdominal pain or bloody urine.
The condition is often treated with steroids.
204. Patient presents with diarrhoea, abdominal pain and finding on
upper GI endoscopy is shown in the photo. What is the pathology?

a) Antigliadin antibody
b) Antimitochondrial antibody
c) Anticentromere antibody
d) Antitopoisomerase antibody

A gliadin antibodies test is used to help doctors diagnose celiac disease. Celiac
disease is an autoimmune disorder in which the body's immune system mistakenly
thinks that gluten — a protein in wheat, barley, rye, and oats

205. Almost odourless Stool seen in-


a) Upper GI bleeding
b) Bacillary Dysentery
c) Ameobiasis
d) Cholestatis stool

206. Hyperkalemia means more than


a) 4.5 meq/l
b) 5.5 meq/l
c) 7.5 meq/l
d) 10.5 meq/l
A typical potassium level for an adult falls between 3.5 and 5.0 millimoles per liter
(mmol/L). Hyperkalemia occurs when levels go above 5.5 mmol/L. A reading above
6.5 mmol/L can cause heart problems that require immediate medical attention

207. Female presents with galactorrhea and amenorrhea and diagnosed


with Pituitary adenoma and now complains of loss of vision which will
be-
a) Bitemporal hemianopia
b) Binasal hemianopia
c) Homonymous hemianopia
d) Superior temporal quadrantopia

Characteristically lesions at the level of the optic chiasm produce a bitemporal


hemianopia. Pituitary adenomas, which grow upward from the pituitary stalk,
compress the chiasm from below, which preferentially involves the inferior, nasal,
and macular nerve fibers.

208. Women presented with on and off headache. On lateral x-ray


suprasellar calcifications are seen and she has heteronymous
hemianopia. Diagnosis;
a) Craniopharyngioma
b) Pituitary macroadenoma
c) Astrocytoma
d) None

Craniopharyngiomas are benign tumors that grow near the pituitary gland. They
can develop as solid tumors or cysts (hollow sacs filled with fluid). Approximately 10
percent to 15 percent of pituitary tumors are craniopharyngiomas.
Craniopharyngioma is a rare type of noncancerous (benign) brain tumor.
Craniopharyngioma begins near the brain's pituitary gland, which secretes hormones
that control many body functions. As a craniopharyngioma slowly grows, it can affect
the function of the pituitary gland and other nearby structures in the brain

209. Which is not a contraindication for OPV?


a) Leukaemia
b) Diarrhoea
c) Malignancy
d) Immunocompromised

False Contraindications
 diarrhea.
 minor upper respiratory illnesses with or without fever.
 mild to moderate local reactions to a previous dose of vaccine.
 current antimicrobial therapy.
 the convalescent phase of an acute illness.
210. Unconscious patient, most common cause of respiratory obstruction;
a) Tongue fall
b) Head injury
c) Respiratory center block
d) Hyoid fracture

The tongue is the most common cause of upper airway obstruction, a situation seen
most often in patients who are comatose or who have suffered cardiopulmonary
arrest. Other common causes of upper airway obstruction include edema of the
oropharynx and larynx, trauma, foreign body, and infection.

211. Hypoxic hypoxia seen in


a) CO poisoning
b) HCN poisoning
c) Ischemia
d) AV shunt

A pulmonary shunt often occurs when the alveoli fill with fluid, causing parts of the
lung to be unventilated although they are still perfused. Intrapulmonary shunting is
the main cause of hypoxemia (inadequate blood oxygen) in pulmonary edema and
conditions such as pneumonia in which the lungs become consolidated.

212. A 32 years’ lady present with shoulder tip pain and diagnosis is
pan coast tumour and presents with meiosis. Diagnosis-
a) Horner syndrome
b) Thoracic outlet syndrome
c) Aberrant right subclavian artery
d) Upper trunk of brachial plexus injury

A disrupted nerve pathway on one side from the brain to the face and eye.
Horner's syndrome is the result of another medical condition, such as a stroke,
tumour or spinal cord injury.
Symptoms include a smaller pupil, drooping eyelid and little or no sweating on the
affected side.
There's no specific treatment for Horner's syndrome, but treatment for the underlying
cause may restore normal nerve function.

213. An 85 year old patient was brought to the ER, BP: 180/100, right
hemiparesis was seen. What is the next best step in management?
a) Reduce BP
b) NCCT
c) MRI
d) Aspirin 300mg and anticoagulants

Noncontrast head CT (NCCT) is the standard radiologic test for patients


presenting with acute stroke. Early ischemic changes (EIC) are often overlooked
on initial NCCT. We determine the sensitivity and specificity of improved EIC
detection by a standardized method of image evaluation

214. Central obesity is seen in;


a) Cushing’s disease
b) Chron’s disease
c) Conns disease
d) Celiac disease

Central obesity in Cushing's syndrome. Fat tissue redistribution in CS leads to


central obesity and metabolic complications. Visceral obesity is associated to
altered adipokine secretion further contributing to insulin resistance, inflammation
and fat accumulation.

215. Earliest finding in the diagnosis of diabetic nephropathy;


a) Microalbuminuria
b) Exudates
c) Hematuria
d) Macroalbuminuria

Microalbuminuria, urine albumin-to-creatinine ratio >30 and less than <300


mg/g has widely been accepted as biomarker of early diabetic kidney disease
(DKD).

216. Reversible dementia seen in


a) Alzheimer’s disease
b) Hypothyroidism
c) Subacute demyelination of spinal cord
d) Pick’s disease

Hypothyroidism is one of the most important causes of potentially reversible


dementia. The present case demonstrates a relatively rapid remission of cognitive
and noncognitive symptoms within a period of a few months.

217. All are included in celiac sprue diet except:


a) Wheat
b) Barley
c) Rye
d) Quinoa

Celiac disease, sometimes called celiac sprue or gluten-sensitive enteropathy, is an


immune reaction to eating gluten, a protein found in wheat, barley and rye. If you
have celiac disease, eating gluten triggers an immune response in your small
intestine.
218. All of the following are causes of hypercalcemia except
a) Hypervitaminosis of vit D
b) Hypepararthyroidism
c) Thiazides
d) Acute pancreatitis

Causes of hypercalcemia
 Overactive parathyroid glands (hyperparathyroidism). This most common
cause of hypercalcemia can stem from a small, noncancerous (benign) tumor
or enlargement of one or more of the four parathyroid glands.
 Cancer. Lung cancer and breast cancer, as well as some blood cancers, can
increase your risk of hypercalcemia. Spread of cancer (metastasis) to your
bones also increases your risk.
 Other diseases. Certain diseases, such as tuberculosis and sarcoidosis, can
raise blood levels of vitamin D, which stimulates your digestive tract to absorb
more calcium.
 Hereditary factors. A rare genetic disorder known as familial hypocalciuric
hypercalcemia causes an increase of calcium in your blood because of faulty
calcium receptors in your body. This condition doesn't cause symptoms or
complications of hypercalcemia.
 Immobility. People who have a condition that causes them to spend a lot of
time sitting or lying down can develop hypercalcemia. Over time, bones that
don't bear weight release calcium into the blood.
 Severe dehydration. A common cause of mild or transient hypercalcemia is
dehydration. Having less fluid in your blood causes a rise in calcium
concentrations.
 Medications. Certain drugs — such as lithium, used to treat bipolar disorder
— might increase the release of parathyroid hormone. Thiazides have several
metabolic effects contributing to higher serum calcium levels, but increased
renal tubular reabsorption of calcium resulting in reduced urine calcium
excretion is the most likely cause
 Supplements. Taking excessive amounts of calcium or vitamin D
supplements over time can raise calcium levels in your blood above normal.

219. Lady exposed to cold, extremities turn to blue associated with which
antibody;
a) Anti scl 70
b) ANA
c) anti ssb
d) anti ssa

Raynaud's disease causes smaller arteries that supply blood flow to the skin to
narrow in response to cold or stress. The affected body parts, usually fingers and
toes, might turn white or blue and feel cold and numb until circulation improves,
usually when you get warm
In a patient presenting with Raynaud phenomenon, a positive ANA test (even in the
absence of other symptoms) warrants more frequent follow-up, urinalysis and
perhaps referral to a rheumatologist.

220. PR interval in ECG means


a) Reflects conduction through AV node
b) Ventricular Depolarisation
c) Ventricular Repolarisation
d) Atrial repolarization

The PR interval is the time from the onset of the P wave to the start of the QRS
complex. It reflects conduction through the AV node

221. Lining of ventricles


a) Ependymal cells
b) Oligodendrocytes
c) Schwann cells
d) Astrocytes

Ependymal cells are mostly known as the cell type lining the brain ventricles. As
non-neuronal cells in the brain and derived from neuroectoderm, they are clearly
defined as a subtype of glial cells.

222. A patient is having continuous tremors and he has tendency to fall.


Lesion is most commonly seen in;
a) Substantia nigra
b) Globus pallidus
c) Caudate
d) Putamen

When the neurons in the substantia nigra are damaged in large numbers, the loss
of dopamine prevents normal function in basal ganglia and causes the motor
symptoms of PD: tremor, rigidity, impaired balance, and loss of spontaneous
movement.

223. Which type of gout is seen in a patient who is on treatment of CML?


a) Secondary gout
b) Pseudogout
c) Acute gout
d) Primary gout
Secondary gout refers to the presence of a recognized cause or precipitating
factor, such as lymphoma (especially following chemotherapy), the excessive use of
alcohol, or the use of diuretics. an acquired rise in the serum urate that is
directly. related to another disorder.
224. Vasopressin works on the principle of;
a) Hyposmolality
b) Hyperosmolality
c) Hypovolemia
d) Hypervolemia

Vasopressin neurons in the supraoptic nucleus of the hypothalamus sense blood


osmolarity. When osmolarity increases (e.g., after a meal, especially a salty one)
vasopressin is secreted, stimulating the kidney to reabsorb more water,
reducing urinary volume.

225. Which is not a component of brown sequard syndrome?


a) C/L pain
b) C/L temperature
c) C/L loss of proprioception
d) I/L paralysis

Brown-Séquard syndrome is a neurologic syndrome resulting from hemisection of


the spinal cord. It manifests with weakness or paralysis and proprioceptive
deficits on the side of the body ipsilateral to the lesion and loss of pain and
temperature sensation on the contralateral side.

226. Absent a waves in JVP is a feature of;


a) Atrial fibrillation
b) Atrial flutter
c) Ventricular tachycardia
d) Ventricular fibrillation

227. 20-year-old male came to clinic and following findings were seen, what
will be the diagnosis?
a) AML
b) ALL
c) CML
d) CLL

Acute myeloid leukemia (AML) is a cancer of the myeloid line of blood cells,
characterized by the rapid growth of abnormal cells that build up in the bone marrow
and blood and interfere with normal blood cell production.
Signs and symptoms of acute myelogenous leukemia include:
 Fever.
 Bone pain.
 Lethargy and fatigue.
 Shortness of breath.
 Pale skin.
 Frequent infections.
 Easy bruising.
 Unusual bleeding, such as frequent nosebleeds and bleeding from the gums.

228. What is the sequence of jaw thrust method?


a) Head tilt, chin lift, jaw thrust
b) chin lift, head tilt, jaw thrust
c) jaw thrust, chin lift, head lift
d) head lift, jaw thrust, chin lift

Head tilt, chin lift, jaw thrust is the correct sequence of jaw thrust method

229. What is the surgeon performing here?

a) Systolic pressure measurement on dorsalis pedis


b) Nerve conduction
c) Muscle tone
d) None
The surgeon is measuring systolic pressure measurement on dorsalis pedis

230. Endotracheal tube in the oesophagus is best assessed by;


a) CO2 Exhalation
b) Chest wall movement
c) Auscultation
d) chest tube passes through vocal cords

Capnography is the most reliable method to confirm endotracheal tube placement in


emergency conditions in the prehospital setting.
Capnography is the monitoring of the concentration or partial pressure of
carbon dioxide (CO. 2) in the respiratory gases. Its main development has been
as a monitoring tool for use during anesthesia and intensive care.

231. Identify the condition

a) Osteosarcoma
b) Ewings sarcoma
c) Osteoid osteoma
d) Osteochondroma

Osteosarcoma is a type of bone cancer that begins in the cells that form bones.
Osteosarcoma is most often found in the long bones — more often the legs, but
sometimes the arms — but it can start in any bone. In very rare instances, it occurs
in soft tissue outside the bone.
The sunburst appearance occurs when the lesion grows too fast and the
periosteum does not have enough time to lay down a new layer and instead
the Sharpey's fibers stretch out perpendicular to the bone. It is frequently
associated with osteosarcoma but can also occur with other aggressive bony lesions
232. What is the diagnosis of the given image?

a) Extradural haemorrhage
b) Subdural haemorrhage
c) Subarachnoid haemorrhage
d) Intracerebral haemorrhage

An extradural haematoma is a collection of blood in the 'potential' space


between the skull and the outer protective lining that covers the brain (the dura
mater). It usually occurs because of a head injury. It is a serious condition and
emergency treatment is needed. An operation to remove the haematoma may be
needed.
In almost all cases, extradural hematomas are seen on CT scans of the brain. They
are typically bi-convex (or lentiform) in shape, and most frequently beneath the
squamous part of the temporal bone. EDHs are hyperdense, somewhat
heterogeneous, and sharply demarcated

233. PA view of chest X-ray is given here. What is the diagnosis?

a) Right Pneumothorax with right tracheal shift


b) Right Pneumothorax with left tracheal shift
c) Left Pneumothorax with left tracheal shift
d) Left Pneumothorax with right tracheal shift

that if one side of the chest cavity has an increase in pressure (such as in the case
of a pneumothorax) the trachea will shift towards the opposing side. The trachea
is the tube that carries air from the throat to the lungs.
234. A boy was riding bicycle, he fell down forward and injury to shoulder
was seen, which nerve injury will be seen in the condition as given in X-
ray?

a) Radial
b) Ulnar
c) Median
d) Axillary

all nerves of the brachial plexus are at risk of injury during glenohumeral dislocation,
the most commonly injured is the axillary nerve

235. Post-menopausal women, fell down in washroom. What is the most


common fracture she may suffer?

a) Colles fracture
b) Smith fracture
c) Monteggia fracture
d) Galeazzi fracture

the Colles fracture is defined as a distal radius fracture with dorsal comminution,
dorsal angulation, dorsal displacement, radial shortening, and an associated fracture
of the ulnar styloid. The term Colles fracture is often used eponymously for distal
fractures with dorsal angulation

236. Football player with knee injury diagnosed as medial collateral


ligament injury. which structure is most commonly associated with this
type of injury?
a) Anterior cruciate ligament
b) Medial meniscus
c) Posterior Cruciate Ligament
d) Lateral lemniscus

A common injury in which forceful twisting causes certain tissue in the knee to tear.
A meniscus tear occurs in the rubbery knee cartilage that cushions the shinbone
from the thighbone. The meniscus can tear with forceful twisting or rotation of the
knee.
Pain, swelling, stiffness and difficulty extending the knee are symptoms.
Treatment includes rest, ice, pain relievers and physiotherapy. Less commonly,
surgery may be required.

237. Avascular necrosis of femur head is due to which of the following


artery?

a) Medial circumflex femoral artery


b) Lateral circumflex femoral artery
c) Obturator artery
d) Profunda femoris artery

The medial circumflex femoral artery (internal circumflex artery, medial femoral
circumflex artery) is an artery in the upper thigh that helps supply blood to the neck
of the femur. Damage to the artery following a femoral neck fracture may lead to
avascular necrosis (ischemic) of the femoral neck/head.

238. The shown apparatus is used for;

a) Ankle knee stabilizer


b) Patella tendon bearing brace
c) Thomas splint
d) None of the above
the brace was considered to off-load tibia, fibula, and foot bones by transmitting
the weight bearing loads via lateral upright
239. Initial finding seen in knee TB
a. Reduction of joint space
b. Narrowing of joint space
c. Degeneration of cartilage
d. None of the above

The Phemister triad was named after American orthopedic surgeon Dallas Burton
Phemister (1882–1951). The three features are juxta-articular osteopenia or
osteoporosis, peripheral osseous erosions, and gradual narrowing of joint spaces—
classically seen with tuberculous arthropathy.

240. A female presents with facial hair, secondary amenorrhea, increased


levels of androgen and deranged LH: FSH ratio. What is the diagnosis?
a) PCOS
b) Endometriosis
c) Androgen insensitivity
d) Testicular feminization syndrome

A hormonal disorder causing enlarged ovaries with small cysts on the outer edges.
The cause of polycystic ovary syndrome isn't well understood, but may involve a
combination of genetic and environmental factors.
Symptoms include menstrual irregularity, excess hair growth, acne and obesity.
Treatments include birth control pills to regularise periods, medication called
metformin to prevent diabetes, statins to control high cholesterol, hormones to
increase fertility and procedures to remove excess hair.

241. A child comes with the complaints of bed


wetting. Treatment of choice-
a) Bed alarm
b) Behavioral therapy
c) Counselling
d) Anti-depressants

Bed alarm is the treatment of choice for nocturnal


enuresis

242. Which is a product of purine metabolism?


a) Uric acid
b) Calcium
c) Phosphate
d) None of the above
Uric acid is the end product of purine metabolism in humans. Two-thirds of uric acid
in the human body is normally excreted through the kidney, whereas one-third
undergoes uricolysis (decomposition of uric acid) in the gut.

243. Giant cell pneumonia is associated with:

a) CMV
b) Herpes Simplex
c) Herpes Zoster
d) Measles

Giant cell pneumonia: A deadly but fortunately rare complication of measles that
tends to strike children who are immunodeficient from leukemia or AIDS. The lung
tissue shows multinucleated giant cells lining the alveoli (air sacs) of the lungs. Also
known as Hecht's pneumonia.

244. Investigation of choice for the following:

a) X Ray
b) CT
c) MRI
d) NCCT
coxa vara usually presents with a limp, a leg length difference, and limited
ability to bring the thigh out to the side. X-rays are used to diagnose coxa
vara. Depending on what caused the condition and how severe the
deformity, reconstructive surgery may be needed to improve the hip
alignment and biomechanics.

245. A 35-year-old anemic female patient came for family planning, she
has 2 living children and history of ectopic pregnancy what is your next
step:
a) Lippe’s Loop
b) Hysterectomy
c) T Piece
d) Progestasert

The Progestasert system combines the advantageous features of IUDs and oral
minidose progestogen preparations. An internal device continuously delivers
progesterone for 1 year to the uterine lumen and endometrium. A T-shaped
Progestasert which releases 65 mcg/day has been selected for wide scale clinical
use.

246. Reduviid bug transmits which disease:

a) Lyme disease
b) Chagas Disease
c) Anaplasmosis
d) Powassan Virus Disease

Chagas disease, also known as American trypanosomiasis, is caused by the


parasite Trypanosoma cruzi. It is transmitted to humans via the reduviid bug (the
“kissing bugs”), and is therefore characterized as a zoonotic disease.

247. Ectopic pregnancy lasts longer at which site:

a) Ampulla
b) Isthmus
c) Infundibulum
d) Interstitium

An ectopic pregnancy occurs when a fertilized egg grows outside of the uterus.
Almost all ectopic pregnancies—more than 90%—occur in a fallopian tube. As the
pregnancy grows, it can cause the tube to burst (rupture). A rupture can cause
major internal bleeding. Very late rupture of tubal pregnancy is seen when the
implantation is at Interstitium.

248. A female came with complain of severe abdominal pain since


morning. She mentions the pain
didn’t subside on medications. She
has a history of amenorrhea for 2
months. Following was viewed by
the surgeon. What is your
diagnosis:
a) Ectopic Pregnancy
b) Chocolate Cyst
c) Ovarian Cyst
d) None of the above

A pregnancy in which the fertilised egg implants outside the uterus.


The fertilised egg can't survive outside the uterus. If left to grow, it may damage
nearby organs and cause life-threatening loss of blood.
Symptoms include pelvic pain and vaginal bleeding.
To prevent complications, treatment is required. In the early stages, medication may
be sufficient. Later stages require surgery.

All the following leads to renal artery stenosis except:

a) Polyarteritis nodosa
b) Fibromuscular dysplasia
c) Atherosclerosis
d) Takayasu Arteritis

the two main causes of renal artery stenosis include:


 Buildup on kidney (renal) arteries. Fats, cholesterol and other substances
(plaque) can build up in and on your kidney artery walls (atherosclerosis). As
these deposits get larger, they can harden, reduce blood flow, cause kidney
scarring and eventually narrow the artery. Atherosclerosis occurs in many
areas of the body and is the most common cause of renal artery stenosis.
 Fibromuscular dysplasia. In fibromuscular dysplasia, the muscle in the
artery wall grows abnormally, often from childhood. The renal artery can have
narrow sections alternating with wider sections, giving a bead-like appearance
in images of the artery.

249. After cotrimoxazole administration patient develops hyperpigmentation


over back as shown below. What is your diagnosis?
a) Fixed Drug Eruption
b) DRESS Syndrome
c) Toxic Epidermal Necrolysis
d) Steven Johnson Syndrome

Fixed drug eruption (FDE) is a distinctive type of cutaneous drug reaction that
characteristically recurs in the same locations upon reexposure to the offending
drug.
Fixed drug eruption typically presents as a single (or small number of) well-
defined, round or oval red or violaceous patch or plaque which may blister or
ulcerate. It is usually asymptomatic but can be itchy or painful.

250. A young male presents with recurrent bouts of diarrhoea, two hours
after drinking milk. Diagnosis?

a) Gastrocolic Reflex
b) Lactase deficiency
c) Inflammatory Bowel Disease
d) IBS

The inability to fully digest sugar (lactose) in dairy products.


Lactose intolerance is usually caused by a deficiency of an enzyme in the body
called lactase.
Symptoms can include abdominal cramps, bloating and diarrhoea.
Treatment focuses on avoidance of dairy products, use of lactose-free products or
the use of lactase supplements.

251. Cotrimoxazole is given for management in all of the following except?

a) Actinomycosis
b) Toxoplasma Gondii
c) Pneumocystis Jiroveci
d) Isospora Belli

In most cases of actinomycosis, antimicrobial therapy is the only treatment required,


although surgery can be adjunctive in selected cases. Penicillin G is the drug of
choice for treating infections caused by actinomycetes

252. Which of the following has a late diastolic murmur?

a) Atrial regurgitation
b) Atrial Myxoma
c) Pulmonary regurgitation
d) Pulmonic Stenosis
Late diastolic (presystolic) murmurs start after S2 and extend up to S1 and have a
crescendo configuration. They can be associated with AV valve narrowing. They
include mitral stenosis, tricuspid stenosis, myxoma, and complete heart block.

253. A 3-year-old patient is having loose motions with weight loss. Small
intestinal biopsy was done. Which of the following tests is best suited for
clinical diagnosis of the patient?

a) Anti-Mitochondrial AB
b) Anti-Gliadin AB
c) Anti-SCL 70 AB
d) Anti-Microsomal AB

A gliadin antibodies test is used to help doctors diagnose celiac disease. Celiac
disease is an autoimmune disorder in which the body's immune system mistakenly
thinks that gluten — a protein in wheat, barley, rye, and oats

254. A 40-year old smoker patient has been diagnosed with infra-renal
aorta aneurysm of 4 cm. What is the best treatment?

a) Urgent Resection and Anastomosis


b) Intraluminal Graft Placement
c) Endoscopy
d) Periodic Size Measurement

255. A 30-year old patient is having weakness due to secretory diarrhoea.


Endoscopy shows large duodenal ulcer. Probable diagnosis is?

a) Gastrinoma
b) Primitive Neuroectodermal Tumour
c) Carcinoid Syndrome
d) Autoimmune Gastritis

A gastrinoma is a gastrin-secreting tumor that can occur in the pancreas, although


it is most commonly found in the duodenum
The signs and symptoms of gastrinomas are related to increased gastric acid
production, this is caused by the excess gastrin hormone that is produced by the
tumour cells. Most patients experience abdominal pain, heartburn, nausea,
diarrhoea, blood in the vomit and/or bleeding from the back passage.

256. Patient presents with MI, STK (streptokinase) was given. What should be
checked in order to monitor toxicity?

a) Bleeding Time
b) Fibrin degradation products
c) Platelet Count
d) Thrombin Time
Careful monitoring for arrhythmia is recommended during and immediately following
administration of Streptase, Streptokinase, for acute myocardial infarction.
Occasionally, tachycardia and bradycardia have been observed.

257. In which of the following conditions JVP increases inspiration?

a) Pericardial Effusion
b) Cardiac Tamponade
c) Aortic Dissection
d) Constrictive Pericarditis

Elevated jugular venous pressure (JVP) is present in virtually all patients that are not
hypovolemic. Pericardial constraint results in the inability of the right heart to
accommodate inspiratory abdominal venous return, translating to an inspiratory
increase in the JVP (Kussmaul's sign)

258. What is the best treatment for a 35-year old AIDS positive patient with
Cryptosporidium Diarrhoea?

a) Nitazoxanide
b) Metronidazole
c) Parmomycin
d) Cotrimoxazole

Nitazoxanide, paromomycin, and azithromycin have activity against


Cryptosporidium. A 3-day course of nitazoxanide oral suspension has been
approved by the FDA for the treatment of cryptosporidiosis-related diarrhea in adults
and in children older than 12 months who do not have HIV infection.

259. Which of the following is Monosynaptic reflex?

a) Withdrawal Reflex
b) Crossed Extensor Reflex
c) Peristaltic Reflex
d) Stretch Reflex

The monosynaptic stretch reflex, or sometimes also referred to as the muscle stretch
reflex, deep tendon reflex, is a reflex arc that provides direct communication
between sensory and motor neurons innervating the muscle. ... This contraction
allows the muscle to resist the force that initially caused the reflex.
FMGE AUG 2020

1 . Choose the correct sequence of muscle contraction in ejaculation:


a ) Ischiocavernosus contraction → urethralis→ Bulbospongiosus
b ) Ischiocavernosus contraction → Bulbospongiosus → urethralis
c ) Bulbospongiosus → urethralis → Ischiocavernosus
d ) Bulbospongiosus Ischiocavernosus → urethralis

During the emission phase, the muscles around the epididymis and ductus
deferens (the tube extending from the epididymis) contract to push the sperm into
the prostate and urethra. During ejaculation, the semen is expelled by strong
spasmodic contractions of the bulbocavernosus muscle, which encircles the corpus
spongiosum (the structure in the penis that encloses the urethra). The whole process
of ejaculation is accomplished by nerve impulses received from the penis; once
ejaculation is started it becomes a reflex reaction that cannot be voluntarily
interrupted.

2 . Deep inguinal ring is a defect in:


a ) External oblique muscle aponeurosis
b ) Internal oblique muscle aponeurosis
c ) Transversus abdominis muscle aponeurosis
d ) Transversalis fascia

The deep (internal) inguinal ring is a defect in the transversus abdominis fascia that
allows the contents to enter the canal and leave the abdomen before extending
obliquely, medially and inferiorly through the canal to exit at the superficial (external)
inguinal ring, a defect in the external oblique fascia.

3 . Name the skin receptor responsible for detecting rapid vibration sense:
a ) Meissner corpuscle
b ) Paccinian corpuscle
c ) Merkel cell
d ) Ruffini's corpuscle

The Pacinian corpuscle or Vater-Pacinian corpuscles or Lamellar corpuscles in the


skin and fascia detect rapid vibrations of about 200–300 Hz.

4 . Which type of joint is present at the arrow mark shown in the diagram:

a) Primary cartilaginous
b) Secondary cartilaginous
c) Fibrous
d) Synovial
Secondary cartilaginous joint
 symphysis pubis between the right and left pubic bones.
 manubriosternal joint between the sternal body and the manubrium.
 intervertebral discs.
 sacrococcygeal symphysis.

5 . A lesion in left sided motor nucleus of facial nerve leads to:


a) Right sided loss of wrinkles on forehead and right side corner of mouth goes
down
b) Right sided loss of wrinkles on forehead and right side corner of mouth goes
up
c) Left sided loss of wrinkles on forehead and left side corner of mouth
goes down
d) Left sided loss of wrinkles on forehead and right side corner of mouth goes up

For example, a left corticobulbar lesion results in paralysis of the muscles that
control the lower right quadrant of the face. By contrast, a lower motor neuron lesion
to the facial motor nucleus results in paralysis of facial muscles on the same side
of the injury.

6. Structure developing in dorsal mesogastrium:


a) Lesser omentum
b) Falciform ligament
c) Spleen
d) Liver

The liver develops in the ventral mesogastrium; the spleen develops in the dorsal
mesogastrium. The liver grows rapidly, pressing against the body wall, and
obliterating these layers of peritoneum. These changes produce this almost separate
pocket behind the stomach, the lesser sac.

7. A patient developed the following deformity, after overhead weight-


lifting exercises. Which of the following muscle is
involved?

a) Latissimus dorsi
b) Trapezius
c) Serratus anterior
d) Supraspinatus
Scapular winging is almost always caused by damage to one of three nerves that
control muscles in your arms, back, and neck: the long thoracic nerve, which controls
the serratus anterior muscle.

8. Contralateral homonymous hemianopia with macular sparing occurs


due to lesion at the level of?
a) Optic nerve
b) Optic tract
c) Optic radiation
d) Orbital cortex

Homonymous hemianopsia can be congenital, but is usually caused by brain injury


such as from stroke, trauma,[5] tumors, infection, or following surgery.
Vascular and neoplastic (malignant or benign tumours) lesions from the optic tract,
to visual cortex can cause a contralateral homonymous hemianopsia. Injury to the
right side of the brain will affect the left visual fields of each eye. The more posterior
the cerebral lesion, the more symmetric (congruous) the homonymous hemianopsia
will be.

9. Posterior tibial artery pulse is felt:


a) Between medial malleolus and medial border of tendo-calcaneus in
inversion
b) Between medial malleolus and medial border of tendo-calcaneus in eversion
c) Between medial malleolus and lateral border of tendo-calcaneus in inversion
d) Between medial malleolus and lateral border of tendo-calcaneus in inversion

The posterior tibial artery enters the sole of the foot by passing deep to the flexor
retinaculum. Its pulsation can be felt midway between the medial malleolus and
the medial border of the tendocalcaneus. The pulsation of the peroneal artery is
felt in front of the lateral malleolus at its medial border.

10. Patient underwent removal of left submandibular salivary gland, and


lingual nerve was damaged. Which of the following statements is NOT correct?
a) Sublingual gland secretions are reduced
b) Previous 2 / 3rd tongue taste sensation is lost
c) Sensation at the floor of mouth lost
d) Tongue deviated to left side
If your lingual nerve sustains an injury, you'll most likely experience any of these
nerve damage symptoms: Changed sensation in the tongue, chin, or lower lip areas
(similar to sensations you feel when your oral cavity is numbed for a dental
procedure or as the anesthesia slowly wears off) Altered ability to taste. If your
lingual nerve sustains an injury, you'll most likely experience any of these nerve
damage symptoms: Changed sensation in the tongue, chin, or lower lip areas
(similar to sensations you feel when your oral cavity is numbed for a dental
procedure or as the anesthesia slowly wears off) Altered ability to taste.

11. Ductus arteriosus develops from which pharyngeal arch artery?


a) 3
b) 4
c) 5
d) 6

Ductus arteriosus itself arises from the left 6th pharyngeal arch artery.

12. Which enzyme deficiency is seen in TaySach's disease?


a) Hexosaminidase A
b) Glucocerebrosidase
c) G-6-PD
d) Hexosaminidase A & B

Tay-Sachs disease is a rare, neurodegenerative disorder in which deficiency of an


enzyme (hexosaminidase A) results in excessive accumulation of certain fats (lipids)
known as gangliosides in the brain and nerve cells.

13. An Alcoholic patient presented with mental confusion, gait problem and
ophthalmoplegia. Which vitamin deficiency is seen?
a) B1
b) A
c) B9
d) B12

Wernicke-Korsakoff syndrome is a condition caused by a deficiency of thiamine or


vitamin B1. Patients classically present with a clinical triad of ophthalmoplegia,
altered mental status, and ataxia.
14. A female patient with macrocytic anaemia, Homocystinuria & normal
methylmalonate levels. Which vitamin therapy can help in this?
a) Folate plus Vitamin B12
b) Folate
c) Vitamin A
d) Vitamin C
Vitamin B12 or B9 (commonly called folate) deficiency anaemia occurs when a lack
of vitamin B12 or folate causes the body to produce abnormally large red
blood cells that cannot function properly. Red blood cells carry oxygen around the
body using a substance called haemoglobin. Additional testing with homocysteine
and MMA determinations may help distinguish between B12 and folate deficiency
states. In folate deficiency, homocysteine levels are elevated and MMA levels are
normal. In vitamin B12 deficiency, both homocysteine levels and MMA levels are
elevated.

15. Macrocytic anaemia case with normal methyl-malonyl with increased level
of homocysteine. What is the diagnosis?
a) B9 deficiency
b) B12, 9 deficiency
c) Iron deficiency
d) Anaemia of chronic ds

Vitamin B12 or B9 (commonly called folate) deficiency anaemia occurs when a lack
of vitamin B12 or folate causes the body to produce abnormally large red
blood cells that cannot function properly. Red blood cells carry oxygen around the
body using a substance called haemoglobin. Additional testing with homocysteine
and MMA determinations may help distinguish between B12 and folate deficiency
states. In folate deficiency, homocysteine levels are elevated and MMA levels are
normal. In vitamin B12 deficiency, both homocysteine levels and MMA levels are
elevated.

16. A heart patient suddenly has chest pain. Which isoenzyme of LDH enzyme is
elevated?
a) LDH -1
b) LDH -5
c) LDH -2
d) LDH -4

It is known that the ratio of isoenzyme 1 to total lactate dehydrogenase (LDH1) in


serum is increased in all patients with acute myocardial infarction within 24 h of the
infarct.
17. Cofactor for Mitochondrial SOD (Superoxide Dismutase):
a) Mn
b) Cu
c) Zn
d) Mb
Manganese superoxide dismutase (MnSOD), an important primary antioxidant
enzyme located in mitochondria, is essential for the removal of superoxide radicals
constantly being generated by the electron transport chain (ETC).

18. Nitric oxide is synthesized from:


a) L-Hydroxy proline
b) Arginine
c) D-Histidine
d) Cysteine
Nitric oxide (NO), synthesized from the amino acid l-arginine by NO synthases, is a
unique type of transmitter in the nervous system.

19. Which of the following is involved in protein folding?


a) Chaperone
b) Ubiquitin
c) Proline
d) Arginine
Chaperones are a group of proteins that have functional similarity and assist in
protein folding. They are proteins that have the ability to prevent non-specific
aggregation by binding to non-native

20. Which of the following is not given in a case of fatty liver disease?
a) Choline
b) Ethanol
c) Folic acid
d) Methionine

Recent studies indicate that additional effects of ethanol impair fat oxidation as well
as stimulate lipogenesis. The effects of ethanol on lipid metabolism result
from inhibition of PPAR-α and stimulation of SREBP-1, resulting in metabolic
remodelling of the liver toward a fat-storing, rather than fat oxidizing organ.

21. 16 Years old girl was kidnapped. Police caught the accused and brought to
the police station. After some time, the accused was found dead in the station.
Inquest in this case should be done by?
a) Executive Magistrate.
b) Judicial Magistrate.
c) Police.
d) No inquest needed.

Death under police custody case is inquested by Judicial Magistrate, also he


inquests
Death in a psychiatric hospital.
Death in jail.
Death as a result of police shooting.

22. Dowry Death inquest is according to which CrPC?


a) 174
b) 175
c) 176
d) 304-B
According to Section 304-B, to make out a case of dowry death, a woman should
have died of burns or other bodily injuries or “otherwise than under normal
circumstances” within seven years of her marriage.

23. Identify the fingerprint pattern shown in image

a) Loop
b) Whorl
c) Arch
d) Composite
24. Identify the test that shows the following crystals.

a) Teichman
b) Takayama
c) Florence
d) Barberios

Takayama test is a confirmation test used to detect blood spots. Based on the
research results this test can still be used to identify dried blood spots on clothing
aged 20 years, and able to detect positively the presence of blood with the formation
of pink crystals.

25. Which of the following is a true statement about dying declaration?


a) If victim survives it is as such inadmissible
b) Oath is administered while recording it
c) Ideal person to record DD is the attending doctor
d) Alteration of terms / phrases of dying person is allowed

A dying declaration is considered credible and trustworthy evidence based upon the
general belief that most people who know that they are about to die do not lie. ... A
person who makes a dying declaration must, however, be competent at the time he
or she makes a statement, otherwise, it is inadmissible.

26. In regard to a fetal death classification, a test was done to determine the
cause which includes dipping the lung in to the water. Identify the test.
a) Foderes
b) Ploquets
c) Hydrostatic
d) Wredins

The hydrostatic test, or floatation test, has historically been used to determine
whether a newborn infant has breathed. It is performed by placing the lungs, still
attached to the heart, in water and seeing whether they float.

27. A patient is brought to casualty with history of snake bite with features
as shown. During course of Treatment, the patient expires. The accused
snake’s photo was captured by relatives as shown in image. What could be the
cause of death in this case?

a) Shock
b) Respiratory paralysis
c) Circulatory failure
d) Rhabdomyolysis

The venom of common kraits contains 3 major neurotoxins (α, β and κ-bungarotoxin)
that can induce failure of neuromuscular transmission resulting in respiratory
muscles paralysis.

28. Identify the method of solvent abuse done with use of a plastic bag
applied to mouth & nose.

a) Bagging
b) Huffing
c) Glading
d) Sniffing
"bagging" — sniffing or inhaling fumes from substances sprayed or deposited inside
a plastic or paper bag

29. Sexual gratification obtained through obscene telephone calls is called


as;
a) Scophophilia
b) Scoptophilia
c) Scatologia
d) Voyeurism

Scatologia, also called Coprolalia, deviant sexual practice in which sexual


pleasure is obtained through the compulsive use of obscene language. The affected
person commonly satisfies his desires through obscene telephone calls, usually to
strangers.

30. A person was found dead in his apartment and the forensic team
observed the scene and gave these noted details regarding injury that caused
death. The injury over the vertebra has clean cut margins but the bones
appeared to be crushed. What could be the correct option in this scenario?
a) Incised
b) Chop
c) Lacerated
d) Stab

31. Rotten egg odour is associated with


a) Arsenic
b) Hydrogen sulphide
c) Aluminium phosphide
d) Phosphorus

Hydrogen sulphide (H2S) is a colourless gas with a characteristic odour of rotten


eggs which being denser than air may pool in low areas in still conditions.

32. Minimum number of Antenatal visits required are;


a) 3
b) 4
c) 5
d) 6
For women whose pregnancies are progressing normally, WHO recommends a
minimum of four ANC visits, ideally at 16 weeks, 24-28 weeks, 32 weeks and 36
weeks

33. A man came for health check-up after his father had a cerebrovascular
accident who died last month. He was having the history of hypertension.
Which type of prevention is this?
a) Primordial
b) Primary
c) Secondary
d) Specific protection

Secondary Prevention - trying to detect a disease early and prevent it from


getting worse. Tertiary Prevention - trying to improve your quality of life and reduce
the symptoms of a disease you already have.

34. Mid-year population is taken on;


a) 01 September
b) 01June
c) 01 April
d) 01 July

The date of the midyear population is July 1. Note that the midyear month is
different from the census month. These differences are taken into account when
calculating.

35. Identify the Statistical diagram shown below

a) Histogram
b) Bar diagram
c) Frequency polygon
d) Scatter diagram
Histogram is a diagram consisting of rectangles whose area is proportional to the
frequency of a variable and whose width is equal to the class interval.

36. Identify the Logo shown below?

a) AGMARK
b) PFA standards
c) Food Standards and Safety Authority of India (FSSAI)
d) Bureau of Indian Standards (BIS)

BIS is the National Standard Body of India established under the BIS Act 2016 for
the harmonious development of the activities of standardization, marking and
quality certification of goods and for matters connected therewith or incidental
thereto.

37. Regarding the School health service guidelines recommendations,


which of the following statements is true?
a) Minimum area 5 sq. ft. per student
b) Minus type desk
c) Can allot 60 students per classroom
d) Light coming from front of the expressed glass

38. Researcher was conducting study in relation of Depression associated


with history of social media usage. One group had social media users with
depression and another group had social media users without depression.
Which type of study design is used?
a) Cohort study
b) Case control study
c) Cross sectional study
d) Random sampling
A cross-sectional study involves looking at data from a population at one specific
point in time. The participants in this type of study are selected based on particular
variables of interest.
39. Delphi technique used for collective opinion of;
a) Population
b) Group
c) Community
d) Individual
The Delphi method is a process used to arrive at a group opinion or decision by
surveying a panel of experts. ... The experts can adjust their answers each round,
based on how they interpret the "group response" provided to them. The ultimate
result is meant to be a true consensus of what the group thinks.

40. A Researcher did study on staff, nurses and junior resident doctors
doing yoga (Divided into male and female). Yoga shows 25% decreased risk of
infections. Which is the best test to test the significance of the result?
a) ANOVA
b) Chi square test
c) Z test
d) Correlation
chi-square test
relating to or denoting a statistical method assessing the goodness of fit between
observed values and those expected theoretically. Here the researcher has 2
groups where 25% decreased risk of infections when they do yoga i.e. 25% less
than the risk expected theoretically.

41. A Researcher undertakes follow-up study for a certain time period


duration, and tells that the people increasingly develop the disease with time,
then it expressed as;
a) Incidence
b) Cumulative incidence
c) Prevalence
d) Relative risk

Cumulative incidence is calculated as the number of new events or cases of


disease divided by the total number of individuals in the population at risk for
a specific time interval. Researchers can use cumulative incidence to predict risk
of a disease or event over short or long periods of time.

42. All of following are Principles of Primary Health Care Except?


a) Community participation
b) Appropriate facility
c) Intersectoral coordination
d) Equitable distribution
Five (5) Principles of Primary Health Care (PHC):
 Social equity.
 Nation-wide coverage/wider coverage.
 Self- reliance.
 Intersectoral coordination.
 People's involvement (in planning and implementation of programs)

43. In different types of graphs, Secular trend of a Disease is best


represented by;
a) Line diagram
b) Bar graph
c) Box and whiskers plot
d) Stem leaf plot

The secular trend describes the occurrence of disease over a prolonged period,
usually years; it is influenced by the degree of immunity in the population and
possibly nonspecific measures such as improved socioeconomic and nutritional
levels among the population and are usually represented by line diagram.

44. Which of the following depicts Severity / lethality of disease?


a) Proportional mortality rate
b) Case specific death rate
c) Case fatality rate
d) Total deaths due to that disease

Case fatality rate, also called case fatality risk or case fatality ratio,
in epidemiology, the proportion of people who die from a specified disease among all
individuals diagnosed with the disease over a certain period of time. Case fatality
rate typically is used as a measure of disease severity and is often used for
prognosis (predicting disease course or outcome)

45. In a hospital, systematic observation and recording of doctors spending


time with patients care and time without patients’ care is calculated for future
management purposes. The type of management technique used here is?
(a) Network analysis
(b) System analysis
(c) Work sampling
(d) Decision making
Work sampling is the statistical technique used for determining the proportion of
time spent by workers in various defined categories of activity (e.g. setting up a
machine, assembling two parts, idle…etc.) It is as important as all other statistical
techniques because it permits quick analysis, recognition, and enhancement of job
responsibilities, tasks, performance competencies, and organizational work flows

46. Definition of blindness, as per WHO;


a) <1/60 in better eye
b) <1/60 in worse eye
c) <3/60 in better eye
d) <3/60 in worse eye

'Blindness' is defined as visual acuity of less than 3/60, or a corresponding visual


field loss to less than 10°, in the better eye with the best possible correction.

47. A 8-month-old child reported to a PHC with runny nose and fever since
last 3 days. Rash started on face and then covered the trunk part of body.
What's the most likely diagnosis?
a) Measles
b) Chicken pox
c) Mumps
d) Rubella

Measles symptoms appear 7 to 14 days after contact with the virus and typically
include high fever, cough, runny nose, and watery eyes. Measles rash appears 3 to
5 days after the first symptoms.

48. Which of the following is NOT constrained by Time or Existing


resources?
a) Vision
b) Target
c) Goal
d) Mission

Goal is not constrained by Time or Existing resources

49. Newborn Health Mission component do not include;


a) Target of infant mortality to double digit by 2030
b) Antenatal care
c) Postnatal visits
d) Care during labor
NHM has six financing components:
 NRHM-RCH Flexipool.
 NUHM Flexipool.
 Flexible pool for Communicable disease.
 Flexible pool for Non communicable disease including Injury and Trauma.
 Infrastructure Maintenance and.
 Family Welfare Central Sector component.

50. Identify the Contraceptive shown in the Photograph given below

a) CuT 7
b) CuT 220B
c) CuT 380A
d) NOVA-T

CuT 380A - It is a T shaped device with a polyethylene frame holding 380 mm2 of
exposed surface area of copper. The IUD frame contains barium sulfate thus making
it radio-opaque.

51. If you are posted as a Medical officer in an area under NVBDCP, and you
will be advised to spray Malathion for malaria prophylaxis. What could be the
probable decision you will take regarding frequency of Malathion spray?
a) Once very fortnight
b) Once every 3 months
c) Twice every3 months
d) Once every month
Frequency of malathion spray under NVBDCP is once every 3 months.
52. A country is suspecting a severe disease. What's the probable time
within which one should report it to WHO?
a) 6 hours
b) 12 hours
c) 24 hours
d) 48 hours

the probable time within which one country should report it to WHO is 24 hrs

53. Least Iron content is present in;


a) Beans
b) Milk
c) Spinach
d) Liver

Both cow's milk and breastmilk are low in iron (around 0.2-0.5 mg iron per litre),
however the iron in breastmilk is bonded to lactoferrin and offers greater
bioavailability.

54. In an operation theatre, the nurse spilled blood on the floor. What can be
used to disinfect it?
a) Cresol
b) 70% Ethyl alcohol
c) Glutaraldehyde
d) 1% Sodium hypochlorite

Wipe the area with water and detergent until it is visibly clean. Saturate the area
again with sodium hypochlorite 1% (10 000 ppm available chlorine). This is a 1:10
dilution of 5.25% sodium hypochlorite bleach, which should be prepared daily. Rinse
off the tongs, brush and pan, under running water and place to dry.

55. In a train accident, there were 74 dead, 64 were severely injured, 20


moderately injured and 32 mildly injured. Color coding in Triage Highest to
Lowest category is
a) Green Yellow Red Black
b) Yellow Red Green Black
c) Black Yellow - Red Green
d) Red - Yellow - Green – Black
RED: (Immediate) severe injuries but high potential for survival with treatment; taken
to collection point first.
YELLOW: (Delayed) serious injuries but not immediately life-threatening.
GREEN: (Walking wounded) minor injuries
BLACK: morbid patients

56. A healthy person get in contact with an infected case, and they have to
separate him not more than the incubation period of disease. This is called as;
a) Absolute quarantine
b) Isolation
c) Interruption of transmission
d) Serial interval

Absolute or complete quarantine: The limitation of freedom of movement of those


exposed to a communicable disease for a period of time not longer than the
longest usual incubation period of that disease, in such manner as to prevent
effective contact with those not so exposed.

57. A male underwent Vasectomy and after 3 months his wife got pregnant.
What advice should have been given to him post-vasectomy?
a) Abstinence for 3 months after the procedure
b) Usage of Barrier methods for 3 months after the procedure
c) OCPS usage for I month
d) Female condom usage for 1 month

Sperms can be seen in semen for up to 3 months’ post ligation, so repeat semen
analysis after 3 months and if azoospermia recorded then he can have
unprotected intercourse

Post-procedural semen analysis at 8 to 16 weeks to document sterility.

58. First investigation for TB Diagnosis?


a) Chest X-ray
b) Sputum smear examination
c) CB-NAAT
d) Liquid culture

A definitive diagnosis of tuberculosis can only be made by culturing Mycobacterium


tuberculosis organisms from a specimen taken from the patient (most often sputum,
but may also include pus, CSF, biopsied tissue, etc.). A diagnosis made other than
by culture may only be classified as "probable" or "presumed". For a diagnosis
negating the possibility of tuberculosis infection, most protocols require that two
separate cultures both test negative

59. In ORS, Sodium is given along with Glucose for


a) Reducing secretions
b) Osmosis
c) Facilitative diffusion
d) Co-transport

Composed of glucose and sodium and other electrolytes, ORS promote sodium and
thus water absorption in part via passive sodium-coupled glucose transport in
intestinal villi.

60. A factory worker had a history of frequent exposure to groundnuts.


Subsequently he develops Hepatocellular carcinoma. Most likely exposure
association is
a) Ergot toxin
b) Aflatoxin
c) Sanguinarine
d) Pyrrolizidine alkaloids

Aflatoxin Alert: Moldy Nuts and Corn Increases Liver Cancer Risk 60-Times If You
Have Hepatitis B. One of the biggest health threats to people living with chronic
hepatitis B is a toxic, nearly invisible mold called aflatoxin found in corn, peanuts,
peanut butter, almonds, Brazil nuts, walnuts and pistachios.

61. A Medical officer examines workers in mines and suspected Silicosis.


What should be his comments?
a) It is reversible once we remove the cause
b) It takes 6-10 years to develop
c) Fibrotic changes in the lungs of the patient can be reversed
d) TB screening should be done at regular intervals

long time workers in mines have high risk of TB and can be suspected to have
silicosis, So TB screening should be done at regular intervals.

62. A person returning from malaria endemic country should continue


chemoprophylaxis for how long?
a) 1 week
b) 15 days
c) 4 weeks
d) 4 months

All prophylactic drugs should be taken with unfailing regularity for the duration of the
stay in the malaria risk area, and should be continued for 4 weeks after the last
possible exposure to infection since parasites may still emerge from the liver during
this period.

63. An infant is given Buffalo milk. It is superior because;


a) High calories and high protein
b) Low calories and high protein
c) High calories and low protein
d) Low calories and low protein

Buffalo milk is rich in vitamin A, has a higher protein value and contains more iron,
calcium and phosphorus than the cow's milk. ... It is better to introduce buffalo milk
slowly.

64. Long term changes / sequelae of a disease are seen in


a) Secular trend of a disease
b) Cyclical trend of a disease
c) Disease changing its traits according to herd immunity in the population
d) Disease changing the symptoms as per seasons

Long term changes of a disease are seen in secular trend. The secular trend
describes the occurrence of disease over a prolonged period, usually years; it is
influenced by the degree of immunity in the population and possibly nonspecific
measures such as improved socioeconomic and nutritional levels among the
population.

65. A 5-year-old child came into OPD with fever, rashes on the body. There
were rashes on the axilla and flexor surface with various macules, papules and
vesicles. Rashes and blisters are in different stages and non- uniform. Most
probable diagnosis is?
a) Chickenpox
b) Smallpox
c) Epstein Barr
d) Measles
Chickenpox is an infection caused by the varicella-zoster virus. It causes an itchy
rash with small, fluid-filled blisters. Chickenpox is highly contagious to people who
haven't had the disease or been vaccinated against it

66. Regular monitoring of visual acuity and fundus examination is required


with the patient is being treated with which of the following DMARD?
a) Hydroxychloroquine
b) Methotrexate
c) Sulfasalazine
d) Leflunomide

All individuals who have taken hydroxychloroquine for greater than 5 years should
receive annual screening for retinopathy. All individuals who have taken
chloroquine for greater than 1 year should receive annual screening for retinopathy

67. A female presented with burning micturition and symptoms of UTI.


Which fluoroquinolones is least likely to be effective?
a) Ofloxacin
b) Moxifloxacin
c) Ciprofloxacin
d) Levofloxacin

Gemifloxacin and moxifloxacin are not effective for the treatment of UTIs because
they do not achieve adequate concentrations in the urine. 5-7 The fluoroquinolones
indicated for UTI are excreted in the urine as greater than 40% unchanged drug
while gemifloxacin is less than 35% and moxifloxacin is only 20%.

68. A patient with heart disease presented with breathlessness the features
of the patient shown in images. Which of the following is used to treatment of
oedema for this patient?
a) ACE Inhibitor
b) Na – K – 2Cl symporter inhibitor in loop of Henle
c) Na – Cl symporter inhibitor in DCT
d) Aldosterone in DCT

Furosemide is a loop diuretic (water pill) that prevents your body from absorbing
too much salt. This allows the salt to instead be passed in your urine. Furosemide
is used to treat fluid retention (edema) in people with congestive heart failure, liver
disease, or a kidney disorder such as nephrotic syndrome.

69. A patient with pancreatic carcinoma is on morphine for treatment of


pain. After a few days, the pain is not relieved as much as earlier. What can be
the likely reason for this?
a) Decrease in absorption of morphine
b) Pain intensity has increased
c) Opioid receptors are up regulated
d) Desensitization of Opioid receptors

Chronic opioid use leads to tolerance, defined as a decrease of the drug response.
It's possible to reproduce in vitro such phenomenon when cellular models expressing
OR are exposed to agonists; in that situation, a decrease of signaling is observed
and is designated as OR desensitization. Some reports distinguish the OR
desensitization from the cellular tolerance.

70. A female patient presented with pain and redness in great toe. Serum
uric acid level is 9.4 mg/dL. The physician prescribed a drug for the treatment.
Which of the following enzyme is inhibited by this drug?
a) Xanthine oxidase
b) Thymidylate synthase
c) phosphoribosyl transferase
d) DHFR

Xanthine oxidase inhibitors are primarily used in the clinical prevention and
treatment of gout associated with hyperuricemia. The archetypal xanthine
oxidase inhibitor, Allopurinol has been shown to have other beneficial effects such as
a reduction in vascular reactive oxygen species and mechano-energetic uncoupling.

71. A patient on anticancer therapy developed infection. Total leucocytes


count of this patient was 2000/mm3, which of the following drug is effective?
a) Oprelvekin
b) Filgrastim
c) Erythropoietin
d) Romiplostim

Filgrastim is a haematopoietic growth factor that primarily acts to stimulate the


proliferation and differentiation of neutrophil progenitor cells. Filgrastim is capable of
reducing the incidence and severity of neutropenia and the complications that
accompany it in patients with cancer or HIV infection.

72. A person presents with acute chest pain. Nitroglycerine was given
sublingually and the pain relieved within 5 minutes. what is the mechanism of
the drug?
a) Release of nitric oxide
b) Release of Endothelin
c) Calcium channel Blocker
d) Beta channel Blocker

nitroglycerin converts to nitric oxide (NO) in the body. NO then activates the enzyme
guanylyl cyclase, which converts guanosine triphosphate (GTP) to guanosine 3',5'-
monophosphate (cGMP) in vascular smooth muscle and other tissues. cGMP then
activates many protein kinase-dependent phosphorylations, which enhances the
reuptake of calcium into the sarcoplasmic reticulum, increases extracellular calcium,
and opens the calcium-gated potassium channel. This ultimately results in the
dephosphorylation of myosin light chains within smooth muscle fibers. This activity
causes the relaxation of smooth muscle within blood vessels, resulting in the desired
vasodilatory effect.

73. A 60-year-old man present to OPD with low backache. His PSA levels are
100 ng/ml. Which of the following drug is indicated?
a) Somatostatin
b) Goserelin
c) Terlipressin
d) Testosterone

Goserelin is a parenterally administered, gonadotropin releasing hormone (GnRH)


agonist which causes an inhibition of estrogen and androgen production and is
used predominantly to treat prostate cancer.

74. A patient with resting tremor was diagnosed to be of parkinsonism.


Which of the following therapy is not indicated?
a) Pramipexole
b) Rivastigmine
c) Dee Brain Stimulation
d) Levodopa + Carbidopa

Rivastigmine is now widely approved for the treatment of mild to moderately severe
dementia in Parkinson's disease (PDD). However, since anticholinergic drugs have a
role in the management of tremor in patients with Parkinson's disease (PD),
concerns have been raised that the use of cholinergic drugs might worsen PD

75. A Pregnant female presented with burning micturition. A diagnosis of


cystitis was made. which of the following drug will be useful for the treatment?
a) Amoxicillin
b) TMP - SMX
c) Ciprofloxacin
d) Doxycycline

Those with asymptomatic bacteriuria can be treated with a single dose of an


antimicrobial to which the organism is susceptible. For those with symptomatic UTI,
we recommend amoxicillin 500 mg t/d for three days.

76. A patient has pain in facial nerve distribution while chewing and
speaking, which of the following drug should be given for treatment?
a) Lamotrigine
b) Phenobarbitone
c) Haloperidol
d) Carbamazepine

Carbamazepine is a medicine used to treat epilepsy. It can also be taken for nerve
pain caused by diabetes (peripheral neuropathy) or if you have a painful condition of
the face called trigeminal neuralgia.

77. Which of the following is a true statement regarding Aliskiren?


a) It is contraindicated in diabetes mellitus
b) It is indirectly increase renin levels
c) It is given in pregnancy
d) It increases the aldosterone levels

Aliskiren is in a class of medications called direct renin inhibitors. It works by


decreasing certain natural chemicals that tighten the blood vessels, so blood vessels
relax and the heart can pump blood more efficiently
78. Which of the following drug is used as radio sensitizer in head and neck
cancers?
a) Cisplatin
b) Amifostine
c) Paclitaxel
d) Doxorubicin

Paclitaxel is a radiosensitizer which may stabilize microtubules, block the G2/M


phase of the cell cycle and thus modulate the radioresponsiveness of tumor cells

79. A 20-year-male is on low dose of oral steroid therapy for the last 10
years, now he develops progressive loss of vision in both the eyes. What is
the reason?
a) Glaucoma
b) Cystoid macular edema
c) Cataract
d) Retinal detachment

Taking steroids can cause a cataract type called posterior sub-capsular cataracts. It
causes a small, cloudy area to form underneath the eye's lens. While cataracts are a
known side effect for some people when taking steroids, they're highly treatable.

80. For the treatment of acute migraine attack, a patient took some
medicines. After that he develops change in colour vision and numbness in
the tip of the thumb. What is likely drug responsible and its treatment?
a) Sumatriptan
b) Dihydroergotamine
c) Propranolol
d) Phenoxybenzamine

Dihydroergotamine is a severe vaso constrictor and it mimics peripheral vascular


disorders when used in acute migraine attack.

81. A child presented to emergency with GTCS since 30 minutes. Airway


has been secured but the child is still having seizures. What is initial treatment
to control the seizures?
a) Valproate
b) Diazepam
c) Phenobarbitone
d) Levetiracetam
In the absence of intravenous access, buccal midazolam or rectal diazepam are
therefore acceptable first‐ line anticonvulsants for the treatment of an acute tonic‐
clonic convulsion that has lasted at least five minutes.

82. A Person from non-endemic area visited India and was given
prophylaxis for malaria. How long the person should continue after travelling
from India?
a) 7days
b) 15days
c) 4weeks
d) 4 months

All prophylactic drugs should be taken with unfailing regularity for the duration of the
stay in the malaria risk area, and should be continued for 4 weeks after the last
possible exposure to infection since parasites may still emerge from the liver during
this period.

83. A child took several iron tablets. What is the antidote for poisoning?
a) BAL
b) EDTA
c) Penicillamine
d) Desferrioxamine

Desferrioxamine injection is used to remove excess iron from the body in anemia or
thalassemia patients who have many blood transfusions. It is also used with other
medicines to treat acute iron poisoning, especially in small children.

84. Adrenaline acts via beta 2 receptors cause bronchodilatation where


histamines act on H1 receptors to cause bronchoconstriction. Which type of
antagonism it is?
a) Physical
b) Chemical
c) Pharmacological
d) Physiological

Physiological antagonism describes the behaviour of a substance that produces


effects counteracting those of another substance (a result similar to that produced by
an antagonist blocking the action of an agonist at the same receptor) using a
mechanism that does not involve binding to the same receptor.
85. A Person having a history of angina and heart rate of 90 bpm and tense
personality developed BP of 150/125. The preferred drug for treatment of
hypertension?
a) Enalapril
b) Amlodipine
c) Atenolol
d) Thiazides

Atenolol belongs to a group of medicines called beta blockers. It's used to treat
high blood pressure and irregular heartbeats (arrhythmia). It can also be used to
prevent chest pain caused by angina. If you have high blood pressure, taking
atenolol helps prevent future heart disease, heart attacks and strokes

86. A boy fell down while playing and the nose is deviated but septum was
found normal, what is the next step done?
a) Lower the edema then close reduction after 7 days
b) Closed reduction and automatically swelling decreases
c) Septo-rhinoplasty
d) Open reduction

Initially medications given to treat the edema and close reduction done after 7 days.

87. A patient underwent removal of submandibular gland and lingual nerve


was damaged during surgery. Which of the following is NOT correct?
a) Rate of sublingual (or submandibular) secretion is reduced.
b) Taste sensation of Anterior 2 / 3rd of tongue is lost.
c) Sensation in floor of mouth lost
d) Tongue deviation.

If your lingual nerve sustains an injury, you'll most likely experience any of these
nerve damage symptoms: Changed sensation in the tongue, chin, or lower lip areas
(similar to sensations you feel when your oral cavity is numbed for a dental
procedure or as the anesthesia slowly wears off) Altered ability to taste. but not
tongue deviation.

88. Syphilis affects which part of the nose?


a) Bony septum
b) Lateral wall of nose
c) Floor of nose
d) vestibule
Syphilis and leprosy are bacterial infections that can have many health implications,
including lesions and ulcers that attack the cartilage in the nose (bony septum). If
left untreated, these infections could cause permanent damage to the nose, resulting
in a saddle nose deformity.

89. A professional trumpet blower presents with left sided neck swelling.
Following is the radiographic findings of the patient. What is the most likely
diagnosis?

a) Laryngocele.
b) Laryngeal cyst
c) Thyroglossal cyst
d) Epiglottits

Laryngoceles can be congenital or acquired, though the majority fall into the latter
category. They occur as a result of increased intralaryngeal pressure such as that
in excessive coughing, playing blowing instruments or due to obstructing lesions
within the larynx.

90. Drug used for treatment of cystitis in pregnancy?


a) Amoxicillin/ Ampicillin
b) Ciprofloxacin
c) Azithromycin
d) Sulfonamides

Those with asymptomatic bacteriuria can be treated with a single dose of an


antimicrobial to which the organism is susceptible. For those with symptomatic UTI,
we recommend amoxicillin 500 mg t/d for three days.
91. Pharyngeal pouch is located between?
a) Superior and middle constrictor
b) Middle and Inferior constrictor
c) Thyropharyngeus and cricopharyngeus
d) Thyroid and cricoids

Pharyngeal pouch is located between Thyropharyngeus and cricopharyngeus. A


Zenker's diverticulum is an outpouching that occurs at the junction of the lower
part of the throat and the upper portion of the esophagus. The pouch forms
because the muscle that divides the throat from the esophagus, the cricopharyngeal
(CP) muscle, fails to relax during swallowing.

92. Endoscopic image is given. Identify the marked structure? (Marked in


YELLOW)

a) Eustachian tube opening


b) Torus tubaris
c) Fossa of rosenmuller
d) None

The fossa of Rosenmüller is a bilateral projection of the nasopharynx just below


the skull base. It is also called the lateral pharyngeal recess or simply the
pharyngeal recess. The fossa is covered by nasopharyngeal mucosa and is the most
common site of origin of nasopharyngeal carcinoma

93. Hypopharyngeal carcinoma with involvement of the same side with no


extension to the opposite side. What is the staging?
a) T3 NO MO
b) T4b NO MO
c) T1 NO MO
d) T4a NO MO
94. Reservoir sign and pain near mastoid antrum is seen in which of the
following condition?
a) Mastoiditis
b) Acute otitis media
c) otitis externa
d) CSOM

Mastoiditis is the result of an infection that extends to the air cells of


the skull behind the ear. Specifically, it is an inflammation of the mucosal lining of
the mastoid antrum and mastoid air cell system inside the mastoid process
95. All are true about the given picture. Except?

a) Its usually a retention cyst


b) Extravasation is not that common
c) It is transluminant
d) All are true

Extravasation is not common and its transluminant from the given picture. but it is
not usually a retention test

96. A 2-month old baby is having watery discharge / fluid leakage from
umbilicus, this is due to
a) Failure of midgut to recanalize
b) Failure of involution of allantoic duct
c) Failure of involution of omphalomesenteric duct
d) Failure of rotation of midgut

Patent urachus refers to a spectrum of umbilical disorders that result from the failure
of involution of normal embryologic tissues that connect the developing bladder to
the umbilical cord. Similarly, presentation is variable and can be readily evident at
birth with umbilical cord abnormalities or diagnosed later in adolescence with the
complaint of persistent umbilical wetness or recurrent umbilical infection.

97. A paediatric patient presented with headache, ear discharge, fever,


reservoir sign is present. what is the diagnosis?
a) CSF otorrhea
b) Acute mastoiditis
c) Acute suppurative otitis media
d) Otitis externa
Mastoiditis is the result of an infection that extends to the air cells of
the skull behind the ear. Specifically, it is an inflammation of the mucosal lining of
the mastoid antrum and mastoid air cell system inside the mastoid process

98. A mother delivered 33 weeks old baby, via normal vaginal delivery with
no complications at birth. What is the preferred method of feeding the
neonate?
a) OGT feeds
b) Direct breast feeding
c) Paladai feeds
d) Parenteral nutrition

The Paladai is a cup-like utensil with a narrow tip that has been used traditionally to
feed babies in India1 when the mother cannot breast feed. It is used to hold
expressed breast milk or other types of liquid or semi-solids. The baby is held
reclining while being fed.

99. A 6-year-old child presented with abdominal pain, hematemesis and


palpable spleen. He had a history of exchange transfusion for neonatal
jaundice. What is the probable diagnosis?
a) Budd Chiari syndrome
b) Cirrhosis of liver
c) Viral hepatitis
d) Portal vein thrombosis

Portal vein thrombosis is blockage or narrowing of the portal vein (the blood
vessel that brings blood to the liver from the intestines) by a blood clot. Most people
have no symptoms, but in some people, fluid accumulates in the abdomen, the
spleen enlarges, and/or severe bleeding occurs in the esophagus.

100. A 1-year-old child presents with 1st episode of wheeze and breathing
difficulty. Chest X-ray shows bilateral hyperinflation. Which of the following
can be a causative organism for this case?
a) RSV
b) S. pneumoniae
c) H. influenza
d) Coxsackie virus

The early phase of RSV in babies and young children is often mild, like a cold. In
children younger than age 3, the illness may move into the lungs and cause
coughing and wheezing. In some children, the infection turns to a severe respiratory
disease.

101. True regarding buffalo milk composition when compared to human


milk?
a) High calorie, high protein
b) Low calorie, low protein
c) High calorie, low protein
d) Low calorie, high fat

Buffalo milk is rich in vitamin A, has a higher protein value and contains more iron,
calcium and phosphorus than the cow's milk. ... It is better to introduce buffalo milk
slowly.

102. A 3-month old child presents with complaints of fever, cough, not able to feed
and breathing difficulty. On examination, respiratory rate was 58 / min, chest
retractions were present. What would be your next step of Management?
a) Severe pneumonia - Give antibiotics and refer immediately
b) Very severe pneumonia and refer urgently
c) Pneumonia -Start antibiotics and send child home
d) No pneumonia

Beta-lactam antibiotics (e.g., amoxicillin, cefuroxime) are preferred for outpatient


management. And is referred immediately to regional hospital

103. All of the following are components of TOF except?


a) Overriding aorta
b) Sub-pulmonic stenosis
c) VSD
d) Tricuspid stenosis

The four components that make up the "tetralogy" include:


 a ventricular septal defect (VSD);
 pulmonary stenosis (sub-valvar, valvar and/or supra-valvar);
 an overriding aorta; and.
 right ventricular hypertrophy.
104. A neonate was found to have Grimace on inserting a catheter into nose,
some flexion of limbs was noted. Heart rate - 120 / minute, Blue extremities
were noted with irregular breathing. Find out the APGAR Score?
a) 4
b) 6
c) 8
d) 10

105. A patient's mother while bathing her child noticed there was no testes in
the scrotum and on examination- one right testes was in Inguinal region and
left was in perineum. What is the DIAGNOSIS?
a) bilateral undescended testes
b) bilateral ectopic testes
c) right undescended and left ectopic testes
d) left undescended and right ectopic testes

from the examination above we can easily deduct that the right testes are
undescended and left testes had become ectopic.

106. A 4-year-old child presented with fever, running nose and following
lesions. Probable aetiology?
a) HSV 1
b) HSV 2
c) Coxsackie virus
d) Streptococci pneumonia
Coxsackievirus can produce a wide variety of symptoms. About half of all kids with
an infection have no symptoms. Others suddenly get a high fever, headache, and
muscle aches, and some also develop a sore throat, abdominal discomfort, or
nausea. a type of coxsackievirus syndrome, causes painful red blisters in the throat
and on the tongue, gums, hard palate, inside of the cheeks, and the palms of hands
and soles of the feet.

107. A patient of 23 years came to the clinic with a known history of myopia
with -2 D in left and -3 D in the right eye. He is sent for retinoscopy. What is the
movement of retinal reflex in the right eye?
a) It moves in the same direction of the retinoscope
b) It moves in the opposite direction of the retinoscope
c) Vertical movement is having wider reflection and moves in the opposite
direction
d) There is no movement of the reflex

it moves in the opposite direction because of the myopia negative i.e. -2D and -3D
respectively

108. Identify the clinical condition from the skin finding as given below:

a) Chicken pox
b) HSV
c) HPV
d) Measles

Chickenpox is a very contagious infection caused by the varicella-zoster virus. It


mainly affects kids, but adults can get it, too. The tell-tale sign of chickenpox is a
super-itchy skin rash with red blisters. Over the course of several days, the blisters
pop and start to leak. Then they crust and scab over before finally healing.
109. Which curvature develops after the baby starts to walk?
a) Primary cervical curvature
b) Secondary sacral curvature
c) Primary sacral curvature
d) Secondary lumbar curvature

As babies, children have a C-shaped spine. Secondary curves in the cervical and
lumbar spine develop as infants become able to lift their heads, sit up, crawl,
stand, and walk. As children grow, their spine continues to develop natural curves
into a normal, mature spine.

110. A mother complains that her baby is not drinking properly from breast
and now complaints of pain and swelling of the breast. Probable condition?
a) Breast abscess
b) Mastitis
c) Breast engorgement
d) Sore nipple

Breast engorgement means your breasts are painfully overfull of milk. This
usually occurs when a mother makes more milk than her baby uses.
Once engorgement occurs gentle massage, frequent feeding, correct positioning
and warm compresses to the breast have been advocated to relieve symptoms
along with analgesia to relieve pain.

Other Causes:
Sore and cracked Nipples
Inverted Nipples
Engorged breasts
Mastitis
Previous surgeries on breast
Neonatal causes
Insufficient/Unsuitable Milk

111. In a paediatric patient with seizure which drug is initially used in


management?
a) Diazepam
b) Phenytoin
c) Phenobarbitone
d) Haloperidol
In the absence of intravenous access, buccal midazolam or rectal diazepam are
therefore acceptable first‐ line anticonvulsants for the treatment of an acute tonic‐
clonic convulsion that has lasted at least five minutes.

112. A 54-year-old female patient with the history of Amenorrhea for last 16
months. What should be the hormone
levels identified in this patient?
a) Increased LH, decreased FSH
b) Increased LH and FSH
c) High FSH, low LH
d) LOW LH AND FSH

 Menopause
• Permanent cessation of menses dated 12 months of amenorrhea
• The avg. age - 51 years (range of 43-57 years)
 Perimenopause
• 2-8 years preceding + 1 year after last menses - fluctuating hormone levels
irregular menstrual cycle & symptoms

When estrogen deficiency occurs in the menopause LH levels increase. Later the
FSH is raised and remains high for the rest of life. This raised FSH and low estrogen
levels appear to cause the characteristic hot flushes.

113. In which of the following condition, MTP is not indicated?


a) Medical: mother’s life is in danger
b) Socioeconomic: Large family size
c) Eugenic: coming baby is handicap
d) Failure of contraception

MTP Act INDICATIONS:


 Continuation of pregnancy constitutes risk to the life or grave injury to
the physical or mental health of woman
 Substantial risk of physical or mental abnormalities in the fetus as to
render it seriously handicapped
 Pregnancy caused by rape (presumed grave injury to mental health)
 Contraceptive failure in married couple (presumed grave injury to
mental health)
 “In determining whether the continuance of pregnancy would involve
such risk of injury to the health (as mentioned above), account may be
taken of the pregnant woman’s actual or reasonable foreseeable
environment”
114. A 40 years old female with severe cardiac compromise, she had to
undergo an operation for which the surgeon gave clearance, which of the
following surgery must not be done?
a) Total hysterectomy
b) Subtotal hysterectomy
c) Vaginal hysterectomy
d) Laparoscopic hysterectomy

Positive pressure pneumoperitoneum with CO2 has deleterious effects on CVS so


Laparoscopic Hysterectomy is not done in this patient

Positive-pressure capnoperitoneum (PPCpn) is the conventional method of


exposure in laparoscopic surgery. This positive intra-abdominal pressure (10–12
mm Hg) is associated with a variable but significant reduction in cardiac output that
may be detrimental in high-risk patients with poor cardiac reserve.
Pneumoperitoneum in excess of 15mmHg has deleterious effects on the
cardiovascular system. The pneumoperitoneum compresses the vena cava and
thus decreases venous return to the heart; this results in blood pooling in the
lower half of the body and a decrease in cardiac output.

115. What is the first sign of MgSO4 toxicity?


a) Respiratory depression
b) Loss of patellar reflex
c) Cardiac arrest
d) Absent urine output

Serum Clinical effects


Mg2+levels(mEq/I)
4-7 Normal therapeutic
level
8-10 Patellar reflex
disappear
10-15 Respiratory depression

12-15 Respiratory paralysis

25 - 30 Cardiac arrest

Calcium gluconate is the antidote for Magnesium Sulfate toxicity.

116. A mother brings her 18-year-old child with complaints of not attained
menarche. GPE shows normally developed breast and pubic hair. On
examination vaginal ending is blind and uterus not palpable. What is the next
line of investigation?
a) USG
b) T3, TSH
c) Estrogen and Progesterone
d) FSH and LH

USG would be most helpful to know presence or absence of uterus.


Karyotype: To differentiate between MRKH and Androgen insensitivity.
Mayer-Rokitansky-Küster-Hauser (MRKH) syndrome is a rare disorder that affects
women. It is characterized by the failure of the uterus and the vagina to develop
properly in women who have normal ovarian function and normal external genitalia.

117. A Female patient came with c/o pruritis, greenish discharge per
vaginum. What is the causative organism?
a) Trichomonas
b) Candida - curdy white discharge + itching.
c) Bacterial vaginosis - Foul smelling greyish white + no itching
d) Chlamydia - mucopurulent endocervical discharge

TRICHOMONAS VULVOVAGINITIS :

• M/c vaginitis
 Etiology - Trichomonas vaginalis –flagellated protozoan
 Mode of Spread: Sexual & non Sexual
 Symptoms - Nature of discharge: Profuse frothy, Colour of discharge:
Yellowish green, Pruritis: Present
 Signs - Discharge: serous & frothy & greenish, pH: > 5, Vulva: inflamed,
Vagina: Strawberry Vagina
 Ix: Wet mount: motile flagellated trichomonas, Clue cells: absent, Whiff test:
negative
• Definitive Dx: culture on Feinberg – Whittington media or Diamonds TYM
media
• Rx - Metronidazole 400mg TID x 5days ‘or’ Metronidazole 2gm single dose
(Mode : both partners)
118. A 14-year-old girl with well-developed secondary sexual characteristics
but presents with amenorrhea. What is the treatment you will prefer next?
a) Wait and Watch
b) Estrogen
c) Give ADH
d) GnRH

• Delayed puberty is said to occur when there is no breast development by the


age of 13 and no menarche by age of 15 years, so the best answer to this
question is to wait and watch.

119. 33-year-old unmarried female presents with the complaint of


amenorrhea and fainting. what is the next step you should perform in this
patient?
a) UPT (urine pregnancy test)
b) Endoscopic Biopsy
c) D and C
d) Hysteroscopy

Always rule out Pregnancy first in case of secondary amenorrhea.

Amenorrhea can occur for a variety of reasons. Some are normal, while others may
be a side effect of medication or a sign of a medical problem.
Causes of 20 Amenorrhea
 Pregnancy
 Breastfeeding
 Menopause
 Contraceptives
 Medications
 Lifestyle factors
 Hormonal imbalance
 Structural problems

120. A female in labor is having occipital posterior position. The cervix gets
8cm dilated, forceps applied in occipitomandibular direction and tried to
rotate, but doctor is not able to rotate. In which type of pelvis, the fetal head
cannot be rotated?
a) Android pelvis
b) Anthropoid pelvis
c) Platypelloid pelvis
d) Gynecoid pelvis
Contracted pelvis: rotation of the head cannot easily occur in android pelvis due to
projection of the ischial spines and convergence of the side walls.
Anthropoid: The elongated shape of the anthropoid pelvis makes it roomier from
front to back than the android pelvis. But it’s still narrower than the gynecoid pelvis.
Some pregnant women with this pelvis type may be able to have a vaginal birth, but
their labor might last longer.

IMPORTANT POINTS
• M/c pelvis – Gynecoid*
• Least common pelvis – Platypelloid pelvis*
• The only pelvis with AP > Tr – Anthropoid pelvis*
• Face to pubis is M/c in – Anthropoid pelvis*
• OP position is M/c in – Anthropoid pelvis*
• DTA is M/c in – Android pelvis*
• Flat bowl shaped pelvis is – Platypelloid pelvis*
• Marked posterior asynclitism is seen in – Platypelloid pelvis*

121. Female having 14 weeks of missed abortion. Which is the agent of


choice to be used in this condition?
a) Mifepristone
b) Misoprostol
c) Oxytocin
d) Methotrexate

Misoprostol is a non-invasive, effective medical method for completion of abortion in


missed abortion.
 Missed Abortion
 When the fetus is dead and retained inside the uterus for a variable period, it
is known as missed abortion
 C/o - no bleed, no morning sickness and regression of breast changes
 O/E - Uterus – lesser than gestational age (arrested at the stage when fetus
was lost) Cx - Internal os is closed
 USG – dead fetus

122. A 35yr old Female presented with 14 week of gestation with bleeding per
vaginum, doctor started misoprostol with dose 200 mcg. How much maximum
dose can be given?
a) 3
b) 4
c) 5
d) 6

123. Cervical cancer is caused by which type of virus?


a) HPV
b) HIV
c) HBV
d) EBV

Long-lasting infection with certain types of human papillomavirus (HPV) is the


main cause of cervical cancer. HPV is a common virus that is passed from one
person to another during sexual intercourse.
HUMAN PAPILLOMA VIRUS
• The M/c etiology - epitheliotropic, oncogenic virus
• 9 major genes - 7 (E1 to E7) + 2 (L1, L2)
• L1(major capsid) & L2(minor capsid protein)
• Viral life cycle – enters genital tract → attaches to the cells → enters basal cells
(endocytosis) →viral DNA injected into human genome→Expression
• Oncogenes - E6 (inhibits p53 anti-oncogene) & E7 (inhibits Rb anti-oncogene)
• High risk serotypes - 16,18,31,33,35, 39,45, 52, 56, 58,59, 68 (strongly
associated –16, 18)
• Low risk - genital warts –6 & 11.
• Immune handling - 80% women at some point infected - infection clears in 9 –15
months - by body’s immunity
• PCR / Southern blot – detects HPV DNA – used as screening along with pap
smear

124. What is shown in the following image?

a) Polyhydraminos
b) Endometriosis
c) Endometritis
d) Mucinous ovarian adenoma

Ovarian mucinous cystadenoma is a benign tumour that arises from the surface
epithelium of the ovary. It is a multilocular cyst with smooth outer and inner
surfaces. It tends to be huge in size.
125. A Young Female is having inserted IUCD 6 months ago. Now she has
presented with the complaints of amenorrhea and fainting since two cycles.
What may be the cause for this condition?
a) PID
b) Endometriosis
c) Ectopic pregnancy
d) Failed contraception

The main risk factor for contraceptive-failure ectopic pregnancy is IUD failure. IUDs
do not increase the absolute risk of ectopic pregnancy, but a pregnancy
occurring with an IUD is more likely to be ectopic than intrauterine.

126. Identify the given non spore forming bacteria?

a) Clostridium difficile
b) Clostridium perfringens
c) Clostridium tetani
d) Corynebacterium diphtheria

Corynebacterium diphtheriae is the leading causing agent of diphtheria. It is a non-


motile, non-encapsulated, non-sporulating gram-positive rod-shaped bacterium with
a high GC-content and occurs in four biovars: gravis, mitis, intermedius, and belfanti,
based on colonial morphology and biochemical profiles.

127. A primigravida is in labor from last 12 hours with tender and rigid
uterus. Fetal head not palpable, loss of Fetal movements. what is the most
appropriate diagnosis?
a) Abruptio placenta
b) placenta previa
c) polyhydramnios
d) Prom
Placental abruption is a complication of pregnancy that happens when the placenta
separates from your uterus before your baby is born. Symptoms or signs can also
include:
 Abdominal pain.
 Uterine contractions that are longer and more intense than average labor
contractions.
 Uterine tenderness.
 Backache or back pain.
 Decreased fetal movement.

128. A female with C-Section delivery, the baby was seeming to be normal
but the placenta was morbidly adherent, what could be the reason for this
condition?
a) absence of nitabuchs membrane
b) absence of decidua parietalis
c) absence of syncytiotrophoblast
d) absence of cytotrophoblast

Morbidly Adherent Placenta


• A morbidly adherent placenta doesn’t separate from the uterine wall upon
attempted manual removal
• It is a condition wherein the placenta is firmly adherent to the uterine wall and
doesn’t separate during the third stage

 Pathological Findings
• Absence of decidua basalis or improper decidua basalis formation
• Absence of Nitabuch's fibrinoid layer
• The trophoblasts completely invade the endometrium and have the potential
to invade the myometrium

129. A 37 weeks of pregnant female came with the complaint of bleeding per
vaginum associated with pain. the fetal heart rate was absent and uterus is
tender. what is the diagnosis and management for this patient?
a) Placental abruption and normal vaginal delivery
b) Placental abruption and LSCS
c) Placenta previa and normal vaginal delivery
d) Placenta previa and LSCS
bleeding per vagina with pain indicates placental abruption and since the fetal heart
rate was absent LSCS would be the best management.

130. Hormone responsible for Lactogenesis?


a) Oxytocin
b) GH
c) Estrogen
d) Progesterone

The two primary hormones that are needed for lactation are prolactin and oxytocin.
Prolactin stimulates milk biosynthesis within the alveolar cells of the breast and
oxytocin stimulates contraction of the myoepithelial cells that surround the alveoli,
causing the milk to be ejected into the ducts leading to the nipple.

131. A pregnant lady delivered a child and after 24 hrs later she came with
complaint of heavy bleeding. What might be the cause for the bleeding in this
patient?
a) Battledore placenta
b) Succenturiate lobe
c) Velamentous placenta
d) Circumvallate placenta

A succenturiate lobe is a variation in placental morphology and refers to a smaller


accessory placental lobe that is separate to the main disc of the placenta. There can
be more than one succenturiate lobe.

Complications
 increased incidence of type II vasa previa
 increased incidence of postpartum hemorrhage due to retained placental tissue
132. Most common symptom seen in endometriosis is;
a) Polymenorrhea
b) Dysmenorrhea
c) HMB
d) Amenorrhea

• Endometriosis Symptomatic - Triad of Dysmenorrhea, Infertility &


Dyspareunia

• Dysmenorrhea - M/c symptom, secondary dysmenorrhoeal, progressive


dysmenorrhea, deep-seated pelvic pain, Associated with premenstrual and
postmenstrual spotting, little correlation /w extent of lesion, correlates well with
the depth of the lesion
• Dyspareunia – POD & rectovaginal septum involvement
• Infertility - 30-40% patients with endometriosis have infertility, 15-30% of
infertile will have endometriosis

133. A 32-year-old female patient complaints on her 4th day of postpartum


delivery of fever, dysuria, foul smelling discharge. what is the probable
diagnosis in this patient?
a) Puerperal sepsis
b) Thrombhophlebitis
c) UTI
d) Endometritis

Puerperal sepsis was defined as infection of the genital tract occurring at any time
between the onset of rupture of membranes or labour, and the 42nd day postpartum
in which two or more of the following are present:

 Fever (oral temperature 38.5°C/101.3°F or higher on any occasion).


 Pelvic pain.
 Abnormal vaginal discharge, e.g. presence of pus.
 Abnormal smell/foul odor of discharge.
 Delay in the rate of reduction of the size of the uterus (involution).

134. Auspitz sign is seen in?


a) Pemphigus vulgaris
b) Psoriasis Vulgaris
c) Pustular psoriasis
d) DLE
The Auspitz sign refers to the appearance of small bleeding points after
successive layers of scale have been removed from the surface of psoriatic
papules or plaques

135. Ulcer "with overhanging / undermined edges” in the neck region as


shown in the image given below.
Diagnosis?

a) Tubercular lymph node


b) Malignant Cervical lymphadenopathy
c) Syphilis
d) Reactive lymphadenopathy

Scrofula refers to painless swelling of the cervical and supraclavicular lymph


nodes that is most often caused by M. tuberculosis in adults and nontuberculous
mycobacteria in children.

136. Which mineral deficiency can lead to following condition?


a) Zinc
b) Iron
c) Calcium
d) Vitamin A

Zinc deficiency occurs in infants when its demand exceeds its supply. It presents
with cutaneous signs which, in severe cases, are associated with diarrhea, alopecia,
and irritability. Genetic and acquired forms of zinc deficiency have been described
and often have overlapping clinical features.

137. First line treatment for acne comedones?

a) Topical steroid
b) Topical antibiotics
c) Benzyl peroxide
d) Topical retinoids

Topical retinoids such as tretinoin or adapalene are effective in many patients with
comedonal acne. Patients with inflammatory lesions benefit from treatment with
benzoyl peroxide, azelaic acid or topical antibiotics. Frequently, the use of
comedonal and antibacterial agents is required
138. Identify the condition shown in the image?

a) Malignant melanoma
b) BCC
c) Squamous cell carcinoma
d) Nevus

Melanoma, also redundantly known as malignant melanoma, is a type of skin


cancer that develops from the pigment-producing cells known as melanocytes.
Melanomas typically occur in the skin, but may rarely occur in the mouth, intestines,
or eye (uveal melanoma).

139. Which organism is responsible for causing erysipelas?

a) Streptococcus
b) Staphylococcus
c) Bacteroides
d) Fungus

Erysipelas is an infection of the upper layers of the skin (superficial). The most
common cause is group A streptococcal bacteria, especially Streptococcus
pyogenes. Erysipelas results in a fiery red rash with raised edges that can easily be
distinguished from the skin around it.

140. Cutaneous horn as shown in the image is associated with which of the
following skin cancer?

a) Squamous cell carcinoma


b) Melanoma
c) BCC
d) Actinic keratosis

Squamous cell carcinoma (SCC) is the second most common form of skin
cancer. It's usually found on areas of the body damaged by UV rays from the sun or
tanning beds. Sun-exposed skin includes the head, neck, chest, upper back, ears,
lips, arms, legs, and hands. SCC is a fairly slow-growing skin cancer.

141. Identify the condition shown in the image?


a) Condyloma acuminata
b) Bowen disease
c) Condyloma lata
d) Hemorrhoids

Condylomata acuminata (CA; singular: condyloma acuminatum), also known


as anogenital warts, are manifestations of HPV infection that occur in a subset of
individuals with anogenital HPV infection

142. Patient suffering from ulcerative colitis presents with ulcer on anterior
leg as shown in the image. Diagnosis?

a) Pyoderma gangrenosum
b) Venous ulcer
c) Tubercular ulcer
d) Chancroid

Pyoderma gangrenosum is a rare condition that causes large, painful sores


(ulcers) to develop on your skin, most often on your legs. The exact causes of
pyoderma gangrenosum are unknown, but it appears to be a disorder of the immune
system.

143. Common complication associated with the following condition?


a) Post streptococcal glomerulonephritis
b) SSSS
c) Encephalitis
d) Pneumonitis

PSGN is a kidney disease that can develop after infections caused by bacteria called
group A Streptococcus (group A strep). These infections include throat and skin
infections like strep throat, scarlet fever, and impetigo. PSGN is not a group A strep
infection of the kidneys.

144. A tourist guide in Mt. Everest, after one of his trips developed blisters on
the feet.
Which is not useful for his treatment?

a) Hyperbaric Oxygen
b) Aspirin
c) Phenylephrine
d) Pentoxifylline
Phenylephrine is used to relieve nasal discomfort caused by colds, allergies,
and hay fever. It is also used to relieve sinus congestion and pressure.
Phenylephrine will relieve symptoms but will not treat the cause of the symptoms or
speed recovery.so it cannot be helpful for the patient with the above condition.

145. Patient presents with history of severe sunburn after only a few minutes
in the sun (photo sensitivity), freckling in sun
exposed areas, dry skin and changes in skin
pigmentation. Diagnosis?
a) Xeroderma pigmentosum
b) Bloom syndrome
c) Warts
d) Melanocytes nevus

Xeroderma pigmentosum (XP) is a very rare skin disorder where a person is highly
sensitive to sunlight, has premature skin ageing and is prone to developing skin
cancers. Xeroderma pigmentosum is caused by cellular hypersensitivity to ultraviolet
(UV) radiation, as a result of a defect in the DNA repair system.

146. Child presents with sores in the mouth and a rash on the hands and feet
as shown in the image below. What is the causative organism for it?

a) Coxsackievirus A16
b) Pox virus
c) Herpes virus
d) Human papilloma virus

Coxsackievirus can produce a wide variety of symptoms. About half of all kids with
an infection have no symptoms. Others suddenly get a high fever, headache, and
muscle aches, and some also develop a sore throat, abdominal discomfort, or
nausea. a type of coxsackievirus syndrome, causes painful red blisters in the throat
and on the tongue, gums, hard palate, inside of the cheeks, and the palms of hands
and soles of the feet.

147. Child presents with fever and pleomorphic rash as shown in the image.
Diagnosis?

a) Chicken pox
b) Small pox
c) Measles
d) Rubella

Chickenpox is a very contagious infection caused by the varicella-zoster virus. It


mainly affects kids, but adults can get it, too. The tell-tale sign of chickenpox is a
super-itchy skin rash with red blisters. Over the course of several days, the blisters
pop and start to leak. Then they crust and scab over before finally healing.

148. A Normal individual underwent an evaluation of his cardiac tissue.


Which isoform of lactate dehydrogenase is predominant in the cardiac tissue?
a) LDH1
b) LDH2
c) LDH3
d) LDH4

The LDH-1 isoenzyme is found predominately in cardiac muscle, LDH-2 is found


primarily in the reticuloendothelial system, LDH-3 predominates in the lungs, LDH-4
in the kidneys, and LDH-5 in the liver and skeletal muscle.

149. A patient has been diagnosed with a solid cancer of the bowel. He also
experienced massive proteinuria after a few months of initial diagnosis of
cancer. which of the following is a likely cause for the development of the
urinary finding?
a) Focal segmental glomerulosclerosis
b) Minimal change disease
c) Membranous Glomerulopathy
d) MPGN

Membranous Nephropathy (MN) is a kidney disease that affects the filters


(glomeruli) of the kidney and can cause protein in the urine, as well as decreased
kidney function and swelling. It can sometimes be called membranous
glomerulopathy as well.

150. Which of the following contributes to the immortality of cancer cells?


a) Topoisomerase
b) Telomerase
c) RNA polymerase
d) Helicase

With each cell division, telomeres shorten until eventually they become too short to
protect the chromosomes and the cell dies. Cancers become immortal by reversing
the normal telomere shortening process and instead lengthen their telomeres

151. A patient died of myocardial infarction and his heart was taken out and
immersed in a solution. The normal area of the heart turned brick red whereas
infarcted portion turned white. Which of the following can be likely component
of the solution?
a) Formalin
b) Triphenyl tetrazolium chloride
c) Ethanol
d) Glutaraldehyde

Triphenyl tetrazolium chloride (TTC) is one of the most conventional stains to


detect infarcted area of the heart in animal experiments. ... Samples with old
myocardial infarction produced clear TTC contrast; normal tissue is vivid red, and
fibrotic myocardium is white discoloration.

152. A patient presents with the presence of a thyroid swelling and weight
loss the physician found presence of cervical lymphadenopathy in the same
patient and decided to go in for a Thyroid biopsy. Histopathology report of the
patient suggested the presence of cells with orphan Annie eye nuclei. Which
of the following is a likely diagnosis for this patient?
a) Follicular thyroid cancer
b) Medullary thyroid cancer
c) Papillary thyroid cancer
d) Anaplastic thyroid cancer

The Orphan Annie-eyed clear nucleus, defined as a large, optically clear nucleus,
devoid of chromatin strands, with sharp chromatin rim, is a more specific feature
than are nuclear grooves or intranuclear cytoplasmic inclusions in papillary thyroid
carcinoma.

153. A patient presented with development of seizures for which the CT


guided biopsy was done. The histopathology reported presence of psammoma
bodies. Which of the following is the most likely diagnosis of this patient?
a) Meningioma
b) Astrocytoma
c) Ependymoma
d) Medulloblastoma

Psammoma bodies (PBs) are concentric lamellated calcified structures,


observed most commonly in papillary thyroid carcinoma (PTC), meningioma, and
papillary serous cystadenocarcinoma of ovary but have rarely been reported in other
neoplasms and non-neoplastic lesions.

154. Which of the following is implicated in the development of


angiosarcoma of the liver?
a) Copper
b) Zinc
c) Arsenic
d) Iron

Hepatic angiosarcoma (HAS) is a rare type of liver cancer that is often fatal, and
arsenic and vinyl chloride monomer (VCM) are two major causal agents.

155. An old man underwent a bone marrow examination which revealed the
presence of the finding shown in the image.
What is the likely diagnosis of the affected
individual?
a) Multiple myeloma
b) ALL
c) Tuberculosis
d) Sickle cell disease

People with smoldering myeloma have some signs of multiple myeloma, such as any
of the following: Plasma cells in the bone marrow between 10% and 60% High
level of monoclonal immunoglobulin (M protein) in the blood. High level of light
chains in the urine (also called Bence Jones protein)

156. A 4-year-old child presents with development of fever, petechial spots


and complaint of fatigue. He is also having presence of pallor, hepato-
splenomegaly as well as sternal tenderness. The clinical situation described
above is most correctly associated with which of the following?
a) AML
b) ALL
c) CLL
d) CML

Acute lymphoblastic leukaemia is the most common childhood cancer. It occurs


when a bone marrow cell develops errors in its DNA.
Symptoms may include enlarged lymph nodes, bruising, fever, bone pain, bleeding
from the gums and frequent infections.

157. A boy presents in the emergency because of development of allergy due


to pollen inhalation. Which of the following cells is important in the
pathogenesis of this condition?
a) NK cell
b) Neutrophils
c) Helper T cell
d) Cytotoxic T cell

Allergic reactions arise from an inappropriate immune response to otherwise


innocuous substances such as pollen, mold, or peanuts, initiated by a group of
immune cells known as the TH2 subset, or type 2 "helper" T cells.

158. Which of the following is the molecule required for presentation of the
antigens by the antigen presenting cell to the TH2 cell?
a) Toll like receptor
b) MHC
c) G protein coupled receptor
d) Lectin receptor

An antigen-presenting cell (APC) or accessory cell is a cell that displays antigen


bound by major histocompatibility complex (MHC) proteins on its surface; this
process is known as antigen presentation. ... APCs process antigens and present
them to T-cells.

159. Which of the following is the mode of inheritance in the patient suffering
from color blindness?
a) Autosomal dominant
b) Autosomal recessive
c) X linked dominant
d) X linked recessive

Examples of X-linked recessive conditions include red-green color blindness and


hemophilia A: Red-green color blindness. Red-green color blindness means that a
person cannot distinguish shades of red and green (usually blue-green), but their
ability to see is normal.

160. A patient has been diagnosed with a condition due to a DNA repair gene
defect. He is having hyper pigmentation on the skin and high risk for
development of skin cancers as per the physician. which of the following is a
likely enzyme defect in the patient?
a) Helicase
b) Nucleotide excision repair defect
c) ATM gene
d) MSH gene

Nucleotide excision repair (NER) is the main pathway used by mammals to remove
bulky DNA lesions such as those formed by UV light, environmental mutagens, and
some cancer chemotherapeutic adducts from DNA. Deficiencies in NER are
associated with the extremely skin cancer-prone inherited disorder xeroderma
pigmentosum.
161. Which of the following is a likely diagnosis of the patient looking at the
karyotype given in the image as Shown

a) Down syndrome
b) Turner syndrome
c) Klinefelter syndrome
d) Edward syndrome

Down syndrome is caused by trisomy 21 — the person has three copies of


chromosome 21, instead of the usual two copies, in all cells. This is caused by
abnormal cell division during the development of the sperm cell or the egg cell.

162. Which of the following amino acids is required for the formation of nitric
oxide in the blood vessels?
a) Citrulline
b) Arginine
c) Histidine
d) Tryptophan

In the body, the amino acid arginine changes into nitric oxide (NO). Nitric oxide is a
powerful neurotransmitter that helps blood vessels relax and also improves
circulation. Some evidence shows that arginine may help improve blood flow in the
arteries of the heart.

163. Macrophages play an important role in phagocytosis and chronic


inflammation. Which of the following cells perform the same function in the
liver?
a) Merkel cell
b) Sinusoidal cell
c) Kupffer cell
d) Hepatocytes

Hepatic macrophages play a central role in maintaining homeostasis in the liver, as


well as in the initiation and progression of liver diseases. Hepatic macrophages are
mainly derived from resident hepatic macrophages called Kupffer cells or circulating
bone marrow-derived monocytes.

164. Which of the following is a characteristic feature associated with


Barrett's oesophagus?
a) Squamous metaplasia
b) Intestinal metaplasia
c) Columnar metaplasia
d) Severe dysplasia

Barrett's esophagus is a condition in which there is an abnormal (metaplastic)


change in the mucosal cells lining the lower portion of the esophagus, from
normal stratified squamous epithelium to simple columnar epithelium with
interspersed goblet cells that are normally present only in the small intestine,
and large intestine. This change is considered to be a premalignant
condition because it is associated with a high incidence of further transition
to esophageal adenocarcinoma, an often-deadly cancer.

165. What is the suggestive diagnosis as per given image?

a) Preseptal Cellulitis
b) Impetigo
c) Orbital cellulitis
d) Conjunctivitis

Orbital cellulitis is an infection of the soft tissues of the eye socket behind the orbital
septum, a thin tissue which divides the eyelid from the eye socket. Infection isolated
anterior to the orbital septum is considered to be Preseptal cellulitis. Orbital cellulitis
most commonly refers to an acute spread of infection into the eye socket from either
extension from periorbital structures (most commonly the adjacent ethmoid or frontal
sinuses (90%), skin, dacryocystitis, dental infection, intracranial infection),
exogenous causes (trauma, foreign bodies, post-surgical), intraorbital infection
(endopthalmitis, dacryoadenitis), or from spread through the blood (bacteremia with
septic emboli).

166. The image indicates?

a) Simple Myopia
b) Irregular astigmatism
c) Regular astigmatism
d) Simple Hyperastigmatism

Each meridian in the regular astigmatic eye has a uniform curvature at every point
across the entrance of the pupil. This is the most common type of astigmatism in
which the symptoms included are blurry vision, headaches, and light sensitivity

167. A 3-year-old child was brought to the ER with unilateral diagnosed as


dacryocystitis. What is the first line of management in the given child
a) Massage
b) Syringing and probing
c) Dacryocystorhinostomy
d) Dacryocystoplasty
The simple method of hydrostatic sac massage has been found to be effective in the
treatment of dacryocystitis in children. The massage is begun by pressure at the
medial canthus, which is effective in blocking the common canaliculus thus
preventing reflux backward through the canaliculi.

168. A 60-year-old patient presented with right sided homonymous


hemianopia with Macular sparing. Localise the neurological site of the lesion
in this condition.
a) Medial geniculate body
b) Optic Tract
c) Optic Chiasma
d) Occipital cortex

Occipital cortex is the neurological site of lesion which causes homonymous


hemianopia with macular sparing.

169. In which chamber of the eye, given keratome is used in Phaco surgery?

a) Anterior chamber
b) Posterior chamber
c) Sclera
d) Lens

 Keratome blade: a surgical instrument used for making an incision


in the cornea in cataract operations.
 Clear corneal incision and it is used to enter the anterior chamber

170. A 60-year-old women comes to the emergency department due to a


sudden onset of severe pain in her left eye with blurred vision, nausea, and
vomiting. the symptoms began a few minutes ago, while was watching a movie
in a theatre. Her temperature is 36.8c (98.4F), blood pressure is 140/90mm.hg,
Pulse is 82/min, and respirations are 14/min, on examination reveals
decreased visual activity. Her left eye appears red, with a hazy cornea, shallow
anterior chamber and dilated, fixed pupil. Her left eye is stony hard to touch,
what is most probable diagnosis?

a) Primary open angle glaucoma


b) Conjunctivitis
c) Acute angle closure glaucoma
d) Anterior uveitis

Acute angle closure is an urgent but uncommon dramatic symptomatic event with
blurring of vision, painful red eye, headache, nausea, and vomiting. Diagnosis is
made by noting high intraocular pressure (IOP), corneal edema, shallow anterior
chamber, and a closed angle on gonioscopy. Medical or surgical therapy is directed
at widening the angle and preventing further angle closure. If glaucoma has
developed, it is treated with therapies to lower IOP.

171. The patient was brought to the emergency. The patient was hit by a
tennis ball…. And noted to have hyphema. What is the source of bleed in this
patient?
a) Anterior ciliary artery
b) Posterior ciliary artery
c) Major arterial circle of iris
d) Minor arterial circle of iris

Hyphema is the collection of blood in the anterior chamber of the eye. The most
common cause of hyphema is blunt trauma, though spontaneous hyphemas can
occur in the setting of sickle cell disease or other increased bleeding states.
Hyphemas are graded based on the degree of blood obscuring the cornea. major
arterial circle of iris is the source of bleed.
172. A 2-year-old Child presented with leucocoria in the right eye since 2
months. On examination a total retinal detachment was the present in the
same eye. Ultrasound B scan revealed a heterogonous sub- retinal mass with
calcification. The most probable clinical diagnosis is:

a) Coats – Disease
b) Retinoblastoma
c) Toxocariasis
d) Retinal tuberculoma

Retinoblastoma is an eye cancer that begins in the retina — the sensitive lining on
the inside of your eye. Retinoblastoma most commonly affects young children, but
can rarely occur in adults. Your retina is made up of nerve tissue that senses light as
it comes through the front of your eye

173. Which of the following is Contra Indicated in Acute Intermittent Porphyria?


a) Thiopentone
b) Propofol
c) Ethomidate
d) Ketamine

hypersensitivity to barbiturates. acute intermittent porphyria Δ (barbiturates


enhance porphyrin synthesis, and therefore are absolutely contraindicated in
patients with a history of acute intermittent porphyria)
174. Identify the Following Instrument.

a) LMA PROSEAL
b) Tight Seal
c) High Seal
d) Classical LMA

ProSeal LMA has a gastric drainage tube, placed lateral to the main airway tube.
The gastric drainage tube forms a channel for regurgitated gastric contents and
prevents gastric insufflation and pulmonary aspiration.

175. Identify the Following Tube.

a) Univalent tube
b) Single Endo tracheal Lumen
c) Double Lumen Endo tracheal Tube
d) Ryles tube
Double lumen tubes play a crucial role in:
1. Airway management during thoracic surgery.
2. Anatomical lung separation or isolating normal from the diseased lung in
situations such as: Massive hemorrhage from one lung. ...
3. Differential lung protective ventilation in case of lung transplantation patients.

176. A lady was taken for C-section operation. the duration of surgery was
more than anticipated and the patient lands into post op hyperventilation.
which of the following could be the reason for that?
a) High Spinal Anaesthesia
b) Malingering
c) Hypoxic Encephalopathy
d) Cranial nerve palsy

High spinal/regional block: spread of local anaesthetic affecting the spinal


nerves. above T4. The effects are of variable severity depending upon the
maximum level. that is involved, but can include cardiovascular and/or respiratory
compromise.

177. A person was admitted into hospital for tibial fracture and a cast was
applied. After 3 weeks the cast was removed and another cast was applied.
The patient was given crutches and asked to walk. What is the stage of healing
of the bone?
a) Inflammation Phase
b) Remodeling Phase
c) Proliferative Phase
d) Infiltration Phase

Remodeling or also known as maturation phase is the fourth and final phase in
wound healing and lasts from 21 days up to 2 years. In this final and longest phase,
collagen synthesis is ongoing in order to strengthen the tissue. Remodeling occurs
as wound continues to contract and fibers are being reorganized.
178. Most common Associated Site of Tb Infection?

a) 1
b) 2
c) 3
d) 4

Bone tuberculosis is simply a form of TB that affects the spine, the long bones, and
the joints

179. Identify the condition?


a) DDH (Developmental Dysplasia of Hip)
b) Ankle Foot Arthrosis Brace
c) Perthes Disease
d) CTEV (Congential Talipes Equino Varus)

In all cases of DDH, the socket (acetabulum) is shallow, meaning that the ball of the
thighbone (femur) cannot firmly fit into the socket. Sometimes, the ligaments that
help to hold the joint in place are stretched. The degree of hip looseness, or
instability, varies among children with DDH.

180. A patient had trauma and was applied cast. On removal of the cast the
doctor pressed the fracture site and described as movable or soft. True among
the following?

a) Patellar Cast with Soft Callus Formed


b) Patellar Cast with Hard Callus Formed
c) Patellar Cast with Bone Remodelling
d) Patellar cast with hematoma.

The callus holds the bone together, but isn't strong enough for the body part to be
used. Over the next few weeks, the soft callus becomes harder. By about 2–6
weeks, this hard callus is strong enough for the body part to be used.
181. All are true except

a) Most Common Infection of Finger


b) Can Progress into Osteomyelitis
c) Treatment Include Incision& Drainage and Antibiotics
d) A Part of Nail Is Removed as Treatment Option

Paronychia is the inflammation of the tissue that immediately surrounds the nail. It is
the most common infection of the hand and it occurs after disruption of the seal
between the nail fold and the nail plate due to penetrating trauma, nail biting,
manicures or a hangnail
182. A parents brought child with the following image of thigh (image has
protruded bone with discharge). Parents confirmed there is history of bone
pain and discharge from the site prior to the protrusion. what could be the
diagnosis?

a) Acute Osteomyelitis
b) Chronic Osteomyelitis
c) Involucrum with Cloaca
d) Osteosclerosis
Chronic osteomyelitis, defined as continuous infection of a low-grade type or of a
recurrent type, is characterized predominantly by bony sclerosis, periosteal new
bone formation, and the presence of sequestra and/or draining sinuses.

183. A 45-year-old female came to outpatient department with complaints of


pain fatigue. Diagnosed tumor of epiphysis

a) GCT
b) Chondrosarcoma
c) Osteosarcoma
d) Ewings Sarcoma

A giant cell tumor is a rare, aggressive non-cancerous tumor. It usually develops


near a joint at the end of the bone. Most occur in the long bones of the legs and
arms. Giant cell tumors most often occur in young adults when skeletal bone growth
is complete.

184. What is the Diagnosis for the given?


a) Gall Stones
b) Carcinoma of Gall bladder
c) Mixed gall stones
d) Porcelain gallbladder

Gallbladder cancer is a relatively uncommon cancer. Most often it is found


after symptoms such as abdominal pain, jaundice and vomiting occur, and it has
spread to other organs such as the liver. It is a rare cancer that is thought to be
related to gallstones building up, which also can lead to calcification of the
gallbladder, a condition known as porcelain gallbladder. Porcelain gallbladder is also
rare. Some studies indicate that people with porcelain gallbladder have a high risk of
developing gallbladder cancer

185. Identify the given image

a) P. Vivax
b) P.falciparum
c) P.malariae
d) P.ovale

Plasmodium falciparum is a unicellular protozoan parasite of humans, and the


deadliest species of Plasmodium that causes malaria in humans. The parasite is
transmitted through the bite of a female Anopheles mosquito and causes the
disease's most dangerous form, falciparum malaria.
186. Identify the give Image

a) Teniasolium
b) Trichuris spiralis
c) Trichuris trichiura eggs
d) Wuchereria bancrofti

Trichuris trichiura eggs are 50-55 micrometers by 20-25 micrometers. They are
barrel-shaped, thick-shelled and possess a pair of polar “plugs” at each end. The
eggs are unembryonated when passed in stool. Trichuris trichiura eggs are 50-55
micrometers by 20-25 micrometers. They are barrel-shaped, thick-shelled and
possess a pair of polar “plugs” at each end. The eggs are unembryonated when
passed in stool.

187. A 30-year-old patient of valvular heart disease is having fever for last 2
weeks. On physical examination the following features are noticed in this
patient. Which of these will be the abdomen examination finding of this
patient?
a) Ascites
b) Portal Hypertension
c) Splenomegaly
d) Renal artery Bruit

An enlarged spleen is the result of damage or trauma to the spleen from any of
several different medical conditions, diseases, or types of physical
trauma. Infections, liver problems, blood cancers, and metabolic disorders can
all cause your spleen to become enlarged, a condition called splenomegaly.

188. A 70-year-old man collapsed in his house and was rushed to the
hospital. On arrival his ECG tracing was as shown below. Which of the
following is the best intervention for this patient?
a) Use of Automated external defibrillator
b) Injection of Vasopressin 40 IU
c) Injection of adrenaline 1 mg iv
d) Injection of atropine 0.5mg iv

Adrenaline 1:1000 (1mg/mL) Solution for injection is used in life threatening


emergencies such as severe allergic reactions or cardiac arrest. 2. you have
atherosclerosis which is a narrowing and hardening of the body's blood vessels

189. A 60-year-old woman presents with weakness in right arm for four
hours’ duration. The weakness gradually became lesser and resolves. She is a
known case of Hypertension and BMI is elevated. Which of the following is the
probable diagnosis of this case?
a) Transient ischemic attack
b) Compressive Neuropathy
c) Transverse Myelitis
d) Stroke

weakness in the right arm for 4 hours, which self resolves for a patient with
elevated BMI and Hypertension is suggestive of stroke

190. A 35-year-old recently divorced male is suffering from palpitations,


headache and diaphoretic episodes on recurrent basis. He has visited the
psychiatrist multiple times. What should be the next best step for management
of this case?
a) He should visit psychologist instead of psychiatrist
b) Check for catecholamines levels to rule out pheochromocytoma
c) Check for hyperthyroidism
d) Increase dose of anti-anxiety medication

High levels of catecholamines, vanillylmandelic acid (VMA), or metanephrine can


mean that an adrenal gland tumor (pheochromocytoma) or another type of
tumor that makes catecholamines is present.
191. A patient has presented with gross abdominal distention and the
following test is being performed. Comment on the finding being elicited?

a) Fluid thrill
b) Shifting dullness
c) Puddle sign
d) Hingorani sign

In medicine, the fluid wave test or fluid thrill test is a test for ascites (free fluid in the
peritoneal cavity). It is performed by having the patient (or a colleague) push their
hands down on the midline of the abdomen. The examiner then taps one flank, while
feeling on the other flank for the tap.

192. An elderly patient with Hypertension fell down in his bathroom. His BP
was 220/110 on admission. Which of the following statements is correct about
this patient?
a) Start anti platelet drugs
b) Platelets are transfused even if the platelet count is not low
c) Reduce the BP to <120:
d) Putamen is the most common site

The putamen is the most common site for hypertensive ICH. Hemorrhages may
remain localized to the putamen; enlarge to involve the internal capsule, corona
radiata, centrum semiovale, or temporal lobe; or rupture into the ventricular system
193. A 70-year-old man has recurrent episodes of exertional syncope and
recurrent episodes of chest pain on climbing stairs. He was diagnosed as a
case of valvular aortic stenosis. Which of the following is the radiological
finding of XRC of this patient?
a) Dilated aortic Root:
b) Widening of aortic knob
c) Widening of vascular pedicle
d) Post stenotic dilatation of aorta

Variable appearance on chest radiographs depending on stage and severity of the


disease.
In early disease, the chest radiograph can be entirely normal or it may
show dilatation of the ascending aorta with a normal heart size. Differentiation
with hypertension can usually be made as in hypertension the entire descending
aorta is enlarged

194. Elderly Chronic alcoholic patient has presented to OPD today. The
following findings were noted in the patient. What is the best investigation to
be done in this patient?

a) Plasma ascorbate levels


b) Factor8 assay
c) RBC transketolase activity
d) Thrombin time

ascorbate is the most effective aqueous-phase antioxidant in human blood


plasma and suggest that in humans ascorbate is a physiological antioxidant of major
importance for protection against diseases and degenerative processes caused by
oxidant stress.

195. A patient presents with complaints of palpitations. On examination her


heart rate is 110 / min with Respiratory rate of 10 / min. She tells you that her
mother has recently expired due to renal cell cancer. What investigation would
you perform to manage this case?
a) Serum Hydroxy Indole Acetic Acid levels (H.I.A.A)
b) Serum Catecholamines
c) 24-hour Urine VMA (Vanillylmandelic acid) for 24 hours
d) 24-hour fractionated Metanephrine levels

 If your 24-hour metanephrine level is between 1 and 2 times the normal


amount, there is about a 30% chance you have a pheochromocytoma or
paraganglioma.
 If your 24-hour metanephrine level is twice the normal level or higher, it's likely
that you have a pheochromocytoma or paraganglioma.

196. Comment on serological report of this hepatitis B patient


IgG Anti HBc Positive
HBsAg Negative
HBeAg Negative
Anti HBs positive

a) Acute hepatitis B, High infectivity


b) Chronic hepatitis B, High infectivity
c) Chronic hepatitis B, low infectivity
d) Previous infection with hepatitis B

HBeAg negative shows that the virus is not actively replicating and the patient is not
infectious
HbsAg negative shows the patient is not currently infected with the virus
Anti HBs, IgG Anti HBc both are positive showing that the antibodies are formed for
the previous infection

197. A 30-year-old female patient presented with pain in joints of hand with
rash on dorsum of hand with extensive skin induration and skin tightening.
Comment on the diagnosis.
a) Rheumatoid arthritis
b) Psoriatic arthritis
c) Osteoarthritis
d) Scleroderma

Psoriatic arthritis is a form of arthritis that affects some people who have
psoriasis — a disease that causes red patches of skin topped with silvery scales.
Most people develop psoriasis years before being diagnosed with psoriatic arthritis.

198. A 40-year-old patient came to OPD with symptoms of excessive


sleepiness and lethargy all the time. On physical examination the clinical
features depicted in the images were seen. Per abdominal examination shows
no free fluid in abdomen. Liver and spleen are not palpable. He tells of regular
Alcohol consumption. Which is the most probable diagnosis?

a) Hepatolenticular degeneration
b) Alcoholic Cirrhosis
c) Portal Hypertension
d) Budd Chiari Syndrome

Alcoholic hepatitis is a condition caused by continued alcohol use that results in


long-term (chronic) inflammation in your liver. Alcoholic cirrhosis is an advanced
stage of alcoholic liver disease that causes your liver to become stiff, swollen,
and barely able to do its job.

199. A 35-year-old woman presented with complaints of palpitations and


weight loss. On examination sinus tachycardia is present,
and the following lesion was noticed on the shin of the
patient. What is the clinical diagnosis?
a) Hypothyroidism
b) Hyperthyroidism
c) Filariasis
d) Milroy disease
Hyperthyroidism, also called overactive thyroid, is a condition where the thyroid
releases high levels of thyroid hormone into the body. This condition can make
your metabolism speed up. Symptoms of hyperthyroidism include a rapid heartbeat,
weight loss, increased appetite and anxiety.

200. On Post-operative day 2, routine X-Ray of patient was performed. The


patient is not complaining of any respiratory distress and is comfortable in
hospital bed. On bilateral auscultation equal air entry is heard. What is the next
step in management of patient?
a) Wait and watch and give supplemental oxygen if required
b) Perform needle thoracotomy urgently
c) Do a thoracostomy with wide bore tube and under water seal
d) Perform elective intubation and positive pressure ventilation

The patient condition is getting better and since there is no complaints of any
respiratory distress and equal air entry is heard in bilateral auscultation, we just
continue the observation of the patient and provide oxygen if needed.

201. A 55-year-old man has chest pain on rest for last 6 hours. ECG was
performed on admission and showed findings of myocardial ischemia. His
biomarkers report was sent to lab for diagnostic workup. Which of the
following will help in diagnosis?
a) LDH1
b) LDH2
c) LDH3
d) LDH4

The LDH-1 isoenzyme is found predominately in cardiac muscle, LDH-2 is found


primarily in the reticuloendothelial system, LDH-3 predominates in the lungs, LDH-4
in the kidneys, and LDH-5 in the liver and skeletal muscle.

202. A 50-year-old man presents with recurrent episodes of chest pain on


exertion. He is diagnosed as a case of chronic stable angina. On routine
monitoring BP is found to be consistently elevated. Which of the following is
the best drug for management of this patient?
a) Enalapril
b) Amlodipine
c) Atenolol
d) Thiazides

Atenolol belongs to a group of medicines called beta blockers. It's used to treat
high blood pressure and irregular heartbeats (arrhythmia). It can also be used to
prevent chest pain caused by angina. If you have high blood pressure, taking
atenolol helps prevent future heart disease, heart attacks and strokes

203. A 30-year-old man has presented with complaints of fever for 3 days
with SOB. On examination on right mammary area bronchial breathing is heard
with normal air entry bilaterally. Comment on the diagnosis.
a) Right lower lobe pneumonia
b) Right middle lobe pneumonia
c) Right sided hydro-pneumothorax
d) Right sided Hydrothorax

Bronchial sounds are high pitched & usually heard over the trachea. Timing includes
an inspiratory phase that is less than the expiratory phase. If bronchial sounds are
heard in the actual lung fields, this may indicate consolidation.

204. What is the impression from the following CXR?

a) Chemo-port device
b) Pacemaker
c) Artifact
d) Travel tract for bypassing accessory pathway
A pacemaker is a device that sends small electrical impulses to the heart
muscle to maintain a suitable heart rate or to stimulate the lower chambers of the
heart (ventricles). A pacemaker may also be used to treat fainting spells (syncope),
congestive heart failure and hypertrophic cardiomyopathy.

205. Prophylactic CNS radiation is given for which of the following cancers?
a) Liver cancer
b) Small cell cancer of lung
c) Prostate cancer
d) Seminoma

Prophylactic cranial irradiation is now known to improve survival to a significant


degree in small-cell lung cancer (SCLC) patients; this is in addition to its established
role in preventing the disabling symptoms of brain metastases.

206. An elderly patient fell in the bathroom in early morning hours and was
rushed to the casualty. On arrival GCS is 9/15 and urgent NCCT head was
performed. Comment on the diagnosis?
a) Extradural hemorrhage
b) Subdural hemorrhage
c) Diffuse axonal injury
d) Intracerebral haemorrhage

A subdural hematoma is a type of bleed inside head. More precisely, it is a type of


bleed that occurs within the skull of head but outside the actual brain tissue. The
brain has three membranes layers or coverings (called meninges) that lay between
the bony skull and the actual brain tissue

207. A young woman presents as shown below, which is triggered by cold


temperatures or emotional stress. Episodes usually affect the fingers and toes
along with numbness and pain in the affected areas. There is no underlying
illness or comorbidity in the patient. Comment on the diagnosis.
a) Primary Raynaud phenomenon
b) Secondary Raynaud phenomenon
c) Harlequin sign
d) Ainhum

Secondary Raynaud's is caused by an underlying disease or condition. It is


especially common in people with connective tissue diseases. Some of these
diseases reduce blood flow to the fingers and toes by causing the blood vessel walls
to thicken and the vessels to constrict too easily

208. Comment on the diagnosis of the Flow volume curve shown below

a) COPD
b) Interstitial lung disease
c) Retrosternal Goiter
d) Tracheal stenosis

The F/V curve is a representation of air stream volume pattern during


inspiration and expiration. It exhibits the obstructive, restrictive and mixed pattern
pathological conditions. ... The classic flow-volume loop shapes describe the
obstructive and restrictive lung pathology as well as anatomical abnormalities.
Patients with chronic obstructive pulmonary disease (COPD) exhibit increases in
lung volume due to expiratory airflow limitation. Increases in lung volumes may
affect upper airway patency and compensatory responses to inspiratory flow
limitation (IFL) during sleep.

209. A 28-year-old pregnant lady presents for routine antenatal check-up.


Which Arterial Blood gas abnormality is expected to be present in this case?
a) Respiratory acidosis
b) Respiratory Alkalosis
c) Metabolic Acidosis
d) Metabolic Alkalosis
Progesterone levels are increased during pregnancy. Progesterone causes
stimulation of the respiratory center, which can lead to respiratory alkalosis. Chronic
respiratory alkalosis is a common finding in pregnant women.

210. Rabies virus will use which kind of molecular motor to cause
involvement of Central nervous system from the site of inoculation into the
host?
a) Dynein
b) Kinesin
c) Kinin
d) Actin

Dynein is a family of cytoskeletal motor proteins that move along microtubules in


cells. They convert the chemical energy stored in ATP to mechanical work.
Dynein transports various cellular cargos, provides forces and displacements
important in mitosis, and drives the beat of eukaryotic cilia and flagella. Rabies virus
uses this to cause involvement of central nervous system from the site of inoculation
into the host.

211. A 31-year-old car driver was involved in an accident. He is brought to ER


with complaints of breathlessness. On physical examination vitals are PR-110 /
min, BP 90/60 mm Hg and temp of 37 degrees. Extensive bruising is noticed on
right side of chest. An urgent bedside CXR was performed. What is the best
management of this case?
a) Thoracotomy
b) Thoracostomy
c) Elective intubation and PPV
d) Fluid resuscitation

The patient is suspected to have hemothorax due to the contusion and so first we go
for elective intubation and PPV

212. A 60-year-old woman a known case of lung cancer is having shortness


of breath. On examination her neck veins are engorged showing gross
elevation of JVP, Heart sounds are distant and breath sounds appear reduced
on left side infra-scapular area. Requisition for CXR is done and is shown
below. Comment on the diagnosis of the patient.
a) Cardiac Tamponade
b) Congestive heart failure
c) Bilateral pleural effusion
d) SVC Syndrome
Cardiac tamponade is a serious medical condition in which blood or fluids fill
the space between the sac that encases the heart and the heart muscle. This
places extreme pressure on your heart. The pressure prevents the heart's ventricles
from expanding fully and keeps your heart from functioning properly.

213. An elderly patient with malignancy develops haematuria with dipstick


performed on urine sample showing 4+ proteinuria. Which of these is the
possible diagnosis of this patient?
a) MGN
b) MPGN
c) FSGS
d) Diffuse Glomerulosclerosis

Membranous glomerulonephritis (MGN) is a specific type of GN. MGN


develops when inflammation of your kidney structures causes problems with
the functioning of your kidney. MGN is known by other names, including
extramembranous glomerulonephritis, membranous nephropathy, and nephritis.

214. In the given tracing shown below, HV interval denotes?

a) AV intranodal conduction
b) Septal Activation of bundle of His
c) Antegrade conduction from Bundle of His to bundle branches
d) Antegrade conduction from Bundle to His to Purkinje fibers

HV interval measures the conduction time through the distal His- Purkinje
tissue and is measured from the onset of the His-bundle deflection to the earliest
ventricular activation.
215. A 60-year-old patient has right sided homonymous hemianopia with
macular sparing. Locate the neurological site of lesion in this patient.
a) Optic chiasma
b) Optic tract
c) Medial geniculate body
d) Occipital cortex

Homonymous hemianopsia can be congenital, but is usually caused by brain injury


such as from stroke, trauma, tumors, infection, or following surgery.
Vascular and neoplastic (malignant or benign tumours) lesions from the optic tract,
to visual cortex can cause a contralateral homonymous hemianopsia. Injury to the
right side of the brain will affect the left visual fields of each eye. The more posterior
the cerebral lesion, the more symmetric (congruous) the homonymous hemianopsia
will be.

216. A 60-year-old man has presented with chronic projectile vomiting and
significant weight loss. Which of the following electrolyte abnormality is seen
in this patient?
a) Hypokalemic hypochloremic Metabolic alkalosis with Hyponatremia
b) Hypokalemic hypochloremic Metabolic alkalosis with Hypernatremia
c) Hypokalemic hypochloremic Metabolic alkalosis with Hypercalcemia
d) Hypokalemic hypochloremic Metabolic alkalosis with Hypomagnesemia

Hypochloremic alkalosis results from either low chloride intake or excessive


chloride wasting. Whereas low chloride intake is very uncommon, excessive
chloride wasting often occurs in hospitalized children, usually as a result of diuretic
therapy or nasogastric tube suctioning.

217. A 35-year-old patient with ulcerative colitis develops ulcer on the


anterior compartment of the leg. Comment on the diagnosis of patient.
a) Venous ulcer
b) Erythema migrans
c) Erythema Nodosum
d) Pyoderma gangrenosum

Pyoderma gangrenosum is a rare condition that causes large, painful sores (ulcers)
to develop on your skin, most often on your legs. The exact causes of pyoderma
gangrenosum are unknown, but it appears to be a disorder of the immune system
218. A patient presented with back pain. Labs show a grossly elevated ESR,
X- Ray skull shows multiple punched out lesions. Which is the most important
investigation to be done for this case?
a) Serum acid phosphatase
b) CT head with Contrast
c) PET scan
d) Serum Electrophoresis

back pain, elevated ESR X-ray showing multiple punched out lesions leading to
multiple myeloma. Serum protein electrophoresis is used to identify patients with
multiple myeloma and other serum protein disorders. Electrophoresis separates
proteins based on their physical properties, and the subsets of these proteins are
used in interpreting the results. Plasma protein levels display reasonably predictable
changes in response to acute inflammation, malignancy, trauma, necrosis, infarction,
burns, and chemical injury.

219. An AIDS Positive 30-year-old male patient has developed fever, vomiting
and meningismus. Which of the following test: will have help in rapid
diagnosis of Cryptococcal meningitis?
a) India Ink preparation of CSF
b) Blood Lateral flow assay for Cryptococcal antigen
c) Blood culture
d) CSF Cultures

he recent development of the Cryptococcal antigen lateral flow assay (LFA), a


commercially available rapid diagnostic test that detects capsular polysaccharide
antigens of the four major cryptococcal serotypes (A and D for C. neoformans and B
and C for C.

220. A 30-year-old man who works in a glass factory presents with complaints of
progressive shortness of breath on exertion. He is predisposed to development of
which of the following?
a) Histoplasmosis
b) Tuberculosis
c) Mesothelioma
d) Pleural calcification

The common Health hazards associated with glass manufacturing processes


include noise, exposures to respirable airborne particulates [especially silica
(SiO2) that leads to chronic effects such as decreased pulmonary function,
lung disease, complicated pneumoconiosis
221. A 12-year-old girl has developed Acute Immune thrombocytopenic
purpura (ITP) and is managed with steroids. There is recurrence of low platelet
count once steroids were stopped. On current physical examination there is
no petechiae or any physical evidence of bleeding. Which is the next line of
treatment?
a) Continue steroids as long they are effective
b) Splenectomy is greater than 2 relapses
c) Wait and watch as self-resolution will occur in most cases of acute ITP
in children
d) Rho Immunoglobulin IV

since the patient was under steroid therapy for Immune thrombocytopenic purpura
(ITP). Since there is no petechiae or any bleeding other than less platelet count ,
most probable next step should be to wait and observe as acute ITP is self-resolved
in most cases of children.

222. What is the primary defect in vitamin D resistant rickets?


a) Defective phosphate absorption from gut
b) Phosphate wasting from proximal renal tubule
c) Defective Calcium absorption from gut
d) Calcium wasting from proximal renal tubules

Hypophosphatemic rickets is a disorder characterized by hypophosphatemia,


defective intestinal absorption of calcium, and rickets or osteomalacia unresponsive
to vitamin D. It is usually hereditary.
223. A 31-year-old alcoholic had his last drink 48 hrs ago. He develops anger
and rage attacks and appears unable to identify faces of family members and
can see inanimate objects. He has been brought to the ER. What is your
diagnosis?
a) Delirium tremens
b) Alcoholic hallucinosis
c) Korsakoff psychosis
d) Wernicke's encephalopathy

Delirium tremens (DTs) is the most severe form of ethanol withdrawal, manifested
by altered mental status (global confusion) and sympathetic overdrive (autonomic
hyperactivity), which can progress to cardiovascular collapse. Minor alcohol
withdrawal is characterized by tremor, anxiety, nausea, vomiting, and insomnia
224. A 25-year-old male patient is having loose stools 18 times per day and
his urine frequency is reduced. What will be the correct finding of this patient?
a) Urine sodium >40
b) Kidney biopsy showing Glomerular damage
c) Hyperosmolar urine
d) Post renal azotemia

Both high and low urine osmolality have several causes. Some of these, such as
dehydration, are relatively easy to treat. Others can be more serious or ongoing.
Here due to severe dehydration, hyperosmolar urine can be found.

225. A 50-year-old lady with Hypertension was admitted with breathlessness


in supine position and palpitations for last 4 days. On physical examination
her heart rate is 120 / min with congested neck veins. Per abdomen shows
liver palpable 3 cm below costal margin. The finding shown in the image below
is?

a) Deep vein thrombosis


b) Superficial venous thrombophlebitis
c) Pitting Pedal oedema
d) Lymphedema

Pitting edema is a specific type of edema that is associated with pitting or indentation
in the affected areas, and is often caused by conditions that lead to the
pooling of blood in the feet or legs. The excess fluid that builds up in pitting edema is
mainly composed of water.
226. A 6-year-old boy has presented with multiple episodes of hematemesis.
On admission his vitals are stable and splenomegaly is noted. The mother tells
that at birth the boy had developed jaundice and an exchange transfusion
procedure had been performed in the same hospital. Diagnosis is
a) Budd Chiari Syndrome
b) Sinusoidal fibrosis
c) Portal vein thrombosis
d) Non cirrhotic portal fibrosis
Portal vein thrombosis is blockage or narrowing of the portal vein (the blood
vessel that brings blood to the liver from the intestines) by a blood clot. Most people
have no symptoms, but in some people, fluid accumulates in the abdomen, the
spleen enlarges, and/or severe bleeding occurs in the esophagus.

227. A 6-year-old child weighs 12 kg. The urine output of the child is only
0.5ml / kg / hour for last 12 hours and serum creatinine has shown a doubling
over the baseline. What is stage of AKI by KDIGO criteria?
a) 1
b) 2
c) 3
d) 4

228. A girl is having honey crust injuries on face which was diagnosed by her
paediatrician as impetigo. She has now been brought your hospital with facial
puffiness and not eating well. When you work up the patient, which of the
following will be the most common finding encountered in this case:
a) RBC in urine
b) Nephrotic range proteinuria
c) Derangement of liver enzymes
d) Focal neurological deficit

Transient proteinuria and hematuria may occur during impetigo and resolve before
renal involvement develops. Antibiotic treatment does not prevent the development
of glomerulonephritis, but it limits the spread of the disease to other individuals.
Other complications may include the following: Scarlet fever.

229. A 61-year-old known case of Diabetes mellitus has been noticed on


laboratory work up to have microalbuminuria. What is the site of reabsorption
of proteins in the ultra-filtrate?
a) Podocytes
b) Proximal convoluted tubule
c) Distal convoluted tubule
d) Collecting duct

The proximal convoluted tubule is where a majority of reabsorption occurs. About 67


percent of the water, Na+, and K+ entering the nephron is reabsorbed in the proximal
convoluted tubule and returned to the circulation.

230. A 35-year-old patient presents with high grade fever, chills and cough
with copious production of purulent sputum. An x-ray examination was made
as shown in the image, what is the most likely diagnosis?

a) Rt middle lobe consolidation


b) Rt lower lobe consolidation
c) Atelectasis
d) Pneumothorax

Consolidation refers to the alveolar airspaces being filled with fluid


(exudate/transudate/blood), cells (inflammatory), tissue, or other material. The list of
causes of consolidation is broad and includes: pneumonia. adult respiratory distress
syndrome (ARDS)

231. An 65 yrs old hypertensive patient on irregular medication has headache


and a BP-220/110, CT head is shown below
a) Intraventricular bleed
b) Putaminal bleed
c) Sub arachnoid hemorrhage
d) Pontine hemorrhage

The putamen is a common site of hypertensive cerebral hemorrhage. Such


hemorrhages show a large range of possible presentations – from enormous
hemorrhages involving the white matter of the hemispheres and the ventricular
system, to cases occurring without causing any symptoms or neurological signs.

232. Pt with History of Recurrent UTI, the IVP image is given, Identify the
location of insertion of ureter arising from left superior pelvis?

a) Below and medial to trigone of bladder


b) Above the dome of the bladder
c) Prostatic urethra
d) Perineum
The trigone (a.k.a. vesical trigone)is a smooth triangular region of the
internal urinary bladder formed by the two ureteric orifices and the internal urethral
orifice.

233. Identify the marked structure on CT head

a) Optic Nerve
b) Optic Chiasma
c) Optic tract
d) P Superior Ophthalmic Artery

The marked part in the CT scan is the optic nerve

234. 5yr old boy is unable to pronate and supinate since childhood?
a) Radioulnar synostosis
b) Radial head dislocation
c) Monteggia fracture
d) Myositis ossificans

Radioulnar synostosis is a rare condition in which the two bones of the


forearm — the radius and the ulna — are abnormally connected. This limits rotation
of the arm. Radioulnar synostosis is usually congenital). It can also occur as the
result of a forearm fracture or trauma.

235. Identify the marked structure shown by the arrow?

a) Pons
b) Medulla
c) Midbrain
d) Spinal cord

The marked structure in the above radiological finding is the mid-brain

236. 45 year old Male with Heart Disease undergoes a chest X-ray, what is the
diagnosis?
a) Prosthetic aortic valve
b) Prosthetic mitral valve
c) Pericardial effusion
d) Implantable pacemaker

An artificial heart valve is a one-way valve implanted into a person's heart to


replace a valve that is not functioning properly (valvular heart disease). The
mechanical prosthetic heart valves are very durable and guarantee 20-30 years of
functioning in ideal conditions. The coating with pyrolytic carbon in mechanical
valves also decreases the chance of blood clotting on the valve surface.

237. The Given Skull X-ray finding is seen in?

a) Hemolytic anemia
b) Multiple myeloma
c) Increased ICT
d) Osteopetrosis

Hemolytic anemia is a disorder in which red blood cells are destroyed faster
than they can be made. The destruction of red blood cells is called hemolysis. Red
blood cells carry oxygen to all parts of your body. If you have a lower than normal
amount of red blood cells, you have anemia.
238. X-ray of a 2-year-old child is given, what is the possible diagnosis?

a) Pneumastosis intestinalis
b) Pneumoperitoneum
c) Subcutaneous emphysema
d) Duodenal atresia

Pneumoperitoneum is the presence of air or gas in the abdominal (peritoneal)


cavity. It is usually detected on x-ray, but small amounts of free peritoneal air may
be missed and are often detected on computerized tomography (CT).

239. Identify the pathology on given Barium enema image?


a) Intussusception
b) Ulcerative colitis
c) Cecal cancer
d) Ileocecal TB

Intussusception is a condition in which one segment of intestine "telescopes"


inside of another, causing an intestinal obstruction (blockage). Although
intussusception can occur anywhere in the gastrointestinal tract, it usually occurs at
the junction of the small and large intestines

240. A 45-year-old female came for post-surgery follow up, A CXR was taken
what is the most likely diagnosis?

a) Malignancy
b) Fibroadenoma
c) Emphysema
d) Left sided mastectomy

In the picture we can clearly absence of left breast.

241. Identify the Given HSG image?


a) Bicornuate
b) Unicornuate
c) Septate
d) Didelphys

Unicornuate uterus is a rare genetic condition in which only one half of a girl's
uterus forms. A unicornuate uterus is smaller than a typical uterus and has only one
fallopian tube. This results in a shape often referred to as “a uterus with one horn” or
a “single-horned uterus.

242. A female patient with dysphagia, Barium swallow image is shown, what is the
likely diagnosis?

a) Diffuse esophageal spasm


b) Esophageal cancer
c) Achalasia cardia
d) Esophagitis

Achalasia Cardia is a rare disorder that makes it difficult for food and liquid to pass
from the swallowing tube connecting your mouth and stomach (esophagus) into your
stomach. Achalasia Cardia occurs when nerves in the esophagus become damaged.
The hallmark of the achalasia pattern is a gradually tapering, smooth, conical
narrowing (bird's beak or rat-tail appearance) of the distal esophageal segment
which extends about 1-3 cm in length

243. A 65-Year-old female fell down in the bathroom &sustained head injury, Ct
scan was performed, what is likely diagnosis?
a) Extradural hematoma
b) Intracranial bleed
c) Thrombosis
d) Subdural hematoma

A subdural hematoma is a type of bleed inside head. More precisely, it is a type of


bleed that occurs within the skull of head but outside the actual brain tissue. The
brain has three membranes layers or coverings (called meninges) that lay between
the bony skull and the actual brain tissue

244. Identify the given IVP image


a) B/L Ureterocele
b) Hydronephrosis
c) Staghorn calculi
d) Bladder calculi

A ureterocele is a swelling at the bottom of one of the ureters. Ureters are the
tubes that carry urine from the kidney to the bladder. The swollen area can block
urine flow. A ureterocele is a birth defect

245. What structure is the arrow pointing to

a) Medial epicondyle
b) Trochlea
c) Capitulum
d) Olecrenon

In human anatomy of the arm, the capitulum of the humerus is a smooth, rounded
eminence on the lateral portion of the distal articular surface of the humerus.

246. Identify the Given Chest Radiograph?


a) Pneumothorax
b) Hydropneumothorax
c) Pericardial effusion
d) Emphysema

A pneumothorax is a collapsed lung. A pneumothorax occurs when air leaks into


the space between your lung and chest wall. This air pushes on the outside of your
lung and makes it collapse. A pneumothorax can be a complete lung collapse or a
collapse of only a portion of the lung.

247. 33 yr. old female with fever, weight loss, elevated levels of ACE levels
with hilar lymphadenopathy on x ray, Probable diagnosis?

a) TB
b) Sarcoidosis
c) Silicosis
d) Small cell cancer

Sarcoidosis is an inflammatory disease that affects one or more organs but most
commonly affects the lungs and lymph glands. As a result of the inflammation,
abnormal lumps or nodules (called granulomas) form in one or more organs of the
body.
248. Identify the structure being pointed by the Arrow?

a) Hepatic artery
b) Portal vein
c) Hepatic vein
d) Inferior venacava

The portal vein (PV) is the main vessel of the portal venous system (PVS),
which drains the blood from the gastrointestinal tract, gallbladder, pancreas,
and spleen to the liver.

249. A patient met with a RTA, is unconscious and the CT scan does not
show any findings except for few petechial hemorrhages, what is the probable
diagnosis?
a) Diffuse Axonal injury
b) Concussion
c) Contusion
d) Subarachnoid hemorrhage

250. A Patient with prostate cancer receives sequential radiotherapy at a


dose of 1.8-2.4 Gy for 5 days every week, for 5-7 weeks this is called as?
a) Regular fractionated radiotherapy
b) Hypofractionated RT
c) Hyperfractionated RT
d) Accelerated RT

Regular fractionated radiotherapy -delivers 1.8–2 Gy single fraction per day, 5


times weekly. • Accelerated fractionation – aims to shorten the treatment time by
delivering larger than standard size fractions five times weekly or more than five
fractions per week of 2 Gy. Multiple fractions may also be given daily.

251. A 35 yrs old patient from Bihar comes with generalized bone pain and
was diagnosed to have Fluorosis. Fluorosis of bone is commonly associated
with?

a) Osteosclerosis
b) Osteomalacia
c) Osteopetrosis
d) Osteochondrosis

Exposure to very high fluoride over a prolonged period results in acute to chronic
skeletal fluorosis Vague, diffuse aches, muscle weakness, chronic fatigue and
stiffness of joints with decreased range of motion are common initial symptoms.
These symptoms may be dismissed as functional but may, in fact, be early signs of
fluoride damage to tendinous insertions and ligaments as well as joint capsules.
During later stages, calcification of the bones takes place, osteoporosis in long
bones and symptoms of osteosclerosis where the bones become denser and
develop abnormal crystalline structure develop.
252. Patient with Right Hypochondrial pain, Water Lily Sign is seen on
Ultrasound, what is the diagnosis?
a) Entamoeba
b) Schistosoma
c) Ascariasis
d) Hydatid cyst

Cystic echinococcosis (CE), also known as hydatid disease, is caused by infection


with the larval stage of Echinococcus granulosus, a ~2–7-millimetre-long
tapeworm found in dogs (definitive host) and sheep, cattle, goats, and pigs. The
water-lily sign, also known as the camalote sign, is seen in hydatid infections
when there is detachment of the endocyst membrane which results in floating
membranes within the pericyst that mimic the appearance of a water lily
253. A 2-year-old child with sudden onset breathlessness, Wheezing and
Decreased breath sounds in right lung, Chest Radiograph shows diffuse
opacity in right hemithorax, what is probable diagnosis?
a) Bronchial asthma
b) Pneumonia
c) Foreign body aspiration
d) Pneumothorax

Aspirated foreign bodies most commonly are lodged in the right main stem and
lower lobe. Aspiration has been documented in all lobes, including the upper lobes,
though with less frequency

254. Prophylactic craniospinal irradiation is done for which cancer?


a) Seminoma
b) Small cell lung ca
c) Prostate cancer
d) Hodgkins lymphoma

Prophylactic cranial irradiation is now known to improve survival to a significant


degree in small-cell lung cancer (SCLC) patients; this is in addition to its established
role in preventing the disabling symptoms of brain metastases.

255. A Patient with Brain Metastasis had to undergo Head and Neck
Irradiation, what is the most common delayed symptom with Head and Neck
Irradiation?
a) Xerostomia
b) Dysphagia
c) Dysgeusia
d) Anhidrosis

The most common long-term complication of radiation therapy (RT) and


chemoradiotherapy for head and neck cancer is xerostomia, which is the result of
damage to the salivary glands. The magnitude of this damage is dose dependent

256. Identify the given X-RAY?

a) Hydropneumothorax
b) Pneumothorax
c) Pneumo-mediastinum
d) Lower lobe pneumonia

Hydropneumothorax is an abnormal presence of air and fluid in the pleural


space. Even though the knowledge of hydro-pneumothorax dates back to the days
of ancient Greece, not many national or international literatures are documented.

257. A Patient with tingling, numbness and paresthesias in his lower


extremities, suggestive of nerve root Compression. From the given
radiological image which disc is likely to be herniated?
a) L1
b) L2
c) L3
d) L4
258. A 35 years old female known case of HIV, with CD4<300, presents with
fever, dyspnea, cough, and x-ray has been taken shows Causative Organism
is?

a) Staphylococcus
b) Streptococcus
c) Histoplasmosis
d) Miliary TB

Miliary TB is a potentially fatal form of TB that results from massive


lymphohematogenous dissemination of Mycobacterium tuberculosis bacilli. The
epidemiology of miliary TB has been altered by the emergence of the human
immunodeficiency virus (HIV) infection and widespread use of immunosuppressive
drugs
259. Identify the organism in the diagram?

a) Microsporium gypseum.
b) Microsporium canis
c) Trichophyton Epidermidis.
d) Trichophyton tonsurans

Microsporum canis is a pathogenic, asexual fungus in the phylum Ascomycota that


infects the upper, dead layers of skin on domesticated cats, and occasionally dogs
and humans. The species has a worldwide distribution
260. A 60-year-old woman, a known case of lung cancer complains of shortness of
breath. On examination her neck veins are engorged which showed a gross
elevation of JVP, heart sounds are distant and breath sounds appear reduced on the
left side infra scapular area. Requisition for CXR is done and is shown below.
Comment on the diagnosis of the patient.

a) Congestive heart failure


b) Bilateral pleural effusion
c) Cardiac tamponade
d) Superior venacava syndrome
Cardiac tamponade is a serious medical condition in which blood or fluids fill
the space between the sac that encases the heart and the heart muscle. This
places extreme pressure on your heart. The pressure prevents the heart's ventricles
from expanding fully and keeps your heart from functioning properly. Conventional
radiographs of the chest in patients with cardiac tamponade may demonstrate
an enlarged cardiac silhouette with or without an epicardial fat pad sign suggesting
a pericardial effusion

261. A boy fell down while playing and the nose is deviated but septum was
found normal, what is the next step done?

a) Lower the edema then closed reduction after 7 days


b) Closed reduction and automatically swelling decreases
c) Septorhinoplasty
d) Open reduction

Initially medications given to treat the edema and close reduction done after 7 days.
262. Where is the marked structure placed in the heart to activate the device
present here?

a) AVN
b) SAN
c) Bundle of His
d) Purkinje fibres

263. A 15-year-old boy playing football has an injury to the leg but now
complains of deep bone pain and high grade fever. A x ray was taken. What is
the diagnosis?
a) Osteochondroma
b) Ewings sarcoma
c) Osteosarcoma
d) Osteochondroma

Typically, radiographic appearance of Ewing's sarcoma is that of a permeative


destruction of bone. It is associated with a soft tissue mass. Lesions can cause a
lamination type of periosteal reaction (onion skinning) or vertical spiculation

264. A 43-year-old male returned from abroad and after a week develops
breathing difficulty with loss of smell and taste. Which is the next best line of
investigation?
a) Sputum examination
b) Nasopharyngeal swab
c) CT chest
d) Bone marrow biopsy

The patient shows the symptoms of SARS COVID-19, the next appropriate step for
the investigation would be nasopharyngeal swab

265. All the following measures are useful against COVID 19 except?
a) 2% Glutaraldehyde
b) 70% ethanol
c) Face mask
d) Hand wash

Hand wash, Face mask, 70% ethanol are all effective against the spread of COVID-
19.
2% glutaraldehyde is the disinfectant used for hospital instruments
266. Identify the image given below?

a) Meter Dose inhaler


b) Capsule inhaler
c) Rotahaler
d) Dry powder inhaler

Rotahaler is a plastic inhalation device which is breath activated. This means


when you inhale, the Rotahaler releases medication from the Rotacap. When inhaled
correctly, the medication has a better chance to reach the small airways. This
increases the medication's effectiveness.

267. In FAST which of the following needs immediate laparo-exploration?

a) 1 and 2
b) 2 and 3
c) 1 and 3
d) All

1 and 3 are the regions which immediate exploration is needed in FAST


DEC 2020
ANATOMY
1. What is the arrow marked structure in the following image?

A. Thalamus
B. Caudate nucleus
C. Lentiform nucleus
D. Claustrum
Answer: C
Explanation:
This is the transverse section of cerebrum
2. Which of the following nerve is involved in the given condition?

A. Dorsal Scapular Nerve


B. Long thoracic Nerve
C. Suprascapular Nerve
Subscapular Nerve Answer: B
Explanation: Winging of Scapula
 Medial border of scapula becomes more prominent
 Injury to long thoracic nerve from nerve root C5, C6, C7
supplying to serratus anterior muscle,
 Injury causes the condition called winging of scapula
3. Nerve involved in injury to the medial Epicondyle of Humerus?

A. Axillary nerve
B. Ulnar nerve
C. Median nerve
D. Radial nerve
Answer: B
Explanation: from the following images of nerves related to humerus, we can
clearly see that the ulnar nerve passes through the medial epicondyle.

4. Muscle attached to the arrow marked structure?

A. Gluteus maximus
B. Gluteus medius
C. Gluteus minimus
D. Iliopsoas
Answer: D
Explanation: the muscle attached to lesser trochanter is Iliopsoas whereas in
the greater trochanter , Gluteus medius
5. Identify the type of joint that is marked by the arrow?
A. Syndesmosis
B. Primary Cartilaginous joint
C. Secondary Cartilaginous joint
D. Synchondrosis
Answer: C
Explanation: Secondary Cartilaginous joint are present in the midline of the
body except Symphysis Menti

6. Which of the tarsal bone is marked by arrow in the image?

A. Cuboid
B. Talus
C. Navicular
D. Cuneiform
Answer: C
Explanation:

7. Which of the following nerve is involved in eversion of foot and loss of


sensation in dorsal aspect of foot?

A. Deep peroneal nerve


B. Superficial peroneal nerve
C. Sural nerve
D. Saphenous nerve
Answer: B
Explanation:
8. Where does the structure marked in the following image ends?

A. Junction of left jugular vein and subclavian vein


B. Brachiocephalic vein
C. Superior vena cava
D. Right subclavian vein
Answer: A
Explanation: the structure marked is the thoracic duct. And it opens at the
junction of left jugular vein and left subclavian vein.
9. ‘X’ marked tendon in the given image is?

A. Flexor digitorum superficialis


B. Flexor digitorum profundus
C. Flexor carpi radialis
D. Flexor carpi ulnaris
Answer: C
Explanation: the tendon marked is in lateral side or radial side and it is Flexor
carpi ulnaris
10. Nerve passing behind the marked structure…

A. Radial nerve
B. Ulnar nerve
C. Median nerve
D. Musculocutaneous nerve
Answer: B
Explanation:
from the following images of nerves related to humerus, we can clearly see
that the ulnar nerve passes through the medial epicondyle.
11. Tingling sensation and paresthesia on medial side of hand. Which structure
most likely to be affected?

A. A
B. B
C. C
D. D
Answer: A
Explanation:
 A is Brachial plexus that is marked here
 B is Subclavian artery and vein
 C is Trachea
 D is Clavicle
BIO CHEM
1. Enzyme deficiency in von gierke’s disease?

(a) Glucose-6-phosphatase
(b) Branching enzyme
(c) Acid maltase
(d) Glycogen Phosphorylase

Ans. (a) Glucose-6-phosphatase

SIGNS AND SYMPTOMS of Glycogen storage disease type 1-


Glycogen accumulation in liver and renal tubule cells(hepatomegaly & renomegaly)
Hypoglycemia
Lactic acidemia
Ketosis
Hyperlipemia
Disease Enzyme Deficient
Carbohydrates
Type I/Von Gierke’s Disease Glucose – 6-phosphatase
Type II/Pompe’s Disease Acid Maltase
Type III/Cori’s Disease Debranching Enzyme
Type IV / Anderson’s Disease Branching Enzyme
Type V/Mc Arlde’s Disease Muscle Glycogen Phosphorylase
Type VI/ Her’s Disease Hepatic Glycogen Phosphorylase
Essential Fructosuria Fructokinase
Fructose Intolerance Aldolase B
Galactossemia Galactose-1-phosphatase uridyl
transferase

2. Patient presents with drying of skin and appears as shown below. Identify the
vitamin deficiency?

(a) Biotin
(b) Niacin
(c) Riboflavin
(d) Ascorbic acid
Ans. (b) Niacin deficiency

Niacin (vitamin B3) deficiency results in a condition known as pellagra. Pellagra


includes the triad of dermatitis, dementia, and diarrhea and can result in death.
Niacin deficiency can occur through genetic disorders, malabsorptive conditions, and
interaction with certain medications.

3. Complex IV of ETC is inhibited by?

(a) Oligomycin
(b) Malonate
(c) Cyanide
(d) Rotenone
Ans. (c) Cyanide

Complex IV, Cytochrome Oxidase:


Components: Cu
Inhibitors: Cyanide, Carbon mono-oxide, H2S

Complex I, NADH-Q Reductase:


Components: NADH DH, FMN (flavin mononucleotide)
Inhibitor: Rotenone (Rat Poison and Insecticide)
Complex II, Succinate-Coenzyme Q Reductase:
Components: succinate dehydrogenase, FAD
Inhibitors: Malonate

Complex III, Cytochrome Reductase:


Components: b cytochromes, Fe-S-protein center, cytochrome c1
Inhibitor: Antimycin A (Antibiotic)

4. A 40-year-old man presented with black pigmentation of pinna and sclera of


eyes. The color of urine changes to black on long standing. What is the
diagnosis?

(a) Alkaptonuria
(b) Phenylketonuria
(c) Maple syrup urine disease
(d) Hartnup’s disease

Ans. (a) Alkaptonuria

Alkaptonuria is a rare genetic metabolic disorder characterized by the


accumulation of homogentisic acid in the body. Affected individuals lack enough
functional levels of an enzyme required to breakdown homogentisic acid. Affected
individuals may have dark urine or urine that turns black when exposed to air.
5. Which of the following is not a micromineral?

(a) Zinc
(b) Copper
(c) Calcium
(d) Iodine

Ans. (c) Calcium

6. Nitrogenous waste is excreted from the body in the form of?

(a) Glutamine
(b) Urea
(c) Uric acid
(d) Nitric oxide

Ans. (b) Urea

Nitrogenous waste is excreted from the body in the form of uric acid. The
nitrogenous waste that is generated in the body tends to produce toxic ammonia if
remained in the body it must be excreted.
Urea and uric acid are the most common nitrogenous waste products in terrestrial
animals; freshwater fish excrete ammonia and marine fish excrete both urea and
trimethylamine oxide.
FORENSIC

1. Patient’s relatives are complaining about patient’s death and filed a case against
the doctor. Which IPC section do you think will be appropriate to punish the
medical practitioner?

(a) IPC 304 A


(b) IPC 304 B
(c) IPC 300
(d) IPC 302

Ans. (a) IPC 304 A

Causing death by negligence. --Whoever causes the death of any person by doing
any rash or negligent act not amounting to culpable homicide, shall be punished with
imprisonment of either description for a term which may extend to two years, or with
fine, or with both.

2. Active principle of the Image shown below?

(a) Datura
(b) Hyoscine
(c) Thebaine
(d) Tetrahydro-cannabinol

Ans. (b) Hyoscine (Image  Datura)

Datura stramonium, known by the common names thorn


apple, jimsonweed (jimson weed), devil's snare, or devil's trumpet, is a species
of flowering plant in the nightshade family Solanaceae.
The active principles are alkaloids such as hyoscine (scopolamine), hyoscyamine,
and atropine.
3. A female who died within 3 years of marriage &this was the case of suspected
dowry. Who should perform the inquest in this case?

(a) Magistrate
(b) Police
(c) Village headman
(d) Judge

Ans. (a) Magistrate


Under section 176 of Criminal Procedure Code, 1973, Magistrate inquest is done in
the following cases: Dowry Death. Rape under police custody or government
custody, Death in Jail. Death as a result of Public Shooting.

4. A dead fetus was found in water. The crown heel length was found to be 25 cm.
Light hairs were present on the skull with ossification center. Approximate age of
fetus is –

(a) 5 months
(b) 7 months
(c) 9 months
(e) 3 months

Ans. (b) 5 months

RULE OF HASSE
 Rule of hasse is used to determine the age of the foetus from crown to heel
length in cm
 During first five months , length in cm = ( age in months ) 2
 After age of > 5months, Age in months = ( length in cm )/5

Crown-rump length (CRL) is the measurement of the length of human embryos


and fetuses from the top of the head (crown) to the bottom of the buttocks
(rump). It is typically determined from ultrasound imagery and can be used to
estimate gestational age

5. A man was found to be agitated and shouting at the airport. Later on he fell down.
On further checking, his bag was found to have enema apparatus and laxatives.
All of the following are true regarding this case except?

(a) Abdominal scan is Indicated


(b) The person is malingering
(c) He is at a risk of poisoning
(d) It’s a case of body packer’s syndrome
Ans. (b) The person is malingering

A body packer is someone who carries drugs such as heroine or cocaine, packed in
rubber or plastic, in his/her body in order to smuggle them. These people can
present with symptoms that vary from mild abdominal complaints to respiratory
insufficiency and even death. Physical examination and additional radiology tests are
helpful for the diagnosis. Any packages can usually be seen on a plain abdominal X-
ray.

6. A person was found dead in bushes with tied arms and legs, cyanosis of the
nails, scratches over face, neck and chin, laceration near lips was noticed. Which
of the following is not the probable mode of death in this case?

(a) Throttling
(b) It’s a Homicide
(c) Asphyxia
(d) Case of a suicide

Ans. (d) Case of a suicide

Suicide is death caused by injuring oneself with the intent to die. A suicide
attempt is when someone harms themselves with any intent to end their life

Homicide is an act of a human killing another person.[1] A homicide requires only


a volitional act that causes the death of another, and thus a homicide may result
from accidental, reckless, or negligent acts even if there is no intent to cause harm

7. A gunshot case brought for autopsy shows an entry wound with inverted margins
& surrounded by blackening and tattooing. Burning & singeing of hairs is noted.
What is the range of firearm weapon?

(a) Near
(b) About 2 meters
(c) Contact
(d) Distant

Ans. (a) Near

Depending on the distance of fire, the wound can also classify as contact wounds –
again divided into firm-contact and loose contact – near-contact or close-range
wounds, mid-range or intermediate-range wounds, distant or far-range wounds, and
indeterminate wounds.
Contact wounds are self-explanatory and are diagnosable by the presence of
muzzle-imprint on the skin. Near-contact or close-range wounds are identifiable
by the presence of flame burns and singeing of hair. Mid-range or intermediate-
range wounds do not have flame burns or singeing of hair but will show the presence
of smoke, gas, and unburnt particles. Distant or far-range wounds will be embedded
with any accompanying components other than the lubricant forming a grease collar.
Indiscriminate wounds are atypical wounds beyond the range of the weapon with an
irregular shape due to yawing (changing in the axis and direction of flight of the
projectile).

8. A piece of cloth was obtained from a crime scene with some spots. On
examination, yellow colored needle shaped crystals were found. Which body
secretions it can be?

(a) Serum
(b) Semen
(c) Saliva
(d) CSF
Ans. (b) Semen

Barberio test was invented by Barberio in the year 1905. When the questioned
stain is allowed to react with picric acid it leads to the formation of yellow
needle shaped spermine picrate crystals, including the presence of seminal stain.

9. Which of the following produces catecholamine storm as seen with scorpion


sting?
(a) Spider
(b) Tick
(c) Sea Urchin
(d) Ciguatera
Ans. (a) Spider

Severe scorpion envenoming causes an autonomic storm producing multi-system


organ-failure (MSOF) and death
Most of the offensive and defensive zootoxins that organisms have evolved for
shutting down the nervous systems of predators and prey e.g., the venoms produced
by spiders, scorpions, snakes, fish, bees, sea snails and others work by plugging ion
channel pores.

Scorpion sting
- More than > 10 Species are present
- Indian red scorpion is highly venomous
- Scientific name → Mesobuthus tumulus
- Nature of venom → resembles snake venom

But fatality is less as the amount is less.

Action:
- Scorpion venom acts on Na+& K+ Channels

It Stimulates Sympathetic & Parasympathetic system



Uncontrolled release of Catelolemines into the circulations

(Autonomic Storm)
1. Pain → the tap sign
2. Paresthesia
3. Systemic Symptoms → Vomiting, Sweating, Salivation, chest pain,
anxiety, cardiac arrhythmias

Rx:-
1. Immobilise
2. Pain relief
3. Prazosin
4. Scorpion antivenom (If available)

10. Dead body of a 12yr old child was found. Which bone would be best for gender
assessment?

(a) Skull
(b) Femur
(c) Pelvis
(d) Mandible
Ans. (c) Pelvis

Krogman’s Table
%age of accuracy in determination of sex from bone.
Pelvis 95%
Skull 90-92%
Long Bones 80%
Pelvis + Skull 98%
All 206 bones 100%
• Best bone for determination of sex is Pelvis

11.A case of RTA was brought for autopsy with stiffness of neck, hands, both upper
and lower limbs. What is this condition called?

(a) Rigor mortis


(b) Cadaveric spasm
(c) Heat stiffness
(d) Cold stiffness
Ans. (a) Rigor mortis

Rigor mortis: Literally, the stiffness of death. The rigidity of a body after death. ...
Rigor mortis is due to a biochemical change in the muscles that occurs several hours
after death, though the time of its onset after death depends on the ambient
temperature

12. A woman tried to fake the pregnancy & delivery. After sometime she brought a
baby claiming it to be her neighbor’s child and trying to blackmail him for getting
benefit in property. Such kind of child is called as?
(a) Suppositious child
(b) Spurious
(c) Superfecundation
(d) Superfetation
Ans. (a) Suppositious child

Supposititious children are fraudulent offspring. These arose when an heir was
required and so a suitable baby might be procured and passed off as genuine.
PATHO

1. A 55-year-old diabetic patient has chronic renal failure and is undergoing


hemodialysis. What is the associated amyloidosis?

(a) AL
(b) AA
(c) Beta 2 microglobulin
(d) ATTR

Ans. Beta 2 micro globulin

Beta-2 ̶ microglobulin (beta-2m) amyloidosis is a disabling condition that affects


patients undergoing long-term hemodialysis (HD) or continuous ambulatory
peritoneal dialysis (CAPD).

2. A person develops pruritic rashes every time on eating sea food within one hour.
Which type of hypersensitivity is this?

(a) I
(b) II
(c) III
(d) IV

Ans. (a) I

Type I hypersensitivity is also known as an immediate reaction and involves


immunoglobulin E (IgE) mediated release of antibodies against the soluble antigen.
This results in mast cell degranulation and release of histamine and other
inflammatory mediators.
Examples include anaphylaxis and allergic rhino conjunctivitis.

3. A nurse got needle prick injury from a HIV positive patient while doing blood
sampling. Which test would be the confirmatory test to rule out the infection?

(a) ELISA
(b) P24 assay
(c) Western blot
(d) Blood culture

Ans. (b) P24 assay

The p24 test identifies actual HIV virus particles in blood (p24 is a capsid
structural protein which makes up a protein 'shell' on the surface of the HIV virus).
However, the p24 test is generally only positive from about two to three weeks after
infection with HIV.

4. A patient comes for organ transplantation. He has a twin brother who is a match
donor for
him. Which type of grafting it would be considered as?

(a) Isograft
(b) Allograft
(c) Autograft
(d) Xenograft

Ans. (a) Isograft

An Isograft is a graft of tissue between two individuals who are genetically


identical(i.e. monozygotic twins). ... Monozygotic twins have the same major
histocompatibility complex, leading to the low instances of tissue rejection by the
adaptive immune system.

5. A 83-year-old female suffered with heart disease and with cardiac atrophy. On
H/E, perinuclear brown colored deposits were seen. What could be the pigment
responsible for this?

(a) Hemosiderin
(b) Melanin
(c) Lipofuscin
(d) Iron

Ans. (c) Lipofuscin

Lipofuscin (age pigment) is a brown-yellow, electron-dense, auto fluorescent


material that accumulates progressively over time in lysosomes of post mitotic cells,
such as neurons and cardiac myocytes.

6. A child presents to OPD with complaints of fatigue. On further examination


anemia and hepatosplenomegaly are seen. His
peripheral blood smear is shown in the given
Image. What is the diagnosis?
(a) Thalassemia major
(b) PNH
(c) AOCD
(d) Sickle cell anemia

Ans. Thalassemia  Image shows Target cells

Thalassemia is an inherited blood disorder that causes your body to have less
hemoglobin than normal. Hemoglobin enables red blood cells to carry oxygen.
Thalassemia can cause anemia, leaving you fatigued.

Numerous target cells are present in this patient with hemoglobin E and beta
thalassemia trait. Target cells, or codocytes, have an excess of cell membrane
relative to cell volume. Macrocytic target cells can be seen in liver disease, and
microcytic target cells may be seen in thalassemia.

7. The Image below shows origin from which of the following cells?

(a) B cells
(b) CD4 T cell
(c) NK cell
(d) CD8 T cell
Reed–Sternberg cells (also known as lacunar histiocytes for certain types) are
distinctive, giant cells found with light microscopy in biopsies from individuals with
Hodgkin lymphoma. They are usually derived from B lymphocytes, classically
considered crippled germinal center B cells.

8. Identify the Image and diagnose accordingly?

(a) IDA
(b) Thalassemia major
(c) Myelodysplastic syndrome
(d) Myelofibrosis

Ans. (d) Myelofibrosis

The presence of teardrop-shaped cells may indicate: Myelofibrosis. Severe iron


deficiency. Thalassemia major. Cancer in the bone marrow.

A marked increase of dacrocytes is known as dacrocytosis. These tear drop cells are
found primarily in diseases with bone marrow fibrosis, such as: primary
myelofibrosis, myelodysplastic syndromes during the late course of the disease, rare
form of acute leukemias and myelophthisis caused by metastatic cancers.

9. Owl eye inclusions are seen in?


(a) HSV
(b) HHV
(c) EBV
(d) CMV

Ans. (d) CMV

CMV is a herpesvirus, has double-stranded DNA, and renders infected cells 2–4
times the size of surrounding cells. These cytomegalic cells contain an eccentrically
placed intranuclear inclusion surrounded by a clear halo, with an “owl's-eye”
appearance.

10. A 50-year-old man presented with gum bleeding. Peripheral smear shows
marked leukocytosis with 70% cells showing MPO positivity. What could be the
diagnosis?

(a) AML
(b) ALL
(c) CML
(d) CLL
Ans. (a) AML

11. Protein in Alzheimer’s?

(a) Apo C17


(b) Tau protein
(c) Keratin
(d) Collagen

Ans. Tau protein

Alzheimer's disease is thought to be caused by the abnormal build-up of proteins in


and around brain cells. One of the proteins involved is called amyloid, deposits of
which form plaques around brain cells. The other protein is called tau, deposits of
which form tangles within brain cells.

12. What is the location of lactate dehydrogenase inside the cell?

Nucleus
Cytoplasm
Mitochondria
Ribosomes

Ans. Cytoplasm

Lactate dehydrogenase (LDH) catalyzes the equilibrium reaction of pyruvate to


lactate. The activity of serum LDH is due to the presence of the enzyme released
from damaged organs and tissues such as liver, heart, skeletal muscle, erythrocytes,
etc. because LDH is located in the cytoplasm of the cells. Therefore, the activity of
LDH is useful for screening for the existence of cell injuries, estimation of damaged
tissues, and evaluation of treatment of diseases.

13. A 10 year old boy presented with a swollen knee after suffering a trauma. Which
of the following is the likely condition seen in this patient?

Von Willebrand disease


Factor 8 deficiency
Immune thrombocytopenia
Vitamin K deficiency

Ans. Factor 8 deficiency

Hemophilia is a disease that prevents blood from clotting properly. A clot helps stop
bleeding after a cut or injury. In factor VIII deficiency (hemophilia A), the body
doesn't make enough factor VIII (factor 8), one of the substances the body needs
to form a clot.

SPM
1. Identify the logo given in the image below:

(a) Allopathic private clinic practitioner


(b) First aid
(c) Red cross emblem
(d) Suraksha clinic
Answer: D
Explanation: the above image is the logo for Suraksha clinic. For RTI (reproductive
tract infection), STI (sexual tract infection) established by NACO (National Aids
Control Organization)

2. Which of the following state or Union territory in India is Rabies free:

A. Sikkim
B. Kerala
C. Jammu & Kashmir
D. Andaman & Nicobar

Answer: D
Explanation: Andaman & Nicobar Islands and Lakshadweep Island from our country
are currently Rabies-free. As import of animals from these are avoided and hence it
was easy to eradicate rabies in these states. Remember water is an effective barrier.

3. You are a medical officer and now you have to check an ANM worker’s ability
to do immunization who has been given training. She is having cotton,
syringe-needle and lunch box after immunization of a child. Which of the
following statement is correct?

A. Cotton swab in yellow bag


B. Food waste in blue box
C. Used vial in translucent white container
D. Syringe with needle in translucent white container
Answer: A
Explanation: Soiled waste like cotton, cloth etc., goes in yellow bag

4. NITI Aayog has replaced which of the following:

A. National Board of Examinations (NBE)


B. National Institute of Health & Family Welfare (NIHFW)
C. Medical Council of India (MCI)
D. National Planning Commission (NPC)

Answer: D
Explanation: NITI (National Institute Transforming India). It is actually new
name of pre-existing NPC (National Planning Commission). It was established
in 1950 but in 1st January 2015 NITI Aayog was formed. Comprising of Prime
Minister as the Chair Person and along with him the chief ministers of all the
states, Lieutenant Governor of Union territories.

5. In a population, a screening test was used for a disease, from the image
below what does B-C represent:

A= Disease onset
B= First possible
detection
C= Critical point
A. Lead time D= Usual time of
B. Screening time diagnosis
C. Incubation time E= final outcome
D. Latent period
Answer: B
Explanation: Screening time
B-D = lead time also known as advantage gained by a screening test.

6. Asha worker in rural areas is designated for how much population?

A. 3000
B. 5000
C. 1000
D. 2000
Answer: C
Explanation: ASHA (Accredited Social Health Activist). A worker under
National Rural Health Mission (NRHM) 2005-2012. From 2013 called as NHM
(national Health Mission)
Was designated for
 1 per village
 1 per 1000
 From 2013, 2 per village of a population of 1000

 She is supposed to be 25–45-year-old female


 Resident of the same village
 Should have passed 10th
 Supposed to get 23 days of training in PHC given by Anganwadi
worker (AWW) & Auxillary Nurse Midwife (ANM)
 She is selected by village panchayat or Gram Sabha
 Major impact in reduction of infant mortality rate

7. Calculate Neonatal mortality rate from the data given below. Total live births
were 4000
Total death among day 0-7 :40
Total death among day 7-28 :40
Total death >than 1 month :40
Perinatal death :40
A. 10
B. 20
C. 30
D. 40
Answer: B
Explanation: A neonate is one who is having an age of 0-28 days. And is
Divided into 2
i. Early Neonatal Period = 0-7 days (40 from question)
ii. Late Neonatal Period = 8-28 days (40 from question)
Therefore, neonatal period = early neonatal period + late neonatal period

Formulae for Neonatal mortality rate (NNMR) = Neonatal deaths X 1000


Live birth
From the question = 40 + 40 X 1000
4000
= 80 X 1000
4000
NNMR = 20.

8. On patients in Rehabilitation, A research study was conducted where impact


of addiction among young adults was assessed on their subsequent divorce.
Among two groups of study subjects, divorced and non-divorced, history of
alcohol/drug use was assessed. What was the type of study done?
A. RCT
B. Cohort study
C. Case Control study
D. Descriptive study
Answer: C
Explanation: here people who are divorced are cases and people who are not
divorced are controls and we go in a backward direction and we ask them
same question. And we can calculate the strength of association (Odds
Ratio).

9. Sample Registration system includes all of the following except:

A. Maternal Mortality rate


B. Infant Mortality rate
C. Birth rate
D. Death rate
E. All of the above
Answer: E
Explanation: all of the options mentioned above are being included in the
sample registration system. So answer is E

10. Mortality is taken into consideration in which of the following?


A. Gross Reproductive rate (GRR)
B. Net reproduction rate (NRR)
C. Total fertility rate (TFR)
D. Completed family size

Answer: B
Explanation:
 Total fertility rate (TFR) = total no of children born to a woman in her
entire reproductive life.
 Gross Reproductive rate (GRR) = total no of girl children born to a
woman in her entire reproductive life.
 Net reproduction rate (NRR) = total no of girl children born to a woman
in her entire reproductive life taking in account their mortality
 Completed family size is simply a synonym for TFR.

11. For disposal of which of the following, a white bag is used?

A. Anatomical waste
B. Sharp waste
C. Glassware waste
D. Plastic waste
Answer: B
Explanation: A white container, puncture proof and sometimes translucent:

12. For a female with Vaginal discharge, STD color kit code would be?
A. Kit 1 Grey
B. Kit 2 Green
C. Kit 3 White
D. Kit 4 Blue
Answer: B
Explanation

13. The STD kit color for Male urethral Discharge:


A. Kit 1 Grey
B. Kit 2 Green
C. Kit 3 white
D. Kit 4 Blue
Answer: A
Explanation:

14. After which phase of clinical trials, a new drug is introduced in the market?
A. Phase 1
B. Phase 2
C. Phase 3
D. Phase 4

Answer: C

Explanation:
Phase 1 For safety and toxicity profile of a Done in HHV (healthy human
drug volunteers)
Phase 2 For effectiveness Done in patients
Phase 3 For comparison trial with existing Also done in patients
drug
Phase 4 For long term, rare side effects Done in patients
 Phase 4 is also called as Post-Marketing Surveillance and is the
longest phase of a trial
 Phase 3 is RCT
 Maximum failure is reported from phase 2
 Maximum tolerated dose (MTD) of a drug is assessed in Phase 1
 Under new guidelines, before phase 1, we have a phase 0, where drug
is tested in very small doses in very small number of healthy human
volunteers to access for its efficacy and non-toxicity profile also known
as phase 0 micro dosing.
15. Parameter used to test for confirmatory diagnosis when a Health worker nurse
got an accidental prick with HIV+ needle would be?
A. ELISA
B. p24 Antigen
C. Western Blot assay
D. PCR
Answer: C
Explanation: when we get needle prick injury, we get 0.1-0.3% of
transmission, it is also a big risk of transmission
-Screening is done by ELISA. It has very high sensitivity, but it lacks
specificity and its main role is to detect antibodies
- if ELISA is positive then confirmatory diagnosis is done by WBA(Western
Blot Assay) according to Indian Program, it is a protein based test and it a
high specificity, it is based on p24 and GP41
-p24 antigen test can be used in window period before seroconversion
-Another test use din window period is HIV RNA Genomic sequencing (NAAT)
Nucleic acid amplification test.

16. One student developed abdominal pain, jaundice and fever. Soon other
students presented with similar symptoms, they are all from same boarding
school and have been eating from the same canteen. Hostel Medical officer
will investigate which of the following parameters to establish diagnosis?
A. IgM for Hepatitis A
B. IgG for Hepatitis A
C. IgM for Hepatitis B
D. IgG for Hepatitis B
Answer: A
Explanation: out of the all types of hepatitis, A & E are the ones with fecal oral
transmission and A is the most common cause of hepatitis in children and
adults, since we are doing immediate test so it has to be IgM antibody of
hepatitis A.
Hepatitis A is caused by Enterovirus 72 which is from Picornaviridae family.
And in children it is sub-clinical. Incubation period of 15-45 days and period of
infectivity is 2 weeks prior to Jaundice till 1 week after. Elisa is used as
confirmatory to find the antibody.
A Patient is presented with rash and diarrhea for 2 months, from the image
given below, find the deficiency
17. A Patient is presented with rash and diarrhea for 2 months, from the image
given below, find the deficiency?

A. Vitamin B12 deficiency


B. Vitamin C deficiency
C. Niacin deficiency
D. Vitamin D deficiency
Answer:
Explanation: patient is having dermatitis, diarrhea for 2 months, the patient
has appearance of CASAL’s Necklace. So, we conclude that it is Pellagra.
Pellagra is characterized by 4D’s
o Diarrhea
o Dermatitis
o Dementia
o Death
Pellagra is caused due to deficiency of vitamin B3 or Niacin. Frequently seen in
Maize/Jowar eaters due to deficiency of Tryptophan which leads to pellagra due to
deficiency of niacin. Excess of leucine inhibits the conversion of tryptophan to niacin
(conversion ratio of tryptophan to niacin is 60:1) in the body. Here the main culprit is
Leucine.

18. Minimum thickness of Lead Apron required is:


A. 0.1 mm
B. 0.2 mm
C. 0.3 mm
D. 0.5 mm
Answer: D
Explanation:
o 0.25 mm used for minimal procedures like mammography
o 0.50 mm is most widely used and is recommended because it
attenuates >90% of scattered radiation
o 1.00 mm becomes very heavy

19. A lactating woman came to OPD 6 weeks after delivery for check-up and to
get advice for the contraception, which of the following is not advised?
A. Combined OCP
B. Norplant
C. IUCD
D. Mini-pill
Answer: A
Explanation: Combined OCP suppresses lactation due to its estrogen
component. So, we advise the mother to opt for mini-pill (lactation pill) and it
has only progesterone in the form of LNG (Levonorgestrel).

20. A low socio-economic female worker who is 22 years-old presents within 8


hours of sexual assault on the 13th day of cycle and asked for emergency
contraception. What will be the best advice for the patient?
A. LNG 1.5 milligram
B. Misoprostol 800 mcg stat
C. Mifepristone 3 doses
D. OCP from 1st day of next cycle
Answer: A
Explanation: 13th day of cycle. So, high chances of getting pregnant, out of the
following options the best one would be LNG 1.5 milligram i.e., a
progesterone only pill-POP (minipill) intraceptive emergency contraceptive
used in the program.
-misoprostol and mifepristone are used in MMA (Medical methods of abortion)
utilized after pregnancy has taken place.
- OCP from next cycle is not recommended because she will be already
pregnant

21. Contraindication of the device from the following image includes:


A. Breast feeding
B. Monogamous
C. Hypertension
D. PID
Answer: D
Explanation:
Contra-Indications of IUCD:
o Absolute contraindications
 Pregnancy
 PID
 Vaginal bleeding
 Ectopic pregnancy
 Cancer of cervix, uterus, adnexa
o Relative contraindications
 Anemia
 Menorrhagia
 H/O PID
 Uterine Malformation
 Unmotivated Female
 Fibroids

22. A child with an IQ level of 55, He is having which of the following?


A. Mild MR
B. Moderate MR
C. Severe MR
D. Normal MR
Answer: A
Explanation:
IQ = Mental Age X 100
Chronological age
Mild MR is the most common form of MR in India and Down Syndrome is the
most common cause.
 Normal IQ is > or = to70
 Mild MR is 50-69
 Moderate MR is 35-49
 Severe MR is 21-34
 Profound MR is < or = to 20

23. In Zika virus, the vector is?


A. Sandfly
B. Aedes aegypti
C. Culex
D. Anopheles
Answer: B
Explanation: Zika is transmitted by 4 ways they are
 Aedes Mosquito.
 Sexual route transmission.
 Blood transmission.
 Mother to child transmission.

24. Used to prevent Covid-19 among health workers, Relative risk for two
vaccines, Relative risk for two vaccines A and B is 0.5 and 2.0 respectively.
Identify the graph(s) which represent them correctly?

A. A & B
B. A & C
C. B only
D. None of the above
Answer: A
Explanation: RR risks of 2 vaccines were given. It is used to measure strength
of association in a Cohort study. It can be
RR > 1, Positive association
RR = 1, No Association
RR < 1, Inverse Association
From the options both graphs A & B are correct
25. What is Urban Heart?
A. Urban Health Equity Assessment and Response Tool
B. Urban Health Evaluation assessment and Response Tool
C. Urban Health Equity Assessment and Response Technique
D. Urban Health Evaluation Assessment and Response Technique
Answer: A
Explanation: Tool used by WHO (World Health Organization) to identify and
reduce health inequities in the city

It has 3 components:
 Sound Evidence
 Intersectoral Action
 Community Participation
It is integrated into local planning cycle

26. The agency that collects data and publishes large scale surveys continuously
regarding morbidity, family planning, vital events?
A. AIIMS
B. Central Bureau of Health
C. National Sample Survey
D. Sample registration system
Answer: C
Explanation: National Sample Survey taken up by NSSO
(National Sample Survey Organization)

-It carries out large scale sample surveys that too in


diverse fields
- Mainly based on Household
-started in 1950 -till date 586+ reports have been shared.
27. Prevalence is associated with which of the following?
A. Sensitivity
B. Specificity
C. Positive predictive value
D. Accuracy
Answer: C
Explanation: -positive predictive value (PPV) is directly proportional to
Prevalence
PPV α Prevalence
-negative predictive value (NPV) is indirectly proportional to
prevalence
NPV α 1
Prevalence
PP
V

NPV

28. Best High-level Disinfectant is which of the following:


A. Phenol
B. Glutaraldehyde
C. Lysol
D. Cetrimide
Answer: B
Explanation:
 High level disinfectants include
o Glutaraldehyde
o 6% H2O2
o Chlorine Di-oxide
o Peracetic acid
o useful for
 Vegetative Microorganisms
 Lipid/Non-Lipid Viruses
 Mycobacteria
 Fungal Spores
 Few Bacterial Spores
 Intermediate level disinfectants also called as Tuberculocides includes
o Sodium Hypochlorite (1:50)
o Phenolic detergent solutions
 Used in Mycobacteria
 Most Viruses/ Bacteria
 Low level Disinfectants
o Quaternary ammonia compounds
o Sodium Hypochlorite (1:500)
o Ethyl isopropyl Alcohol
 Used for some viruses and bacteria

29. Newborn birth rate is 25 per 1000 population in a village with total population
of 5000. What is the total number of pregnancies per year?
A. 125
B. 138
C. 145
D. 250
Answer: B
Explanation: First we calculate
Expected births for 1000 = 25
So, for 5000 = 125 births
But in question total no of pregnancies is asked
In India we have about 10% of pregnancy wastage so we add that along with
125 here

So, 125 + 12.5 = 137.5 pregnancies that is from options


138 would be the exact answer

30. Under ESI after certification by medical officer, beneficiaries get 70% wages
for 3 months under?
A. Medical Benefit
B. Sickness Benefit
C. Disablement Benefit
D. Dependent’s Benefit
Answer: B
Explanation:

Employees State Insurance (ESI)


 Runs under the Ministry of labour
 Union Minister of labour is the chair person
 Employer (3.25%) and Employee (0.75%) both need to
contribute to avail the benefits under ESI
 Benefits
 For income < 21,000 per/month are beneficiaries
 Medical Care
 Sickness benefit (70% wages for 91 days or 3
months)
 If extended it becomes (80% of wages for 2
years)
 Disablement benefit is 90% wages depending on
temporary or permanent disability
 Dependent’s benefit is also 90%
Under ESI Funeral expense 15,000 is also given

31. In a group of 200 Hypertensive patients. First group of 100 follow medical
advice whereas second group of 100 follow lifestyle advice. There is test for
both the groups after 3 months. Which of the following is best test for
significance to compare Blood pressure levels in both the groups?
A. Paired Student’s T test
B. Unpaired Student’s T test
C. Fischer test
D. Chi-Square test
Answer: B
Explanation: Comparing Quantitative data in two groups, we use Unpaired
Student’s T test
If Comparing Quantitative data in same group before and after an intervention
we use Paired Student’s T test

32. Which of the following is included in NPCDCS?


A. Bronchial Asthma, Hypertension, CVD
B. Diabetes, Hypertension, Bronchial Asthma
C. CVD, Diabetes, Bronchial Asthma
D. CVD, Cancer, Diabetes, Stroke

Answer:
Explanation: Full form National Program for prevention and control of Cancer
Diabetes CVD’s Stroke (NPCDCS)
Through formation of NCD clinics, this was established in 100+ districts in
India
Now 450 districts in country have this and they have included COPD and CKD

33. A doctor is teaching an intern about knee reflex demonstration. What type of
learning is this?
A. Cognitive learning
B. Affective learning
C. Psychomotor learning
D. None of the above
Answer: C
Explanation: skills is associated with psychomotor learning

34. Under ICDS, Intersectoral coordination is found with?


A. NREGA
B. NRHM
C. NLEP
D. NACP
Answer: B
Explanation: National rural health mission
-ICDS-integrated child development services through Anganwadi
-Maternal and child health care is the main aim
-Beneficiaries are NRHM
-Gives nutritional support

35. Blood smear for malaria will be collected by which of the following health
worker?

A. Multipurpose worker Female


B. ANM
C. ASHA
D. Multipurpose worker Male

Answer: D
Explanation: Multipurpose health worker male has the main duty of collecting
blood smear for malaria.
ANAESTHESIA
1. Which is the following I.V. anesthetic drug used for day care surgeries?

(a) Ketamine
(b) Propofol
(c) Thiopentone
(d) Etomidate

Ans. (b) Propofol

Explanation: Day care anesthesia is involved in small procedures that are done in
one day, that is he has to be admitted, operated, recovered and discharged on the
same calendar day, so the anesthetic agent should be short acting, easily
metabolized and excreted and that is why we choose propofol. It is the anesthetic
agent of choice for day care. It has anti-emetic and anti-pruritic properties and has
not much side effects.
 Opioid of choice for day care: Remifentanil
 Inhalational agent of choice in day care: Sevoflurane

2. Which of the following drug is causes post-operative delirium and


hallucination?

(a) Ketamine
(b) Fentanyl
(c) Thiopentone
(d) Halothane

Ans. (a) Ketamine

Explanation: Waking up from ketamine anesthesia, the name of the phenomenon


we see in this condition is Emergence Delirium. Ketamine maintains heart rate, blood
pressure, and airway reflexes so can be used in stomach full patients and in
emergency during trauma

3. A patient after Gunshot injury presented to emergency department with


hypovolemia. Which of the following cannulas can be used for most rapid
blood transfusion?

(a) Grey
(b) Orange
(c) Pink
(d) Green
Ans. (b) Orange
Explanation:

4. Local anesthetic agent works by which of the following mechanism?


a. Blocking Sodium voltage gated channel
b. Blocking of Potassium channel
c. Opening Sodium voltage gated channel
d. Opening of potassium channel
e. Ans. (a) Blocking Sodium voltage gated channel
f. Explanation:
5. What type of anesthesia is required for dental surgery?

a. Field block
b. Nerve block
c. Infiltration
d. All of the above

Answer: D
Explanation: All of the above

 All these kinds of anesthesia can be given in dental surgery


 Local anesthetic agent given just on the site is infiltrative
 Or nerve block for example inferior alveolar nerve block which is
given in cleft palate
A field can also be blocked little above the level when we nee to block a bunch of
nerves
PSYCHIATRY

1. A child with an IQ level of 55, He is having which of the following?


E. Mild MR
F. Moderate MR
G. Severe MR
H. Normal MR
Answer: A
Explanation:
IQ = Mental Age X 100
Chronological age
Mild MR is the most common form of MR in India and Down Syndrome is the
most common cause.
 Normal IQ is > or = to70
 Mild MR is 50-69
 Moderate MR is 35-49
 Severe MR is 21-34
 Profound MR is < or = to 20

2. A person with anxiety has a felling that his parents will die one day and he will
have sleepless nights, which of the following disorders describes his
condition?
A. Generalized anxiety
B. Severe depression
C. PTSD
D. Adjustment disorder
Answer: A
Explanation: Generalized anxiety disorder: in generalized anxiety disorder
We have major and minor criteria for making a diagnosis of generalized
anxiety disorder we need 2 major and 3 minor for 6 months. Treatment would
be SSRI and behavior therapies
Major criteria
 Excessive worry
 Not able to control
Minor criteria
 Restlessness
 Muscle tension
 Irritability
 Sleep disturbance
 Easy fatigability
 Decreased concentration

3. A kid with normal learning skills but poor mathematical skills. His IQ is
appropriate to his age. He passed the exams with poor marks. What could be
the probable diagnosis?
A. Exam phobia
B. ADHD
C. Specific learning disorder
D. Asperger’s syndrome
Answer: C
Explanation: in Specific learning disorder (Dyslexia)
 We have reading, writing, mathematical skills, mixed types
 For making a diagnosis of Dyslexia, IQ should be normal for the child
 H/o migration should be ruled out

4. Patient with 6 episodes of depression, hypermania, hypomania in a year with


recovery phase in between. which of the following is appropriate diagnosis?
A. Severe depression
B. Cyclic dysthymia
C. Rapid cyclic bipolar disorder
D. Mania
Answer: C
Explanation: by definition, Patient must have at least four different episodes in
last 1 year is rapid cycling bipolar disorder.

5. Patient after getting operated for appendicitis developed confusion and


hallucination. His wife informed that he takes alcohol occasionally. what could
be the most probable cause for his condition?
A. Delirium
B. Alcohol hallucination
C. Acute transient psychotic disorder
D. Mania
Answer: A
Explanation: Delirium
 Most common organic disorder
 Diagnosis of delirium
-Impairment or clouding of consciousness
-disorientation to time place and person
-memory deficit
-remote memory will remain intact
-perceptual abnormalities in the form of illusion and visual
hallucination
- transient/fragmentary delusion
-Sun downing phenomenon
-Increased psychomotor activity
 Confusion assessment method is used
 Treated by etiology, low dose of haloperidol(to decrease psychomotor
activity)
6. A patient having unilateral headache associated with nausea and vomiting,
which drug can be used to decrease headache episodes?
A. Propranolol
B. Alprazolam
C. Fluoxetine
D. None of the above
Answer:
Explanation: patient has Migraine
 Unilateral, pulsating type associated with Nausea, Vomiting,
photophobia, phonophobia
 Precipitated by sun light
 Travelling and lack of sleep, even hunger precipitates too
 Treatment – for acute- to abort the attack- Nsaids, tryptans
-for prophylaxis- Propranolol, sodium valproate,topiramate

ORTHO

1. Which of the following is a radio sensitive tumor?


A. Melanoma
B. Ewing’s sarcoma
C. Osteosarcoma
D. Hepatoma
Answer: B
Explanation: from the mnemonic WELMS we know the radio sensitive tumors
are
 Wilms
 Ewing’s sarcoma
 Lymphoma
 Myeloma
 Seminoma/Dysgerminoma
We also can remember radio resistant tumors from the HOMP
 Hepatoma
 Osteosarcoma
 Melanoma
 Pancreatic cancers
So, from the above explanation we can conclude from the options that
Ewing’s sarcoma is the radiosensitive tumor.
2. What is the following image?

A. Taylor brace splint


B. Milwaukee brace
C. Scoliosis brace
D. Von Rosen splint
Answer: A
Explanation: The given brace is Taylor Brace splint, given for immobilization
of thoracic and lumbar spine and in used in traumatic or infected spine

3. Identify the condition given in the image?

A. Galeazzi fracture
B. Monteggia deformities
C. Monteggia Fracture
D. Green stick fracture
Answer: A
Explanation: this a fracture involving disruption of distal radial-ulnar joint with
damage to the interosseous membrane and triangular Fibro cartilage
component (TFCC). this is Galeazzi Fracture.
-Monteggia fracture is a fracture of ulnar with dislocated radial head

4. A patient comes after injury, from the following X-ray, What is the probable
diagnosis?

A. Tibial Fracture
B. Patellar Fracture
C. Knee loose body
D. Femur fracture
Answer: B
Explanation: the image clearly shows fracture of Patella and the treatment is
Tension Band wiring

5. Identify the condition shown in the following image?

A. Osteosarcoma
B. Chondrosarcoma
C. Chondroblastoma
D. Ewing’s sarcoma
Answer: B
Explanation: - since osteosarcoma is a lytic type
- Chondroblastoma occurs in epiphysis
- Ewing’s sarcoma occurs in the diaphysis
- Chondrosarcoma is the answer because it is the calcified
lesion shown in the image and femur is the most common
site

6. A 30-year-old patient has a history of trauma and pain in right hip with flexion,
abduction and external rotation, what is the most likely diagnosis?
A. Hip posterior dislocation
B. Hip anterior dislocation
C. Fracture neck femur
D. Intertrochanteric fracture
Answer: B
Explanation: we conclude Hip anterior dislocation because
o Hip anterior dislocation: From FABER Flexion Abduction
External Rotation is seen
o Hip posterior dislocation: From FADIR Flexion Adduction
Internal Rotation is seen
o Fracture of neck of femur: Intracapsular and will have shortening
and external rotation
o Intertrochanteric fracture: extracapsular and will have more
shortening and more External rotation

7. Patient has multiple fractures along with blue sclera, what is the most likely
diagnosis?
A. Osteogenesis Imperfecta
B. Rickets
C. Scurvy
D. Paget’s disease
Answer: A
Explanation: Osteogenesis Imperfecta
Osteogenesis imperfecta (OI) is an inherited (genetic) bone disorder that is
present at birth. It is also known as brittle bone disease. A child born with OI
may have soft bones that break (fracture) easily, bones that are not formed
normally, and other problems. Blue sclera is also a feature of Osteogenesis
Imperfecta. It is the commonest cause of multiple fractures at birth.
8. 60-year-old female patient complaints of pain in knee bilaterally, the pain
increases while going up and down the stairs. The x-ray shows Bilateral
reduced joint space, what is the most likely diagnosis?
A. Osteoarthritis
B. Rheumatoid Arthritis
C. Ankylosing spondylitis
D. Hemophiliac arthropathy

Answer: A
Explanation: Osteoarthritis
Osteoarthritis is the most common form of arthritis, affecting millions of
people worldwide. It occurs when the protective cartilage that cushions the
ends of the bones wears down over time. Although osteoarthritis can
damage any joint, the disorder most commonly affects joints in your
hands, knees, hips and spine.

9. The following clinical image shows?

A. Winging of scapula
B. Erb’s palsy
C. Klippel Feil Syndrome
D. Fracture Scapula
Answer: A
Explanation: the image clearly shows winging of scapula, The term 'winged
scapula' is used when the muscles of the scapula are too weak or paralyzed,
resulting in a limited ability to stabilize the scapula. As a result, the medial or
lateral borders of the scapula protrudes from back, like wings.
10. A child with widening of bone has the following x-ray, what is the aetiology?
A. Scurvy
B. Rickets
C. Gout
D. Salter Harris type 2 Injury
Answer: B
Explanation: Rickets will have a wide skeleton, the bones have mineralization
defect and cupping and splaying of the bone can be clearly seen and the
margins point out called as flaring/fraying, classical characters of Rickets
11. Which of the following nerve is related to medial epicondyle?
A. Median nerve
B. Ulnar nerve
C. AIN
D. PIN
Answer: B
Explanation: Ulnar nerve
o In upper limb around upper part of humerus= axillary nerve
o Around the shaft =Radial nerve
o Behind the medial epicondyle=Ulnar nerve
o And head of radius=Posterior interosseous nerve

12. The spot diagnosis for the following image is?

A. Scoliosis
B. Disc prolapses
C. Klippel Feil Syndrome
D. Torticollis
Answer: C
Explanation: short neck, lower hairline, short and high scapula which is a
classical feature of Klippel Feil Syndrome
SURGERY
1. In the hospital, a patient presents with the following swelling on standing
which tends to reduce when he lies down. What could be the probable
diagnosis?

A. Para-umbilical Hernia
B. Umbilical Hernia
C. Epigastric Hernia
D. Omphalocele
Answer: A
Explanation:

2. A 5-year-old child was brought to the clinic with the swelling shown below
which keeps on increasing in size. What should be the appropriate
management for this condition?

A. Intralesional steroids
B. Excision
C. Reassurance
D. Conservative
Answer: B
Explanation: A dermoid cyst is a collection of tissue under the skin. It may
contain hair follicles, oil, and sweat glands. In some cases, it may contain
bone, teeth, or nerves. A dermoid cyst may appear at birth or soon after.
Dermoid cysts are often found on the head, neck, or face, most often around
the eyes.
 Surgical Excision is the only mode of treatment for dermoid cyst.

3. A 45-year-old female presents with Rapidly enlarging lump in her breast. Firm
and mobile on examination with Bosselated surface. Lymph nodes are not
involved. What could be the diagnosis?

A. Fibroadenoma
B. Phyllodes tumor
C. Ductal carcinoma
D. Ductal papilloma
Answer: B
Explanation: Phyllodes tumor
 A phyllodes tumor is a rare tumor of the breast. Phyllodes tumors grow
in the connective tissue of the breast, called the stroma. This includes
the tissue and ligaments that surround the ducts, blood vessels, and
lymph vessels in the breast.

Most phyllodes tumors are benign. They may look very much like
common benign breast tumors called fibroadenomas. Often, the
pathologist needs to look at the whole tumor under the microscope to
make a diagnosis. This is why surgery to remove a phyllodes tumor is
recommended, even if it is thought to be benign.

4. An Incision was made over the patient’s chest for a surgery and after
sometime, he comes with the scar shown in the
image. What is the diagnosis?
A. Keloid
B. Hypertrophic scar
C. Contracture
D. Abscess
Answer: A
Explanation: Keloid scar

5. After RTA, a patient presented to emergency and following is done to


maintain the airway. What is the procedure shown in the image?

A. Intubation
B. Tracheostomy
C. Thoracotomy
D. Radical neck dissection
Answer: B
Explanation: Tracheostomy

6. A bed ridden patient developed fever, pain and pedal edema, what could be
the diagnosis?

A. Cellulitis
B. DVT
C. DIC
D. Pressure ulcer
Answer: B
Explanation: DVT only can cause low grade fever, remaining all in the options
can only cause high grade fever.
7. A Patient presented with long standing pain in the anal region. On
examination the finding shown is given below. What is the most important
thing to look for on per rectal examination of this patient?

A. Internal os of the fistula


B. International haemorrhoids
C. Anal Fissure
D. Fistula in ano
Answer: D
Explanation: Perianal fistula
 A perianal fistula (fistula-in-ano) refers to an abnormal connection
between the anal canal and the perianal skin. The majority are
associated with anorectal abscess formation, with one third of patients
with an anorectal abscess having an associated perianal fistula at the
time of presentation.

8. Which instrument is shown in the image?
A. Allis Forceps
B. Sponge holder
C. Babcock forceps
D. Retractor
Answer: C
Explanation: The image showed is Babcock forceps
9. What procedure is done in the following image?

A. Phallen’s sign
B. Tinel’s sign
C. Prayer sign
D. Schamroth sign
Answer: C
Explanation: A positive prayer sign can be is when on examination the patient
is unable to approximate the palmar surfaces of the phalangeal joints while
pressing their hands together
Seen in diabetes
10. A woman was presented with the following lesion in the image through which
purulent discharge was coming out. The most probable diagnosis?

A. Hydradenitis Suppurativa
B. Tuberculous lymphadenitis
C. Branchial cyst
D. Bezold abscess
Answer: B
Explanation: Tubercular Lymphadenitis
 Tuberculous lymphadenitis is a chronic, specific granulomatous
inflammation of the lymph node with caseation necrosis, caused by
infection with Mycobacterium tuberculosis or related bacteria.

11. A man came to the hospital presented with vomiting, nausea and fever. He
was admitted for appendectomy after diagnosing appendicitis. During surgery,
surgeons found an incidental finding as shown in the image below which was
present at a distance of 2 feet from ileocaecal junction. What is the most likely
diagnosis?

A. Intussusception
B. Intestinal obstruction
C. Meckel’s diverticulum
D. Diverticulitis
Answer: C
Explanation: Meckel’s diverticulum
 Most common congenital anomaly of small bowel
 It is remnant of Vitellointestinal duct
 Connects between midgut and yolk sac
 Normally disappears in 7th week of intrauterine life
 Present in 2% of population
 Lies in ileum 2 feet from ileocaecal junction
 Measures 2 inches in length
 Has a separate blood supply
 May contain ectopic gastric, pancreatic or colonic mucosa

12. Identify the procedure in the


following image?

A. Colostomy
B. Loop ileostomy
C. Ascending colostomy
D. Appendectomy
Answer: B
Explanation: loop Ileostomy
 To form a loop ileostomy, a loop of small intestine is pulled out through
a cut in your abdomen. This section of intestine is then opened up and
stitched to the skin to form a stoma. The colon and rectum are left in
place.

13. A cricket player was injured with the ball and suddenly lost consciousness.
When one of the teammate comes for helping, he gets up by himself and
starts playing. During tea break, the person was again unconscious while his
friend saw him, immediately he has been rushed to the hospital. What is the
most probable diagnosis?

A. EDH
B. SDH
C. SAH
D. Intracerebral haemorrhage
Answer: A
Explanation: EDH
 The player was in Lucid interval and it is a classical feature of EDH
 CT-scan will show Biconvex shaped hematoma

14. In which of the following is Dohlman’s procedure used for?

A. Cricoid webs
B. Pharyngeal pouch
C. Achalasia Cardia
D. Diffuse esophagal spasm
Answer: B
Explanation: used in pharyngeal pouch or Zenker’s Diverticulum
15. A young male a chronic smoker, after developing the following condition of
finger tips as seen in the Image coms to the clinic. What could be the
condition?

A. Medium vessel vasculitis


B. Buerger’s disease
C. Scleroderma
D. APLA syndrome

Answer: B
Explanation: Buerger’s disease
 Buerger's disease is a rare disease of the arteries and veins in the
arms and legs. In Buerger's disease — also called thromboangiitis
obliterans — blood vessels become inflamed, swell and can
become blocked with blood clots (thrombi)
 This eventually damages or destroys skin tissues and may lead to
infection and gangrene. Buerger's disease usually first shows in
your hands and feet and may eventually affect larger areas of your
arms and legs.
 The exact cause of Buerger's disease is unknown. While tobacco
use clearly plays a role in the development of Buerger's disease, it's
not clear how it does so. It's thought that chemicals in tobacco may
irritate the lining of your blood vessels, causing them to swell.

16. A patient presented with the following condition as shown below. what could
be the diagnosis?

A. Arterial ulcer
B. Venous ulcer
C. Neuropathic ulcer
D. Pressure ulcer

Answer: B
Explanation: venous ulcer
 Are seen most commonly seen in medial malleolus (Gaiter area)
 They are usually sloping ulcers and floor is covered by granulation
tissue
 If it bleeds, we ask the patient to lift the limb and we do not compress
17. Identify the Anomaly shown in the picture?

A. Pyloric stenosis
B. Pancreatic Divisum
C. Annular Pancreas
D. Duodenal Atresia

Answer: C

Explanation: Annular Pancreas


 It is due to the failure of rotation of Ventral bud of pancreas.
 Annular pancreas is the most common birth defect seen in the
pancreas. The pancreas is an organ that plays an important part in
your digestive process. The term annular pancreas means that a ring
of extra pancreatic tissue covers the first part of the small intestine
(duodenum).

18. A patient has an USG finding of a gall stone abutting the cystic duct with
dilatation of Common hepatic duct. What is the most likely diagnosis?

A. Acute cholecystitis
B. Porcelain GB
C. GB polyp
D. Mirizzi Syndrome

Answer: D
Explanation: Mirrizi Syndrome
 Mirizzi syndrome is defined as common hepatic duct obstruction
caused by extrinsic compression from an impacted stone in the cystic
duct or infundibulum of the gallbladder
19. Which of the following fluid is avoided in Intracranial Head Injury?

A. NS
B. RL
C. 5% dextrose
D. Hypertonic solution

Answer: C
Explanation: The ideal fluid is Mannitol, even any hypertonic solutions can be
used.
But we cannot use any hypotonic solution or glucose containing solutions
because they will cause cerebral edema causing an increase in intra-cranial
pressure.

20. A male came to the OPD with an abdominal lump in periumbilical region that
moves at right angle to attachment of mesentry. What is your diagnosis?

A. Chylolymphatic cyst
B. Pancreatic Pseudocysts
C. Retroperitoneal Sarcoma
D. Peritoneal cyst

Answer: A
Explanation: Chylolymphatic cyst
 A chylolymphatic cyst is a rare variant of a mesenteric cyst [1,2]. These
cysts present within the mesentery, lined with a thin endothelium or
mesothelium and filled with chylous and
lymphatic fluid.
21. A new born is found to have herniation of bowel and liver through the
Umbilicus, which is covered with a membrane. What is the diagnosis?

A. Gastrochisis
B. Omphalocele
C. Epigastric Hernia
D. Ectopia Vesica

Answer: B
Explanation:

PEDIA
1. A 5-year-old child of 17 kg weight with diarrhea and unable to take the feed.
What is the amount of maintenance fluid required in 24 hours?

(a) 1000 ml
(b) 1700 ml
(c) 1350 ml
(d) 2000 ml

Ans. (c) 1350 ml

17 kg ----- (10 x 100) + (7 x 50) = 1000 + 350 = 1350 ml

Calculation of fluids by HOLIDAY SEGAR REGMIN


First 0-10 kg = 100ml/kg
Next 10-20 kg= +50 ml/kg
>20 kg = +20 ml/kg

TREATMENT PLAN FOR DEHYDRATION: ( Diarrhea Treatment )


No Dehydration Some Dehydration Severe dehydration (lethargy)
PLAN A PLAN B Plan C
 ORS 100-  Rehydration (75ml/kg  Start IV fluids
200ml/loose in 1st 4 hrs) +  Ringer lactate >>Normal saline
stool (ongoing maintenance
losses 10- (replacing on going Give 100 ml/kg as :
20ml/kg) loss- 10-20ml/kg) Age 30ml/kg 70ml/kg
<1yr 1hr 5 hr
>1yr 30 min 2 ½ hr

2. A mother with the new born is worried for her child as the newborn hasn’t
passed the meconium till 6 hours of birth. How much time should she wait for
the meconium to pass out?

(a) 48 hours
(b) 24 hours
(c) 12 hours
(d) 72 hours

Ans. (a) 48 hours

Maximum time to wait in a newborn to pass out -

24 hours – urine
48 hours – meconium

Delay in passage of meconium:

Hirschsprung disease (Aganglionsis)


Imperforate anus
Hypothyroidism
Meconium ileus [cystic fibrosis]
Small left colon syndrome [Associated with Infant of diabetic mothers]
3. A boy presents with hemarthrosis of left knee after a history of fall.
Deficiency of which factor can lead to the following condition?

(a) Von-willebrand disease


(b) Vitamin K deficiency
(c) Thrombocytopenia
(d) Factor VIII deficiency

Ans. (d) Factor VIII deficiency

Hemophilia is a disease that prevents blood from clotting properly. A clot helps stop
bleeding after a cut or injury. In factor VIII deficiency (hemophilia A), the body
doesn't make enough factor VIII (factor 8), one of the substances the body needs
to form a clot.

Hemophilia:
X linked Recessive
Causes severe forms of bleeding
M/C joint affected: ANKLE
Characteristic lab feature: increased APTT

MEDICINE
1. A 45-year-old female having arthralgia, swallowing difficulty with dry food and
gritty feeling under eyelid. She is suffering from NCNC(Normocytic normochromic
anemia) with Increased ESR, positive for anti-ANA, Anti-Ro and Rheumatoid
factor. Which of the following could be the diagnosis?

(a) Sjogren’s syndrome


(b) Rheumatoid arthritis
(c) Serum sickness
(d) SLE
Ans. (a) Sjogren’s syndrome

Sjogren's syndrome is a disorder of your immune system identified by its two


most common symptoms — dry eyes and a dry mouth. The condition often
accompanies other immune system disorders, such as rheumatoid arthritis and
lupus.

2. Diagnose the given ECG?

(a) Ventricular tachycardia


(b) Ventricular fibrillation
(c) Atrial tachycardia
(d) Atrial fibrillation

Ans. (a) Ventricular tachycardia


3. Identify the anomaly shown in the Image?

(a) Pyloric stenosis


(b) Pancreatic divisum
(c) Annular pancreas
(d) Duodenal atresia

Ans. (c) Annular Pancreas

Annular pancreas is a morphological anomaly that results in pancreatic tissue


completely or incompletely encircling the duodenum. This condition can cause
duodenal obstruction
he pancreas develops from a single dorsal and two ventral buds, which appear as
outgrowths of primitive foregut at 5 weeks of gestation. The ventral buds fuse rapidly.
In the 7th week of gestation, the duodenum expands, and rotates the ventral bud
from right to left, and fuses with the dorsal bud. The ventral bud forms the inferior
part of uncinate process and inferior head of pancreas while the dorsal bud gives
rise to tail and body of pancreas.

Annular pancreas develops due to failure of the ventral bud to rotate with duodenum,
causing encasement of duodenum.

4. A patient comes to emergency with complaints of acute abdominal pain after


alcohol intake. Pain was radiating to his back. On further investigation, he was
found to have raised level of AST, ALT and amylase. What could be the
diagnosis?

(a) HCC
(b) Hepatitis
(c) Acute pancreatitis
(d) Gall bladder stones

Ans. (c) Acute Pancreatitis


The most common causes of acute pancreatitis are gallstones and alcohol-induced
pancreatitis. This patients liver enzymes especially ALT and bilirubin level are
elevated

Mnemonic for Causes of Acute Pancreatitis

 I: Idiopathic
 G: Gallstones
 E: Ethanol (alcohol)
 T: Trauma
 S: Steroids
 M: Mumps (and other infections) / malignancy
 A: Autoimmune
 S: Scorpion stings/Spider bites
 H: Hyperlipidemia/Hypercalcemia
 E: ERCP
 D: Drugs

5. A patient with ptosis, anhidrosis and slight drooping of eyelid is presented to the
clinic. What do you think the patient is suffering from?
a. Horner syndrome
b. Keratoconjunctivitis
c. Dacryocystitis
d. Orbital cellulitis

Ans. Horner syndrome


Horner syndrome is a combination of signs and symptoms caused by the
disruption of a nerve pathway from the brain to the face and eye on one side of
the body. Typically, Horner syndrome results in a decreased pupil size, a drooping
eyelid and decreased sweating on the affected side of your face.
6. Which fluid should not be given in severe head injury?

(a) 5% dextrose
(b) Normal saline
(c) Hypertonic saline
(d) Mannitol

Ans. (a) 5% dextrose

Mannitol, a hypertonic crystalloid solution, is commonly used to decrease brain water


content and reduce intracranial pressure (ICP). Hypertonic saline solutions also
decrease brain water and ICP while temporarily increasing systolic blood pressure
and cardiac output. Hypo-osmolar solutions, such as 5% dextrose in water,
reduce serum sodium and increase brain water and ICP so Hypotonic, low
sodium and dextrose-containing fluids should be avoided
7. Purple striate on the abdomen as shown in the picture given below is a
characteristic feature of which of the following given condition?

(a) Cushing’s syndrome


(b) Hypothyroidism
(c) SIADH
(d) Addison’s disease

Ans. (a) Cushing syndrome

Cushing's syndrome is a disorder that occurs when your body makes too much
of the hormone cortisol over a long period of time. Cortisol is sometimes called
the “stress hormone” because it helps your body respond to stress.
8. A boy presented with blue sclera who has a history of frequent fractures. What
could be the diagnosis?

(a) Scurvy
(b) Rickets
(c) Osteogenesis Imperfecta
(d) Osteomalacia

Ans. (c) Osteogenesis Imperfecta

Osteogenesis Imperfecta (OI), also known as "brittle bone disease", is


an autosomal dominant bone disorder characterized by fragile bones that break
easily.

Specifically, Osteogenesis Imperfecta is caused by defects in Type I Collagen, a


key component of bone. Patients present with recurrent fractures (distinguish vs.
child abuse), blue sclera (translucency with visible choroidal veins), tooth
abnormalities, and hearing loss (due to ossicle involvement).

Treatment involves bisphosphonates (such as alendronate), which prevent the loss


of bone density and help reduce the risk for fracture.
10. Identify the ECG given below?

(a) Atrial flutter


(b) Ventricular tachycardia
(c) Ventricular fibrillation
(d) MI

Ans. (a) Atrial flutter


11. Identify the condition shown below?

a. Nail pterygium
b. Clubbing
c. Koilonychia
d. None of the above

Ans. Clubbing
Causes of Nail Clubbing - Mnemonic CLUBBING
Cyanotic Heart Disease, Cystic Fibrosis
Lung Cancer, Lung abscess
Ulcerative Colitis
Bronchiectasis
Benign Mesothelioma
Infective Endocarditis, Idiopathic Pulmonary
Neurogenic Tumors
Gastrointestinal Disease

12. A 68-year-old Diabetic patient develops which of the following macro vascular
complications after 20 years?

(a) Retinopathy
(b) Neuropathy
(c) Nephropathy
(d) Myocardial Infarction
Ans. (d) Myocardial Infarction

Macrovascular complications of T2DM include coronary heart disease,


cardiomyopathy, arrhythmias and sudden death, cerebrovascular disease and
peripheral artery disease. Cardiovascular disease is the primary cause of death in
diabetic patients. Many clinical studies have shown a connection between T2DM and
vascular disease, but almost always other risk factors are present in diabetic
patients, such as hypertension, obesity and dyslipidemia.

13. Tracheal shift towards right with left sided pneumothorax. What is the
management?

(a) Left sided thoracostomy


(b) Right sided thoracostomy
(c) Conservative
(d) Wrapping the chest tightly

Ans. (a) Left sided thoracostomy

When tracheal deviation is present, the trachea will be displaced in the direction of
less pressure. Meaning, that if one side of the chest cavity has an increase in
pressure (such as in the case of a pneumothorax) the trachea will shift towards the
opposing side.
Thoracostomy is a minimally invasive procedure in which a thin plastic tube is
inserted into the pleural space — the area between the chest wall and lungs — and
may be attached to a suction device to remove excess fluid or air. A chest tube may
also be used to deliver medications into the pleural space.
14. A HIV patient presents with following presentation. What is the staging of HIV?

(a) I
(b) II
(c) III
(d) IV

Ans. (c) III

15. JVP Image shows deep x descent and a rapid y descent and elevated over all.
What could be the diagnosis?
(a) Constrictive pericarditis
(b) Dilated cardiomyopathy
(c) HOCM
(d) Tamponade

Ans. (a) Constrictive pericarditis

16. A student who was eating the food of his hostel mess found to have developed
hepatomegaly, jaundice. On the same time all other students who were eating
from the same mess developed similar symptoms. Which investigation is more
appropriate to be done to find out the cause?

(a) IgM for Hepatitis A


(b) IgG for Hepatitis A
(c) IgM for Hepatitis B
(d) IgG for Hepatitis B

Ans. (a) IgM for Hepatitis A

Hepatitis A immunoglobulin M (IgM anti-HAV) antibody test: When a person is first


infected with hepatitis A, the body produces IgM anti-HAV antibodies. These
antibodies are usually detectable from two weeks after symptoms begin to around
six months later.

Hepatomegaly is an enlarged liver, which means it's swollen beyond its usual size. A
swollen liver usually is a symptom of another health condition, such as hepatitis.

Depending on the cause of your enlarged liver, you may notice symptoms like:

 Yellowing of the skin or eyes (jaundice)


 Fatigue and weakness
 Nausea
 Weight loss
17. A male patient presented with high grade fever and headache. He was found to
have protein level 220 mg/dl and glucose 35mg/dl in CSF. He develops rashes as
shown below. What should be the probable cause?

(a) E coli
(b) Listeria
(c) Neisseria meningitides
(d) Cryptococcus

Ans. (c) Neisseria meningitides

Typical CSF abnormalities in meningitis include the following:


 Increased opening pressure (>180 mm water)
 Pleocytosis of polymorphonuclear leukocytes (white blood cell [WBC] counts
between 10 and 10,000 cells/µL, predominantly neutrophils)
 Decreased glucose concentration (< 45 mg/dL)
 Increased protein concentration (>45 mg/dL)
 The bacteria rapidly multiply and produce toxins which travel around the body
causing damage to blood vessels and organs. As the blood vessels get
damaged, blood starts to 'leak' into the surrounding tissue, often causing
what looks like a 'rash' to appear on the skin.

18. A patient who is normal at rest but has discomfort on activities. Normal activities
are slightly affected. The patient comes under which category of NYHA
classification?

(a) NYHA I
(b) NYHA II
(c) NYHA III
(d) NYHA IV
Ans. (b) NYHA II

19. A patient comes to emergency with fever and headache. O/E he has neck
stiffness, CSF analysis was done and opening pressure increased, proteins mild
increased, glucose normal, lymphocytes increased. What is the most likely
diagnosis?

(a) Tubercular meningitis


(b) Purulent bacterial meningitis
(c) Viral meningitis
(d) Cryptococcus

Ans. (c) Viral meningitis


20. A woman after hysterectomy presents with distended abdomen. What is
electrolyte imbalance seen?

(a) Hyperkalemia
(b) Hypokalemia
(c) Hypernatremia
(d) Hyponatremia

Ans. (b) Hypokalemia

The symptoms of hypokalemia are nonspecific and predominantly are related to


muscular or cardiac function. Weakness and fatigue are the most common
complaints. The muscular weakness that occurs with hypokalemia can manifest in
protean ways (eg, dyspnea, constipation or abdominal distention, exercise
intolerance).

21. Patient with previous history of MI 3 years ago now presenting with tachypnea,
tachycardia, S3 heard. What could be the diagnosis?

(a) MI
(b) Pericarditis
(c) Congestive heart failure
(d) Pulmonary embolism

Ans. (c) Congestive heart failure

Results from increased atrial pressure leading to increased flow rates, as seen
in congestive heart failure, which is the most common cause of a S3.

Risk factors for heart failure include:


 Coronary artery disease. Narrowed arteries may limit your heart's supply of oxygen-
rich blood, resulting in weakened heart muscle.
 Heart attack. ...
 Heart valve disease. ...
 High blood pressure. ...
 Irregular heartbeats. ...
 Congenital heart disease. ...
 Diabetes. ...
 Some diabetes medications.
22. Identify the condition due to the Image shown below?

(a) Hypercholesterolemia
(b) Hypertriglyceridemia
(c) Hyper alpha lipoproteinemia
(d) Hyper beta lipoproteinemia

Ans. (a) Hypercholesterolemia

Hypercholesterolemia, also called high cholesterol, is the presence of high levels


of cholesterol in the blood.

Some types of hypercholesterolemia lead to specific physical findings. For example,


familial hypercholesterolemia (Type IIa hyperlipoproteinemia) may be associated
with xanthelasma palpebrarum (yellowish patches underneath the skin around the
eyelids), arcus senilis (white or gray discoloration of the peripheral cornea),
and xanthomata(deposition of yellowish cholesterol-rich material) of the tendons,
especially of the fingers. Type III hyperlipidemia may be associated with xanthomata
of the palms, knees and elbows

23. A 12-year-old child having proteinuria 4+ and edema and features associating
with nephrotic syndrome without hematuria. Which drug would you prefer?

(a) Prednisolone
(b) Cyclosporine
(c) Cyclophosphamide
(d) Methotrexate

Ans. (a) Prednisolone

The term "nephrotic syndrome" refers to a distinct constellation of clinical and


laboratory features of kidney disease. It is specifically defined by the presence
of heavy proteinuria(protein excretion greater than 3.5 g/24 hours),
hypoalbuminemia (less than 3.5 g/dL), and peripheral edema.

Nephrotic syndrome is a condition where the kidneys leak protein from the blood into
the urine. When untreated, children can suffer from serious infections. In most
children with nephrotic syndrome, this protein leak resolves with corticosteroid drugs
(prednisone, prednisolone) reducing the risk of serious infection

24. A 65-year-old patient presents with dementia and loss of cognition. Which is the
most common cause for this condition?

(a) Alzheimer’s
(b) Pick’s
(c) Parkinson’s
(d) CJD

Ans. (a) Alzheimer’s

Alzheimer disease is a neurodegenerative disease that is signified by cognitive


decline, memory loss, and erratic behavior. Alzheimer's disease is the most
common type of dementia. It is a progressive disease beginning with mild memory
loss and possibly leading to loss of the ability to carry on a conversation and respond
to the environment. Alzheimer's disease involves parts of the brain that control
thought, memory, and language

25. A patient is allergic to wheat and unable to eat it. O/E, he was tested positive with
Anti-Endomysial Antibodies. What could be the diagnosis?

(a) Celiac sprue


(b) Acrodermatitis enteropathica
(c) Tropical sprue
(d) Thiamine deficiency

Ans. (a) Celiac sprue

IgA anti-endomysial antibodies are found in 90% coeliac patients. They are a very
specific & sensitive marker for coeliac disease and dermatitis herpetiformis. The
anti-endomysial antibody test has been suggested to be 98% sensitive and 98%
specific for clinical or subclinical coeliac disease.
Celiac disease, also known as celiac sprue or gluten-sensitive enteropathy, is a
chronic disorder of the digestive tract that results in an inability to tolerate gliadin,
the alcohol-soluble fraction of gluten. Gluten is a protein commonly found in wheat,
rye, and barley.
26. A 75-year-old female has a problem in walking long distances. O/E bruit heard in
knee. On X-ray imaging, reduced joint space and osteophytes were seen. Most
probable diagnosis is?

(a) Osteoarthritis
(b) Rheumatoid arthritis
(c) Psoriatic arthritis
(d) Gouty arthritis

Ans. (a) Osteoarthritis

Osteoarthritis is the most common form of arthritis, affecting millions of people


worldwide. It occurs when the protective cartilage that cushions the ends of the
bones wears down over time.

Osteoarthritis symptoms often develop slowly and worsen over time. Signs and
symptoms of osteoarthritis include:

 Pain. Affected joints might hurt during or after movement.


 Stiffness. Joint stiffness might be most noticeable upon awakening or after
being inactive.
 Tenderness. Your joint might feel tender when you apply light pressure to or
near it.
 Loss of flexibility. You might not be able to move your joint through its full
range of motion.
 Grating sensation. You might feel a grating sensation when you use the joint,
and you might hear popping or crackling.
 Bone spurs. These extra bits of bone, which feel like hard lumps, can form
around the affected joint.
 Swelling. This might be caused by soft tissue inflammation around the joint.

 X-rays. Cartilage doesn't show up on X-ray images, but cartilage loss is


revealed by a narrowing of the space between the bones in your joint. An X-
ray can also show bone spurs around a joint.
27. Comment on the image shown below?

a) Prayer sign
b) Phalen sign
c) Tinel Sign
d) Schamroth sign

Ans. Prayer sign


Inability to extend the proximal interphalangeal joints of the fourth and fifth fingers -
Prayer sign
“Prayer sign,” a feature of diabetic cheiroarthropathy. It is also known as diabetic stiff
hand syndrome or limited joint mobility syndrome. It is found in both type 1 and type
2 diabetic patients. The prevalence increases with duration of diabetes. This
condition is associated with and predictive of diabetic microvascular complications.

28. . An old man presents with complaints of breathlessness and occasional


episodes of chest pain. General physical examination shows high-volume pulse
and BP of 150/70 mm Hg. On cardiovascular examination, the apex beat
displaced 1.5cm lateral to Mid clavicular line. On auscultation diastolic murmur is
heard. What is the diagnosis?

a) Aortic insufficiency
b) Mitral insufficiency
c) Pulmonary insufficiency
d) Tricuspid insufficiency

Ans. Aortic insuffiency

Lateral and/or inferior displacement of the apex beat usually indicates enlargement
of the heart, called cardiomegaly.

Aortic regurgitation (AR), also known as aortic insufficiency, is a form of valvular


heart disease that allows for the retrograde flow of blood back into the left ventricle.
Aortic regurgitation is characterized by cardiomegaly on the chest radiographs,
which is predominantly caused by LV enlargement
29. A patient was asked to copy the clock face made by the physician. The image on
right is made by the
patient. Which is correct about this image?

a) Apraxia
b) Agraphia
c) Hemianopia
d) Hemi-neglect

Ans. Apraxia

Apraxia is a neurological disorder characterized by the inability to perform learned


(familiar) movements on command, even though the command is understood and
there is a willingness to perform the movement. Both the desire and the capacity to
move are present but the person simply cannot execute the act.

The clock-drawing test is used for screening for cognitive impairment and
dementia and as a measure of spatial dysfunction and neglect. It was originally used
to assess visuo-constructive abilities but we know that abnormal clock drawing
occurs in other cognitive impairments.

30. A 30-year-old patient presents with acute abdomen. X Ray Abdomen was done in
standing position. Which of the following is correct about management of this
condition?

a) Drain the peritoneal fluid


b) X Ray guided drainage of peritoneal accumulation
c) Exploratory laparotomy
d) Insert Flatus tube and repeat X ray
Ans. Exploratory Laparotomy

Exploratory laparotomy has been the main stay of the treatment in patients
with acute abdomen. Laparoscopic management has previously been considered
as a relative contraindication for acute small bowel obstruction and peritonitis.

31. The following CT abdomen shows

a) Pan Cake Kidney


b) Polycystic kidney
c) Horse Shoe kidney
d) Renal cell cancer

Ans. Horseshoe Kidney


Horseshoe kidney when the 2 kidneys join (fuse) together at the bottom to form a U
shape like a horseshoe. It is also known as renal fusion. The condition occurs when
a baby is growing in the womb, as the baby's kidneys move into place. Horseshoe
kidney can occur alone or with other disorders.

32. Comment on the diagnosis based on X Ray abdomen image shown here

a) Congenital hypertrophic pyloric stenosis


b) Duodenal atresia
c) Congenital diaphragmatic hernia
d) Transient Tachypnoea of newborn

Ans. Duodenal Atresia


Duodenal atresia is a condition in which the first part of the small bowel (the
duodenum) has not developed properly. It is not open and cannot allow the
passage of stomach contents.
The double bubble sign is seen in infants and represents dilatation of the
proximal duodenum and stomach. It is seen in both radiographs and ultrasound, and
can be identified antenatally

33. A 60-year-old with restricted ambulation and prolonged bed rest due to acute
lumbago presented with severe pain in left leg below the knee. On examination
discoloration of left calf with significantly increased girth is noted. What is the
clinical diagnosis?

a) Acute lymphangitis
b) Superficial migratory thrombophlebitis
c) Deep vein thrombosis
d) Milroy disease

Ans. Deep vein thrombosis

Deep vein thrombosis (DVT) occurs when a blood clot (thrombus) forms in one or
more of the deep veins in your body, usually in your legs. Deep vein thrombosis can
cause leg pain or swelling but also can occur with no symptoms.

You can get DVT if you have certain medical conditions that affect how your blood
clots. A blood clot in your legs can also happen if you don't move for a long time,
such as after you have surgery or an accident, when you're traveling a long distance,
or when you're on bed rest.

DVT signs and symptoms can include:

 Swelling in the affected leg. Rarely, there's swelling in both legs.


 Pain in your leg. The pain often starts in your calf and can feel like cramping or
soreness.
 Red or discolored skin on the leg.
 A feeling of warmth in the affected leg.

34. A cricket player was hit on the head with the cricket ball and was fine and then
collapsed suddenly. He woke up to play the match brilliantly but was later found
unconscious in the dressing room.Which of the following is correct about this
presentation?
a) Extradural hemorrhage
b) Subdural hemorrhage
c) Cortical venous thrombosis
d) Sub arachnoid hemorrhage

Ans. Extradural Hemorrhage

THE LUCID interval following head trauma and unconsciousness is described


classically in epidural hematomas, the lucid interval occurs after the patient is
knocked out by the initial concussive force of the trauma and then temporarily
recovers, before lapsing into unconsciousness again when bleeding causes the
haematoma to expand past the extent for which the body can compensate.

35. Boy baby is brought at 3 weeks of age with recurrent episodes of non-bilious
vomiting. Diagnosis is?

a) Duodenal atresia
b) Divarication of recti
c) CHPS
d) Peri-umblical hernia

Ans. CHPS

Congenital Hypertrophic pyloric stenosis (CHPS) is the idiopathic thickening of


gastric pyloric musculature which later results in progressive gastric outlet
obstruction.

Clinical presentation

Symptoms typically clinically manifests between 2 to 12 weeks of age. The


presentation is typical with non-bilious projectile vomiting. The hypertrophied pylorus
is palpated as an olive-sized mass in the right upper quadrant. A succussion splash
can be heard hours after the last meal. Due to the loss of hydrochloric acid in the
gastric contents from persistent vomiting, there is electrolyte imbalance, specifically
the characteristic hypochloremic metabolic alkalosis.
The baby in early stage of the disease remains hungry and sucks vigorously after
episodes of vomiting. Delay in diagnosis can lead to dehydration, malnutrition,
metabolic alterations, and lethargy. The pyloric stenosis can be associated with
Turner syndrome, tracheo-esophageal fistula, esophageal atresia and trisomy 18.

Diagnosis
The diagnosis is easily made if the presenting clinical features are typical, with
projectile vomiting, visible peristalsis, and a palpable pyloric tumor. An enlarged
pylorus, described as an "olive," can be palpated in the right upper quadrant or
epigastrium of the abdomen.
Ultrasound is the modality of choice, and Muscle wall thickness 3 mm or greater and
pyloric channel length 14 mm or greater are considered abnormal in infants younger
than 30 days.
Barium meal Upper GI study reveals delayed gastric emptying, peristaltic waves
(caterpillar sign), elongated pylorus with a narrow lumen (string sign). The entrance
to the pylorus looks like beak (beak sign)

36. An 8-year-old child presents with a localized painless swelling slowly growing in
size since childhood. Which is best treatment for this condition?

a) Excision
b) Chemotherapy
c) Intralesional steroids
d) Radiotherapy

Ans.Excision

A dermoid cyst is a congenital choristoma of the orbit. A choristoma is a benign


tumor consisting of histologically normal cells occurring in an abnormal location.
Dermoid cysts consist of keratinized epithelium and adnexal structures such as hair
follicles, sweat glands, and sebaceous glands. They slowly enlarge with age.
Signs

 Palpable mass
 Ptosis
 Proptosis
 Strabismus
 Pulsating proptosis with mastication
 Globe displacement
 Restriction in extraocular movements
 Inflammation
 Orbitocutaneous Fistula
Small, asymptomatic cysts may not require treatment. They may stabilize or even
decrease in size over years. However, some surgeons opt for early excision to avoid
the risk of traumatic rupture in the future

37. A young Guy comes to your OPD with the complaint that he develops pain in
penis during erection and cannot have sex. Which is the likely diagnosis?

a) Paraphimosis
b) Phimosis
c) Hypospadias
d) Peyronie disease

Ans. Peyronie disease

Peyronie's disease is a noncancerous condition resulting from fibrous scar


tissue that develops on the penis and causes curved, painful erections.
Peyronie's disease is caused by repeated penile injury, typically during sex or
physical activity.
Penises vary in shape and size and having a curved erection isn't necessarily a
cause for concern. In Peyronie's disease, the bend is significant, and may occur
along with pain or interfere with sexual function.

PHARMACOLOGY

1. Which of the following is a SERM?

(a) Ethyl estradiole


(b) Tamoxifene
(c) Anastrozole
(d) Mifepristone

Ans. (b) Tamoxifen

SERM -DRUGS
 Tamoxifen
 Tamoxifen analogs
 Toremifine, Droloxifene, Idoxifene
 Fixed ring compounds :
 Raloxifene, Lasofoxifene, Aroxif, Miproxifene, Levormeloxifene, EM652

2. Which among the following is a monoclonal antibody used for treatment of


cancer?
(a) Cisplatin
(b) Rituximab
(c) Vincristine
(d) Cyclophosphamide

Ans. (b) Rituximab

Rituximab is the first MAb approved for cancer therapy. Clinical trials indicate that
rituximab is efficacious and safe for recurrent or chemotherapy-resistant, B-cell, low-
grade NHL.

3. Find out the best treatment for the given condition?

(a) Valacyclovir

(b) Imipenem

(c) Carbapenem

(d) Penicillin

Ans. (a) Valacyclovir

Valacyclovir is used to treat herpes virus infections, including herpes labialis (also
known as cold sores), herpes zoster (also known as shingles), and herpes simplex
(also known as genital herpes) in adults. It is also used to treat chickenpox and cold
sores in children.

4. A patient was taking Theophylline for Bronchial asthma. After starting


another drug, symptoms of theophylline toxicity started appearing. Which
is the likely drug to cause theophylline toxicity?

(a) Rifampicin
(b) Erythromycin

(c) Valproate
(d) Phenobarbitone

Ans. (b) Erythromycin


Using erythromycin together with theophylline may increase the effects of
theophylline

5. Antidote used for heparin toxicity?

(a) Vitamin K
(b) Protamine sulphate

(c) Ascorbic acid


(d) N acetylcysteine

Ans. (b) Protamine sulphate

Despite of the low therapeutic index, protamine is the only registered antidote of
heparins. The toxicology of protamine depends on a complex interaction of the high
molecular weight, a cationic peptide with the surfaces of the vasculature and blood
cells.

6. A 65-year-old patient presented with dribbling of urine with urgency. He


was started on prazosin therapy, but started developing postural
hypotension. Which of the following is the better alternative drug for this
patient?

(a) Tamsulosin
(b) Phenoxybenzamine

(c) Terazosin
(d) Timolol

Ans. (a) Tamsulosin

 With prazosin and other alpha1 (postsynaptic) adrenergic blockers, first


dose hypotension due to arterial and venous dilatation is well recognised and is
minimised by using a low starting dose. With chronic use, postural
hypotension may persist.
 Tamsulosin is well tolerated and has minimal effects on blood pressure;
tamsulosin 0.4 mg has the lowest potential to reduce blood pressure and causes
less symptomatic orthostatic hypotension than terazosin.

7. A patient presented with hospital acquired pneumonia. On testing he was


found to be allergic to Penicillin G. Which of the following drug is likely to
be safe in this patient?

(a) Methicillin

(b) Cefotitan
(c) Imipenam

(d) Aztreonam

Ans. (d) Aztreonam

Aztreonam can safely be used in all patients with a penicillin or cephalosporin


allergy with the exception of ceftazidime as they share a similar side chain.

8. A new drug is introduced in the market after which phase of clinical trial?

(a) I
(b) II
(c) III
(d) IV

Ans. (c) III

New Drug Application (NDA) is done following the Phase III Clinical Trials, the
drug manufacturer analyzes all the data from the studies and files an NDA with the
FDA (provided the data appear to demonstrate the safety and effectiveness of the
drug). The NDA contains all of the data gathered to date about the drug.

9. A mother is suspected to have a premature delivery. Which of the following


drug could be helpful to attain the lung maturity before birth?

(a) Aspirin
(b) Dexamethasone
(c) MgSO4
(d) NSAID

Ans. (b) Dexamethasone

Betamethasone and dexamethasone are the most widely studied corticosteroids,


and they generally have been preferred for antenatal treatment to accelerate fetal
organ maturation.

10. All of the following conditions get worsened after timolol maleate 0.5%
administration except?

(a) Bronchial asthma


(b) Hypertension
(c) Depression
(d) Diabetes/Glaucoma
Ans. (b) HTN

It works by blocking the action of certain natural chemicals in your body such as
epinephrine on the heart and blood vessels. This effect lowers the heart
rate, blood pressure, and strain on the heart. This medication is also used to
prevent migraine headaches. So timolol doesn’t worsen hypertension but used
to lower it .

11. Which anti-tussive should not be prescribed for the patients who are
driving?

(a) Codeine
(b) Ambroxol
(c) Diphenhydramine
(d) Hydrocodone

Ans. (c) Diphenhydramine

First-generation antihistamines, such as diphenhydramine, are known to affect


driving performance.

PHYSIOLOGY

1. Identify the movement taking place in the Intestine?

(a) Segmented movement


(b) Peristaltic movement
(c) Passive diffusion
(d) Intersegmental contraction

Ans. (b) Peristaltic movement

Peristalsis, involuntary movements of the longitudinal and circular muscles,


primarily in the digestive tract but occasionally in other hollow tubes of the body, that
occur in progressive wavelike contractions. Peristaltic waves occur in the
esophagus, stomach, and intestines.

2. A patient after trauma came to the ER. On neurological examination, he was


found to have normal handwriting skills but could not speak. Where could
be the lesion for his clinical condition?

(a) Wernicke’s area


(b) Temporal area
(c) Broca’s speech area
(d) Primary motor cortex

Ans. (c) Broca’s speech area

Symptoms of Broca’s aphasia include:


 poor or absent grammar
 difficulty forming complete sentences
 omitting certain words, such as “the,” “an,” “and,” and “is” (a person with
Broca’s aphasia may say something like “Cup, me” instead of “I want the
cup”)
 more difficulty using verbs than nouns correctly
 difficulty articulating sounds and words
 difficulty repeating what has been said by others
 trouble with writing sentences
 difficulty reading
 problems with full comprehension
 difficulty following directions
 frustration

3. Pressure – volume graph given and asked to find out A accordingly:

(a) Aortic valve closing


(b) Aortic valve opening
(c) Mitral valve opening
(d) Mitral valve closing
Ans. (b) Aortic valve opening

4. Which part of nephron does water reabsorption takes place more:

(a) A
(b) B
(c) C
(d) D

Ans. (c) C

The proximal convoluted tubule is where a majority of reabsorption occurs. About


67 percent of the water, Na+, and K+ entering the nephron is reabsorbed in the
proximal convoluted tubule and returned to the circulation.
5. Inhibition of Na +/ K+ ATPase pump results in which of the following?

(a) First depolarize and then hyperpolarize


(b) Hyperpolarisation
(c) Depolarisation
(d) Repolarisation

Ans. (c) Depolarisation

Na +/ K+ ATPase pump is essential for the maintenance of Na+ and K+


concentrations across the membrane. ... Inhibition of this pump, therefore,
causes cellular depolarization resulting not only from changes in Na+ and
K+ concentration gradients, but also from the loss of an electrogenic
component of the resting membrane potential.

MICROBIOLOGY

1. A patient complaints of frequent perianal pruritus. On microscopic


examination, following Image was seen. Find out the most probable
causative organism?
(a) Ascaris lumbricoides
(b) Enterobius vermicularis
(c) Trichuris Trichura
(d) Ancylostoma duodenale

Ans. (b) Enterobius vermicularis

Enterobius vermicularis, also called pinworm, is one of the most common nematode
infections in the world. Humans are the only natural host for this infection. Enterobius
vermicularis is an organism that primarily lives in the ileum and cecum.
Once E. vermicularis eggs are ingested, they take about 1 to 2 months to develop
into adult worms which happens in the small intestine. These do not usually cause
any symptoms when confined to the ileocecal area. The female adult worms and ova
migrate to the anal area mostly at night time and deposit thousands of eggs in the
perianal area. This migration causes pruritus. Eggs hatch near the anal area causing
perianal pruritus.

2. A patient with history of travel to Ohio shows lung symptoms. What could
be the most common cause?

(a) Blastomycosis
(b) Histoplasmosis
(c) Mucormycosis
(d) Coccidiodomycosis

Ans. (a) Blastomycosis

 Blastomycosis is an infection caused by a fungus called Blastomyces. The


fungus lives in the environment, particularly in moist soil and in decomposing
matter such as wood and leaves. Blastomyces mainly lives in areas of the
United States and Canada surrounding the Ohio and Mississippi River valleys
and the Great Lakes.
 Blastomycosis is a fungal infection caused by inhaling Blastomyces
dermatitidis spores. If it involves only the lungs, it is called pulmonary
blastomycosis. Only about half of people with the disease have symptoms,
which can include fever, cough, night sweats, muscle pains, weight loss,
chest pain, and feeling tired.

3. Rice water stools are characteristic of:

(a) Shigella
(b) Cholera
(c) E.coli
(d) Salmonella
Ans. (b)Cholera

The passage of profuse "rice-water" stool is characteristic of severe cholera. The


stool is watery with flecks of mucous and has the appearance of water in which rice
has been washed.

4. Which of the following is high level disinfectant?

(a) Gluteraldehyde
(b) Alcohol
(c) Lysol
(d) Phenol

Ans. (a) Gluteraldehyde

High-level disinfectants/liquid sterilants are to be used as immersion products and


not for surface disinfection. Examples are glutaraldehydes, special hydrogen
peroxide, and special peracetic acid products. Items submerged should be limited to
those that cannot be heat-sterilized (such as heat-sensitive plastic items).

5. Identify the organism stained with India Ink stain?

(a) Chlamydia
(b) Histoplasma
(c) Cryptococcus
(d) Aspergillus

Ans. (c) Cryptococcus

India ink capsule stain is used to demonstrate cell capsules through microscopic
examination. This procedure is used to detect presence of encapsualted species,
e.g. Cryptococcus neoformans, which causes cryptococcosis in humans.

6. A pregnant woman with malaria presents in front of the doctor. What could
be the causative organism?
(a) P. vivax
(b)P. falciparum
(c) P. ovale
(d) P. malariae

Ans. (b) P. falciparum

Pregnant women are susceptible to malaria during pregnancy. Plasmodium


falciparum, which sequesters in the placenta, causes the greatest disease,
contributing significantly to maternal and infant mortality.

7. PrP deposition is seen in?

(a) CJD
(b) Rabies
(c) Parkinson’s disease
(d) Alzheimer’s disease

Ans. (a) CJD

Creutzfeldt-Jakob disease (CJD) is caused by an abnormal infectious protein in


the brain called a prion. (prion protein PrP)

8. A patient presented with colicky abdominal pain, diarrhea and no history of


fever. What could be the diagnosis?

(a) E. coli
(b) Entamoeba histolytica
(c) Shigella
(d) Giardia lamblia

Ans. (b) Entamoeba histolytica

Amoebiasis or amoebic dysentery is an infection caused mainly by the protozoan


parasite Entamoeba histolytica. Symptoms may include lethargy, loss of
weight, colonic ulcerations, abdominal pain, diarrhea, or bloody diarrhea.

9. A male patient presented with bloody diarrhea. On Investigation of stool


sample, the organism was found to be gram negative, non-motile and
oxidase negative. What could be the cause?
(a) Shigella
(b) E. coli
(c) Salmonella
(d) Yersinia pestis

Ans. (a) Shigella

Shigellae are Gram-negative, nonmotile, facultatively anaerobic, non-spore-forming


rods.

10. A post covid-19 patient started to develop nasal congestion, black crusts in
nose, and blackish discoloration of skin on nasal and cheek area. Which of
the following micro-organisms is the causative agent ?

(a) Aspergillus
(b) Fusarium
(c) Mucormycosis
(d) Pencillium

Ans. (c) Mucormycosis


Mucormycosis is a fungal infection caused by a group of microorganisms
belonging to the phylum Glomeromycota. Once classified under rare fungal disease,
Mucormycosis is regrouped under emerging pathogens. They are ubiquitous, found
mainly in soil and decaying organic matter.
During the second wave of the COVID-19 pandemic in India, an unprecedented
surge in cases of mucormycosis was observed: immune dysregulation caused by the
SARS-CoV-2 and the use of broad-spectrum antibiotics and corticosteroids—
particularly in patients with poorly controlled diabetes with ketoacidosis

ENT

1. An infant presented with inspiratory stridor and the clinical picture shows
omega shaped epiglottis. The probable diagnosis is?

(a) Acute epiglottitis


(b) RLN palsy
(c) SLN palsy
(d) Laryngomalacia

Ans. (d) Laryngomalacia

This is the well-known "omega shaped" epiglottis in laryngomalacia.


Laryngomalacia is a congenital softening of the tissues of the larynx (voice box)
above the vocal cords. This is the most common cause of noisy breathing in
infancy. Another common finding of laryngomalacia involves the posterior or back
part of the larynx, where the arytenoid cartilages or the mucosa/tissue over the
arytenoid cartilages can collapse into the airway and cause airway obstruction.
2. Identify the Image shown below:

(a) Rhinophyma
(b) Rhinosporidiosis
(c) Rhinoscleroma
(d) Rhinosinusitis

Ans. (a) Rhinophyma (aka potato nose)

Rhinophyma is a skin disorder characterized by a large, red, bumpy or bulbous


nose. It can occur as part of phymatous rosacea. The exact cause of rhinophyma is
unknown, but it's considered a subtype of severe rosacea. This condition is
significantly more common in men, especially between the ages of 50 to 70 years.

3. What is the best management for Atrophic Rhinitis?

(a) FESS
(b) Young’s operation
(c) Nasal steroid spray
(d) Immunotherapy

Ans. (b) Young’s operation

Young's operation is a surgery designed for the treatment of atrophic rhinitis, first
described by Austen Young in 1967. Young's operation is a surgery designed for
the treatment of atrophic rhinitis, first described by Austen Young in 1967.

4. True statement about BERA?

(a) Used to detect early hearing impairment


(b) Invasive test
(c) It is a subjective test
(d) None of the above
Ans. (a) Used to detect early hearing impairment

Brainstem-evoked response audiometry (BERA) is a simple, noninvasive, objective


test for early identification of hearing impairment in children and neonates. It
can be used as a screening test and is useful in newborns, infants, and other
difficult-to-test patients.

5. Laryngeal crepitus is seen in all except?

(a) Tongue cancer


(b)Pneumonia
(c) Laryngitis
(d) Post cricoid carcinoma

Ans. (d) Post cricoid carcinoma

Cause of decreased bone conduction on audiogram?

(a) Ossicular dislocation


(b) Cochlear pathology
(c) Fixation of foot plate of stapes
(d) EAC pathology

Ans. (b) Cochlear pathology

6. The cause of Nasopharyngeal carcinoma is?

(a) HSV
(b) EBV
(c) HZV
(d) Parainfluenza virus

Ans. (b) EBV

The nasopharynx is located at the very back of the nose near the Eustachian tubes.
Nasopharyngeal carcinoma is more common in Southeast Asia and is frequently, but
not always, caused by Epstein-Barr virus (EBV).
7. Identify the procedure shown below?

(a) Hemlich maneuver


(b) Epleys maneuver
(c) Semonts maneuver
(d) Cottles maneuver

Ans. (d) Cottles maneuver

The Cottle's maneuver (CM) is a test in which the cheek on the side to be
evaluated is gently pulled laterally with one to two fingers to open the valve.
This test is used to determine if the most significant site of nasal obstruction is at the
valve or farther inside the nasal cavity.

8. What is the most common causative agent in croup?

a) Measles virus
b) Parainfluenza virus
c)Respiratory syncytial virus
d) Corynebacterium diphtheriae

Ans. (a) Parainfluenza Virus

Croup is commonly caused by virus (75% of cases), less commonly by bacteria. In


most cases, the causative agent is parainfluenza virus type 1 and 2 (a member
of the paramyxovirus family). Croup can also be caused by adenovirus ,measles
virus, respiratory syncytial virus. The most common bacteria associated with croup
are Staphylococcus aureus, Haemophilus influenzae, Streptococcus pneumonia and
Moraxella catarrhalis.

9. A patient presented to ER after fracture of base of skull with CSF


Rhinorrhea. On further investigation the following image was obtained.
What is the name of this sign shown below?
(a) Schwartz sign
(b) Brown sign
(c) Halo sign
(d) Phelp sign

Ans. (c) Halo sign

A “halo” or “ring” sign, occurs when cerebrospinal fluid (CSF) mixes with blood on
an absorbent surface. The blood forms a spot in the center and a lightly stained
ring forms a halo around it.

10. The most appropriate investigation for JNA is?

(a) PET
(b) CT
(c) Angiography
(d) X-ray

Ans. (b) CT

To confirm a diagnosis of Juvenile nasopharyngeal angiofibroma, your doctor


will order a CT scan or MRI. These scans help doctors determine the exact location
of the tumor and see how far it has spread.

11. A child presented with a high arched palate and mouth breathing and
failure to thrive. The probable diagnosis is?
(a) Tonsil hypertrophy
(b) Adenoid hypertrophy
(c) Lingual tonsil hypertrophy
(d) Turbinate hypertrophy

Ans. (b) Adenoid hypertrophy

Adenoid hypertrophy is an obstructive condition related to an increased size of


the adenoids
Adenoid facies refers to the atypical appearance of facial features, and can result
from persistent adenoid hypertrophy. Commonly, adenoid facies is known as “long
face syndrome”, and is characterized by a long, lean face with an open mouth. With
adenoid facies, individuals typically present with increased mouth breathing,
an arched palate, underdeveloped upper jaw bones (i.e. hypoplastic maxilla), a
short upper lip, elevated nostrils, and dental crowding of the front teeth. It most
commonly occurs in children and often presents with chronic nasal obstruction.

12. A patient presented with pulsation of the tonsil. The cause is?

(a) Aneurysm of the common carotid


(b) Aneurysm of the external carotid
(c) Carotid body tumor
(d) ICA

Ans. (d) ICA

Pulsating Tonsil Due to Medial Displacement of the Internal Carotid Artery

RADIOLOGY

1. Thickness of Lead Apron?

(a) 0.1
(b) 0.2
(c) 0.5
(d) 0.01

Ans. (c) 0.5

Lead aprons are the primary radiation protective garments used by personnel during
fluoroscopy. The radiation protection provided by a lead apron is approximately the
same as 0.25- to 1-mm thick lead. An apron with 0.5-mm thickness can attenuate
approximately 90% or more of the scatter radiation.
2. A female visits OPD with her 6-week child presenting with vomiting and
abdominal pain. X-ray of the infant shows?

(a) Jejunal atresia


(b) Diaphragmatic hernia
(c) Duodenal atresia
(d) Pyloric stenosis

Ans. (c) Duodenal Atresia → Image shows Double Bubble Sign

The double bubble sign is a classic radiographic manifestation of duodenal


obstruction, the cause of which could be intrinsic (such as duodenal atresia,
duodenal stenosis or duodenal web) or extrinsic (such as annular pancreas or
rotational anomalies).

3. A two-weeks child brought to the emergency room by her mother with the
symptoms of non-bilious vomiting and lump in the epigastric area. What is
the most appropriate diagnosis?

(a) Congenital pyloric stenosis


(b) Jejunal atresia
(c) Duodenal atresia
(d) Esophageal atresia

Ans. (a) Congenital pyloric stenosis

Pyloric stenosis should be suspected in any young infant with severe vomiting.
Infants with pyloric stenosis classically present with projectile, non bilious vomiting.
Vomiting may be intermittent or occur after each feedingOn physical exam, palpation
of the abdomen may reveal a mass in the epigastrium. This mass, which consists
of the enlarged pylorus, is referred to as the 'olive’.
4. Patient comes to emergency room with vomiting and colicky abdomen
pain. On investigation following Image is obtained. Likely diagnosis is?

(a) Ectopic kidney


(b) Horse-shoe kidney
(c) Pan cake kidney
(d) Cross fused kidney

Ans. (b) Horse-shoe kidney

Horseshoe kidney is a condition in which the kidneys fuse (bind) together at the
bottom, forming a “U” shape or horseshoe shape. Children who have horseshoe
kidney have one “fused” kidney instead of 2 separate kidneys.
5. A patient presents with hard palpable gall bladder with pain in right
hypochondrium. On performing USG following Image was seen. What could
be the diagnosis?

(a) Multiple gall stones


(b) Choledocolithiasis
(c) Hydatid cyst
(d) Gall bladder cancer
Ans. (a) Multiple gall stones

Gallbladder with multiple gallstones. Arrows indicate stones with associated posterior
acoustic shadowing

6. Identify the radiograph shown below?

a) Popcorn calcification
b) Egg shell calcification
c) Ground Glass Appearance
d) Miliary opacities

Ans. Egg shell calcification

Peripheral calcification of lymph nodes, "eggshell calcification," commonly occurs in


patients with sili
cosis and coal-worker's pneumoconiosis. Sarcoidosis, postirradiation Hodgkin
disease, blastomycosis, and scleroderma are other reported causes.

7. A 5 year old child has been brought to hospital with complaints of


discomfort in the hip joint, limping while walking and decreased range of
motion in hip. The following radiograph was taken, what will be the
diagnosis ?

(a) Slipped capital femoral epiphysis


(b) Perthes disease
(c) Myositis ossificans
(d) Meyer’s dysplasia

Ans. (b) Perthes disease

Perthes disease is a rare childhood condition that affects the hip. It occurs when
the blood supply to the rounded head of the femur (thighbone) is temporarily
disrupted. Without an adequate blood supply, the bone cells die, a process called
avascular necrosis.
8. A 6-day-old neonate with respiratory distress showed the following findings
on the chest X-ray. What is the diagnosis?

(a) Congenital cystic adenomatoid malformation


(b) Bronchogenic cysts
(c) Hyaline membrane disease
(d) Congenital diaphragmatic hernia

Ans. (d) Congenital Diaphragmatic Hernia

Congenital diaphragmatic hernia (CDH) occurs when there is a hole in the


diaphragm, which is the thin sheet of muscle separating the chest from the
abdomen. When this gap forms during a fetus's development in the womb, the
bowel, stomach or even the liver can move into the chest cavity.
9. A patient presenting with breathlessness and pedal edema gives a history
of MI few years back. A chest radiograph is taken and shown below. Likely
diagnosis is?

(a) Pericardial effusion


(b) Constrictive pericarditis
(c) Cardiac tamponade
(d) Dilated cardiomyopathy

Ans. (a) Pericardial effusion

Pericardial effusion is the buildup of extra fluid in the space around the heart. If
too much fluid builds up, it can put pressure on the heart. This can prevent it from
pumping normally. A fibrous sac called the pericardium surrounds the heart. This sac
consists of two thin layers.

Pericardial effusion symptoms include:

 Shortness of breath or difficulty breathing (dyspnea)


 Discomfort when breathing while lying down (orthopnea)
 Chest pain, usually behind the breastbone or on the left side of the chest
 Chest fullness
 Swelling in the legs or abdomen
10. A patient of NPC (Nasopharyngeal carcinoma) presented with the following
finding as shown below. Which of the following sign is marked by the
arrow?

(a) Griesenger sign


(b) Phelp sign
(c) Holman miller sign
(d) DODD sign

Ans. (c) Holman miller sign

The Holman-Miller sign (also called the antral sign) is seen in juvenile
nasopharyngeal angiofibroma; it refers to the anterior bowing of the posterior
wall of the maxillary antrum as seen on a lateral skull radiograph or cross-
sectional imaging
11. Identify the diagnosis on Imaging as shown below?

(a) Incomplete mole


(b) Hydatidiform mole
(c) Retained products of conception
(d) PCOD

Ans. (b) Hydatidiform mole

A molar pregnancy also known as hydatidiform mole is a rare complication of


pregnancy characterized by the abnormal growth of trophoblasts, the cells that
normally develop into the placenta. There are two types of molar pregnancy,
complete molar pregnancy and partial molar pregnancy.

Radiographic features

Ultrasound
 enlarged uterus
 may be seen as an intrauterine mass with cystic spaces without any associated
fetal parts,the multiple cystic structures classically give a "snowstorm" or
"bunch of grapes" type appearance.
 may be difficult to diagnose in the first trimester
o may appear similar to a normal pregnancy or as an empty gestational sac
o <50% are diagnosed in the first trimester
 bilateral theca lutein cysts may also be seen on ultrasound
 color Doppler interrogation may show high velocity with a low impedance flow

A 27 year old married women comes to hospital with complains of irregular


menstruation, hirsutism and not able to conceive ,the following usg image was
observed in that patient?
(a) Ovarian hyperstimulation syndrome
(b) Ovarian cyst
(c) PCOS
(d) Endometriosis

Ans. (c) PCOS

 Polycystic ovary syndrome (PCOS) is a condition in which the ovaries


produce an abnormal amount of androgens, male sex hormones that are
usually present in women in small amounts. The name polycystic ovary
syndrome describes the numerous small cysts (fluid-filled sacs) that form in
the ovaries.
The string of pearls (or beads) sign in the ovary is the ultrasound sign of multiple
small (<5 mm) follicles peripherally located in the gonad. It is typically seen in
polycystic ovaries, and is thought to represent abnormal accumulation of immature
follicles.

12. A cattle handler from a village with cough, hemoptysis and chest pain
underwent a chest radiograph and is shown below. What is the probable
diagnosis?
(a) Hydatid cyst
(b) Anthrax
(c) Byssinosis
(d) Abscess/Aspergillosis

Ans. (a) Hydatid cyst

 Cystic echinocccosis (CE), also known as hydatid disease, is caused by


infection with the larval stage of Echinococcus granulosus, a 2–7
millimeter long tapeworm found in dogs (definitive host) and sheep, cattle,
goats, and pigs (intermediate hosts).
 Cough, hemoptysis, and chest pain are the most common symptoms in
patients with hydatid disease of the lungs.

OPHTHALMOLOGY

1. A tumor in the anterior pituitary causing pressure over optic chiasma will
present as?

(a) Homonymous hemianopia


(b) Bitemporal hemianopia
(c) Heteronymous hemianopia with central sparing
(d) Monocular vision loss

Ans. (b) Bi-temporal Hemianopia

When there is compression at optic chiasm, the visual impulse from both nasal
retina are affected, leading to inability to view the tempora, or peripheral, vision.
This phenomenon is known as bitemporal hemianopsia.
2. Ophthalmic surgical Instrument shown in the Image is used for surgery of ?

(a) Entropion
(b) Ectropion
(c) Chalazion
(d) Bitot spot

Ans. (c) Chalazion

A chalazion is a red bump on your eyelid. It is sometimes called an eyelid cyst or a


meibomian cyst. It slowly forms when an oil gland (called a meibomian) becomes
blocked.

A chalazion forceps is a self-retaining type of thumb forceps that has discoid ends.
One arm has a flattened plate at its end, which is round and solid. The other arm has
a matching ring that fits into the opposite arm’s solid plate. There is an adjustable
screw at the middle of the plates, which can be tightened until it has a firm and fine
grip to the cyst or lesion.

3. Corneal transparency is maintained by?

(a) Chondratin sulphate


(b) Keratan sulphate
(c) Hyaluronic acid
(d) Dermatan sulphate
Ans. (b) Keratan sulphate

Keratan sulfate (KS) also plays a role in maintaining the transparency of the cornea
by regulating the organization of collagen fibrils and maintaining corneal
hydration. Alteration of corneal KS may lead to reduced hydration and corneal
opacification. KS is also implicated in corneal wound repair.

4. 5-year-old suffering with chronic epiphora has appeared in the clinic. What
is the next step of management?

(a) Laryngeal probing and massaging


(b) Dacrocystorhinostomy
(c) Tarsal plate fracture
(d) Lacrimal sac massage

Ans. (b) Dacryocystorhinostomy

 Epiphora is an overflow of tears onto the face, other than caused by normal
crying. It is a clinical sign or condition that constitutes insufficient tear film
drainage from the eyes, in that tears will drain down the face rather than
through the nasolacrimal system.
 Dacryocystorhinostomy (DCR) is a surgery that creates a new path for
tears to drain between your eyes and your nose. You may need this
surgery if your tear duct has become blocked.

5. Beta Blocker is avoided in the below conditions except?

(a) Glaucoma
(b) Peripheral vascular disease
(c) COPD
(d) Diabetes

Ans. Glaucoma

 Glaucoma is a group of eye conditions that damage the optic nerve, the
health of which is vital for good vision. This damage is often caused by an
abnormally high pressure in your eye.
 Beta Blockers decrease the pressure inside your eyes by reducing how much
fluid (aqueous humor) is produced in the eyes. Reducing pressure in the
eyes, slows down optic nerve damage which greatly decreases the rate of
vision loss.
6. Patient wearing contact lens since two years develops redness and
decreased vision. What could be the cause?

(a) Bacterial corneal ulcer


(b) Giant cell papillary
(c) Fungal keratitis
(d) Conjunctivitis

(e) Ans. Bacterial corneal ulcer

 Contact lens wear is a common predisposing factor in Bacterial keratitis and


is one of the two preventable risk factors for corneal infection in a working age
population.
 A bacterial corneal ulcer, or bacterial keratitis, is an infection of the corneal
stroma that causes rapid visual loss and pain. Infectious corneal ulcers
need to be treated as soon as possible to preserve vision.

7. Identify the condition?

(a) Chronic melanoma


(b) Retinoblastoma
(c) HSV keratitis
(d) Geographic ulcer

Ans. (b) Retinoblastoma


Retinoblastoma is an eye cancer that begins in the retina — the sensitive lining on
the inside of your eye. Retinoblastoma most commonly affects young children, but
can rarely occur in adults.

8. A patient presented with anterior uveitis in OPD. Which of the following can
be associated with his condition?

(a) HLA B5
(b) HLA B27
(c) HLA B7
(d) HLA DR4

Ans. (b) HLA B27

Acute anterior uveitis (AAU) is the most common form of uveitis, accounting for
approximately 90% of all cases. Half of all cases of AAU are HLA-B27 positive.
The disease is typically acute in onset, unilateral, no granulomatous inflammation
involving the iris and ciliary body, with a tendency to recurrent attacks.

9. A patient with miosis, anhidrosis and mild drooping of eyelid is presented


to the clinic. What is the suspected diagnosis?
(a) III cranial nerve palsy
(b) Horner’s syndrome
(c) VI cranial nerve palsy
(d) IV cranial nerve palsy

Ans. (b) Horner’s syndrome


Horner syndrome is a combination of signs and symptoms caused by the
disruption of a nerve pathway from the brain to the face and eye on one side of
the body. Typically, Horner syndrome results in a decreased pupil size, a drooping
eyelid and decreased sweating on the affected side of your face

10. What is not correct regarding following Image?

(a) Occurs more commonly in sunny areas


(b) Amniotic membrane transplant
(c) It causes myopia
(d) Iron deposition is found

Ans. (c) It causes myopia

The give condition in the image is pterygium.

Pterygium :
 Triangular overgrowth of subconjunctival tissue.
 Wedge shaped, pink -white translucent membrane with apex extending into
cornea.
 Stocker's line - iron line at leading edge
Risk factors - Exposure to UVrays, hot, dry weather, wind, dust
Symptoms
 Irritation, lacrimation, FB sensation, decreased vision , difficulty in contact lens
wear
Treatment
 Excision with conjunctival autograft
 Amniotic membrane transplant with fibrin glue / sutures.
11. A 15-year-old boy presents to clinic with difficulty in coordination of
movements and abdominal pain. On further examination copper deposition
in a ring form is seen in cornea. Most likely diagnosis?

(a) Keratitis
(b) Munson sign
(c) Wilson’s disease
(d) Neuroblastoma

Ans. (c) Wilson’s disease

Wilson's disease is a genetic disorder in which excess copper builds up in the


body.
Individuals with Wilson disease, copper deposits in the front surface of the eye (the
cornea ) form a green-to-brownish ring, called the Kayser-Fleischer ring.

OBG
1) A 55-year-old female presents with complaints of post coital bleeding. She is mother of 6 children
and has a history of early marriage. On colposcopic examination, inflammed cervix is seen which
bleeds on touch. What is the most probable diagnosis?

(a) Cervical cancer


(b) Cervical cyst
(c) Endometrial carcinoma
(d) Vulval carcinoma

Ans. (a) Cervical Cancer

Cervical Cancer is a malignant tumour of the lower-most part of the uterus (womb) that can be
prevented by PAP smear screening and a HPV vaccine.
Symptoms include bleeding in between periods and after sexual intercourse. Foul smelling white
discharge and low back pain or lower abdominal pain may also occur. In some cases there may be no
symptoms.
2) Identify the procedure being done in the Image?

(a) Leopold’s Ist maneuver


(b) Leopold’s IInd maneuver
(c) Leopold’s IIIrd maneuver
(d) Leopold’s IVth maneuver

Ans. (c) Leopold’s IIIrd maneuver

Maneuver I: The uterine contour is outlined. The fundus is palpated with the fingertips of both hands
facing toward the maternal xiphoid cartilage. This should allow the identification of the fetal parts in
the upper pole (fundus) of the uterus.

Maneuver II: Once an assessment is made of the fetal part present in the uterine fundus, the hands
are placed at either side of the maternal abdomen. With this maneuver, the examiner will be able to
determine the location of the fetal back.

Maneuver III: Using one hand, the examiner will grasp the presenting part between the thumb and
fingers. This is done on the lower abdomen, a few centimeters above the symphysis pubis. This will
allow the examiner to develop a further identification of the presenting part and assessment of its
engagement.

Maneuver IV: This last maneuver resembles the first one, but instead of facing the fundus, the
examiner faces the pelvis of the patient. The palms of both hands are placed on either side of the
lower maternal abdomen, with the fingertips facing toward the pelvic inlet. This maneuver should
allow the identification of the fetal parts in the lower pole of the uterus.
3) Look at the Image and diagnose the condition?

a. Kleinfelter
b. Down’s
c. CAH
d. Turner’s

Ans. D - Turner’s syndrome

Turner syndrome is a genetic disorder affecting girls and women. The cause of Turner syndrome
is a completely or partially missing X chromosome. Turner syndrome symptoms include short stature
and lack of breast development and periods. Treatment for Turner syndrome may include hormone
therapy.

 Broad chest.
 Cubitus valgus, where the arms point out slightly at the elbows.
 Dental problems.
 Eye problems, such as a lazy eye or drooping eyelids.
 Scoliosis, when the spine curves sideways.
 Low hairline at the back of the neck.
 Many skin moles.
 Missing knuckle in a particular finger or toe, making the digit shorter.
 Narrow fingernails and toenails.
 Small lower jaw.
 Swelling of the hands and feet.
 Unusually short, wide neck or webbed neck (extra skin folds).
4) An 8-weeks pregnant lady with spotting since the start of her pregnancy. On USG, gestational sac
corresponds to gestational age. What is the probable diagnosis?

(a) Missed abortion


(b) Threatened abortion
(c) Inevitable abortion
(d) Incomplete abortion

Ans. (b) Threatened abortion


 Threatened Abortion
 It is a clinical entity where the process of abortion has started but has not progressed to a state
from which recovery is impossible
 C/o - Slight bleeding
 O/E - Uterus - corresponds to gestational age, Cx - Internal os is closed
 USG - live foetus with subchorionic haemorrhage

5) Treatment for Abortion within 9 weeks?

(a) Mifepristone + Misoprostol


(b) Methotrexate
(c) Misoprostol
(d) Danazol

Ans. (a)Mifepristone + Misoprostol

METHODS OF PERFORMING MEDICAL TERMINATION OF PREGNANCY


 FIRST TRIMESTER
 Medical
 Mifepristone and Misoprostol (PGE,)
 Surgical
 Menstrual regulation
 Vacuum Aspiration (MVA/EVA)
 Suction evacuation and/or curettage
 Dilatation and evacuation (Rapid method/Slow method)

 SECOND TRIMESTER
 Medical
 Prostaglandins PGE1 (Misoprostol), 15 methylPGF2a (Carboprost), PGE2 (Dinprostone)
and their analogues (used-intravaginally, intramuscularly or intra-amniotically)
 Surgical
 Dilation and evacuation
 Intrauterine instillation of hyperosmotic solutions
- Intra-amniotic hypertonic urea (40%), saline (20%)
- Extra-amniotic—Ethacrydine lactate, Prostaglandins
- (PGE2, PGF2a)
 Hysterotomy (abdominal)— less commonly done

 Most suitable method for 2nd trimester abortions is by prostaglandins

 Mifepristone + Misoprostol
 200 mg of mifepristone is given orally on day 1 followed 2 days / 48 hrs later by
 Vaginal misoprostol 800 mcg (4 tablets of 200mcg each) is placed in the posterior fornix of vagina

 The combipack of 1 tablet mifepristone and 4 tablets misoprostol (i.E. 5 tablets) is approved by
DGHS, govt, of india.

 Mifepristone blocks progesterone receptors in the endometrium which leads to disruption of


the embryo, production of prostaglandins and a decrease in human chorionic gonadotropin
levels.
 Combination of mifepristone and misoprostol is effective for inducing abortion within 63 days
of amennorhea

6) 35-year-old female, working in a factory presents within 8 hours after a sexual assault by a male
colleague. She is at day 13 of her menstrual cycle. What should be the emergency contraceptive
of choice in her case?

(a) Levonorgesterol of 1.5 mg single dose


(b) DMPA
(c) Misoprostol followed by methotrexate
(d) Ethyl estradiol

Ans. (a) Levonorgesterol of 1.5 mg single dose

The American College of Obstetricians and Gynecologists recommends a single dose of


levonorgestrel 1.5 mg for emergency contraception within 72 hours of unprotected intercourse as one
option. As another option, they recommend two doses of levonorgestrel 0.75 mg may be effective
when taken 12-24 hours apart.

Levonorgestrel 1.5mg tablets can be used at any time during the menstrual cycle unless menstrual
bleeding is overdue.

7) Identify the Image?

a. Hydrosalpinx
b. Long para ovarian cyst
c. PID
d. Bicornuate uterus
Ans: (a) Hydrosalpinx

Hydrosalpinx is the blockage of a woman's fallopian tube caused by a fluid buildup and dilation of
the tube at its end. Most often it occurs at the fimbrial end of the tube next to the ovary, but it can also
occur at the other end of the tube that attaches to the uterus.

8) A G2P1 presents with history of child with down’s syndrome. Now her POG is 10 weeks. What is
the best possible method of Investigation?

(a) Amniocentesis and Chorionic villi sampling


(b) Triple marker
(c) Quadraple marker
(d) USG

Ans. (a) Amniocentesis and chorionic villi sampling

Karyotyping of cells obtained by either amniocentesis or CVS(Chorionic villus sampling) is the


standard and definitive means of diagnosing aneuploidy in fetuses. The risk that a woman will give
birth to an infant with Down syndrome increases with age.
Chorionic villus sampling can reveal whether a baby has a chromosomal condition, such as Down
syndrome, as well as other genetic conditions, such as cystic fibrosis.
Amniocentesis — Amniocentesis is used most commonly to identify chromosomal problems
such as Down syndrome. When the fetus is known to be at risk, it can detect other genetic diseases
like cystic fibrosis, Tay-Sachs disease and sickle cell disease.

9) Which of the following strain of HPV is more commonly associated with cervical cancer?

(a) HPV 6
(b) HPV 11
(c) HPV 16
(d) HPV 8

Ans. (c)HPV 16

Two HPV types (16 and 18) cause 70% of cervical cancers and pre-cancerous cervical lesions. There
is also evidence linking HPV with cancers of the anus, vulva, vagina, penis and oropharynx.
10) 25 year old married man came to infertility clinic, having azoospermia. normal sized testis, FSH
and testosterone levels also normal. What could be the reason?

(a) Obstruction of Vas Deferens


(b) Kallman syndrome
(c) Klinifelter syndrome
(d) Impotency

Ans.(a) Obstruction of Vas Deferens

Obstructive azoospermia: This type of azoospermia means that there is a blockage or missing
connection in the epididymis, vas deferens, or elsewhere along your reproductive tract. You are
producing sperm but it's getting blocked from exit so there's no measurable amount of sperm in your
semen.

In some men, the epididymis becomes blocked, preventing sperm from entering the vas deferens
and getting into the ejaculate. A blockage can occur on one side or on both sides. If a man has an
epididymal obstruction on one side, he may suffer a lowered sperm count.

11) A female in the labor room - head has not reached to the ischial spines with 3 cm cervical
dilatation in 3 hours will be considered at which stage of labor?

(a) Latent phase


(b) Active phase
(c) Second stage
(e) Third stage

Ans. (a) Latent phase

Latent phase

• It is the preparatory
phase of cervical
dilatation
• It is 8 hrs in primi and
4hrs in multi*
• The cervix dilates only
3 cms during the
latent phase*
12) Image showing a contraceptive device in uterus. The placement is best seen with:

(a) Laparoscopy
(b) Hysteroscopy
(c) Laparotomy
(d) Hysterectomy
Ans. (b) Hysteroscopy

A hysteroscopy is a procedure used to examine the inside of the womb (uterus). It's carried out
using a hysteroscope, which is a narrow telescope with a light and camera at the end. Images are
sent to a monitor so your doctor or specialist nurse can see inside your womb.
The hysteroscope is inserted into the vagina, through the cervix, and into the uterus, allowing the
surgeon to view, take a biopsy or to use special instruments to treat the underlying condition.
Hysteroscopy may also be used to remove adhesions, polyps or a malpositioned
IUD (intrauterine device).

13) A woman with secondary amenorrhea with history of curettage for abortion. FSH 7 IU/ml. What is
your likely diagnosis?

(a) Pituitary failure


(b) Ovarian failure
(c) Uterine synechiae
(d) Pregnancy

Ans. (c) Uterine synechiae

Uterine Synechiae are usually a consequence of uterine trauma caused by: Infections. Prolonged
use of intrauterine devices (IUDs) Post-miscarriage or postpartum curettage.
Uterine synechiae are associated with increased risk of preterm premature rupture of membranes,
placental abruption, and malpresentation. Uterine synechiae are intrauterine adhesions around
which chorioamniotic membranes wrap during pregnancy to produce amniotic sheets or shelves.

14) Identify the Image given below?

(a) Dichorionic Monoamniotic


(b) Monochorionic Monoamniotic
(c) Dichorionic Diamniotic
(d) Monochorionic Diamniotic
Ans. (c) Dichorionic Diamniotic

15) Drug used for Endometriosis?

(a) Raloxifene
(b) Danazole
(c) Gosarelin
(d) tamoxifene

Ans. (b) Danazole

Endometriosis is an often painful disorder in which tissue similar to the tissue that normally lines the
inside of your uterus — the endometrium — grows outside your uterus. Endometriosis most
commonly involves your ovaries, fallopian tubes and the tissue lining your pelvis.

TREATMENT
• Treat all patients - regardless of clinical profile – even asymptomatic pts – progressive disease
– asymptomatic →symptomatic (30-60% patients in 1 yr.
• Chronic disease with ↑recurrence rate (even with hormonal and surgical treatment)

 MEDICAL MANAGEMENT
• Aim - Relief from pain
o Pseudo pregnancy drugs - decidualization of endometrium – COC (Combined Oral Contraceptive),
Progesterone
Pseudomenopause drugs - atrophy of endometrium - Danazol, GnRH agonists

16) A patient presented with secondary amenorrhea after 5 years of her marriage. O/E she was found
to have retroverted uterus with some tenderness and swelling near the posterior wall of vagina.
What is the appropriate diagnosis?

(a) Posterior wall fibroid


(b) Cervical fibroid
(c) Endometriosis
(d) Adenomyosis

Ans. (c) Endometriosis

A retroverted uterus (tilted uterus, tipped uterus) is a uterus that is oriented posteriorly, towards
the back of the body.
A retroverted uterus is due to a disease such as endometriosis
The other factors that can cause the uterus to become retroverted. Pelvic surgery, pelvic
adhesions, endometriosis, fibroids, pelvic inflammatory disease, or the labor of childbirth can change
the position of the uterus to retroverted.

17) Women with ovarian malignancy, what are all the procedures that can be done except?

(a) Debulking surgery


(b) iv chemotherapy
(c) Intraperitoneal chemo therapy
(d)radiotherapy

Ans. (d) Radiotherapy

Radiation therapy in ovarian cancers has been considered an outdated concept for many years,
mainly due to toxicity and failure to show benefit in terms of survival. Chemotherapy has been
extensively used after surgery for these cancers and it has almost replaced radiation therapy as an
adjuvant treatment.

18) A 14-year-old girl presented with primary amenorrhea, complains of urinary retention, pain in the
abdomen. On history talking, she explains that she has been suffering from cyclical abdominal
pain for last 6 months. O/E following image is seen. What is the diagnosis?

(a) Imperforate hymen


(b) MRKH
(c) Klinifelter syndrome
(d) CAH

Ans. (a) Imperforate hymen

Imperforate hymen is when the hymen covers the whole opening of the vagina. Imperforate
hymen is the most common type of blockage of the vagina. Imperforate hymen is something a girl is
born with
19) A 30-year nulliparous lady being married for 5 years came to infertility center for treatment. She is
having secondary amenorrhea for last 45 days. Doctor had done test for her ovulation in which
she is ovulating regularly and normally. Under what parameters you can confirm that ovulation
has taken place?

(a) High FSH


(b) Basal body temperature < 0.5
(c) High LH
(d) High progesterone

Ans. (d) High Progesterone

In conception cycles, properly timed mid-luteal progesterone levels are over 10 ng/mL.
Progesterone is also often used to assess ovulation after induction of ovulation.

20) A 14-year-old girl presented with heavy bleeding per vaginum for last 10 days. What should be
done to know the cause other than pregnancy test?

(a) Coagulation test


(b) LFT
(c) KFT
(d) D-dimer test

Ans. (a) Coagulation test

Coagulation tests measure your blood's ability to clot, and how long it takes to clot. Testing can
help your doctor assess your risk of excessive bleeding or developing clots (thrombosis) somewhere
in your blood vessels.

21) A woman who is lactating came to OPD 6 months after delivery for checkup and to get advice for
contraception. She still wants to continue breast feeding for her child. Which is not to be adviced?

(a) Combined OCP


(b) POP
(c) IUCD
(d) Mini pill

Ans. (a) Combined OCP

Contraceptives which contain estrogen have been linked to reduced milk supply and early
cessation of breastfeeding even when started after milk supply is well established and baby is
older.

22) A 32-year-old female came for regular check up found with ovarian cyst of 6*6 cm. What’s the
next line of management?

(a) Laparoscopic cystectomy


(b) CA-125 follow up
(c) Laparotomy and Ovariotomy
(d) Combined OCP and follow up
Ans. (d) Combined OCP and follow up

Ovarian cysts are growths that develop on or inside the ovaries. There are several types of cysts.
Functional cysts are common, not related to a disease and form as a result of ovulation. Disease-
related cysts can include those that result from polycystic ovary syndrome and ovarian cancer.

Functional ovarian cysts generally go away without treatment. Your healthcare provider may give
you medications containing hormones (such as birth control pills) to stop ovulation and prevent future
cysts from forming. If you do not ovulate, you will not form functional cysts.

23) A primigravida at 24-weeks of gestation comes to the hospital with an appointment with a report
of RBS of 126mg/dl. Fasting and OGTT after 2 hours was found to be 200mg/dl. On performing
the USG the fetus was found to be 2.5kg in weight. What should be the best management for
her?

(a) Reassure
(b) Call her to check again at 34 weeks
(c) Admit, start insulin and check for response
(d) Start insulin and send her home

Ans. (c) Admit, start insulin and check for response

Gestational diabetes mellitus (GDM), defined as any degree of glucose intolerance with onset or
first recognition during pregnancy, is associated with adverse outcomes in mothers and offspring

Target glucose values in women with GDM are ≤ 95 mg per dL (5.3 mmol per L) with fasting, ≤ 140
mg per dL (7.8 mmol per L) one-hour postprandial, or ≤ 120 mg per dL (6.7 mmol per L) two-hour
postprandial.

GDM Pregnant women should be managed by Medical Nutrition Therapy (MNT), and insulin
therapy/ metformin as required and check for response. In the postpartum period, OGTT should be
repeated at 6 weeks after delivery, if blood sugar <140 mg/dL, then women should be referred to NCD
clinic for Post Prandial Blood Sugar (PPBS) testing annually.

24) What is the contraindication of the given device?

(a) HTN
(b) PID
(c) Obesity
(d) Anemia

Ans. (b) PID


ABSOLUTE CONTRAINDICATIONS

 Pregnancy :current, recurrent, or recent (within past 3


months)
 Pelvic inflammatory disease (PID) or sexually transmitted
infection (STI)
 Puerperal sepsis
 Immediate post-septic abortion
 Severely distorted uterine cavity
 Unexplained vaginal bleeding
 Cervical or endometrial cancer
 Malignant trophoblastic disease
 Copper allergy (for copper IUDs)
25) A 40cc fetal material was detected on USG and planned for evacuation. But the catheter goes
deep into the uterus without resistance. What should be the next step?

(a) Mifepristone and misoprostol


(b) Wait and watch
(c) Continue the procedure
(d) Immediate laparoscopy to be done

Ans. (d) Immediate laparoscopy to be done

Uterine perforation is a potential complication of all intrauterine procedures and may be


associated with injury to surrounding blood vessels or viscera (bladder, bowel). In addition, uterine
perforation and associated complications can result in hemorrhage or sepsis. So in order to control
the bleeding immediate laparoscopy should be done.

26) Which of the following is true about true conjugate?

(a) From sacral promontory to posterior border of pubic symphysis


(b) From sacral promontory to anterior border of pubic symphysis
(c) From sacral promontory to upper border of pubic symphysis
(d) From sacral promontory to lower border of pubic symphysis

Ans. (c) From sacral promontory to upper border of pubic symphysis

THE TRUE PELVIS


It is composed of inlet, cavity, and outlet.
The Pelvic Inlet (Brim)

Boundaries
 Sacral promontory,
 alae of the sacrum,
 sacroiliac joints,
 iliopectineal lines,
 iliopectineal eminencies,
 upper border of the superior pubic rami,
 pubic tubercles,
 pubic crests and
 upper border of symphysis pubis.

Diameters
 Antero -posterior diameters:
o Anatomical antero-posterior diameter (true conjugate) = 11cm
from the tip of the sacral promontory to the upper border of the symphysis pubis.

27) Which among the following is the most dependent part in a sitting female?

(a) Hepatorenal pouch


(b) Paracolic gutter
(c) Pouch of douglas
(d) Recto-uterine pouch
Ans. (c) Pouch of douglas

The broad ligaments reflect over the uterus, fallopian tubes, and parametrial uterine vessels and
serve as the anterior boundary of the rectouterine pouch of Douglas. The cul-de-sac or the pouch of
douglas is the most dependent portion of the peritoneal cavity and collects fluid, blood, abscesses,
and intraperitoneal drop metastases.

28) Identify the usg image shown below?

a) Hydatiform mole
b) PCOS
c) Milliary TB
d) Ectopic pregnancy

Ans. (a) Hydatidiform mole

Hydatidiform mole (HM) is a rare mass or growth that forms inside the womb (uterus) at the
beginning of a pregnancy. It is a type of gestational trophoblastic disease (GTD).
Snowstorm sign in obstetric imaging is classically seen in complete Hydatiform mole. It is
characterized by the presence of many hydropic villi which gives the ultrasonographic appearance of
a central heterogeneous mass having a solid, hyperechoic area and interspersed with a multitude of
cystic areas and filling the entire uterine cavity.

DERMATOLOGY

1. Which sign is shown in the given picture?

A. Auspitz sign
B. Dermographism
C. Koebner’s Phenomenon
D. Nikolsky’s sign
Answer: C
Explanation: The Koebner phenomenon describes the appearance of new skin lesions of a pre-
existing dermatosis on areas of cutaneous injury in otherwise healthy skin. It is also known as
the Köbner phenomenon and isomorphic response.

2. From the given image is shown which nail finding that is the characteristic feature of
Psoriasis Vulgaris?

A. Subungual keratosis
B. Auspitz sign
C. Oil drop sign
D. Onycholysis

Answer: A
Explanation: Subungual hyperkeratosis refers to the accumulation of scales under
the nail plate, which is detached and uplifted. The nail bed often appears thickened.
The condition results from excessive proliferation of keratinocytes and failure to shed
off from the stratum corneum.
The nail findings in Psoriasis can be remembered by a simple mnemonic POISON
-P= Pitting
-OI=Oil drop sign
-S=Subungual hyperkeratosis
-O=Onycholysis
-N=Nail plate dystrophy

3. Identify the following Image:

A. Leishmania Cutanea
B. Lepromatous Leprosy
C. Post Kala Azar Dermatitis
D. Tuberculoid Leprosy

Answer: B
Explanation: There is saddle nose
deformity and loss of eyebrows (lateral
madarosis) and we can see multiple
lobular lesions over the face leading us to
the answer Lepromatous Leprosy.
4. Which of the following is best treatment for the following condition?

A. Valacyclovir
B. Imipenem
C. Carbapenem
D. Penicillin

Answer: A
Explanation: we can see from the image that the lesion has clear midline demarcation
and is vesicular lesions on erythematous base leading us to the diagnosis Herpes
zoster
 It is a reinfection of varicella virus
 Painful condition
 Appears as a vesicular lesion on erythematous base on dermatomal
distribution
 Treatment is Acyclovir, Valacyclovir

5. A patient who has anti-convulsant medication presented with rashes which is shown
in the following image. What
could be the diagnosis?

A. Urticaria
B. Fixed drug eruption
C. Angioedema
D. Cellulitis

Answer: B
Explanation: in the question there is a history of drug intake, and the patient
developed rashes after taking a drug leading us to a diagnosis of Fixed drug eruption.
In angioedema and urticaria we can see a much bigger edematous swelling
 In fixed drug eruption, after taking drug a rash develops that heals with
hyperpigmentation
 And after when the drug is retaken, the rash appears on the same site and this
will heal with hypopigmentation.
6. Identify the image given below?

A. Carbuncle
B. Follicular abscess
C. Cellulitis
D. Erysipelas

Answer: A
Explanation: a furuncle is infection of hair follicle, single pus forming infection of a
single hair follicle but whereas a carbuncle is also a pus forming infection of
contiguous hair follicles

7. A patient presented with hypopigmented rash with satellite lesions as shown in the
image given below?
A. BT
B. BL
C. LL
D. TT

Answer: A
Explanation: satellite lesions and finger like projections (Pseudopodia) are seen in BT
type of leprosy
-In BB, we can see Swiss Cheese appearance or inverted saucer appearance
-other dermatological disorders where we can see satellite lesions are
 BT Leprosy
 Sporotrichosis
 Candidiasis
8. A person is with chronic alcoholism, which deficiency
causes the following as shown in the image?

A. Riboflavin
B. Ascorbic acid
C. Biotin
D. Niacin

Answer: D
Explanation: patient is having dermatitis, diarrhea for 2 months, the patient has
appearance of CASAL’s Necklace. So, we conclude that it is Pellagra.
Pellagra is characterized by 4D’s
o Diarrhea
o Dermatitis
o Dementia
o Death
Pellagra is caused due to deficiency of vitamin B3 or Niacin. Frequently seen in Maize/Jowar
eaters due to deficiency of Tryptophan which leads to pellagra due to deficiency of niacin.
Excess of leucine inhibits the conversion of tryptophan to niacin (conversion ratio of
tryptophan to niacin is 60:1) in the body. Here the main culprit is Leucine.

9. Identify the following image?

A. Verrucous vulgaris
B. Elephantiasis
C. Leprosy
D. Leishmaniasis
Answer: B
Explanation: we can see unilateral swelling in lower limb,
 In elephantiasis, there will be a chronic lymphatic obstruction
 Transmitted by culex
 Once the microfilaria enters the body, they convert into adult worms and they
finally destruct the lymphatics causing unilateral, non-pitting kind of an edema
of the lower limbs
 Skin changes: Hypertrophy, verrucous changes and pigmentary changes

10. A truck driver with history of lesions on penis undergoing treatment presented with
rashes. What is the diagnosis?

A. Secondary syphilis
B. Drug eruption
C. TEN
D. Steven Johnson syndrome
Answer: A
Explanation: these rashes are typical for secondary syphilis along with the history of
lesions in the penis is leading us to the diagnosis of Secondary syphilis.
 In (Primary) 10 Syphilis – Hard Chancre, Painless indurated genital ulcer
 If not treated properly patient will have (secondary) 20 Syphilis
JUNE 2021
1. Identify the muscle marked 'X' in the given image?

a) Styloglossus
b) Genioglossus
c) Hyoglossus
d) Palatoglossus

Ans. B
Muscles of Tongue
Intrensic M/s 4 pairs Extrinsic M/s 4 pairs
 Superior Longitudinal M/s  Genioglossus 

 Inferia Longitudinal M/s  Styloglossus


 Transverse M/s  Palato glossus (vagus nerve)
 Verticle M/s  Hyoglossus
Q1. Largest Muscle Genioglossus
Q2. Safty muscle of tongue Genioglossus
Q3. Not a true Muscle of tongue Palatoglossus

2. Arrange a, b, c, d in sequence?
a) A - Cordate nucleus B - Thalamus C - Lentiform Nucleus D – External capsule
b) A-Cordate nucleus B-lentiform nucleus C-thalamus D-external capsule
c) A-Thalamus B-lentiform nucleus C-cordate nucleus D-external capsule
d) A- Thalamus B-cordate nucleus C-lentiform nucleus D-external capsule

Ans. A

3. Injury to bulgar urethra leads to extravasation of urine in all except ?


a) Deep perineal pouch
b) Superficial perineal pouch
c) Scrotum
d) Thigh

Ans. A

4. All of the following are correct except?

a) Superior thoracic artery


b) Thoracoacromial artery
c) Lateral thoracic artery
d) Posterior circumflex humeral artery

Ans. D

1) given artery : AXILLARY ARTERY


Axillary artery is divided into 3 parts by PECTORILIS MINOR MUSCLE
1ST PART
Superior thoraxic artery
2nd part
Thoraco acromial artery
Lateral thoracic artery
3rd part
Anterior circumflex humeral artery
Posterior circumflex humeral artery
Sub scapular artery
Corticospinal tract crosses at
a) Mid brain
b) Pons
c) Lower medulla
d) Upper medulla

Ans. C

Functions:
Motor Functions
Skilled Voluntary monents (coriting)

6. Sutures was done to the finger during the surgery procedure. after the patient complains that he
is un able to
flex the middle finger. this can be due to injury of which of the following?
a) Median nerve
b) Flexor pollicis longus
c) Flexer digitorum profundus
d) Flexor retinaculum

Ans. A
Also called : LABOURES NERVE
Root value: C5 C6 C7 C8 T1
Sign:pointing index Benedictis hand sign
Test for nerve: PEN TEST : Muscle- abductor pollicis brevis

7. A new born present with the congenital anomaly of cleft lip. Which is the embrynological
development deftct?
a) Non fusion of palatine shelves
b) Non fussion of medial nasal process &maxillary process
c) Non fussion of lateral nasal process & maxillary process
d) Separation of nasal septem&vomer from palatine process

Ans. B

8. A patient came to the physician with complains of chest pain which is radiating to the left arm ,
which nerve will
causes pain?
a) Musculocutaneous nerve
b) Intercostobrachial nerve
c) Medial cutaneous nerve of arm
d) Medial cutaneous nerve of forearm

Ans. B
. A patient of covid-19 requires oxygen therapy, which of the following cylinders should be used to
provide
oxygen to this patient?

a. b. c. d.

a) O2 [Black body with white shoulders] pressure 2000 psi (140 kg/cm2)
b) N2O [Blue body] 760 psi
c) Entonox [Blue body with white shoulder]  2000 psi
d) CO2 [Brown]

Ans. A
10. A copd patient in ICU was on oxygen therapy with mask with 2 liter of oxygen. He was found to
be Hypo-
ventilating. His PCO2 on blood gas analysis was 64mmHg. Which of the following should be done?
a) Increase the FIO2
b) Decrease the FIO2
c) Start BIPAP
d) Intubate and start on ventilator

Ans. C

 ABG [ Blood gas analysis ] po2 : < 50 ( normal )


pco2 :>50 ( normal ) ( PH < 7.3 ) causes tachypnea

 Then RR will be :> 35 bpm [ normal : 12-20 bpm]
tidal volume :< 5ml/kg [ normal : 6-8 ml/kg]

 Ventilator

Non – Invasive Invasive [Intubate]

Non-invasive ventilator :

1. Continuous positive airway Pressure (CPAP)  (Alveoli always) open so that gas exchange takes place
continuously
even during expiration]

2. Bilevel – positive airway Pressure [BI – PAP] positive pressure during inspiration and expiration 

(Means better distribution of gases)

3. pressure support ventilation ( PSV )

 NOTE : BI-PAP >> CPAP

Contraindications to NPPV Predictors of Success of NPPV


Cardiac or respiratory arrest Ability to cooperate
Hemodynamic instability Good mask fit/minimal air leak
Inability to protect airway or clear secretions Moderate hypercarbia (PaCO2 45-92 mmHg)
Uncooperative or agitated Moderate acidemia (pH <7.35)
Facial surgery, deformity, or trauma Clinical improvements within first two hours
11. A young patient underwent surgery under GA with O2, N2O and Desflurane. Just after
extubation, the patient
started to desaturate. On auscultation, there are crepitations. What can be the most common
cause?
a) Pleural effusion (No crepitations)
b) ARDS (long term intubation)
c) Negative pressure pulmonary edema
d) Atelectasis (no crepitations)

Ans. C

 It happened because of laryngospasm (Upper airway obstruction)due to secretions


 Rx: by giving positive pressure breath & Suxamethomium (to extubate secretions )

12. Which of the following anesthetic agent is best for cardiac patient?
a) Halothane
b) Isoflurane  it minimally  CO
c) Nitrous oxide
d) Sevoflurate

Ans. B

INHALATIONAL INTRAVENOUS MUSCLERELAXANT


1.Cardiac patient Isoflurane Ethomidate Vecuronium

2.Asthma/COPD Sevoflurane  Propofol Any one


(max dilution) (controlled)
 Ketamine
(uncontrolled)

3.Hepatic Sevoflurane Propofol Cis – atracurium/


(minimally decrease Atracurium
the Hepatic blood flow)

Desflurane
4.Renal Propofol Cisatracurium /
Atracurium

sevoflurane
5.Neuro surgeries Thiopentone Any one can use

13. Which of the following structure is not encountered during epidural anesthesia?
a) Supraspinous ligament
b) Interspinous ligament
c) Ligament flavum
d) Dura
Ans. D

 Epidural:- 5 parts pierced


 Spinal: 5 parts + Dura &Arachorid

14. In endotracheal tube what determines the resistance?


a) Curvature
b) diameter
c) Texture
d) Length

Ans. B
 Resistance to flow  R = 8nL/ Π r4 resistance length , radiusresistance

( L- length , r – radius )

15. 7 A patient was given halothane for general anaesthesia. He developed rigidity and
hyperthermia. These
symptoms are most likely due to which ion?
a) Na+
b) k+
c) ca2+
d) cl-

Ans. C
 Malignant hyperthermia :
 Malignant hyperthermia is a hypermetabolic muscle disorder due to genetic mutation in
the ryanodine receptor
 Mutation is seen most commonly in chr19
 Mutation causes uncontrolled release of ca+ from sarcoplasmic reticulum causing
intense muscle contractions and subsequent complications of hypermetabolism
16. Macrocytic anemia case with normal methyl-malonyl with increaselevel of homocysteine.
Diagnosis?
a) B9 deficiency
b) B12, 9 deficiency
c) Iron deficiency
d) Anemia of chronic disease

Ans. A
 Vit B12 Vit B12  Homocystein methyle  Megaloblastic only B12 – source : mineal pratus
(Cobalamine) transferavo anemia B12 daf seen in Vegetarian
Ilium – (termaind)  Methu melonyle Co-A  Peripheral
Vit B9 / (Folic mutain nspropathy Megablastic anemia
Acid) B12 B9
Methameture N
Folate acid methameture
DHF DHF Reductars acid
  THF  DNA &  Megaloblastic
H2 RNA Anemia
H2  Neural tube defects
Jejurum

17. A newborn develops vomiting, irritability, and lethargy several days afterbirth. His mother
states that his
diapers smell like "burned sugar". Laboratory studies confirm the diagnosis, and the patient is
started on the
appropriate dietary restrictions with subsequent improvement in his symptoms. The defective
enzyme
responsible for this patient's condition
a) Branched chain keto acid dehydrogenase
b) Pyruvate dehydrogenase complex
c) Glutathione peroxidase
d) Glucose-6-phosphatase

Ans. A

18. A 28 year old male has undergone surgery where would debriment wasperformed and (wound
was not
healing which one of the following vitamindeficiency is related with delayed would healing?
a) Vitamin C
b) Vitamin B2
c) Vitamin A
d) Vitamin D

Ans. A

19. LeschNyhan syndrome is reported due to enzyme defect of?


a) Adenosine Deaminase
b) Xanthine Oxidase
c) Hypoxanthine Guanine Phospho Ribosyl Transferase
d) Purine Nucleoside Phosphorylase

Ans. C
20. Which of the following protein doesn't have a quaternary structure?
a) Hemoglobin
b) Collagen
c) Insulin
d) Myoglobin

Ans. D

21. A 8 month old child presents to with hypoglycemia in between feeding. On examination there
was hepatomegaly
and liver was palpable1 cm below subcostal margin. The child also presents with lacticacidosis,
ketosis and
xanthomas on the buttocks. What's the most likely enzyme deficiency leading to this?
a) Branching enzyme
b) Glucose 6 phosphatase
c) C. Glycogen synthase
d) Phosphofructokinase

Ans. B
SIGNS AND SYMPTOMS of Glycogen storage disease type 1-
Glycogen accumulation in liver and renal tubule cells(hepatomegaly &renomegaly)
Hypoglycemia
Lactic acidemia
Ketosis
Hyperlipemia
Disease Enzyme Deficient
Carbohydrates
Type I/Von Gierke’s Disease Glucose – 6-phosphatase
Type II/Pompe’s Disease Acid Maltase
Type III/Cori’s Disease Debranching Enzyme
Type IV / Anderson’s Disease Branching Enzyme
Type V/Mc Arlde’s Disease Muscle Glycogen Phosphorylase
Type VI/ Her’s Disease Hepatic Glycogen Phosphorylase
Essential Fructosuria Fructokinase
Fructose Intolerance Aldolase B
Galactossemia Galactose-1-phosphatase uridyl transferase

22. A newborn develops vomiting, irritability, and lethargy several days afterbirth. His mother
states that his diapers smell like "burned sugar". Laboratory studies confirm the diagnosis,
and the patient is started on the appropriate dietary restrictions with subsequent
improvement in his symptoms. The defective enzyme responsible for this patient's condition
a) Aromatic acid decarboxylase
b) Phenylalanine Hydroxylase
c) Branched Chain Alpha keto acid dehydrogenase complex
d) isovaleryl CoA dehydrogenase

Ans. C
Protein’s
Phenylketonuria (Mousy/Musty odour) Phenylalanine Hydroxylase

Alkaptonuria / Black Urine Disease Homogentisate Oxidase


Maple Syrup Urine Disease (Burnt Sugar Odour) Branched Chain Keta Acid
Dehydrogenase/Decarboxylase
23. A boy presents with the stretchable skin & flexible fingers which he can reach his forearm
following. What's the most likely condition?
a) Osteogenesis imperfecta
b) Ehlers Danlos Syndrome
c) Chondrodysplasia
d) Marfan syndrome

Ans. B

24. Most common type of Collagen seen in fibrous cartilage of joints?


a) Type I
b) Type II
c) Type IlI
d) Type IV

Ans. B
Types Locations
Type I Bone, skin, dentin, cornea, blood vessels,
fibrocartilage and tendon
Type II Cartilaginous tissues
Type III Skin, ligaments, blood vessels and internal organs
Type IV Basement membrane in various tissue
Type V Blood vessel wasll, synovium, corneal stoma,
tendon, lung, bone cartilage and skeletal muscle

25. A 73-year-old woman comes to the physician because of a 2-monthhistory of diffuse weakness
and tingling of
her arms and legs Neurologicexamination shows weakness of the extensor and flexor muscles
of thelower extremities. Sensation to vibration and position is decreased in allextremities.
This patient most likely has a deficiency of which of the following vitamins?
a) Vitamin B1
b) Vitamin B2
c) Vitamin B6
d) Vitamin B12
Ans. D
Vitamin Name Deficiency
Vitamin B1 Thiamine Deficiency causes beriberi. Symptoms of this disease of the
nervous system include weight loss, emotional disturbances
Vitamin B2 Riboflavin Deficiency causes ariboflavinosis. Symptoms may include
cheilosis (cracks in the lips), high sensitivity to sunlight,
Vitamin B3 Niacin Deficiency causes pellagra. Symptoms include aggression,
dermatitis, insomnia, weakness, mental confusion, and diarrhea
Vitamin B5 Pantothenic Acid Deficiency can result in acne and paresthesia, although it is
uncommon
Vitamin B6 Pyridoxine Deficiency may lead to microcytic anemia (because pyridoxyl
phosphate is the cofactor for heme synthesis, depression,
dermatitis
Vitamin B7 Biotin Deficiency does not typically cause symptoms in adults but may
lead to impaired growth and neurological disorders in infants
Vitamin B9 Folic Acid Deficiency results in a macrocytic anemia, and elevated levels of
homocysteine. Deficiency in pregnant women can lead to birth
defects

26. Xeroderma Pigmentosa, the type of DNA repair defect


a) Nucleotide excision repair
b) Base excision repair
c) Double stranded break repair
d) Transcription coupled repair

Ans. A
DNA REPAIR MECHANISMS:

1. Exonucleolytic Proofreading
2. Nucleotide Excision Repair -
a. Xeroderma Pigmentosa
3. Strand Directed Mismatch Repair -
a. Hereditary NON Polyposis Colon
b. Cancer (Lynch syndrome)
4. Base Excision Repair
5. Double Strand Break Repair
6. Transcription Coupled Repair
a. Cockayne's syndrome

27. A 25 old female presents to medicine OPD with acuteyearabdominal pain for which almost
exploratory
laparotomy was done. Heis also accompanied by neuropsychiatric symptoms and the urinesample
collected
turns to black with time in room temperature) Mostlikely diagnosis is
a) Porphyria cutanea tarda
b) Acute intermittent porphyria
c) Congenital erythropoietic porphyria
d) Variegate porphyria

Ans. B
28. Storage form of neutral lipids in our liver is
a) Cholesterol
b) Cholesterol ester
c) Triglycerides
d) Fatty acids

Ans. C

29. A 25 year male presents with hypercholesterolemia and increasedrisk of heart disease. His
elder brother also
had similar findings andsuccumbed to this. Workup reveals a congenital LDL receptordeficiency,
The most
likely defect will be?
a) Primary familial hypercholesterolemia
b) Abetalipoproteinemia
c) Familial combined Hyperlipidemia
d) Apolipoprotein E

Ans. A
30. Which of the following is a cofactor for Glutathione Peroxidase?
a) Zinc
b) Copper
c) Selenium
d) Calcium

Ans. C
31. A 35 years old male returns back to India from China. On immigrationhe was found to have
slightly elevated
temperature. One of the followingscreening test was used for this?

a) CBNAAT
b) Dimer
c) Real time RT-PCR
d) CRP

Ans. C

32. Marasmus is reported in Indian Children mainly due to?


a) Due to decreased intake of carbohydrate
b) Due to decreased intake of protein
c) Due to decreased intake of fat
d) Due to decreased intake of vitamins

Ans. B
33. Which vitamin deficiency leads to cognitiveimpairment in old age?
a) Vitamin A
b) Vitamin B 6
c) Vitamin B 12
d) Vitamin C

Ans. A
Vitamin A – Retinol/Retinoic acid
Vitamin B 6 - Seizures in children
Vitamin B 12 - Methylcobalamine& cognitive impairment
Vitamin C – Iron absorbtion
34. An alcoholic patient with neurological symptoms. There is deficiency of vitamin ?
a) B1
b) A
c) B9
d) B12

Ans. A

 B1- Thiamine
 Deficiency seen in chronic alcoholics
 Wernicks Encephalopathy
 G-global confusion
 O-opthalmoplegia
 A-ataxia
 Korsakoff psycosis
 B9-folic acid
 B12-methylcobalamine
35. A patient presented with the following lesion. Likely diagnosis?

a) Hypomelanosis of ito
b) Vitiligo (segmental)
c) Nevus of ito
d) Becker's nevus
Ans. B
Segmental vitiligo
Vitiligo vulgaris Most common type
Segmental vitiligo Seen in children
Most frequent in trigeminal nerve distribution
Generalized vitiligo Extensive lesions
Acrofacial vitiligo Periorificially and acral parts
Lip- tip vitiligo Lips, tip of penis, vulva, nipples
Vitiligo universalis Widespread ,multiple endocrinopathies

36. A female patient presented with a multiple warty lesions around vulva. The lesions are
gradually increasing.
On examination they are soft and sessile and didn’t bleed on touch. What is the lilely diagnosis?

Condyloma accuminata

a) Condyloma acuminata
b) Bowen disease
c) Condyloma lata
d) Hemorrhoids

Ans. A
Condyloma accuminata : Caused by HPV 6 and 11
Occurs on penis and vulva, in the urethra and vagina, at the uterine cervix, and in the perianal region.
May become large and invasive
Condyloma lata: Seen in secondary syphilis.

37. A female student presented with utricaria following intake of seafood.she wants a non-
sedating drug since this
is her exam time.which is the preferred antihistamine for this patient?
a) Diphenhydramine
b) Fexophenidine
c) Promethazine
d) Chlorphendimethate

Ans. B
Fexophenidine

38. What could be the diagnosis for this image where exclamation mark is seen in trichoscopy?
a) Alopecia areata
b) Telogen effluvium
c) Trichotillomania
d) Anagen effluvium

Ans. A
Alopecia areata
Patchy alopecia with exclamation mark hair (image)
Subtypes:
 Ophiasis→ alopecia along margin of scalp
 Alopecia totalis→ loss of hair on total scalp
 Alopecia universalis→ loss of hair on whole body

Features:
 Sharply defined alopecia
 Exclamation mark hair
 Sparing of grey /white hair
 Going white over night phenomenon
 First site affected: scalp
 Nails: hammered brass( fine stippled pitting)

Treatment: few lesions → topical minoxidil, topical corticosteroids, topical psoralen + UVA
Extensive lesion → oral corticosteroids, oral psoralen+ UVA

39. A 70 years old patient develop the following lesion over face since 2 years. What is the
diagnosis?

a) Basal cell carcinoma


b) Squamous cell carcinoma
c) Malignant melanoma
d) keratoacanthoma

Ans. A
Basal cell carcinoma
Underlying cause :
 Albinisum
 Arsenic
 Psoralens + UVA
 UV rays
 Tar
 Xeroderma pigmentosum
C/F:
Middle aged/ elderly males
Rodent ulcer ulcerated nodule with rolled, pearly, beaded edges.
Superficial BCC
Morphoeic BCC
Pigmented BCC

Site: FACE (except superficial BCC)


Palisading arrangement
Treatment: surgical excision
Keratoacantoma: This shows keratin deposits in the center.

40. A young lady presents with lacy lesion in the oral cavity. What is the diagnosis?

a) Candidiasis
b) Psoriasis
c) Lichen planus
d) Oral leukoplakia

Ans. C
Oral lichen planus→Asymptomatic/ complain of intolerance of spicy food.
White, reticulate lacy pattern on the buccal mucosa, tongue and gingival.
Erosive gingivitis.

41. Patient presented with skin lesions and crusting.


What kind of organism caused this lesion?
a) Gram positive cocci
b) Gram negative cocci
c) Gram positive bacilli
d) Gram negative bacilli
Ans. A
Impetigo →streptococcus aureus and streptococcus pyogens
Characteristic feature: honey -coloured crust

42. Which of the following is an unpaired cartilage of larynx?


a) Arytenoid
b) Thyroid
c) Cuneiform
d) Corniculate

Ans. B

* Paired Unpaved
Arytenoid ● Thyroid
Cuneiform ●Cricoid:- Complete ring cartilage
Curricular * Epiglottis

43. Which of the following laryngeal cartilage doesn’t calcify with age:
a) Thyroid
b) Cricoid
c) Epiglottis
d) Croniculate

Ans. C
Epiglottis is a fibro elastic / soft cartilage will not ossify with age
44. A 40 years old trumpet blower has develoved a right sided swelling in upper part of neck. The
swelling keeps
changing in size . the image shown in x-ray find the appropriate diagnosis of the condition?

a) Throglossal cyst
b) Brachial cyst
c) Laryngocoele
d) Pharyngeal pouch

Ans. C

Laryngocele
 It is the dilation of the saccule of laryngeal ventricle
 On imaging, these lesions are generally characterized as well-defined,thin-walled, fluid or air-
filled cystic lesions in the paraglottic space

45. Which of the following is the m/c site of head and neck cancer in India?
a) Nasopharynx
b) Oropharynx
c) Oral cavity
d) Hypo pharynx

Ans. C
M/c site of oralcavity ca in India:-
Buccal mucosa (Gingio buccal sulcus)
Head &Neck:- Oral cavity cancer
Over all:- Brest cancer

46. Identify the structure market:-


a) Spheroid sines
b) Frontal sines
c) Maxillary sines
d) Ethmoid sines

Ans. C
Maxillary sines is the largest sines of all 4 sines
Maxillary sines drains into MIDDLE MEATUS

47. Identify the name of this x – ray shown in given they:-

a) Cold well’s view


b) Waters view
c) Schuller’s view
d) Towne’s View

Ans. B
* Water view ⭢Occipto mental view (also Done with closedmouth)
*Cald well’s view Occipto frontal view (Frontal & ethmoid sinus more clearly)

* Schuller’s View
D x – ray mastoid view
* Towne’s View
48. Which of the following is the most common craw of neonatal and paediatric inspiratory
stridor?
a) Laryngesmalacia
b) Laryngeal web
c) Congenital subglottic – stenosis
d) Laryngeal cyst

Ans. A
 Laryngomalacia is m/c congenitaialanomaly of laryngs
 Weakness of supraglottis which leads to lnspiroy stridor shaped larynges omega

49. Which of the following laryngeal muscle is the abductor of vocal cord:-
a) Cricothyroid
b) Thyroartenoid
c) Posterior crico arytenoid
d) cateralcricoartenoid

Ans. C

NERVE SIPPLY

Only: 1 abductor:- Posterior crico arytenoid Recurrent laryngeal nerve

Adductor:- Cricothyroid Superior laryngeal nerve


Lateral cricoarytenail
Interaryteroid Recurrent laryngeal nerve
Thyroarytenoid (ext part)

Tenser muscrr :- Cricothyroid


Vocals (int part of thyroarytenoid)

50. What is he exact anatomical location of Killian’s dehiscence?


a) Between superior and inferior constrictor muscle
b) Between Thyropharyngeus and cricopharynges muscle fibers
c) Inferior constrictor muscle
d) Between middle and inferior constrictor muscle

Ans. B
51. A 4 years old child has respiratory distress, fever, inspiratory stridor, x – ray neck lateral view
shows view
shows thumbsign on neck what could be causing organism?
a) Harmophilus influenza
b) Streptococcus pneumonia
c) Para influenza virus
d) Staphylococcus aureus

Ans. B

Dx: ACUTE EPIGLOTTITIS

Epiglottitis
 THUMB-PRINT SIGN is seen
 Epiglottitis may be seen radiographically on a lateral neck x-ray
 Epiglottic swelling(supraglottic) of three to four times normal is seen at the base of the tongue and
appears as if someone left a thumbprint

52. 7 years old child is complaining of hearing loss and sense of full ness in the ear. The
tympanometey show type
B curve what is the diagnoris in this child?
a) Chronic suppurativeotive media
b) Secretory otitis media
c) Acute suppurative otitis media
d) Otitis externa

Ans. B
M/c:- in school going children
2 – Prominent symptonr:- CHL
Fullners in the ear
TYPE A curve: NORMAL
TYPE B curve: serous /secretor otitis media
TYPR As curve:oto Sclerosis
TYPE C curve : Carcinoma (middle ear obstruction)
TYPE Ad curve: oscoles Disruption
53. A 17 year old female patient developed earache on right side post tonsillectomy. Which nerve
is responsible
for the ear pain?
a) Glossophrayngealnerve
b) Vague Nerve
c) Trigeminal nerve
d) Facial nerve

Ans. A
Ear ache right side post tonsillectomy – Glossophrayngeal nerve
Nerve lies in the bed tonsil Glossophrayngeal nerve
Sensory supply to middle ear – Tympanic branch Glossophrayngeal nerve

54. A neonate suffering from bilateral complete choanal atresia has respiratory distress since
birth. What is the
cause of respiratory difficulty in this neonate?
a) Less lung Volume
b) Associated vocal cord palsy
c) Newborns are obligatory nasal breathers
d) Insufficient lung surfactant

Ans. C

55. A 40year old diabeticwoman tested covid 19 positive and she started to have nasal congestion,
black crusts in
nose, and blackish discoloration of skin on nasal bridge and cheek area. What could be the cause?
a) Mucormycosis
b) Allergic fungal sinusitis
c) Nasal vestibulitis
d) Maxillary sinusitis

Ans. A
zygomycetes→seen in terminally ill patients
R → Rhizopus
A→Absidia
M→Mucor
Tissue biopsy :H&E stain (aseptate hyphae, obtuse angled)
Biopsy material on SDA→ cultural growth + LPCB→ Rhizopus
Mucor mycosis is commonly seen in:
DM
Frequent blood tansfusion
Chronic dialysis

56. A patient of Presbyacusis has been found to have high frequency sensorineural hearing loss on
audiometry.
Which of the following part of inner ear is affected in this patient?
a) Apexof cochlea
b) Basal turn of cochlea
c) Utricle
d) Saccule

Ans. B

57. Which of the following arteries is the main blood supply of Tonsil?
a) Descending palatine artery
b) Ascending palatine artery
c) Tonsillar branch facial artery
d) Ascending pharyngeal artery

Ans. C

58. A 60 year old diabetic patient has presented with severe ear ache and blood stained
eardischarge. The
examinationshows granulations in the external auditory canal What is the possible diagnosis in
this case?
a) Malignant otitis externa
b) Diffuse otitis externa
c) Otomycosis
d) Chronic suppurative otitis media
Ans. A

Young diabetic : black nose – MUCORMYCOSIS


Old diabetic : EAR – MALIGNENT OTITIS EXTERNA

m/c/c –PSEUDOMONOS

59. A child aged 2 years, presented with severe SNHL was prescribed hearing aids, but showed no
improvement. Which is the next line of management?
a) Fenestration surgery
b) Conservative
c) Stapes mobilization
Cochlear implant

Ans. D

60. Cause of death in Judicial hanging?


a) Asphyxia
b) Brain hypoxia
c) Cervical vertebral fracture
d) Vagal inhibition
Ans. C
Judicial hanging Hang man’s # C2 – C3 vertebra#
C 2:Pars inter articular part of vertebra

61. Cherry red colour post-mortem staining is associated with which poisoning ?
a) H2s
b) CO
c) Nitrate
d) Hydrogen cyanide

Ans. B
CO: Cherry red

H2s: Bluish greennitrate :redish brown:


Hydrogen cynide:BRIGHT RED

62. A baby patient comes to casualty with history of unknown poisoning by some seed/drug, on
examination
patient presented with Dilated pupil, Dry mouth, Dilirium and decreased, secretions, skin
scratches,
Hallosination poisoning should the Dr suspect
a) Dhatura
b) Poppy seeds
c) Mushroom
d) Cannabis
Ans. A
 Dathura contains anti-cholinergic substances that cause these symptoms
 Mushroom contains cholinergic substances
 Poppy seeds contains opium- morphine like substance

Symptoms Resemble Atropin poisoning



Active principle(hyoscine) in Dhatura poisoning

63. A person died due to starvation, which is correct in such case during autopsy?
a) Enlarged heart
b) Enlarged stomach
c) Distended gall bladder
d) Expanded lungs

Ans. C
When a person have his food. When if reaches Duodenium it stimulates to release of CCK which helps in
contraction of gall bladder and Bile is released.
 As the person is in starvation:- bile secritons will not enter to deodinum so, gall bladder
distended.
 All visceral organ size , Except BRAIN
 Lungs will get collaps
 Cause of death is starvation:- Circulatory failure

d/t:- Brown atrophy of heart

64. A person has committed a criminal offence, caught by police & later brought to a criminal court
but the court
directs police to transfer the case to Juvenile courts if it was proved that accused is of ___________
years of age?
a) 21
b) 17
c) 25
d) 19

Ans. B
Child < 18  according to Juvinile Justice Board.
Abdament in act:- any Juvinal of 16-18 years do’s Heanes offence like (Rape, murder) will come under
adult.
65. In Barberious test, yellow colourneede shaped crystals are seen under microscope. This is
done to detect
which stain?
a) Sputum
b) Semen
c) Blood
d) CSF

Ans. B

Boy friend:

1) Barberious
2) Florena = Semen

Blood:-T-Tetamans
T – Takayama

66. A farmer was brought to casualty with pinpoint pupil increased secretions and garlicky ordor,
which drug to
be used by physiation for treatment?
a) Oximes
b) Atropine
c) N-acetylcyteine
d) Physostigmile

Ans. B
 organophosphate poisoning(OP): pinpoint pupil,garlicodour,increase secretions
 DOC for OP: atropine
 DOC for angiodiazepine: Flumazenil
 Paracetmolpoisoning : N-acetylcysteine is used
 Note: oximes are used in OP but it is not the drug of choice

OPC – poisonics (y? :- farmer, pinpoint pupil, garlicky order)


, Ach at Receptor level
Controlled by:-Atropine

Physostigmine:- Doc:- DHATHUR poisoning


Oximes :- may be used but in 2nd choice
N – acetyl cysteine:- Antidote :- for Paracetmolo poisoning / Acetamrinaphin

67. A childaccidentaly consumes iron syrup used by the mother, Antidote to be used in such
poisoning is?
a) BAL
b) EDTA
c) Desferrioxamine
d) Trientene

Ans. C
Chelatorfor :copper poisoning &wilsons disease: TRIENTENE

68. Gutter fracture of skull bone is associated with impact of?


a) Sharp object
b) Blunt object
c) Fall from height
d) Bullet

Ans. D
Tanguntial gun shot:- # of only outer table of skull

69. In an unconscious patient with head injury requiring emergency Sx. If there are no relatives to
give consent the doctor should?
a) Operate without consent
b) Should not operate
c) Wait for the arrival of relatives
d) Take consent from the hospital authority

Ans. A
IPC: 92 state DOCTORCANoperate with out consent in case of emergency

70. A 40-years old patient was brought by police in unconscious state to casualty with history of
unknown poisoning, on examination patient had needle track ulcers over upper limbs, Bugs
crolling on skin, patient had Hallusions with scratch marks, increased blood pressure,
increased heart rate and respiratory rate of 30/ minute, which poisoning should Dr. suspect?
a) Cocaine
b) Heroin
c) Cannabis
d) LSD
Ans. A
Option we can easily eliminate by:
HEROIN : respiratory rate decreases
CANNABIS: BP decreases (due to :vomiting&diatthia)
LSD : is taken through oral rout so percutianeous ulcers are not found over upper limb

71. Partial hanging mimics like ligature striangulation, How?


a) Hyper extended neck
b) Transverse mark over neck
c) Hand touches ground
d)Glow & stocking marks

Ans. B
LIGATURE MARK DIFFERENCE BETWEEN

PARTIAL HANGING COMPLETE HANGING

72. First tooth to appear?


a) Upper lateral incisors
b) Lower lateral incisors
c) Upper central incisors
d) Lower central incisors

Ans. D
Lower central incisors :6-10 months
Upper central incisors :8-12 months
All teeth appear by 2.5 years

73. Which of the following immunoglobulins is a pentamer?


a) IgM
b) IgA
c) IgE
d) IgG

Ans. A
74. Identify the organism:

a) HIV
b) Herpes
c) Hepatitis B
d) Adenovirus

Ans. A
Family :Retroviridae
Subfamily: lentivirnae
It is a ssRNA virus
3 structural gene
→gag : matrix protein
→pol: polymerase
→Env : envelop, Gp41,Gp120
HIV affects CD4+ve T-cells by coreceptors CXCR4
HIV affects macrophages by coreceptors CCR5
Most sensitive test for HIV: ELISA
Most specific test for HIV: HIV RNA PCR ,WESTERN BLOT
Mother to child transmission –30%
Blood transfusion –90%
Needle stick injury—0-5%

75. A 5 year old child presents with fever, bull’s neck as shown in the image. Identify the disease.

a) Mumps
b) Lyme’s disease
c) Diphtheria
d) Measles
Ans. C
Diphtheria( klebs loeffler’s bacilli)
Gram positive bacilli
Chinese letter pattern/cuneiform arrangement is seen.

Staining—ponder’s, albert’s, neisser’s ( contents of albert’s stain: toludine blue,malachite green, glacial
acetic acid, kl + l2)
Pseudomembrane formation is seen → greyish site →M/C site faucial (tonsillar)
Bull’s neck( neck edema)
Chronic complication : myocarditis, polyneuropathies
Dx:
LSS( loeffler’s serum slope) ,culture media,enriched media(used for early diagnosis of diphtheria)
Potassium tellurite agar( best selective media)
Macleod’s media ( diphtheria toxin)

76. Most common viral agent causing pneumonia in < 1 year of age:
a) RSV
b) Parvovirus
c) HHV-6
d) Influenza virus

Ans. A
RSV( Respiratory Syncitial Virus) –RTI in <1 year of age.
Diagnosis :cytopathic effect (synctium form)—MN giant/mØ.
DOC—Ribavarin.

Safety-pin appearance is seen in

Yersinia pestis

Explanation: yersinia pestis causes Plague


Other organisums which show safety-pin appearance:
1) Klebsiella granulomatis
2) Burkholderia mallei
3) Burkholderia pseudomallei
4) Vibrio parshemolyticus
5) Haemophilus ducreyi

77. The agent of the disease shown in the image is not found in which body fluid?

a) Saliva
b) Urine
c) Fluid from the leg
d) Ascitic fluid

Ans. A

78. Which among the following infection doesn’t form a membrane like covering over the tonsils?
a) Diphtheria
b) Infectious mononucleosis
c) Streptococcal tonsilitis
d) Ludwig angina

Ans. D
Ludwig's angina is a rare skin infection that occurs on the floor of the mouth, underneath the tongue. This
bacterial infection often occurs after a tooth abscess, which is a collection of pus in the center of a tooth.

79. A female with the following symptoms of sudden high fever, associated with headache and
vomiting along with
nuchal rigidity, presented to the hospital. CSF analysis of this patient showed the growth of this
organism aa
gram negative diplococci shown in the image,identify the organism:

a) Neisseria meningitidis
b) Streptococcus pneumoniae
c) Staphylococcus aureus
d) Mycobacterium tuberculosis
Ans. A
Neisseria meningitidis: Gram negative cocci(catalase/oxidase positive)
Ferrments glucose and maltose( capsulated)
Cultured media : thayer martin media, muller hinton agar
Transport media : stuwart amile media
Complications: water-house friderichsen syndrome( B/L adrenal haemorragic necrosis)
Deficiency of C5-C9(MAC): Prone for neisserial infections.
DOC: Ciprofloxacin( carrier), ceftrioxan(cases)

80. A covid patient presented with fever along with nasal congestion and unilateral headache.
Later he developed
facial pain and numbness along with blackish discharge, KOH amount reveals nonseptate hyphae
at right
angkes or obtuse angle, what is the most probable diagnosis?

a) Mucormycosis
b) Aspergillosis
c) Bacterial orbital cellulitis
d) Cavernous sinus thrombosis

Ans. A

81. Identify the organisum given in the smear image.

a) Plasmodium Falciparum
b) Plasmodium vivax
c) Plasmodium ovale
d) Plasmodium malariae
Ans. A
Plasmodium vivax Plasmodium falciparum
Young aged RBC involved All age RBC are involved
↑ in size of RBC Normal size RBC
Schuffner’s dot Maurer’s dot
Gametocyte stage Banana shaped gametocytes
Single ring with in RBC Multiple ring within RBC
Reddish brown pigment Dark brown pigment

82. Identify the organisum causing diarrhea

a) Rotavirus
b) Giardia lamblia
c) Cryptosporidium
d) Adenovirus

Ans. B
Giardia lamblia → fatty foul smelling diarrhea and fat malabsorption.
Feco-oral route transmission
Dx: string test
Falling leaf motility
4 pairs of flagella , tennis racket shape
DOC: Metronidazole, tinidazole
83. Orchitis is an important complication of which infection?
a) Mumps virus
b) Adeno virus
c) Polio virus
d) Pox virus

Ans. A
Myxo virus
Period of communicability: 4-6 days before and 1 week after.
Secondary attack rate: 86%
Mode of transmission : droplets
C/F: pain, swelling in parotid region, earache.
Complication: aseptic meningitis( children), orchitis( M/C extra salivary manifestation in adults), leading
cause of pancreatitis in children.
Vaccin: jerry-lynn stain, live attenuated

84. An immunocompromised patient came to the hospital with severe diarrhea. Flectal swab
revealed the
following image. Identify the causative organism.

a) Cryptosporidium parvum
b) Rotavirus
c) Giardia lamblia
d) Adenovirus

Ans. A
Cryptosporidium parvum → causes diarrhea in AIDS patients.
Oocysts and cyst are infective
Dx:Acid fast oocyts
DOC: Cotrimoxazole

85. A 10 year old child is diagnosed with amblyopia in one eye. What will be the best treatment ?
a) Observation
b) Penalisation
c) Occlusion
d) None of the above

Ans. C
Occlusion of the good eye to force the use of lazy eye.
86. A 3 months old baby presents with mucous exuding from eye on pressing the lacrimal sac.
What treatment
should be given?
a) Lacrimal sac massage
b) Syringing
c) Probing
d) DCR

Ans. A
Lacrimal sac /Pediatric massage/Crigglar sac massage: only till 1 year

1-2 years: syringing


After 2-3 years : probing (bowman’s lacrimal probe)
After 3 years : DCR

87. Which lesion of the visual pathways produces a macular sparing field defect?
a) Optic chiasma
b) Occipital lobe
c) Optic nerve
d) Optic tract

Ans. B
Occipital lobe is supplied by 2 arteries: posterior cerebral artery and middle cerebral artery in which
posterior cerebral artery is more often got damaged. And middle cerebral artery still supplies so we see
macula is spared.

88. A child with history of malnutrition is examined and the following finding is noted. What is the
most likely
diagnosis?

a) Megalocornea
b) Buphthalmos
c) Anterior staphyloma
d) Corneal degeneration
Ans. C

Due to malnutrition corneal perforation occurs which further leads to staphyloma.


Protrusion of the outer tunic eyeball w.r.t equator :
Anterior to equator —anterior staphyloma
Posterior to equator – posterior staphyloma
Along the equator – equatorial staphyloma

89. A young male complains of watery eyes and itching and on eversion of upper eyelid the
following picture is
seen. What is the diagnosis?

a) Bacterial conjunctivitis
b) Trachoma
c) Vernal keratoconjunctivitis
d) Angular conjunctivitis

Ans. C
In question-- Itching is mostly seen in allergic conditions and we see cobble stone appearance in image,
above features are seen vernal keratoconjunctivitis.
Other options:
Bacterial conjunctivitis : no itching is seen
Trachoma: folicule formation is seen
Angular conjunctivitis : seen in chronic malnurised patients.

90. Which of the following is the gold standard measuring IOP?


a) Schoitz
b) Tonopen
c) Pulse air tonometer
d) Goldmann applanation tonometer

Ans. D

Goldmann applanation tonometer is used to measure IOP by touching the


cornea.
91. What is the treatment of choice for recurrent pterygium?
a) Simple excision
b) Observation
c) Excision with conjunctival auto graft
d) Treatment with Mitomycin C

Ans. C
Excision with conjunctival auto graft

92. A lady present with a sudden painful loss of vision which worsens on eye movement. RAPD is
present on
perimetry a central scotoma is revealed. What is the diagnosis?
a) Optic neuritis
b) Papilledema
c) Retinoblastoma
d) CRVO

Ans. A

In optic neutritis (blurred disc margin) In retrobulbar neutritis( clear cut margin)

Other options:
Papilledema –no loss of vision.
Retinoblastoma – causes leukocoria, squint, glaucoma.
CRVO – painless loss of vision.

93. A patient presents with history of trauma with vegetable matter a few days back, with pain and
photophobia.
What is the most likely cause?
a) Herpes simplex keratitis
b) Acanthamoeba keratitis
c) Bacterial keratitis
d) Fungal keratitis
Ans. D

Other options:
Herpes simplex – painless ulcer
Acanthamoeba keratitis – painfull but due to contact lens

94. A child presents with proptosis, pain and loss of vision. O/E the following picture is seen. What
is the most
likely diagnosis?

a) Orbital cellulitis
b) Stye
c) Preseptal cellulitis
d) Blepharitis

Ans. A
Orbital cellulitis— proptosis, chemosis, congestion, pupil dilation and inflammation are seen in image.
Generally orbital cellulitis starts with a simple cold and cough→ ethmoid sinusitis →medial wall of orbit.

95. A boy gets punched in the eye. Which of the following is the most likely occurrence?
a) Subluxation of the lens
b) Inferior orbital wall fracture
c) Medial wall fracture
d) Lateral wall fracture

Ans. B
Blow out fracture-- Inferior wall gets fractured more often because it gets supported by maxillary sinus
which is cavernous.
(Tear- drop sign)
96. A patient with VA ˃1/60, but <3/60 in better eye. What type of blindness does he have?
a) Low vision
b) Economic blindness
c) Social blindness
d) Manifest blindness

Ans. C

97. In which of the following, V-Y procedure is used?


a) Ectropion
b) Entropion
c) Symblepharon
d) Ankyloblepharon

Ans. A

Ectropion

Ectropion Outward turning of lid margin.


Entropion Inward turning of lid margin

Symblepharon Adhesion between palpebral and bulbar


conjunctiva

Ankyloblepharon adhesion of the edges of upper eyelid with the

lower eyelid

98. A patient presents with history of DM of more than 20 years duration and painless loss of
vision with the given
fundus findings. What is the most likely diagnosis?

a) NPDR
b) PDR
c) CRVO
d) CRAO

Ans. A
NPDR
Other options:
PDR— not the answer because no new blood vessels seen in the image.
CRVO—no tomato splash appearance in image.
CRAO—could see ocular spot.
99. A lady presents with ptosis, on eating and sipping of fluids her ptosis decreases. What is the
most likely diagnosis?
a) Mechanical ptosis
b) Horner’s syndrome
c) Complicated ptosis
d) Blepharophimosis syndrome

Ans. C
Complicated ptosis—Marcus Gunn Jaw- winking syndrome: a retraction of the ptotic lid in conjunction
with stimulation of the ipsilateral pterygoid muscle

100. A newborn child presents with watering and photophobia. O/E the following picture is seen.
What is the
diagnosis?

a) Buphthalmos
b) Congenital cataract
c) Megalocornea
d) Congenital dacryocystitis

Ans. A
Other options:
Congenital cataract—absence of water and megalocornea
Megalocornea—no watering
Congenital dacryocystitis—absence of megalocornea

101. A patient presents with complicated cataract and on slit lamp findings are shown in the given
image. What is
the type of cataract seen in this case?
a) Anterior subscapular cataract
b) Nuclear cataract
c) Posterior subscapular cataract
d) Cortical cataract

Ans. C

102. Refractive error found here is:

a) Myopia and astigmatism


b) Hypermetropia
c) Myopia
d) Astigmatisum

Ans. A
Myopia and astigmatisum due to irregular surface.

103. All of the following are true about below procedure, except:

a) It can be done with patient being in any position


b) There is no stereopsis
c) Can be used in cases of hazy ocular media
d) The retinal periphery cannot be examined.

Ans. C
The image shown here is direct ophthalmoscopy.
104. A patient presents with ocular motion defect. The position of the eyeball is shown in the
image below.
Identify the nerve involved in following defect.

a) Occulomotor nerve
b) Optic nerve
c) Trochlear nerve
d) Abducens nerve

Ans. A

105. Person with one month H/o hip pain, true limb shortening and diagnosed as a case of
tuberculosis. Which
stage is it ?
a) Late arthritis
b) early arthritis
c) early synovitis
d) advanced arthritis

Ans. A
 Stage 1 : synovitis stage : hip in same poture ( i.e abduction and external rotation)
 Stage 2 : early arthritis : hip is in adduction and internal rotation
 Stage 3 : late arthritis : joint space distructed ( TRUE LIMB SHORTENING )
 Stage 4 : advance arthritis : acetabulum will move freely

106. X-ray and clinical photo is given, which nerve is most commonly damaged ?

a) Deep peroneal nerve


b) Sciatic nerve
c) Femoral nerve
d) Saphenous nerve

Ans : B
Posterior hip dislocation:
 FADIR flexion,adduction,internal rotation
 Limb shortening
 Sciatic nerve damage
 Lesser trochanter is not seen clearly
Anterior hip dislocation:
 FABERflexion,abduction, external rotation
 Femoral nerve damage
 Lesser trochanter is seen clearly

1)POSTERIOR HIP DISLOCATION 2)ANTERIOR HIP DISLOCATION

107. Hormones and endogenous substances that are involved in regulation of calcium balance in
the body are ?
a) Vitamin D,PTH,Insulin
b) Vitamin D,PTH, Calcitonin
c) Vitamin D,Calcitonin,TSH
d) Vitamin D,PTH,TSH

Ans. B
ca + po4
I. Vit d 
II. PTH 
III. Calcitonin 

108. A patient from bihar present with teeth and skeletal image as shown. most likely cause is ?

a) Osteropetrosis
b) Osteoporosis
c) Osteosclerosis
d) Osteomalacia

Ans. C
 Dental flourosis
 Normal value of flourosisis : 0.5 - 0.8 ppm
 Upper limit : 1- 1.5 ppm
109. A alcoholic patient slept with compression. Around his arm region and had injury to radial
nerve. Most likely
this is which type of injury ?
a) Neuropraxia
b) Neurotmesis
c) Axonotmesis

Ans: A

110. A 40 year old female patient having history of bee allergy came to the emergency room with
complaints of
swelling of face and dyspnea after a bee sting . Which of the following inflammatory mediators
plays a major
role
a) Serotonin
b) Bradykinin
c) Histamine
d) Prostaglandin

Ans. C
111. When stained with Congo red and visualized under the polarising microscopewhich of the
followingsubstance would show apple green birefringence
a) Lipids
b) Amyloid
c) Collagen
d) Calcium

Ans. B
112. 24year oldfemale came with complains ofneurological deficits. Examination reveals
pallorHerblood
picture is given below. What is the most like diagnosis

a) Folate deficiency
b) Vitamin B12 deficiency
c) Thalassemia
d) Iron deficiency anemia

Ans. B
Vitamin B12 deficiencyanaemia occurs when a lack of vitamin B12 causes the body to produce
abnormally large red blood cells that cannot function properly.

113. A 35 year old lady presented with slow growing thyroid swelling for the past 10 months .
Histological
examination was done and the microscopic image of the biopsy had the presence of Orphan Annie
eye nuclei.
What is the patient suffering from
a) Follicular carcinoma of thyroid
b) Papillary carcinoma
c) Anaplastic carcinoma
d) Medullary carcinoma

Ans. B
114. Supravital staining is used for ?
a. Red blood cells
b. Platelets
c. Reticulocytes
d. White blood cells

Ans. C

115. Small or medium sizedsterile vegetations found in either or both sides heart valve commonly
associated with SLE patients is characteristic of which of the following disease?
a) Non bacterial thrombotic endocarditis
b) Rheumatic heart disease
c) Infective endocarditis
d) Libmann Sachs endocarditis
Ans. D
116. A person with hepatic jaundice, deranged liver function tests and presence of a clinical
finding as shown
alongside

a) Huntington chorea
b) Wilson's disease
c) Leigh syndrome
d) Hemochromatosis

Ans. B
117. A teenager presents with features of liver disease, tremor and poor incoordination, mask like
facies, and
psychiatric symptoms. Which of thefollowing is implicated in the pathogenesis of this condition?
a) ATP 7A gene on chromosome 13
b) ATP 7B genechromosome 13
c) ATP 7C gene on chromosome 13
d) ATP 7B gene on chromosome 6

Ans. B

HNF 1-alpha- Hepatic Adenoma


ATP7B- Wilsons disease
Fibrillin- Marfan syndrome
COL1A1- Osteogenesis imperfecta

WILSON DISEASE-ATP 7B genechromosome 13


HEMOCHROMATOSIS-HFE genechromosome 6P
ALPHA1-ANTITRYPSIN DEFICIENCY- SERPINA 1 genechromosome 14
CYSTIC FIBROSIS – CFTR genechromosome 7

118. A child presented with history of pallor with fatigue, petechial bleeding and fever since last 7
days. On
examination he was havingenlargement of liver and spleen with sternal tenderness. Which of the
following
is the likely explanation for the symptoms in the child?
a) Aplastic anemia
b) Chronic myeloid leukemia
c) Acute lymphoblastic leukemia
d) Acute myelogenous leukemia

Ans. C
119. A person is working with 30 year history of working in cardboard manufacturing and
develops
breathlessness and likely to be associated withmottling in lungs is
a) Bysinossis
b) Bagasossis
c) Asbestosis
d) Nasopharyngeal carcinoma

Ans. B

Bagassosis is occupational disease of lung caused by inhalation of Bagasse or sugarcain dust

 First reported in India in cardboard manufacturing firm near Kolkatta


 Asbestosis: Cement, roof tiling
 Byssisinosis: Cotton industries
Agent Disease Typical Exposure

Coal Dust Anthracosis Coal miners

Asbestos Asbestosis Mining, installation of insulation

Silica Silicosis Sand blasting, Stone cutting

Bagasse Bagassosis Cardboard, paper and sugar


industry

Cotton, hemp Byssinosis Textile manufacturing

120. A tall male teenager presented with gynecomastia . On examination,absence of secondaryhe


has average
cognition characteristics a deep voice, beard, and male distribution of pubic hair. His genetic
analysis was
performed and he had the presence of abarrbodv. What is the likely diagnosis of this condition?
a) Turner syndrome
b) Klinefelter syndrome
c) Swyer syndrome
d) Down syndrome

Ans. B
Klinefelter-xxy-and-turner-xo-syndrome
121. A 16 year old boy had abdominal discomfort on the left side andhe went to his physician. His
father had a
similar condition during his teenage and later developed some GI malignancy. He then underwent
a
colonoscopy which is shown alongside. What doyou conclude about his condition?

a) Familial adenomatous polyposis


b) Cancer colon
c) Ulcerative colitis
d) Lynch syndrome

Ans. A
FAP
 ˃100 polyps are seen.
 Autosomal dominant
 Mutation in APC gene (5q21)

122. A 30 year male patient has a complaint of a painless testicular mass. His blood sample
revealed non elevated
alpha fetoprotein levels. Which of the following is the most likely cause of his testicular mass?
a) Yolk sac tumor
b) Classical seminoma
c) Teratoma
d) Choriocarcinoma
Ans. B

123. An elderly male has been working in the dye industry for 40years.He develops
somefrequency and urgency,
Bladder symptoms like hematuria. What is the likely subtype of the conditionpresent in him?
a) Transitional cell cancer
b) Adenocarcinoma
c) Squamous cell cancer
d) Small cell cancer

Ans. A

124. Which of the following is the likely diagnosis in a lady with extremely low TSH and high T3
and T4?
a) Subclinical hypothyroidism
b) Subclinical hyperthyroidism
c) Graves disease
d) Reidel thyroiditis
Ans. C

125. A middle aged lady presents with weight gain, constipation, hoarseness of voice and lethargy.
On
examination, she is found to be having anti TPO antibodies. What is the likely diagnosis in her?
a) Reidel's thyroiditis
b) Graves' disease
c) Hashimoto thyroiditis
d) De quervain thyroiditis

Ans. C

126. A lady has been diagnosed with breast cancer. Her biopsy report revealing the presence of
tumor cells with
signet ring appearance, mucin deposition and "Indian file pattern" is most likely suggestive of
a) Invasive lobular carcinoma
b) Invasive ductal carcinoma
c) Medullary cancer
d) Mucinous

Ans. A
127. Histological examination of a bone tumor is as follows . What is thelikely diagnosis?

a) Osteosarcoma
b) Giant cell tumor
c) Ewings sarcoma
d) Chrondrosarcoma

Ans. B

Giant cell tumor


 The typical age group 20-40years.
 Morecommon in females
 Usually occurs at epiphysis
 Most common location is distal femur>proximaltibia
 Usually solitary
 Malignant transformation seen in less than 5% cases.

128. Which of the following anti-diabetic drug does not require dose reduction in renal failure?
a) Linagliptin
b) Sitagliptin
c) Vildagliptin
d) Saxagliptin

Ans. A
 Normally liptins are avoided in renal failure except: linagliptin is the only drug safe in renal failure
 Liptins are dipeptidyl peptidase-4 inhibitorthey inhibit breakdown of GLPGLPrelease
insulinblood sugar
129. Which of the following drugs used in emergency management of asthma, which doesnot act by
causing
bronchodilation?
a) Salbutamol
b) Ipratropium
c) Deriphylline
d) Hydrocortisone

Ans. D
 There are 3 major types of bronchodilators:
I. Beta2 agonists [eg:salbutamol,salmetrol]
II. ACE inhibitors [eg:ipratropium,thiotropium]
III. PDE inhibitors [eg:theophylline,deriphylline,aminophylline]

130. Which of the following is the action of dopamine at low doses ?


a) It increases renal blood flow
b) It causes vasoconstriction
c) It has positive chronotropic action
d) It causes hypotension

Ans. A
 Low dose: <1-2μg/kg/min
 At low dose dopamine stimulates only D1 receptors
 D1 receptors are present on renal blood vessels
 So when dopamine stimulate D1 receptors  renal vasodilation  increase in renal blood
flow
 Intermideatedose : 2-10μg/kg/min
 High dose :>10μg/kg/min

131. A female patient presented with pain and redness in great toe.serum uric acid level is
9.6mg/dl. A part from
prescribing analgesics for relieving pain , the physician prescribed a drug which can decrease the
formation
of uric acid. Which of the following enzyme is likely to be inhibited by this drug ?
a) Xanthine oxidase
b) Thymidylate synthase
c) Phosphpribosyl transferase
d) Dhfr

Ans. A
 Uric acid formed from purines  which are metabolized to generate xanthine  and convert
xanthine into uricacid by xanthineoxidase

132. Vitamin that increases the absorption of iron from stomach is ?


a) Vitamin A
b) Vitamin B12
c) Vitamin C
d) Vitamin D

Ans. C
 2 major substances help in increase in absorption of iron :
I. Hydrochloric acid
II. Vitc/ascorbic acid
 2 substances which decrease absorption of iron:
I. Phytates
II. Oxalates
III. Tannates

133. longest acting phosphodiesterase inhibitor among the following drugs is ?


a) Sildenafil
b) Vardenafil
c) Tadalafil
d) Phentolamine

Ans. C
PDE inhibitor
USES:
 Erectile dysfunction
 Pulmonary HTN

134. Which of the following statements about these drugs is true?

a) Drug D is most potent


b) Drug A and B have equal efficacy but B is more potent than drug B
c) Drug C is least potent
d) Drug B has more potency but less efficacy than drug a

Ans. B
 Potency: It defines left and right side of the curve
 more the drug is on left side more potent the drug
 more the drug is on right side less the potent the drug
 Efficacy: It defines by height of the curve
 Higher the curve more efficacy
 Lower the curve less efficacy

135. Which of the following antimicrobials is contraindicated in a patient with seizure disorder?
a) Ampicillin
b) Ofloxacin
c) Moxifloxacin
d) Cefixime

Ans. B
 2 major groups of anti-microbials contraindicated in seizure disorders:
I. Flouroquinolol [floxacindrugs] - ofloxacin > moxifloxacin
II. Carbapenems [imipenem drugs]

136. Which of the following drug is preffered for closure of ductus arteriosus in a preterm baby
with PDA ?
a) Indomethacin
b) Ibuprofen
c) Mefenamic acid
d) Paracetamol

Ans. B
 NSAIDS are used to close ductus arteriosus
 NSAIDS are: ibuprofen[best],indomethacin,aspirin

137. A patient on long term digitalis therapy develops arrythmia. What will be the next step in the
management
a) Adenosine
b) Amiodarone
c) Atropine
d) Lignocaine

Ans. D
 Lidocaine: drug of choice for ventricular arrhythmias
 Atropine: drug of choice for atrial arrhythmias
 Amidarone: it causes toxicity
 Digoxin toxicity can emerge during long-term therapy as well as after an overdose. It can occur even
when the serum digoxin concentration is within the therapeutic range.Toxicity causes anorexia,
nausea, vomiting and neurological symptoms. It can also trigger fatal arrhythmias.
"Digitalis" arrhythmias were treated with beta-adrenergic blockers: Inderal, Viskene, Eraldin, Trasicor
and Aptin. These drugs proved effective in most cases with atrial arrhythmias and in some--with
ventricular arrhythmias. Lidocain was more effective in cases of ventricular arrhythmias. Effective
drugs of a broad spectrum are also Aimalin, Pulsenorma and Ritmodan.

138. A patient on treatment of myasthenia gravis presents with increasing muscle


weakness.which of the
following drug is used to differentiate myasthenia gravis from cholinergic crisis ?
a) Edrophonium
b) Pilocarpine
c) Atropine
d) Pyridostigmine

Ans. A
Explanation:
 Edrophonium is ACHesterase inhibitor
 It act only for 10min
 By giving edrophonium if the condition improves then it diagnose as Myasthenia gravis
 By edrophonim if the condition worsens then it diagnose as cholinergic crisis
139. Which of the following drugs can be used for the treatment of undescended testes?
a) Testosterone
b) GnRH
c) Cold Water Compress
d) Anti -MIH Therapy

Ans. B
 Pulsatile GnRH analogues [leuprolide] testosterone  help in movement of testis to scrotum

140. An elderly male developed the symptoms of dementia and was diagnosed as Alzhiemer’s
disease. Which
neurotransmitter level is reduced in this disease ?
a) Dopamine
b) Acetylcholine
c) Adrenaline
d) Serotonin

Ans. B
 Alzhiemer’s dementia occurs in basal nucleus of MEYNERT in brain
 MEYNERT function: is to maintain memory, this area contains plenty of Ach
 So if there is any problem in MEYNERT nucleus then Ach level will decrease

141. Which of the following combination of seizure and its first line drug is not correct ?
a) Generalised seizure – Valproate
b) Myoclonic- Topiramate
c) Focal- Levetiracetam
d) Absence- Ethosuximide

Ans. B
 Myoclonic -valproate

142. Orthotoulidine test is used to measure?


a) Free chlorine & combined chlorine
b) Free chlorine / Residual chlorine
c) Chlorine demand
d) Combined chlorine

Ans. D
Measure:- Total chlorine – free chlorine A/M  combine chlorine
O T test: Measures free chlorine (within 10 seconds) and free + combined chlorine (15-20 mint) in water.

143. Low stationary phase of demographic cycle?


a) Phase 1
b) Phase 2
c) Phase 3
d) Phase 4

Ans. D
Stage 1: (High stationary):- High BR
High DR

Stage 2: (Early expanding): Birth Rate Unchange


Death Rate decrease

Stage 3: (late Expnading): BR: decreases


DR: decreases

Stage 4: (late stationary): BR: low


DR: low

Stage 5: (Declining): BR < DR


Population decreases

Minimum demographic gap: stage 1 & stage 4


Maximum demographic gap: late part of stage 2
Negative D. graph in stage 5
India is in: stage 3 (late expanding stage)

144. A health worker goes house to house for every 15 days taking fever history and preparing
malarial slide.
Which form of surveillance is this?
a) Active
b) Passive
c) Sentinel
d) Any of the above

Ans. A
Malarial surveillance was done by MPW (Male)Subcenter level
 Passive: Hospital reporting  Most convenient way of sample
 Sentinel: Done for air & water quality

145. Two vaccines MB and Pentavalent used for immunization round given for the worker which
was open. Which
of the following is vaccine is discarded?
a) MR discard, USE pentavalent
b) Pentavalent
c) Both MR & Pentavalent
d) MR can be used, Pentavalent was discarded

Ans. A
VACCINES WHICH WILL NOT FOLLOW OPEN VIAL POLICY
 BCG
 MEASLES
 MR
 JE
 YELLOW FEVER

146. Recent transmission of Malaria in community is measured by?


a) Annual Parasite incidence
b) Infant Parasite rate
c) Speen rate
d) Slide positivity rate

Ans. B
IPR: Most sensitive index for recent transmission of Malaria

Annual parasite index:


comformed cases during 1 yearx 1000
Population under survelliance

Measures:- Burden of Malaria


Slide positivity rate: % of slides found positive for malaria parasite irrespective of the type of species

147. According to Bio medical wastage compare. Which Bag of Bio medical wastage can be
incinerated?
a) Red
b) Blue
c) Yellow
d) White

Ans. C
Yellow: Incinevates
Red
Blue Disinfected and later discarded
White

148. Given mosquito causes which disease ?

a) JE
b) Dengue
c) Zika virus
d) Malaria

Ans. A
GIVEN IMAGE :.CULEX
Causes:Failarisis
JE

.ANOPHELES

Causes:
Maliaria

.ADES
Causes :
Dengue ,Riftvally fever, Yellow fever, Chicken gunia

149. The average number children that would be born to a woman over her reproductive life span
refers to?
a) Total fertility rate
b) Gross reproduction rate
c) Net reproduction rate
d) General fertility rate

Ans. A
TFR:- Give magnitude of complete family size
GRR: Measures the no. of daughters a woman would have in her life time
NRR: No. of daughters a newborn girl will bear during her life time assuming fixed age specific fertility
and mortality rate
NRR: 1
To active NRR=1, CPR> 60%
GFR:- No. of CB for 1000 people who are in reproductive age group

150. A pregnant lady was bit by a rabid dog, what should be the next step?
a) Treat the wound locally
b) Give vaccine immediately
c) Local treatment of wound + immunoglobulin + vaccine
d) Local treatment of wound + vaccination

Ans. D

151. Under Ayushman Bharat what is the insurance package provided?


a) Rs. 1 lk/family/year
b) Rs. 5lk/family/year
c) Rs. 3 lk/family/year
d) Rs. 2lk/family/year

Ans. B
Image:- PM-JAY
Full form :Pradhan mantra janarogyayojana

152. School health is a responsibility by which level of healthin RURAL areas?


a) Sub center
b) PHC
c) CHC
d) Sub district hospital

Ans. B
Population covered by PHC
Plain :30000
Hilly: 20000
Population covered by Sub center
Plain :5000
Hilly: 3000
Population covered by CHC
Plain :120000
Hilly: 80000
CHC is the first referral level

153. All are principals of Primary Health care EXCEPT?


a) Appropriate technologies
b) Community participation
c) Inter sectoral coordination
d) Universal Health coverage

Ans. D
4th principle :Equity of distribution

154. Survey of 4 cities. A, B, C, D reported IFR as 2.1, 2.4, 2.6, 2.8. which city had achieved desire
goal of TFR?
a) 2.1 (A)
b) 2.4 (B)
c) 2.6 (C)
d) 2.7 (D)

Ans. A
TOTAL FERTILITY RATE :- Single most important factor in population growth

155. Which vaccine follows open vial policy?


a) BCG
b) Pentavalent
c) JE
d) MMR

Ans. B
Open vial vaccines can be used for 4 weeks
Vaccines follows open vial policy are:
.PENTAVALENT .OPV
. HEP B . IPV
. DPT . Td
156. Which vaccine has seroconversion time less than incubation period for that disease?
a) Flu
b) Measle
c) Rubella
d) Mumps

Ans. B
 Flu:- I.P:- 7 days after vaccine AB develops > 7 days
 Measles: I.P:-10-14 days:- after exposure If we give vaccines Ab will develops within 7 days
 Rubella:- I.P:- 14-21 days: AB take time: 2-4 weeks

Measles vaccine:
a) Pre exposure:
 Live attenustaved
 Strain: Edmostron ZAEREB
 Powered: We use: distilled water
 0.5 ml s/c to Rt shoulder
 2 dose 1) = 9-12 months
2) = 1 1/2 year (16-24 months)
b) Pos Exposures
1) Measles vaccine
Within 3 days of exposure
2) Ig (Immuno compromised child)
0.25 mg/kg
IM

157. A person reported to ophthalmic OPD with vision of <3/6 in RT eye and finger movement at 1
meter distance
in LT eye. His state of vision is?
a) Severe visual impairment
b) Economical Blindness
c) Social Blindness
d) Manifest Blindness

Ans. C
Normal: 6/6 – 6/18

Low vision: < 6/18 – 6/60


Economic Blindness:-< 6/60 – 3/60

Social Blindness: <3/60 – 1/60

158. A man travelled to Assam and after returning back he developed fever, giddiness, and on
investigation
confirmed to be a case of PI falciparum, Tx of choice?
a) Doxycline
b) Chloroquine
c) I/V artemether
d) Artesunale, Salfadoxine, Pyridethamine

Ans. C
P. vivax:-Chloroquin in all trimesless
No primaquive

P. falciparum: Quinine  1sttrment ACT( artemisinin combination therapy)


NE state: ACT-L
Otur: ACT – SP 2nd& 3rdtrimes
No primaquin

159. A worker in the wheat section of a food facility eats peanutsregularly and develops hepatic
carcinoma after
many years.Which of the following is implicated in the pathogenesis of his condition?
a) BOAA
b) Aflotoxin
c) Pyrolizidine
d) Sanguanarine

Ans. B

BOAA:-Latherysm
Aflatoxin:- Aspergillus flavours fungus : causes :hepatocellular carcinoma
Pyrocizidine:- Endemic asitis
SANGUANARINE:- Epidemic drop (mustard oil+ argenomonc oil)

160. In a hospital the management wants to know that how much time that each doctor is
spending with patient
on an average. They want to know increase or decrease in the number of doctors as per
requirement, to
Improve the quality of care. Which management technique should be used?
a) FGD
b) Systemic Analysis
c) Network Analysis
d) Work Sampling

Ans. D

161. A case control study was done in a group of people who are ordering food and who are
preparing food to see
in which group chances of Obesity is more what sample should the investigator should take?
a) Person with or without ordering food
b) Person with or without exerces
c) Person with and without ordering food
d) Person without ordering food
Ans. C

162. Demographic Bonus is?


a) Population density
b) Reduction in population in last few years due to contraception
c) Fertility rate
d) Literacy

Ans. B

163. Reproductive health promoting agencies in world is?


a) UNDP
b) UNICEF
c) UNFPA
d) UNESCO

Ans. C

UNFPA: United nation fund


Year of formation: 1967
HEADQUATORS: NEW YORK ,USA

164. Which agency gives support to active sustandable development goals?


a) WHO
b) UN
c) UNESCO
d) UNDP

Ans. D

UNDP: UNITED NATIONS DEVELOPMENT PROGRAM


Year of Formation :1965
Head quators; NEW YORK.USE

165. Which agency is funding for TB program in India?


a) SIDA
b) WHO
c) WORLD BANK
d) GLOBAL FUND

Ans. A

166. Study the graph given below of different occupational group’s. Analyse and mark who has the
highest chance
of developing fever?

a) Industrial Worker
b) White collar Worker
c) Farmer
d) Housewives

Ans.

167. A person with depression prescribed a serotonin and norepinephrine reuptake inhibitor.
Identify the drug?
a) Fluoxeitine
b) Venlafaxine
c) Sertraline
d) All of the above

Ans. B
Serotonin nor-epinephrine reuptake inhibitors:
 Venlafaxine
 Desvenlafaxine
 Duloxetine
Selective serotonin reuptake inhibitor:
 Escitalopram
 Paroxetine
 Fluoxetine

168. Earliest symptoms of alcohol withdrawal is:


a) Tremors
b) Hallucinations
c) Seizures
d) Delirium

Ans. A
Alcohol withdrawal
Symptoms
 After 6-8 hours: coarse tremors (m/c), nausea, vomiting, anxiety.
 After 12- 24 hours: alcoholic hallucinations (auditory).
 After 24-48 hours: alcohol withdrawal seizures (GTCS)—comes in clusters.
 After 48-72 hours: delirium tremens—(lillputian hallucination-patient sees everything small)

169. A chronic alcoholic patient develops delirium tremens after stopping alcohol. Patient should
be treated with?
a) I/V methycobalamine
b) I/V thiamine
c) I/V thiamine + lorazepam
d) I/V methylcobalamine + lorazepam

Ans. C

170. Not a symptom of panic attack:


a) Suicide attempt
b) Sudden anxiety
c) Episodic anxiety
d) Feeling of impending doom

Ans. A
Panic attack →sudden, discrete period of intense fear or discomfort that are accompanied by significant
physical and cognitive symptoms.
C/F: palpitations, chest discomfort, dizziness,discomfort, sweating, trembling, shaking.
Cognitive symptoms : disorganized thinking, irrational fears, depersonalization, decreased ability to
communicate.
Fear of death: mimics heart attack
Peak at 10 minutes, lasts upto 30 minutes.

171. A 32 years old women presented with symptoms of mild depression first time. What should
be the initial
treatment?
a) Cognitive behaviourtherapy(CBT)
b) Antidepressant
c) CBT+ antidepressant
d) Electro convulsive therapy

Ans. A
Treatment of depression:
Mild depression
For 1st time:
 CBT- Multiple sessions maybe needed.
 Exercise-early morning
 Active monitoring
If not getting better start anti-depressants
Antidepressents:
SSRI’S, SNRI’S, TCA’S, MAOI’S, Atypical antidepressant (choosen on the basis of side effects).

172. A patient suffering from RTA 6 hrs back he was brought to the emergency with altered
sensorium. his
Glasgow coma score was 12. CT scan of head done which showed the following. he is most likely
suffering
from –
a) Sub dural hematoma
b) Epidural hematoma
c) Subarachnoid hemorrhage
d) Intracerebral hematoma
Ans. B
Explanation: EPIDURAL HEMATOMA

 Location: lies between skull and outer endosteal layer of duramater


 Vessels involves:middle meningeal artery
Anterior ethmoidal artery
Transverse or sigmoid sinus
Superior sagittal sinus
 Presentation:
Symptoms develop soon after injury Lucid interval in 20% of cases
 CT appearance: Biconvex lens

173. Barium swallow image demonstrates


a) Achalasia cardia
b) Zenker’s diverticulum
c) Corkscrew esophagus
d) Cancer esophagus

Ans. A
ACHALASIA CARDIA

 Rat-tail appearance/bird-beak tapering


 Dilated sigmoid esophagus
174. What is the probable cause of the condition shown in the given X-ray?

a) Pleural effusion
b) Perforation peritonitis
c) Sigmoid volvulus
d) Intestinal obstruction

Ans. B
PERFORATION PERITONITIS
 Air under diaphragmpneumoperitoneum

175. Following image is a suggestive of

a) Congenital diaphragmatic hernia


b) Pneumothorax
c) Cardiac tamponade
d) Tracheo esophageal fistula

Ans. A
Congenital diaphragmatic hernia
 Depending on the location and size of the defect retroperitoneal or intra-abdominal organs and
tissues can prolapse intothoraciccavity due to negative intra-thoracic pressure
176. Barium enema image demonstrates

a) Meckel's diverticulum
b) Diverticulosis of colon
c) Sigmoidvulvulus
d) Colon cancer

Ans. B
Explanation: DIVERTICULOSIS
 SAW-TOOTH APPEARANCE
 Description: serrated/concertina-like appearance
 Mc site: sigmoid colon
 Seen in: Diverticulitis

177. A chronic smoker presented to the emergency . His X-ray chest was done which showed the
following.
Possible diagnosis in this patient
a) Pneumothorax
b) Tension pneumothorax
c) Hemothorax
d) Hydropneumothorax

Ans. B
TENSION PNEUMOTHORAX

 Bronchovascular margins are absent


 Lung collapse towards hilum
 Heart shifted towards right side
178. Alcoholic male presented with pain in epigastrium radiating to the back.what is the likely
diagnosis in the
given CT image.

a) Amoebic liver abscess


b) Pancreatic pseudocyst
c) Acute cholecystitis
d) Hydatid cyst

Ans. B
PANCREATIC PSEUDOCYST
 Duration:>4weeks
 Fibrous wall is seen
 Homogenous collection is seen

179. T1 and T2 images in MRI

a) Caudate
b) Thalamus
c) Lentiform nucleus
d) Insular cortex
Ans.
 T2 is the time taken for hydrogen atoms to diphase
 T1 is the time taken for return to relaxed state ( and release energy)
T1 Image T2 Image
Time to echo (TE) 40 ms 120 ms
Time to repetition (TR) 400 ms 1000 ms
Useful for Demonstrating anatomy Demonstrating pathology
Bright for Fat, methaemoglobin, contrast, Fluid (oedema), fat
calcium, blood

180. What is most responsive to radiotherapy?


a) Mucosa
b) Bone
c) Nerves
d) Muscle

Ans. A

181. A 65 year old female presented with imbalance while standing. On examination there is loss of
vibration and
proprioception. What will be next line of treatment
a) Iron
b) Vitamin B12
c) Niacin
d) Folic acid

Ans. B
Vitamin B12deficiency symptoms include:
 strange sensations, numbness, or tingling in the hands, legs, or feet.
 difficulty walking (staggering, balance problems)
 anemia.
 a swollen, inflamed tongue.
 difficulty thinking and reasoning (cognitive difficulties), or memory loss.
 weakness.
 fatigue

182. A patient presented with complaints of fatigue, dyspnea on exertion.Lab tests shows
macrocytic RBCS. Serum
methylcobalamin level is within normal limits.What will be next line of treatment
a) Iron
b) Folic acid
c) Vitamin B12
d) Pyridoxine

Ans. B
The symptoms of anemia that occur due to folate deficiency include:
 Persistent fatigue.
 Weakness.
 Lethargy.
 Pale skin.

 Shortness of breath.
 Irritability.
183. Which of the following presentations has worst prognosis
a) Paralysis
b) Parasthesia
c) Pallor
d) Pulselessness

Ans. D
Pulseless electrical activity (PEA), a cardiac arrest rhythm scenario with an associated poor prognosis, is
defined as cardiac electrical activity without a palpable pulse.

184. Central venous pressure is increased in which type of shock


a) Hypovolumic
b) Septic
c) Neurogenic
d) Cardiogenic

Ans. D
Central venous pressure (CVP) has a normal range of 5–7 mm Hg in an adult spontaneously breathing
patient while supine. The CVP is elevated in obstructive or cardiogenic shock, while it is decreased in
septic, neurogenicand hypovolemic shock.
185. A patient diagnosed to have empyema. Which of the following is the best treatment
a) Antibiotics only
b) Thoracotomy + antibotics
c) Pleurodesis + antibiotics
d) Intercostal chest tube drainage + antibiotics

Ans. D
Empyema is a collection of pus in the space between the lung and the inner surface of the chest wall
(pleural space).
Treatment is aimed at removing the pus and fluid from the pleura and treating the infection. Antibiotics
are used to treat the underlying infection. The specific type of antibiotic depends on what type of bacteria
is causing the infection. The method used to drain the pus depends on the stage of the empyema.

186. A female presented with symptoms of lethargy, weight gain, constipation, hair loss, dry skin
and intolerance
tocold. What is the most likely diagnosis
a) Hypothyroidisim
b) Hyperthyroidism
c) Hypoparathyroidism
d) Hyperparathyroidism

Ans. A

187. An elderly male presented with dyskinesia and rigidity. There was some pathology in
substantia nigra.Which
neurotransmitter is most likely involved
a) Acetylcholine
b) Serotonin
c) Glycine
d) Dopamine
Ans. D
The substantia nigra (SN) is an area of deeply pigmented cells in the midbrain that regulates movement
and coordination. ... Neurons of the SNc produce Dopamine, which stimulates movement. In contrast,
GABAergic neurons of the SNr can stimulate or inhibit movement depending on the input signal.

188. Elderly male with low backache and sclerotic lesions in vertebra is most likely having which
condition
a) Ca prostate
b) Multiple myeloma
c) TB of vertebra
d) Waldenstrom macroglobulinemia

Ans. A
Osteoblastic lesions are asso- ciated with increased bone formation and appear sclerotic.
Bone lesions resulting from prostate cancer are primarily osteoblastic, but are also associated with
increased bone resorption

189. A patient has serum K of 3.2 meq / L. What will be next line of treatment
a) IV KCl infusion
b) Oral KCI
c) IV Calcium gluconate
d) IV Insulin

Ans. B
Potassium supplements are generally prescribed for low potassium levels. If the situation is
severe, potassium might be given as an intravenous (IV) solution.

Category & Clinical presentation


State Serum K level Clinical presentation
Mild 3-3.5 mEq/L Asymptomatic
Moderate 2.5-3 mEq/L Cramping, weakness, malaise,
and myalgias
Severe < 2.5 mEq/L ECG changes arrhythmias
ECG charges: ST-segment depression of flattering. T-wave inversion, and U-Wave elevation

190. A young female with malar rash and fever.On Echocardiography there are vegetations on both
sides of mitral
valve. What is the likely diagnosis
a) A.RHD
b) B. Infective endocarditis
c) C. Non bacterial thrombotic endocarditis
d) Libman Sacks endocarditis

Ans. D
191. A patient had diarrhea associated with blood in stools. Physician diagnosed shigella as the
etiological agent of diarrhea. Which clinical feature supported shigella over cholera
a) Abdominal pain
b) Fever
c) Vomiting
d) Stool frequency

Ans. B
Symptoms Cholera = Shigella =
acute watery diarrhoea acute bloody diarrhoea
Stool > 3 loose stools per day. Watery > 3 loose stools per day, with
like reice water blood or pus
Fever No Yes
Abdominal cramps Yes Yes
Vomiting Yes a lot No
Rectal pain No Yes

192. A young boy was found to have wing beating tremors.Which is the most likely diagnosis
a) Sydenham Chorea
b) Wilson disease
c) Huntington chorea
d) Neurodegenative disease

Ans. B
Wing-beating tremor is a coarse, irregular tremor that is most prominent when the limbs are held
outstretched, reminiscent of a bird's flapping its wings; due to up and down excursion of arm at abducted
shoulder. Seen mainly with Wilson disease.

193. What deformity is present as shown in the figure


a) Boutonniere deformity
b) Swan neck deformity
c) Mallet finger
d) Z deformity
Ans. B

194. Which of the following agent doesn't cause bronchodilatation still used in emergency
management of bronchial asthma
a) Salbutamol
b) lpratropium
c) MgSO4
d) Hydrocortisone

Ans. D
Corticosteroids reduce the mucus secretion by inhibiting the release of secretagogue from macrophages.
Corticosteroids inhibit the late phase reaction by inhibiting the inflammatory response and interfering with
chemotaxis. This action may be due to the inhibition of LTB4 release.

195. Corticospinal pathway crosses to opposite side at what level


a) Midbrain
b) Pons
c) Upper medulla
d) Lower medulla

Ans. D
The corticospinal tract, along with the corticobulbar tract, form two pyramids on either side of the
medulla of the brainstem—and give their name as pyramidal tracts. ... The lateral corticospinal
tract neurons cross the midline at the level of the lower medulla of medulla oblongata, and controls the
limbs and digits.

196. A young patient brought to the ED after road traffic accident. CT head shows evidence of raised
intracranial
pressure. What will be the next line of management
a) Mannitol
b) Furosemide
c) Acetazolamide
d) Glycerol

Ans. A
197. A 30 year old presented with vomiting and pain epigastrium which is radiating to back.On
examination the
findings asshown in the figure are noted.What is the diagnosis

a) Acute pancreatitis
b) Acute cholecystitis
c) Acute hepatitis
d) Ectopic ruptured pregnancy

Ans. A
Cullen's sign is superficial edema and bruising in the subcutaneous fatty tissue around the
umbilicus . It is named for gynecologist Thomas Stephen Cullen (1869–1953), who first described the
sign in ruptured ectopic pregnancy in 1916. This sign takes 24–48 hours to appear and can predict acute
pancreatitis, with mortality rising from 8–10% to 40%

Symptoms of acute pancreatitis


 Fever.
 Higher heart rate.
 Nausea and vomiting.
 Swollen and tender belly.
 Pain in the upper part of your belly that goes into your back. Eating may make it worse, especially
foods high in fat.

198. A patient with deep X descent and rapid Y descent in JVP is most likely having
a) Constrictive pericarditis
b) Tamponade
c) Acute pericarditis
d) D.TR

Ans. A

199. A middle aged male presented with fatigue and weakness more in evening.His symptoms
improved after
administeringedrophonium.What is the most likely diagnosis
a) ALS
b) LES
c) Myasthenia Gravis
d) Botulism

Ans. C
The Tensilon test uses the drug Tensilon (edrophonium) to help you to diagnose myasthenia gravis.
Tensilon prevents the breakdown of the chemical acetylcholine, a neurotransmitter that nerve cells release
to stimulate your muscles.

200. A patient while being operated for a thoracic surgery have a sudden cardiovascular
collapse.There are
decreasedbreath sounds on right side.What is the likely cau
a) Tension pneumothorax
b) ARDS
c) Cardiac tamponade
d) Atelechesis

Ans. A
Tension pneumothorax is a major life‐threatening condition that must be recognized by clinical findings
and immediately treated. Air from a leak in the thoracic wall or the lung becomes trapped in the pleural
space, decreasing venous return to the heart and collapsing the opposite lung.
201. A male patient presented to ED with plasma glucose of 350 mg / dl and positive urine ketones.
Serum K 3.8
mmol / L,serum na- 130 mmol / L. Patient is dehydrated. You start normal saline. What will be
added in normal
saline
a) Glucose
b) NAHCO3
c) Insulin
d) KCI

Ans. C
Diabetic ketoacidosis is a serious complication of diabetes that occurs when your body produces high
levels of blood acids called ketones. The condition develops when your body can't produce enough insulin

202. A patient presented with these eye examination findings. He has behavior abnormalities and
poor hand
writing. Whichgene mutation is responsible for this presentation

a) ATP 7A gene on chromosome 13


b) ATP 7B gene chromosome 13
c) ATP 7C gene on chromosome 13
d) ATP 7B gene on chromosome 6

Ans. B

Wilson disease Symptoms:


 Fatigue, lack of appetite or abdominal pain.
 A yellowing of the skin and the whites of the eye (jaundice)
 Golden-brown eye discoloration (Kayser-Fleischer rings)
 Fluid buildup in the legs or abdomen.
 Problems with speech, swallowing or physical coordination.
 Uncontrolled movements or muscle stiffness.

203. The definition of steroid resistant nephrotic syndrome is absence of remission even after
taking steroids for
a) 4 weeks
b) 6 weeks
c) 8 weeks
d) 12 weeks

Ans. A
Steroid-resistant nephrotic syndrome (SRNS) is defined as NS resistant to steroid therapy, defined by
the absence of complete remission after four weeks of daily prednisone therapy at a dose of 60 mg/m2 per
day
204. Which of the following is true about severe vitamin D deficiency
a) Low calcium, high PTH
b) Low calcium, Low PTH
c) Normal calcium, High PTH
d) High calcium, high PTH

Ans. A
When vitamin D level is low, the absorption of calcium in the intestines becomes less, which then causes
the level of calcium in the blood to go down. As a consequence the parathyroid glands become more
active and produce more PTH that causes calcium to come out of the bones, therefore weakening the
bones.

205. A 5 year old boy has hyperflexible fingers and can touch back of hand with wrist extended.
What is the most
likelydiagnosis
a) Marfans syndrome
b) Ehler Danlos syndrome
c) Alport syndrome
d) Homocystinuria

Ans. B
Ehlers-Danlos syndrome is a group of inherited disorders that affect your connective tissues — primarily
your skin, joints and blood vessel walls. Connective tissue is a complex mixture of proteins and other
substances that provide strength and elasticity to the underlying structures in your body.

206. A 22 year old male presents with history of recurrent fall and difficulty in ambulation.On
examination
atrophied legmuscles are observed, Elder brother has same complaints.What is the diagnosis

a) Duchhene muscle dystrophy


b) Becker's muscle dystrophy
c) Charcot Marie Tooth disease
d) Myotonic dystrophy

Ans. C
A group of hereditary disorders that damage the nerves in the arms and legs.
Charcot-Marie-Tooth is a degenerative nerve disease that usually appears in adolescence or early
adulthood.
Muscle weakness, decreased muscle size, decreased sensation, hammer toes and high arches are symptoms.
The main treatments are physiotherapy and occupational therapy. Medication may reduce pain.
Muscular: abnormality walking, flaccid muscles, instability, lack of coordination, muscle weakness,
rhythmic muscle contractions, or loss of muscle
Foot: hammer toe, high foot arches, constantly walking on tip toe, or difficulty raising the foot
Sensory: pins and needles or reduced sensation of touch
Also common: hand contractures, slow reflexes, or tremor

207. A young female presents in her first trimester of pregnancy. She is taking warfarin for
prosthetic heart
valve. What should be next appropriate step
a) Continue warfarin
b) Add vitamin K
c) Stop warfarin and switch to heparin
d) MTP

Ans. C
Warfarin therapy should be avoided during pregnancy. If warfarintherapy is essential, it should be
avoided at least during the first trimester (because of teratogenicity) and from about 2 to 4 weeks before
delivery to reduce risk of hemorrhagic complications.

Upto 12 weeks - LMWH


12- 36 weeks - Warfarin
 36 weeks - LMWH

208. What is the arthropathy in diabetes with given x ray of ankle known as

a) Osteomyelitis
b) Charcoat arthropathy
c) Gout
d) Osteoarthritis

Ans. B
Charcot foot, also called Charcot arthropathy, is a disease that attacks the bones, joints, and soft tissue in
your feet. When it starts, you may not realize something's wrong. But eventually, it can cause painful sores
or change the shape of your foot

209. What is the acid base abnormality in patient presented with multiple episodes of vomiting
a) Metabolic acidosis
b) Metabolic alkalosis
c) Respiratory alkalosis
d) Respiratory acidosis

Ans. B
Vomiting or nasogastric (NG) suction generates metabolic alkalosis by the loss of gastric secretions,
which are rich in hydrochloric acid (HCl). Whenever a hydrogen ion is excreted, a bicarbonate ion is gained
in the extracellular space.

210. An arterial blood gas is performed and reveals-pH-7.2, paCO2-68 mmHg, and
serum HCO3- 26 mmol / L,
Diagnosis?
a) Respiratory acidosis partially compensated
b) Respiratory acidosis fully compensated
c) Metabolic acidosis with respiratory acidosis
d) Respiratory acidosis uncompensated

Ans. D
211. What is the GCS in a patient with eye opening to pain, inappropriate words and limb withdrawl
is
a) E2, M4,V3
b) E2, M3.V4
c) E2, V2,M4
d) E1, V3, M3

Ans. A

212. A female presents with chronic diarrhea, fatigue and dyspnea on exertion. On examination the
findings are
shown inthe figure. Lab test shows low Hb. What will be the appropriate treatment
a) Doxycycline
b) Folic acid
c) High protein diet
d) Iron

Ans. D
Koilonychia affects the shape of nails. The nails start to curve like a spoon. Possible causes include
nutritional deficiencies and autoimmune conditions.
Iron deficiency is the most frequent cause of koilonychia. Iron deficiency anemia is the world’s most
common nutritional deficiency disease. It most commonly affects children and women of childbearing age.
Koilonycia is a nail finding seen in iron deficiency anemia, syphilis, ischemic heart disease.

213. All the following conditions are associated with the sign shown in figure except

a) Pregnancy
b) Hypo estrogen state
c) Rheumatoid arthritis
d) Cirrhosis

Ans. B
Palmar erythema is a skin condition that makes the palms of your hands turn red, palms have turned red
because of dilated capillaries, which are the smallest blood vessels in the body.Mostly associated with
Pregnancy,Liver disease,Autoimmune diseases,Thyroid disease and it is a hyper estrogenic state.
214. Where is this pacemaker lead located ?

a) LA
b) RA
c) LV
d) RV

Ans. D
There are three basic kinds of pacemakers:
1-Single chamber: One lead attaches to the upper or lower heart chamber.
2-Dual-chamber: Uses two leads, one for the upper and one for the lower chamber.
3-Biventricular pacemakers(used in cardiac resynchronization therapy).

215. A patient presented with recurrent syncope episodes. ECG strip shows the following findings.
What is the most
appropriate treatment option for him

a) Pacemaker
b) ICD
c) Anticoagulant
d) Antiarrhythmic drugs

Ans. A
Sinus bradycardia is a type of slow heartbeat. A special group of cells begin the signal to start your
heartbeat. These cells are in the sinoatrial (SA) node. Normally, the SA node fires the signal at about 60 to
100 times per minute at rest. In sinus bradycardia, the node fires less than 60 times per minute.
Treatment of post infectious bradycardia usually requires permanent pacing. In patients with
hypothermia who have confirmed sinus bradycardia with a pulse, atropine and pacing are usually not
recommended because of myocardial irritability.
216. A 30-year-old patient was having breathingdifficulty on doing day to day activity. His
physiciannoticed
dullness onpercussion in right infra-axillaryarea with reduced air entry. CXR showed
pleuraleffusion and
pleural tapping was done. What findings will suggest an exudative pleural effusion?
a) Pleural fluid protein = 3.5 gm% and LDH= 100 IU
b) Pleural fluid protein=4.5 gm% and glucose= 30mg%
c) Pleural fluid LDH =90 IU and glucose=60mg %
d) Pleural fluid protein=3.5gm% and glucose=90 mg%

Ans. A

217. In ED a patient becomes unresponsive and ECG showed arrhythmia. DC shock is immediately
given. But he is
still unresponsive and arrhythmia is persisting. What is the next step
a) Repeat DC shock AV
b) CPR
c) Start bag and mask ventilation
d) Check carotid pulse

Ans. B
218. In ICU a patient is on mechanical ventilator. His ABG shows features of hypoventilation leading
to respiratory
acidosis. What will you do next to improve the ventilation of the patient
a) Increase FiO2
b) Increase respiratory rate
c) Increase PEEP
d) Increase expiratory time

Ans. B
Respiratory failure is divided into type I and type II. Type I respiratory failure involves low oxygen,
and normal or low carbon dioxide levels. Type II respiratory failure involves low oxygen, with high
carbon dioxide.
Type 1 respiratory failure may require only supplementary oxygen, but type 2 failure may require
additional support such as continuous positive airway pressure (CPAP) or biphasic positive airway
pressure (BiPAP) to increase exchange of both gases and, where possible, reverse any causes for low tidal
volumes or low respiratory rates.
219. Which blood vessel is involved in Type II Takayasu arteritis
a) Arch of aorta
b) Arch of aorta and ascending aorta
c) Arch of aorta and descending aorta
d) Abdomen aorta

Ans. B
Takayasu arteritis can be divided into the following six types based on angiographic
involvement :
 Type I - Branches of the aortic arch
 Type IIa - Ascending aorta, aortic arch, and its branches
 Type IIb - Type IIa region plus thoracic descending aorta
 Type III - Thoracic descending aorta, abdominal aorta, renal arteries, or a combination
 Type IV - Abdominal aorta, renal arteries, or both
 Type V - Entire aorta and its branches

Type Vessel Involvement


Type I Branches from the aortic arch
Type IIa Ascending aorta, Aortic arch and its branches
Type IIb Ascending aorta, aortic arch and its branches, thoracic descending aorta
Type III Thoracic descending aorta, abdominal aorta, and / or renal arteries
Type IV Abdominal aorta and / or renal arteries
Type V Combined features of types IIb and IV
220. Which of the following will show pulsus paradoxus
a) Obstructive sleep apnea
b) Pleural effusion
c) Pulmonary hypertension
d) Acute exacerbation of asthma

Ans. D
Pulsus paradoxus (a decrease in the systolic blood pressure during inspiration) results from a decrease
in cardiac stroke volume with inspiration due to greatly increased left-ventricular afterload. Pulsus
paradoxus can be observed in cardiac tamponade and in conditions where intrathoracic pressure swings
are exaggerated or the right ventricle is distended, such as severe acute asthma or exacerbations of chronic
obstructive pulmonary disease.

221. 20-year-old Boy presented with pain in scrotum with red discoloration of scrotal skin. (On
lifting the testis pain was decreased) Diagnosis?
a) Torsion testis
b) Inguinal hernia
c) Hydrocele
d) Epididmyo-orchitis

Ans. D
Prehn's sign is an evaluation used to determine the cause of testicular pain. It is performed by lifting the
scrotum and assessing the consequent changes in pain. A positive Prehn's sign indicates relief of pain upon
elevation of the scrotum and is associated with epididymitis but exacerbates the pain of torsion.

222. A Patient has been admitted to the ICU since he is having fever for past (4 days with new onset
of breathlessness which was unresponsive to incremental flow rate of oxygen and saturation is
progressively decreasing. What can be the diagnosis?
a) Atelectasis
b) Idiopathic Pulmonary fibrosis
c) ARDS
d) Mediastinitis chest pain

Ans. C

Refractory hypoxemia, and this term usually considered when there is inadequate arterial oxygenation
despite optimal levels of inspired oxygen.
A hallmark of COVID-19 is the rapid development of refractory hypoxemia with a poor response to oxygen
supplementation, suggesting intrapulmonary shunting. Refractory hypoxemia in Covid 19 patients
occurs due to moderate to severe ARDS.

223. A 5-year-old boy starts choking at dinner table and is not able to cough or speak. What is the
immediate
management to be done in this child?
a) Abdominal Thrust
b) Drink water / Carbonated drinks fast
c) Give back blows
d) Identify the food consumed
Ans. C

224. 46 xx, absent uterus, vagina, normal secondary sexual characteristics. Ovary present
diagnosis, fsh and Lh
within normal limits?
a) MRKH
b) AIS
c) TURNER syndrome
d) Kliniefilter Syndrome

Ans. A
Bilateral mullerian duct agenesis:
 Mullerian duct is absent  so no fallopian tube ( distal FT is present ) , uterus, cervix, vagina 2/3
 Gonad  ovary
 Lower 1/3 rd of vagina is present ( bcz it is from urogenital sinus )
 Phenotype : female
 Karyotype : 46xx
 Clinical features :primary amennorhea
no mensus( no uterus )
no cyclic abdominal pain
normal secondary sexual characters ( breast development )
 Examination :per vaginal : blind pouch vagina
USG : gonads present , no uterus
 Gold standard investigation :laproscopy
 Treatment : vaginoplasty ( done just before or after marriage ) Williams , Mc indoe’s

225. A 25 year old female attends gynae OPD with c/o secondary amenorrhea. She has history of
previous d and c
and her FSH levels are 6 IU/L . The probable cause of amenorrhea is
a) Sheehan syndrome
b) Ashermann syndrome
c) Premature ovarian failure
d) Pregnancy

Ans. B
Ashermansyndrome :
 It is a rare acquired condition of the uterus
 Scar tissue or adhesions form due to some form of trauma
 Causes :c- section
fibroids/ polyps

 Symptoms :no periods , pain but no bleeding bcz the blood is unable to leave the uterus bcz the
exit is blocked by scar tissue
 Asherman syndrome increases risk during pregnancy : placenta previa
placenta increta
excesssive bleeding
 Diagnosis : Hysteroscopy
 Treatment : operative hysteroscopy : small instruments are attached to the end of the hysteroscope
and used to remove the adhesions

226. 16 yr old girl with primary amenorrhea. Ssc well developed. On examination there is bluish
bulging
hymen. What is the best management?

a) Cruciate incision and drainge of blood


b) long term treatment with estrogen
c) B / l gonadectomy
d) OCP

Ans. A
Imperforate hymen :
 Means when the hymen covers whole opening of vagina
 Symptoms :cryptomenorrhea
lack of menstrual cycle
secondary sexual characters are present
cyclic abdominal pain
 Diagnosis : on P/V : bluish hymen is seen
 Treatment : cruciate incision : x-shaped cut made in the hymen

227. A 55 year old women was found to have a carcinoma cervix figo stage 3b. What is the
management?
a) Concurrent chemoradiotherapy
b) Radiotherapy + hpv vaccine
c) Schauta procedure
d) Intracavitary brachytherapy followed by ebrt

Ans. A

Management of cervical cancer :


 Surgery : is done for stages up till 2a2 except if size of tumour is >4cm (eg : 1b3,2a2 )
 Radiotherapy: done in stage 2b onwards + stages 1b3 , 2a2.
Cisplatin( radiosensitizer): is given before radiotherapy chemoradiation

228. A couple presented with infertility semen analysis is normal, female has menorrhagia. On
hsg there is distal
dilatation with fimbria. What is the likely cause?
a) Tubal polyp
b) Tubal endometriosis
c) Tubal spasm
d) Tubal salphingitis

Ans. D
Gonorrhea : distal dilatation of fimbria is seen
 Etiology :neisseriagonorrheae ( gram –ve diplococcic)
 m/c site : endo cervix >bartholinglan> urethra
 clinical features : asymptomatic
mucopurulent endocervical discharge
urethral syndrome dysuria
pelvic pain
post coital bleeding
 Investigation: culture : MC COY / HELA CELLS
NAAT ( nucleic acid amplification test ) - best test
urine test ( 1st in the mrng )
 complications : 1.FITZ-HUGH CURTIS SYNDROME
2.REITER’S SYNDROME
3.ACUTE SALPINGITIS
4.ECTOPIC PREGNANCY
5. INFERTILITY
 Treatment : doxycycline or azithromycin
 In pregnancy cyclines are contraindicated so RX of choice in pregnancy azithromycin

229. Women with history of previous cs presented with prolonged labor. Patient is
hypotensive. Fhr is not
audible, fetal parts are easily palpable on pa examination, what is the diagnosis?
a) Abruption
b) Uterine rupture
c) Fetal intrauterine death
d) Polyhydramnios

Ans. B
 Most common cause is previous caesarean section
 Mcc innon scarred uterus is obstructed labor
 Sign of impending rupture is fetal tachycardia
 Sigh of rupture is fetal bradycardia

Management:
 Uterine rupture : immediate laprotomy if not possible do hysterectomy
 Impending rupture : immediate caesarean section

230. P212 postpartum presents with a few weeks of spotting and history of bleeding
per vaginum since 3 months
now comes with heavy bleeding. She gives h / o d and c in some other hospital but still continues
to
bleed. What could be the reason?
a) Normal Lochia
b) DIC
c) Retained bits of placenta
d) Gestational trophoblastic neoplasia

Ans. D
Normal lochia- is present for only first 14 days (no sudden bleeding will be seen in this case)
Retained bits of placenta is removed by Dilation and curettage (D&C)
Continuous bleeding per vaginum beyond 12weeks or beyond
Puerperium suspects Choriocarcinoma or Gestational Trophoblastic Neoplasia.

231. Complication associated with iv bolus oxytocin?


a) Hypotension
b) Hypoglycemia
c) Hyperglycemia
d) Hypertension
Ans. A
Oxytocin is given to women during Caesarean section to decrease blood loss. When given as a rapid i.v.
bolus, it causes hypotension and tachycardia.

232. Which of the following peaks is responsible for ovulation

a) A
b) B
c) C
d) D

Ans. B
Usually around the midpoint of your cycle — LH secretion surges to really high levels(LH peak). This
hormone surge is what triggers ovulationabout 24 to 36 hours later.

233. A 23 year old primigravida presents at 19 weeks of pregnancy gestation for the first time. She
ahs no
significant medical history. Her family history is significant for diabetes running in the family. Her
ecg is
normal. Urinalysis is normal
Her FBS- 126 mg/dl
Hba1c- 6.5
Likely diagnosis is
a) Pregestational diabetes
b) Gestational diabetes
c) Normal pregnancy
d) Glucose intolerance

Ans. A
Women with pregestational diabetes mellitus defined by fasting ≥7 mmol/L (126 mg/dL) or 2 h ≥11.1
mmol/L (200 mg/dL),hba1c >6.5

234. False statement regarding following image?


a) Do not use electricity
b) Creates a negative pressure of 660 mm hg
c) Capacity is 30cc
d) Can be used up to 12 weeks

Ans. C
Manual vacuum aspiration (MVA) is a way of surgically treating miscarriage when there is pregnancy
tissue remaining within the womb. MVA uses a narrow tube to enter and empty the womb
using aspiration (gentle suction).There are two methods of vacuum aspiration (also called suction
aspiration): Manual vacuum. This procedure can be used around 5 to 12 weeks after the last menstrual
period (early first trimester)

235. A 24 year old primigravida attends anc clinic with h / o bp 160/120 mmhg. Her lab findings
shows increased
liver enzymes and increased LDH. Low platelet count. Diagnosis?
a) AFLP
b) HELLP
c) Hepatitis B
d) Obstetric cholestasis

Ans. B
HELLP syndrome is a potentially life-threatening disorder that’s usually associated with preeclampsia
 Hemolysis
 EL: elevated liver enzymes
 LP: low platelet count

236. Which type of pelvis is associated with persistent op position?


a) Android pelvis
b) Anthropoid pelvis
c) Gynecoid pelvis
d) Platypelloid pelvis

Ans. B
Android pelvis- MC OP position and associated with deep transverse arrest, it’s a triangular or a male type
pelvis.
Anthropoid pelvis- its anterio posteriorly oval which is associated with persistent op position and face to
pubis delivery.
Gynecoid pelvis- MC and the best pelvis.
Platypelloid pelvis- Rarest pelvis, it is Flat and transversely oval, its associated with face presentation and
Asynclitism

237. An intern conducting delivery observes mother developing breathlessness, hypotension,


tachycardia and
collapsing. Shock is out of proportion to bleeding what is the cause?
a) PPH
b) Uterine inversion
c) DIC
d) Amniotic fluid embolism

Ans. B
Uterine inversion means the placenta fails to detach from the uterine wall, and pulls the uterus inside-
out as it exits.Uterine inversion is an unusual and potentially life‐threatening event occurring in the third
stage of labour. It is associated with significant blood loss, and shock, which may be out of proportion to
the haemorrhage, When managed promptly and aggressively, uterine inversion can result in minimal
maternal morbidity and mortality.

238. A 25 year old primigravida female is on lithium for treatment of her psychiatric illness for
past 2 years . The
m/c anomaly seen in fetus whose mother is on lithium during pregnancy is
a) VSD
b) PDA
c) Ebstein anomaly
d) NTD

Ans. C
 Lithium causes Ebstein’s anomaly if used during pregnancy
 Ebstein’s anomaly is a congenital cardiac malformation
239. A 25 year old women visits Iabour room at 37 weeks. Pt has mild labor pain for 10 hrs and
cervix is
persistently 1 cm dilated and uneffaced. Management?
a) LSCS
b) Arm
c) Sedation and wait
d) Augmentation with oxytocin

Ans. C
After sedation false labor pain subsides
Braxton Hicks contractions are the "false" labor pains that a pregnant woman might have before
“true” labor. They're your body's way of getting ready for the real thing. But they don't mean labor has
started or is about to begin.Before "true" labor begins, you may have "false" labor pains.

240. A pregnant women with prosthetic valve on warfarin tests positive for pregnancy. What is
the advice to be
given?
a) Discontinue warfarin
b) To replace warfarin with heparin
c) Add vit k with warfarin
d) To continue warfarin throughout pregnancy
Ans. B

Warfarin therapy should be avoided during pregnancy. If warfarintherapy is essential, it should be


avoided at least during the first trimester (because of teratogenicity) and from about 2 to 4 weeks before
delivery to reduce risk of hemorrhagic complications.

Upto 12 weeks - LMWH


12-36 weeks - Warfarin
> 36 weeks - LMWH

241. A G4 P0+3 , 24 year old female presents at 22 weeks with previous history of three
midtrimester abortions,
On usg her cervical length is 20 mm What could be the most probable cause of her recurrent
pregnancy loss
a) Cervical Incompetence
b) Uterine Anomaly
c) Hormonal Disturbance
d) Chromosomal Anomalies

Ans. A
Common causes of miscarriage in the second trimister include cervical insufficiency (the premature
dilation of the cervix) or preterm labor (also known as premature birth). With cervical insufficiency (also
known as an incompetent cervix), the baby may be born too early to survive.

242. A pregnant women comes for checkup at 18 weeks. On examination it was found that the
uterine size
corresponds to only 16 weeks. On usg, oligohydrominos was found. Which of the following is likely
cause?
a) Fetal Anemia
b) Anencephaly
c) Renal agenesis
d) DM

Ans. C
243. Which is most cost effective cervical cancer screening test?
a) VIA
b) HPV DNATesting
c) PAP smear
d) Colposcopy

Ans. A
Cost effective
1. Lugol iodine
2. Visual inspection of Acetic acid
BEST :Pap smear

244. A 55 year old post menopausal female attends obgy clinic with history of chronic pelvic pain.
An Xray pelvis
showed the following image The most likely diagnosis is

a) Ovarian mass
b) PID
c) Calcified fibroid
d) Bladder stone

Ans. C
leiomyoma is a benign lesion in postmenopausal age group. It causes diagnostic confusion with
solid calcified adnexal mass and large bladder calculi at the pelvic region. A calcified fibroid is when
a fibroid has reached the final stage of degeneration, or cell death and calcium deposits develop on the
remaining fibroid tissue Clinical and radiological diagnoses were confirmed by histopathology of the
hysterectomy specimen.
245. Which of the following have a false positive Barr body
a) Turners syndrome
b) Trisomy 21
c) Klinefelter syndrome
d) Androgen insensitivity syndrome

Ans. C
A Barr body (named after discoverer Murray Barr) is an inactive X chromosome in a cell with more than
one X chromosome, rendered inactive in a process called lyonization, in species with XY sex-
determination.
Turners syndrome-45XO
Klinefelter syndrome-47XXY
Androgen insensitivity syndrome-46XY

246. What organs are removed in hysterectomy ?


a) Ovary
b) Fallopian tube
c) Cervix
d) Vagina

Ans. C
A total hysterectomy is the removal of the uterus and cervix. A total hysterectomy with bilateral salpingo-
oophorectomy is the removalof the uterus, cervix, fallopian tubes (salpingo) and ovaries (oophor)

247. A female after delivery complains of cramps during breast feeding Which of the following
hormone is
responsible for it
a) Prolactin
b) Oxytocin
c) Estrogen
d) Progesterone

Ans. B
Nipple stimulation during breastfeeding causes a hormone known as oxytocin to be released into your
bloodstream. This hormone causes the contraction of all smooth muscles and helps your uterus contract
back into its pre-pregnancy shape and size. These contractions also help reduce postpartum blood loss,
so although you may be uncomfortable, this cramping is helping your body heal.

248. After binge drinking of alcohol a young male presented with vomiting and upper
Glbleed.Most likely cause
is-
a) Boerrhave syndrome
b) Mallory Weiss tear
c) Duodenal ulcer perforation
d) Tension pneumothorax

Ans. B
In Mallory Weiss syndrome, vigorous vomiting produces a vertical split in the gastric mucosa,
immediately below the squamocolumnar junction at the cardia in 90% of the cases. In only 10% is the
tear in the oesophagus.
Mallory Weiss tear occurs just below GE junction, partial tear involving mucosa and submucosa.
Patient present with painless hematemesis.

249. Following stage is done for which of the following malignancies?

a) Oral cancer - AJCC


b) Bladder cancer- WHO
c) Gastric cancer - Bormann
d) Color cancer -Duke 's

Ans. C

Type 4 ( diffuse infiltrate so known as Linnitisplastica aka leather bottle stomach)


250. A 50year old woman presents to the emergency room for nausea, vomiting, loss of appetite and
abdominal pain. She has a history of crohn’s disease, on infliximab, but she reports that this did not
feel like a flare of her disease. On physical examination, there is tenderness to palpitation of her
abdomen without any peritoneal signs. An abdominal radiograph shows valves of kerekering. She
is started in intravenous fluids, given anti-emetics, and put on bowel rest. What is the diagnosis?

a) Distal obstruction in the small intestine


b) Proximal obstruction in the large intestine
c) Pseudo obstruction
d) Distal colonic obstruction

Ans. A
In image we can see dilated bowel loops with
valvulae conniventes. Valvulae conniventes is seen in jejunem means obstruction is in ileum. Bowel loops
are placed centrally (in large bowel obstruction bowel lops are placed peripherally and we can see
haustration)

Small bowel obstruction Large bowel obstruction


 Colicky pain  Colicky pain
 Pain in peri-umbilical region  In hypogastrium
 Multiple episodes of bilious  Distention is seen
vomiting
 Obstipation present  Obstipation present

251. A 35 year old patient presented with a one year history of low grade fever, anorexia and weight
loss. During
investigation with a contrast evaluation of the bowel, following images were seen. Possible
diagnosis?
a) Ileocaecal tuberculosis
b) Volvulus
c) Cancer colon
d) Diverticulosis of colon

Ans. A
In image the position of caecum is subhepatic (almost located in the right hypochondrium).
Generally the ileo-caecal angle is 900. In tuberculosis we see pulled up caecum i.e., obtuse ileo- caecal
angle which is known as GOOSE NECK DEFORMITY.

252. A female patient presented to the hospital with complaint of vomiting, diarrhea, wheezing,
flushing and palpitations. On laboratory evaluation, high serotonin was found. What is the
diagnosis?
a) VIPoma
b) Somatostatinoma
c) Pheochromocytoma
d) Carcinoid syndrome
Ans. D
carcinoid syndrome can produce number of vasoactive peptides, most
commonly 5- hydroxytryptamine (serotonin), but also histamine,
prostaglandins and kallikrein. When they metastasise to the liver, the
‘carcinoid syndrome’ can become evident, because the vasoactive
substances escape the filtering actions of the liver. The clinical
syndrome itself consists of reddish- blue cyanosis, flushing attacks,
diarrhea, borborygmi, asthmatic attacks and, eventually, pulmonary and
tricuspid stenosis. Classically, the flushing attacks are induced by
alcohol.
other options-
VIPoma →is aka Vernal Morrisons syndrome/WDHA syndrome.
Somatostatinoma→ these patients have DM, steatorrhea and gall stones.
Pheochromocytoma→ headache + diaphoresis + palpitations.

253. In an asymptomatic patient, on routine screening a small mass was detected in the rectum.
Carcinoma rectum was suspected. What is the next step in the management?
a) Colonoscopy with lesion biopsy
b) Sigmoidoscopy with lesion biopsy
c) CT pelvis
d) Barium enema

Ans. B
Rigid sigmoidoscopy can be performed in the outpatient clinic and is useful to identify the neoplasm and
possibly obtain biopsies. However, it requires the rectum to be empty of faces and may require a prior
rectal enema, which may not be practical in the outpatient setting. As colonoscopy is almost always
required to visualize the whole colorectum, it is often easier and safer to obtain biopsies at this time.

Proctoscope 10-12 cm
Rigid sigmoidoscope 25 cm
Flexible sigmoidoscope 60 cm
Colonoscope 160

254. What is the site of maximum pain in acute appendicitis


a) McBurney's point
b) Flank pain
c) C)Right upper abdomen
d) D)Umbilicus

Ans. A
255. Patient presented with history of fever and pain in right lower abdomen for last 5 days. On
palpitation, painful lump was palpable in right iliac fossa. On laboratory investigations,
leukocytosis was found. What is the most probable diagnosis?
a) Hydronephrosis
b) Appendicular lump
c) C.Tuberculosis
d) D.Carcinoma colon

Ans. B

256. A 34 year old female presented with a mass in the inguinal region which produced gurgling
sound on reduction. The mass reduced completely on pressing it through the deep inguinal ring.
Possible diagnosis in this lady?
a) Indirect inguinal hernia
b) Direct inguinal hernia
c) Femoral hernia
d) Pantaloon hernia

Ans. A
Indirect inguinal hernia
Other option—
Direct inguinal hernia (not seen in females)
Femoral hernia (hernia occurs in femoral ring)
Pantaloon hernia (have dual sac)

257. Very old male presented with a ballotable mass in the left flank region. On examination
painless hematuria,
RBC's were found in urine, while there were no WBC's in urine. Most probable diagnosis in this
patient?
a) Hydronephrosis
b) Pyelonephritis
c) Renal cell carcinoma
d) D.Polycystic kidney disease

Ans. C
mass+ pain( in lumbar/loin region) + hematuria
258. An old man presented to hospital with low back pain and sclerotic lesion in vertebrae and
pelvic bone. What
is the most likely diagnosis in this patient?
a) Testicular tumor
b) Lung cancer
c) Colon cancer
d) Prostate cancer

Ans. D
Prostate cancer metastasis to lumbar vertebra.( pelvic bones are also involved)
RCC, colon cancer—lungs
259. Type of bladder cancer seen in the patient with history of working with dye- related
industry?
a) Adenocarcinoma
b) Squamous cell carcinoma
c) Transitional cell carcinoma
d) Papillary carcinoma

Ans. C

260. A patient with history of trauma has ruptured bulbar urethra. What is the location for
accumulation of urine?
a) Superficial pouch
b) Deep pouch
c) Retrouterine pouch
d) Uterovesical pouch

Ans. A
In bulbar urethral injury there is superficial extravasation.
Urethral injury is of 2 types
Anterior and posterior ( C/F- urinary retention, blood in meatus and hematoma)
In anterior urethral injury – perineal hematoma
In posterior urethral injury – pelvic hematoma

261. 37 years old women came to the hospital with the folloing presentation (image) what is the t-
staging ofof
the cancer?

a) T4a
b) T4b
c) T4c
d) T4d

Ans. B
262. 60 years old lady presented with a hard breast mass of 5x6cm in size, single lymph node mass
present. There
was no evidence of metastasis? What should be the ideal treatment?
a) Radical mastectomy with chemotherapy
b) Modified radical mastectomy followed by adjuvant chemotherapy
c) Neoadjuvant chemotherapy with Modified radical mastectomy with radio therapy
d) Palliative mastectomy.

Ans. A

263. A women with breast malignancy as shown in image,it is not associated with lymph node
metastasis, not movable and attached to the rib ,what is the diagnosis?

a) Phyllodes tumour
b) Brest abscess
c) Inflammatory breast cancer
d) Leiomyosarcoma

Ans. A
264. A lactating female with history of breast feeding for 15 days presented to emergency with
fever. On examination, breast was red as shown in the image .what is the probable diagnosis?

a) Breast abscess
b) Milk fever
c) Tuberculosis
d) None of the above

Ans. A

265. Which of the following radio-isotop is used for carcinoma thyroid?


a) I-123
b) I-125
c) I-127
d) I-131

Ans. D
I-123 =13 Hrs used for diagnosis RAI(radio active iodine up take scan)
I-131 =8 days used for diagnostic and therapitic
Also used for Radio active ablation

266. A patient came with the pain in right hypochondrium .bloodinvestigantion shows AST-22 IU/l
and ALT-27 IU/ L,serum bilirubine levels -6mg% and ALP-200 mg % what should be the
investingation in thus patient?
a) USG abdomen
b) CECT abdomen
c) Hepatitis A and Hepatitis E markers
d) HIDA scan

Ans. A

267. A female patient presented to the hospital with history of recurrent pain in the right
hypochondrium for 1 year. what is the diagnosis?
a) Hydatid cyst
b) Acute pancreatitis
c) Acute cholecystitis
d) Amobic liver abscess

Ans. C
268. A 60 years old male who is heavy alcoholic present with 12 hours history of sharp central
abdominal pain and vomiting, preferred to nsit, rather lying down as pain increases when lying
down. pulse rate 132/min, bp 118/82 mmhg, temperature is 38.3 c. on examibnation he has mild
jaundice. There is periumbilical and flank discoloration as shown in the image and minimum
tenderness in epigastrium. What is the diagnosis?
a) acute pancreatitis
b) pyelonephritis
c) acute viral hepatitis
d) perforated duodenal ulcer

Ans. A
key points
BLUISH DISCOLERATION OF THE FLANKS =GREY TURNER SIGN
DIS COLERATION AROUND UMBILICUS =CULLEN SIGN

269. Alcoholic male presented with pain in epigastrium radiating to the back. What is the likely
diagnosis in the given CT image ?

a) Amoebic liver abscess


b) Pancreatic pseudocyst
c) Acute cholecystitis
d) Hydatid cyst

Ans. B

270. What is the name of surgery, whose image is given below?


a) Pancreatiojejunostomy
b) Whipples surgery
c) Pancreaticoduodenostomy
d) Duodenojejunostomy

Ans. A
also called : PUESTOW’S PROCEDURE LATERAL PANCREATIOJEJUNOSTOMY
Involves a retrocolic side-to-side roux-en-ypancreaticojejunostomy

271. What is the name of procedure, whose image is given below?

a) Pancreaticojejunostomy
b) Whipples procedure
c) Radical pancreatectomy
d) Duodenojejunostomy

Ans. B

272. In a new born male baby, mother found empty scrotum.on examination, left testis is in
inguinal region and
right testis is in perineum. Which of the following is correct?
a) Right testis is undescended and left testis is ectopic
b) Left testis is undescended and right testis is ectopic
c) Both are undescended
d) Both are ectopic

Ans. B
left – is at inguinal region –undescended
right is at perienguinal- ectopic

273. In a new born bady with empty scrotum, testis was was found to be laying close to the
peritoneum.which of
the following helps the testis to reach the scrotum?
a) GnRH Analogues
b) Testosteron
c) Cold compression
d) Manually bring testis down to the bottom of scrotum

Ans. A
Maternal chorionic gonadotrophin growth of the testis and may stimulate its migration so we give baby
GnRH analog that helps the testis to reach the scrotum
274. 18 years old boy presented in agony with sudden onset of severe pain in the groin region,
associated with
redness and swelling of the scrotum. On examination the right testis as gone up. Possible
diagnosis in this
boy is?
a) Testicular torsion
b) Hydrocele
c) Strangulated inguinal hernia
d) Acute epididymo-orchitis

Ans. A
Testicular torsion:

Hydrocele:

Strangulated inguinal hernia:


epididymo-orchitis:

275. A 35 years old male presented with scrotal enlargement on investigation, LDH was significantly
high but the HCG and alpha fetoprotin were within normal limits .histopathology image is shown
below what is the most
likely diagnosis?
a) Teratoma of testis
b) Seminoma of testis
c) Choriocarcinoma
d) Yolk sac tumor

Ans. B

276. A young male underwent lumbar sympathectomy and bilateral L1 ganglions were removed.
What is the most commion side effect?
a) Impotence
b) Retrograde ejaculation
c) Bladder dysfunction
d) Incontinence

Ans. B
patients with burgers diseas who have rest pain we perform lumbar sympathectomy,
in this we remove L1 and L2 ganglion on one side
if we remove both sides the patient will have retrograde ejaculation
it can be prevented by sparing of L1 at one side

277. A patient under went surgery, from which she was immobilized.patient started complaining
of leg pain on the 5th post operative day, on examination Homans sign was found to be positive.
What is the most probable
diagnosis?
a) Deep venous thrombosis
b) Urinary sepsis
c) Peripheral vasculsar disease
d) Lymphangitis

Ans. A
278. A patient with old burn scar develop symptoms over burnt area . it was indicative of
malignancy.what is the type of malignancy?

a) Basal cell carcinoma


b) Squamous cell carcinoma
c) Keratocanthoma
d) Sebaceous horn

Ans. B
given condition-MARJOLINS ULCER

279. A patient with blunt trauma was brought to the emergency with the pulse rate 120/min
bp80/50. What is the next best step?
a) FAST
b) DPL
c) Exploratory laprotomy
d) CECT

Ans. A
Patient whether stable or unstable first FAST should be done
(FAST-focused assessment with sonography for trauma)

280. A patient of blunt abdominal trauma was brought in the emergency with cold clammy skin
and increased puls rate. What is the next step of management?
a) CECT
b) Exploratory laprotomy
c) Resuscitation with blood products
d) Resuscitation with intervenous crystalloids

Ans. D
given only one liter of warm isotonic crystalloids for adults
child <40kg given 20ml/kg of warm isotonic crystalloids

281. A patient under went a surgery for ingunial hernia 5 days before. Now comes to the hospital
with complaints of fever. What can be the cause?
a) Atelectasis of lung
b) Surgical site infection
c) Thromboembolism
d) Chest infection

Ans. B
cause of post-op fever
POD 1-2: Atelectasis
POD 2-3: UTI
POD 3-7: Wound infection (ssi)
POD 5-7: DVT/Thrombophlebitis
POD >7: drug fever
282. Identify the movement taking place in the intestine?

a) Segmented movement
b) Peristaltic movement
c) Passive diffusion
d) Intersegmental contraction

Ans. B
GI – Motility
 Types
2- types
Food
Major inititoing response
for persislassis is – local
stretching of gest
Yes No
Fed state pattern food Fasting state pattern
Para & of each

 Food F
Contraction felaxation 2

1 Peristalasis ‘food’ Contraction Relaxation


Oral Anal 1 Arch – I Nitric Oxide (No)
food 2 Substance – P
Balance
 Ach
 LES  Absence of
No
Rx :
Inhibits Arch  Botulinum toxin Achalasa Cardia
283. Pressure volume graph given and asked to find out B accordingly?

a) Opening of aortic valve


b) Opening of mitral valve
c) Closing of aortic valve
d) Closing of mitral valve

Ans. A

284. Which part of nephron does water reabsorption takes


place more?

a) A
b) B
c) C
d) D
Ans. A
About 2/3rd of the filtered water is reabsorped in proximal convoluted tubule along with salt
absorption.other molecules absorped here are bicarbonates, glucose, amino acids and phosphates.

285. Inhibition of Na+ - K+ ion channel results in which of the following?


a) First depolarise and then hyperpolarise
b) Hyperpolarisation
c) Depolarisation
d) Repolarisation

Ans. C

286. Hormone that regulate serum calcium levels


a) Calcitonin
b) Vitamin D
c) Parathyroid hormone
d) All

Ans. D
Calcium Regulation :
3 HORMONES 3 organs modal
1. Parathormone 
2. Vit D Ca2+ 1. Bone (Bank for Ca2+)
3. Calcitonin   Ca2+ 2. GIT
3. Kidneys

Parathormone (Actions)
1. Bone  2 cells PTH PTH
Osteo blast Osteo.Clast  Bone resorption
PTH receptor    + Ca2+ - PO42-
RANK ligand Crystal
Rx : Osteoporosis

Rank Ligand Blocker
DENOSUMAB  Ca2+  PO42-
Bone (PTH)
2) Kidneys G PTH
 Excreation of Ca2+
 Excreation of PO42-
PTM
 Phosphaturic action
Over all action
PTH  PO42- Terminating Hormone

3) GIT   Vit D   Ca2+ absortion


&
PTH + Indiract action

287. Why can't hemoglobin pass through glomerulus?


a) Large molecule bounded with protein
b) Cone of protein decrease glomerular capillaries
c) Negative charge of bowmans capsule
d) None of the above

Ans. A
Protiens  not filted by GC Glossullary capillary
3 reason

1. Charge Repals
-Ve
Protein Anions
Gc
?
Heparan, sulfate
Heparan SO42-
(Proteoglycans)

2. Size
GC – Small gap
Fenestrations  Fenestrated Endothelium
10 – 90 nm

3. Slit diaphram :
Structure
Proteins
i. Neprin  NPHS 1 geno
mutation
Congenital. N syndrome = (protein Excretion)

Finish type

ii. Podocin  NPHS2


 gene
Mutation
AR. Steroid resistant Nephrotic syndrome ARSRNS

iii. Alpha actin :


 Mutation
FSGS. Focal Segmeatal glomullory sclerosis

288. In gestational life surfactants are produced by which cells?


a) Type I pneumocytes
b) Type II pneumocytes
c) Basal cells

Ans. B
Type I pneumocytes Type II pneumocytes
Large Small
Less in number More in number
More area SURFACTANT
Stem cells

Surfactant :
Lipids (major—lecithin ), (minor –spingomyelin)
Protein :A,B,C,D
In amniotic fluid—bubble test, shake test, clement test.
Function of surfactant:
 ↓ in surface tension
 Prevent collapse of alveoli
 Stability of alveoli
 SP : A and D → lung maturity
Surfactant stored in Lamellar body.
Regulation of surfactant :
↑surfactant : steroids, glucocorticoids, cortisol, T3 and T4.

289. In child which appears first?


a) Palmar grasp
b) transfering objects
c) uni dexterous approach
d) bi dexterous approach

Ans. A
Palmar grasp (immature) should logically also appear first followed by transferring objects
Bi dexterous appear by 4th month
Unidextrous appear by 6th month

290. A 4 years old child,presented with thisty,drinks eagerly,irritable with dehydration, what the
treatment ?
a) ORS
b) Ringer lactate
c) Plain normal saline
d) Refer to district hospital

Ans. A
DEHYDRATION:
Skin pinch test

NO dehydration No sign / < 1 sec


symptoms
Some Thirst slow <2 sec
dehydration ++,Irritable +
Severe lethargic, very slow >2 sec
dehydration inactive

Calculation of fluids by HOLIDAY SEGAR REGMIN


First 0-10 kg = 100ml/kg
Next 10-20 kg= +50 ml/kg
>20 kg = +20 ml/kg

TREATMENT PLAN FOR DEHYDRATION:

No Dehydration Some Dehydration Severe dehydration (lethargy)


PLAN A PLAN B Plan C
 ORS 100-  Rehydration (75ml/kg in 1st  Start IV fluids
200ml/loose stool 4 hrs) + maintenance  Ringer lactate >>Normal saline
(ongoing losses 10- (replacing on going loss-
20ml/kg) 10-20ml/kg) Give 100 ml/kg as :
Age 30ml/kg 70ml/kg
<1yr 1hr 5 hr

>1yr 30 min 2 ½ hr

Patient comes with an enlarged leg with sulphur crystals coming out (madura foot as shown in the
image). Organisum responsible is:

Explanation :
Actinomycotic mycetoma: Sinus ,swelling,granules(discharging)

Actinomycetoma Eumycetoma
 Numerous sinus and swelling  Single sinus and swelling
 Colour of granules→ yellow/white except  Black →madurella mycetomatis
in Actinomadural pellitri where they have  White
red granules  Acremonium
 Respond to antibiotics  Pseudallescheria boydii
 Amputation

CTEV

 Dennis brown splint used to correct foot


 Uses: C A V E C: cavus
A: adduction
V: varus correction
E: equines ( tibia and thalus involved )
 Keep the foot in over corrected position
Right Midble Lobe Consolidation

 Image 1: Area of consolidation middle lobe which silhouette right heart border
 Image 2: cardiac shadow is seen on the middle lateral lobe

Diffuse Esophageal Spasm

 Cork-screw/rosary bead esophagus on barium studies

Q. Implantable Cardioverter-Defibrillator [Icd]

Plain radiograph
 For implantable cardio-defibrillator ( ICD) , a chest x-ray should be obtained to confirm lead
position and examine lead integrity
 Most pulse generators, both pacemakers and ICD’s, can be identified from their appearance
under radiography: most helpful method of identifying unknown devices
Q. What is the radioisotope used in PET scan ?
Ans: 18FDG
ANNEXURE 5. HALF – LIVES OF RADIOACTIVE ISOTOPES
Substance Half – life
Iodine – 132 2.3 hours
Technetium – 99 6 hours
Iodine – 123 13 hours
Gold – 198 2.7 days
Gollium -67 3.3 days
Iodine – 131 8 days
Phosphorus -32 14.3 days
Iodine -125 59 days
Iridium – 192 74 days
Cobalt -60 5.3 days
Strontium – 90 29.1 years
Caesium -137 30 years
Radium – 226 1600 years
Uranium – 238 4.5 billion years

 Half- life: 110min

Q. Oneliner: To know correct position of IUCD USG is best

Q. Identify the cysyt?

Q. Identify the fibroid ?


ARISE
MEDICAL ACADEMY
Building Better Doctors

All
the
Best

Arise Medical Academy


Add us on FB Page : - /arisemedacademy
www.arisemedicalacademy.com
ARISE
MEDICAL ACADEMY
Building Better Doctors

Arise Medical Academy


Add us on FB Page : - /arisemedacademy
www.arisemedicalacademy.com
ARISE
MEDICAL ACADEMY
Building Better Doctors

Arise Medical Academy


Add us on FB Page : - /arisemedacademy
www.arisemedicalacademy.com
RISE
FMGE iNSiGHT

Follow us on
f cy d
ARISE - Hyderabad
+91 7680929292
040 2351 5252
# 2ⁿ Floor, Above Indian Bank,
Opp. Olive Hospital,
Kakatiya Nagar Colony,
Mehdipatnam, Hyderabad - 500 028
arisemedicalacademy@gmail.com
ARISE - Kerala (Kannnur) ARISE - Delhi
+91 8136932666 +91 9100728811
+86 15778411369
# 2ⁿ Floor, Kingdom Tower, Goutham Nagar,
Manna, Taliparamba,
Kannur, Kerala.
arisemedicalacademy.kerala@yahoo.com arisemedicalacademy@gmail.com

Arise Medical Academy


Add us on FB Page : - /arisemedacademy
www.arisemedicalacademy.com

You might also like

pFad - Phonifier reborn

Pfad - The Proxy pFad of © 2024 Garber Painting. All rights reserved.

Note: This service is not intended for secure transactions such as banking, social media, email, or purchasing. Use at your own risk. We assume no liability whatsoever for broken pages.


Alternative Proxies:

Alternative Proxy

pFad Proxy

pFad v3 Proxy

pFad v4 Proxy